Emergency Medicine EOR

Lakukan tugas rumah & ujian kamu dengan baik sekarang menggunakan Quizwiz!

Heat edema; heat rash; heat cramps; heat syncope; heat exhaustion; heat stroke. ______is a life-threatening form of heat illness that typically occurs in patients with core temperatures greater than 41C/106F. What is the preferred method for rapid cooling.

Heat stroke; ice water immersion

AFib

Irregularly Irregular Rate Control Rhythm Conversion Cardioversion if unstable

Neurogenic shock

Hypotensive situation resulting from the loss of sympathetic control of vital functions from the brain. Happens with pt with injury above the 6th thoracic vertebra. - Loss of sympathetic tone below the level of injury

What are causes of orthostatic hypotension?

Hypovolemia Dehydration Meds (Diuretics) Endocrine, neurologic, metabolic disorders

Three types of hypernatremia are hypovolemic, isovolemic, or hypervolemic. Give examples of etiologies of each type.

Hypovolemic hypernatremia - lack of access to water, excessive sweating Isovolemic hypernatremia - renal water losses - diabetes insipidis, fever, burns Hypervolemic hypernatremia - seawater ingestion, cushings, exogenous steroids

What is the number one question you want to ask if a patient present with vaginal bleeding?

Is she pregnant? - Pregnancy-related complications are the most common cause of abnormal vaginal bleeding in women of reproductive age

______is acute inflammation of the lacrimal gland. Tx: observed and treated with abx if needed. Superior temporal region. ______is infection of the lacrimal gland. Tx: antibiotics and relief of the obstruction. Inframedial region.

Dacryoadenitis; Dacryocystitis

abscess tx

I and D followed by Bacitracin

Influenza - caused by influenza A or B. Antivirals can shorten the course. Rimantadine and Amantadine can be used to shorten course of ______. Zanamivir and Oseltamivir can shorten the course of _____.

Influenza A; Influenza A or B.

Pt presents with anesthesia of the maxillary division (V2) of the trigeminal nerve, what are you thinking?

Infraorbital nerve entrapment

______is a rare complication of otitis media. CP with pain, swelling, tenderness, and redness behind the ear in the area of the mastoid bone. Treat the same as otitis media but what is the difference?

Mastoiditis; treat for 3-4 weeks

is a holosystolic murmur is heard, think of what 3 conditions

Mitral regurg. Tricuspic regurg. Ventricular Septal Defect

______is a pansystolic, blowing in nature, high pitch, musical sound. Radiates to left axilla.

Mitral regurgitation

Describe the pathophysiology of right sided heart failure

RV can't pump so there is a back up of blood from the periphery

What are different kinds of SVT?

Reentrant SVT - present with PSVT Ectopic SVT

EKG changes with an NSTEMI?

ST segment depression Q-wave

What is the difference between systolic and diastolic HF?

Systolic HF is a pumping problem due to decreased contractility which = decreased EF. Diastolic HF is a filling problem due to the heart being stiff (Ex: LVH). 40% of patients have diastolic HF. EF is preserved.

name four classes of drugs that are commonly overdosed on resulting in coma

TCAD, benzos, opiates, antiepilectics

Clinical features of cardiac tamponade?

Tachycardia Tachypnea Narrow pulse pressure JVD Pulsus paradoxus (breathing in causes drop in BP)

which type of line needs to be placed in shock patients

central (femoral)

CHF - Diastolic dysfunction means a problem with the ____.

compliance or relaxation of the heart during ventricular filling

major ddx for stridor and drooling in a child

epiglotitis (child looks very anxious/toxic), Croup (this will have a cough whereas epiglotitis will not), foreign body aspiration (sudden onset of coughing and choking; the acute nature differentiates from croup)

patent ductus arteriosus Tx

indomethacin

What is the major jone's criteria for rheumatic fever

migratory arthritis, carditis, valvulitis, CNS involvement (chorea), erythema marginatum, subcutaneous nodules

high voltage electrical shocks/burns are those that are over 1000J. what is one of the complications of this to watch out for

minimally compartment syndrome

indications for CABG

minimally three vessel dz or dz of LAD (although I'm less sure on this latter)

utiliity of neck xrays for patient suspected to have epiglotitis

only order if you are unsure of the diagnosis; generally you don't need em' as presentation says it all, may be able to see the epiglotits without xray

HPI of spontaneous pneumothorax

sudden onset pleuritis chest pain typically of affected side, SOB, typically in a tall male that smokes. oxygen sat is nice to look at but only has positive predictive value, sinus tachycardia is the most common presentation (which is also the case for PE)

What is the treatment for constrictive pericarditis?

pericardiectomy --- otherwise NSAIDs and Steroids

A ______presents with asymmetric swelling of peritonsillar soft tissue and deviation of the uvula AWAY from affected side -- CP: sore throat, drooling, hot potatoe voice, trismus.

peritonsillar abscess

Paroxysmal supraventricular tachycardia is a reentry tachycardia, commonly noted in elderly patients with underlying heart disease. What treatment may be helpful before using adenosine ie. the drug of choice?

vagal maneuvers or antianxiety medication

acyanotic cardiac defect that results in increased pulmonary blood flow; left-to-right shunts; most common congenital heart dz (25%)

ventricular septal defect

pt seen in ED after being hit in the eye w/ a wrench. didn't have any pain; what does this say about corneal abrasion?

very unlikely as this almost always has associated pain

what can cause cavitary lung lesions - generally speaking

Cavitary lesions of the lung have multiple causes, including both infectious and non-infectious etiologies. These include bacterial pneumonia, fungal disease, tuberculosis, malignancies and some pulmonary vascular disease.

PE and CXR of pleural effusion

dec breath sounds dull percussion decrease tactile fremitus CXR: blunting of the costophrenic angle

define pulsus paradoxus

decrease in systolic blood pressure by at least 10mmHg upon inspiration; can occur minimally w/ cardiac tamponade, asthma, obstructive sleep apnea, croup,

What are clinical features of Impetigo?

"Honey colored crust" Erythematous macules, vesicles, bullae Often occurs in pre-existing skin conditions such as eczema

What are clinical features of a aortic dissection?

"Ripping or "tearing" chest pain radiating to the back Severe back, abdominal, flank pain Hypotension/shock

Septal Hematoma

- Collection of blood between the mucoperichondrium and the cartilage of the septum - Appears as a blue, boggy, tender area of swelling along the nasal septum

HR range for simple tachyarrhythmia

100-150bpm

Prostatitis can be acute bacterial, chronic bacterial, or chronic nonbacterial. For acute bacterial, fluoroquinolones or Bactrim are indicated for _____. For chronic bacterial, fluoroquinolone or Bactrim are indicated for ______.

4 weeks; 1-3 months

Endotracheal tube should be placed about ____ cm from the carina

5 cm

When can fetal heart tones be appreciated?

9-12 weeks

What is primary amenorrhea?

Absence of menstruation by age 16

What must be managed first in any trauma pt?

Airway!

Best imaging modality for an orbital wall fx?

CT scan

Head trauma imaging modality of choice

CT scan

Cardiac Markers: _____appears 3-6 hours after MI and stays elevated for 2-4 days. Specific to heart muscle.

Creatine kinase Mb

What are the two classes of aortic dissection?

Debakey Stanford

How is an acute asthma exacerbation treated?

Duoneb tx (albuterol + ipratropium) q 20 minutes x 3

How is VVC diagnosed? Treatment?

Dx: KOH shows yeast buds Tx: Nystatin, miconazole, or clotrimazole

coarctation of the aorta Dx

ECHO is diagnostic diagnosis also made by PE confirmed by 4-extremity blood pressures

tetralogy of fallot Dx

EKG > RAD, RVH CXR-boot shaped heart

What should be on your DDx with vaginal bleeding and a positive pregnancy test?

Ectopic pregnancy Spontaneous abortion Placental abruption Placenta previa Post-partum hemorrhage

Blood coming from the tooth indicated what type of fx?

Ellis III

Imaging test of choice for maxillary trauma?

Facial CT scan

What three things are hypercoagulable states that increase risk of DVT?

Factor V Leiden; cancer; HELLP (hemolysis, elevated liver enzymes, and low platelets in pregnancy)

What causes erysipelas?

GAS

Mild TBI

GCS 14-15

What are the 4 most common causes of primary amenorrhea?

Gonadal dysgenesis (Turner syndrome) Hypothalamic-pituitary insufficiency Androgen insensitivity Imperforate hymen

What is the Rule of 9's?

Head and neck - 9% Each arm - 9% Each leg - 18% Anterior trunk - 18% Posterior trunk - 18% Perineum - 1%

_______sign is pain with dorsiflexion of the foot. May be positive with DVT.

Homan's

What is Asherman syndrome?

Intrauterine adhesions usually due to trauma (usually from D&C)

What causes central vertigo?

Issue with balance centers of the brain

Nearly all cases of epistaxis originate anteriorly within the mucosa of the nasal septum in a region known as ____. Why is posterior epistaxis harder to manage?

Kiesselbach plexus (anterior) vs. Woodruff plexus (posterior); posterior is harder to manage because application of direct pressure does not compress area

What is burn shock?

Loss of fluid from the intravascular space as a result of burn injury which causes "leaking capillaries" that require crystalloid infusion

What is chronic venous insufficiency?

Loss of wall tension of veins which causes stasis. Associated with history of DVT, leg injury, or varicose veins.

How does MR sound? Where is it heard and where does it radiate to?

Loud, BLOWING, high-pitched, PANSYSTOLIC at apex radiating to axilla

Duke Criteria for dx of infective endocarditis Dx with 2 major or 1 major and 3 minor

Major a. positive blood cultures b. endocardial involvelment on echo Minor a. predisposing condition: cardiac or IV drug use b. fever c. vascular phenomenon d. immunologic phenomenon

Second most common facial fracture?

Mandibular Fx

How do you treat BV?

Metronidazole 500mg PO BID for 7 days or gel (5 days) Clindamycin cream x 7 days

What is the pregnancy rate with mild endometriosis? Moderate? Severe?

Mild - 75% Moderate 50-60% Severe - 30-40%

Which heart blocks require pacing?

Mobitz Type II 3rd degree

Most frequently fractured bone in the face?

Nasal bone

What causes Gonorrhea?

Neisseria gonorrhoeae - a Gram neg intracellular diplococcus transmitted by sexual activity. MC in 15-29 year olds. Incubation is 2-8 days after exposure.

What are some compensatory mechanisms for CHF?

Neuro-hormonal system RAAS Ventricular hypertrophy

What are clinical features of endocarditis?

Non-specific symptoms (fever, cough, dyspnea, arthralgias, GI complaints, pallor) Splenomegaly Murmur that may change PATHOGNOMONIC findings (occur in 25% of pts): Palatal, conjunctival, or subungual petechia Splinter hemorrhages Osler nodes (painful, violaceous, raised lesions of the fingers, toes, feet) Janeway lesions (painless red lesions or palms or soles) Roth spots (exudative lesions of the retina)

Who is most commonly affected by endometriosis?

Nulliparous women in their 20s and early 30s

ED tx for anaphylaxis?

O2, prepare to intubate if necessary Limit exposure to allergen Epinephrine - first line med IVF with NS Steroids H1+H2 antihistamines (Benedryl + Zantac) B2 agonists or duo-neb

What is Peripheral arterial disease (PAD)?

Obstruction of arteries NOT within the carotids, brain, or aortic arch. Commonly caused by atherosclerosis.

Pulmonary Embolism

Occlusion of pulmonary artery Most originate from thrombi in pelvis/LE >400K per year Current or previous , Malignancy, obesity, estrogen, immobility, trauma, surgery are risk factors S/Sx: Dyspnea, pleuritic chest pain, hemoptysis (late), tachypnea, tachycardia

How does mitral stenosis sound?

Opening SNAP Low-pitched mid-diastolic rumble (because blood is trying to get in to LV)

35yo male presented to the ED w/ a fluctuant mass diagnosed as an abscess...

PA inquired about status of tetanus as the abscess has to originate somewhere... whether or not an abscess can start w/ a wound I do not know, but it does serve as a good reminder to inquire about tetanus status in a patient with a wound

Hallmark of placenta previa?

PAINLESS vaginal bleeding is hallmark

What are characteristics of second degree superficial burns?

Red, painful Wet with blisters Most common cause: Scalding Heals 14-21 days May or may not have scarring

Tx for pericarditis?

Steroids NSAIDs Antibiotics if bacterial Pericardiectomy if constrictive pericarditis

most common etiological agent of pneumonia

Strep pneumo

T/F Trauma pts generally need a CT of Head, chest, and abdomen because of significant mechanism of injury

TRUE

What is Placenta previa?

The placenta partially or completely covers the cervical os

What is concealed abruption?

There is blood retained between the detached placentae and the uterus

If a pt presents with a flattened cheek bone, what type of fracture are you thinking?

Tripod fracture

Cardiac Markers: _____is the test of choice and appears 2-6 hours after MI and stays elevated for 5-10 days.

Troponin

Torsed Gonad

Twisting of ovary, testicle, or fallopian tube around is vascular pedicle Hx of enlarged ovary, recent physical activity, Sudden, severe, unilateral pain, *irreversible ischemia after about 6 hours*

How do you treat a black widow spider bite?

Tx: Antivenin (rarely indicated and not readily available) Diazepam Calcium gluconate

FAST Scan

Ultrasound used to evaluate deep, blunt abdominal trauma at the bedside in the ER - Study of choice in trauma setting

_______refers to inflammation of the urethra. Gonococcal produces a thick, purulent urethral discharge and burning with urination. Nongonococcal presents with dysuria and scant urethral discharge. May be suggested if urine dipstick test for leukocyte esterase is positive and no bladder infection is present.

Urethritis

Gross hematuria is indicative of?

Urologic injury (Bladder)

What is initial treatment for PSVT in a stable patient?

Vagal maneuvers: Valsalva, ice water bath, carotid sinus massage Adenosine - 6mg IV bolus BB/CCBs - for narrow SVT Procainamide - for wide complex SVT

there is one arrhythmia that adenosine is absolutely contraindicated in... what is this arrhythmia

WPW

When should Rhogam be given in pregnancy?

When the mom is Rh-negative and the baby is Rh-positive. Mom may develop antibodies to the Rh antigen and cause hemolysis of the fetal red blood cells in subsequent pregnancies

Treatment for brown recluse?

Wound cleansing Analgesia

What are risk factors for primary dysmenorrhea?

Young age Menarche before 12 Irregular or heavy menses BMI >20 Smoking

name two severe pediatric airway infections

acute epiglotitis and croup

mainstay of tx in acute epiglottitis

airway management

anatomical structure that harbors pneumonia

alveoli

a note about upper abdominal pain

always consider this as possibly referred cardiac origin... ask about ACS ssx

What is the number 1 cause of syncope?

arrhythmias

What is the minor jones criteria for RF?

arthralgia, fever, elevate ESR or CRP, prolonged PR interval

which class of drug should be given to patients w/ hypertrophic cardiomyopathy?

beta blocker; goal is to increase preload

where does air collect in a pneumothorax

between the potential space of the visceral and parietal pleura

always ask patients presenting w/ CP about a hx of HA... why?

both can be caused by vasospasm... if you get vasospasm in one area (i.e. heart aka Prinzmetal angina) you're likely to get it elsewhere (i.e. HA) thus hx of migraines reduces likelihood of true angina

the first thing that needs to be assessed in a patient w/ abdominal pain is possibility of surgical abdomen. which two abdominal conditions are candidates for abdominal surgery

bowel obstruction, peritonitis

i know well that miosis is a sign of opioid intoxication. what are some others?

bradycardia, respiratory depression

Treatment of acute symptomatic hyponatremia is with hypertonic saline (3%); what can happen if saline is given too rapidly?

cerebral edema or central pontine myelinolysis

CRAO: Meds such as timilol, acetazolamide, and mannitol may help to reduce intraocular pressure. What does the macula appear like on exam?

cherry red spot

in the evaluation of a pt that just had a MVC, check for the seatbelt sign. presence of a seatbelt sign is an indication for what imaging modality?

chest abdomen and pelvic CT according to Jared Campbell

simple definition of angina

chest pain w/ exertion

pathogenesis of cardiac tamponade

collection of fluid (blood) in the pericardial space which consequently compresses the heart and thus get poor systemic circulation

MMC of transudative pleural effusion

congestive heart failure (other causes: cirrhosis, nephrotic syndrome, pulmonary embolism)

Patients with branch retinal artery occlusion and central retinal vein occlusion present with what classic physical exam finding?

cotton wool spots

pericarditis itself isn't too worrisome; what are the feared complications?

could progress to restrictive pericarditis or cardiac tamponade

tetralogy of fallot Sx

cyanosis murmur squatting "tet" spells (rapid breathing & increased cyanosis) loud systolic ejection murmur at LUSB, right ventricular heave, S2 is always single

synonym for encephalopathy

delirium (although in my opinion delirium is probably a symptom of encephalopathy)

utility of a FAST exam

demonstrates collection of fluid in potential spaces. a negative FAST exam means nothing, so it is used for its positive predictive value. if the first is negative, need to do repeat(s)

ECG manifestations of pericarditis

diffuse ST segment elevation

electrical tx for wide complex Vtach

electrical shock w/ 200J

ddx for RUQ pain

gall bladder issues, lower lobe pneumonia (I keep forgetting about this one), probably others, I just wanted to emphasize PNA

severe HTN tx

get that blood pressure down w/ IV vasodilators such as Nitroglycerin, esmolol, labetolol. this should be done w/ an emergency neurosurgery consult

initial tx for a tension pneumothorax

go with a clinical diagnosis and do a needle decompression before ordering an Xray as time is of the essence (Tintinalli text)

the general tx for a PE is antiplatelet therapy. however, name the indication for a thrombolytic such as tPA

hemodynamic instability

Pulmonary ____occurs when resistance to flow across the pulmonary vasculature increases. - may see prominent upper lobe pulmonary veins, increased density in the central lung fields, and Kerley B lines.

hypertension

most common cause of sudden cardiac death (SCD) in the young athelete

hypertrophic cardiomyopathy

Secondary hypertension is HTN due to an ______.

identifiable cause ie. renovascular disease, coarctation of the aorta, primary aldosteronism, Cushing's, Pheochromocytoma, OSA, renal parenchymal hypertension

an artificial airway is indicated for any kid presenting w/ ssx consistent w/ epiglotitits, minimally until age 6 (this is a controversial subject); when should it be used in adults?

if over 50% of the airway is obstructed

No nitroglycerin in which kind of MI?

inferior

autoimmune disease of inflamed medium-sized vessels (coronary artery aneurysms)

kawasaki disease

if you were to order one, just one, test to make the diagnosis of sepsis, what would it be?

lactate. that's lactate, not lactate dehydrogenase. this is even better when ordered sequentially/trending. drawback is that it lacks specificity and thus needs to be combined with clinical judgment

ST Elevations in I, aVL, V4, V5, V6

lateral wall MI, left circumflex artery

bicarb supplementation is appropriate minimally in patients w/ DKA or HHS if the arterial pH drops below a certain value... what is this value?

less than 6.90

which lab has better sensitivity and specificity for pancreatitis--amylase or lipase?

lipase

minimally two HTN meds that work well in black folks

lisinopril, HCTZ

presentation of PCP pneumonia

may have SOB in an AIDS patient, doesn't have the typically lung consolidation but instead has bilateral fluffy infiltrates and a low pulse ox reading

there are two beta blockers that can be used for ACS patients... what are they?

metoprolol, carvedilol

anatomical location at which tension pneumothorax receives needle decompression

mid clavicular space of second intercostal

2 flavors of Vtach

monomorphic, polymorphic

What is the imaging study of choice for kidney stones? Stones less than ____mm have a 90% chance of passing spontaneously.

noncontrast helical CT; 4mm

Beck's triad

pathonomonic for cardiac tamponade... hypotension, muffled heart sounds, JVD

acute rheumatic fever major Jones criteria

polyarthritis carditis subcutaneous nodules erythema marginatum syndeham chorea

_____is the separation of the sensory retina from the underlying pigmented epithelium. What is the most common location? What are the clinical manifestations?

retinal detachment; superior temporal area; "curtain came down over eye"/flashers & floaters/blurry vision

On lateral neck x-ray, prevertebral soft-tissue swelling and forward displacement of the esophagus and trachea are seen with _____. It is believed to originate from infected lymph node becoming cellulitic and ultimate abscess formation.

retropharyngeal abscess

heart failure can be divided into right heart or left heart failure. peripheral edema, JVD, hepatosplenomegaly, DOE are ssx of which: left or right?

right

tetralogy of fallot commonly associated w/ ___

right aortic arch DiGeorge syndrome Down's syndrome assoc. w/ AVSD

What nerve injury is associated with an acetabulum fracture?

sciatic

define sepsis, particularly how it differs from SIRS

sepsis meets at least two of the SIRS criteria and has an identifable source of infection

fluid repletion w/ normal saline is the first step in mgmt of DKA or HHS. potassium levels also need to be managed, typically using 20-40mEq in normal saline. potassium is only given though if the patient's potassium is outside of a certain range. what is that range?

should be 3.3-5.3

what complication needs to be considered in any seizure patients (aside from etiology)

shoulder dislocation... posterior shoulder dislocations tend to happen w/ seizures, so inquire about shoulder pain

tachy brady syndrome is aka

sick sinus syndrome

Tricyclic antidepressants cause WHAT life-threatening cardiac dysrhythmias? What does treatment include?

sinus tach; prolongation of PR, QRS, and QT intervals; sodium bicarbonate as a Na channel antagonist, activated charcoal for gastric decontamination, benzos for seizures

risk factors for CAD

smoking, HTN, HLD (LDL>100, >70 for DM), male gender, age over 60 (this value is debatable), family hx heart conditions, sedentary lifestyle

presence of Howell Jolly Ab suggests what?

splenic dysfunction or asplenia

atrial septal defect Tx

spontaneous closure if small w/n 1st yr of life transcatheter device closure if larger

Thyroid storm is a rare life-threatening complication of thyrotoxicosis/hyperthyroidism characterized by fever, tachycardia, and dysfunction of the CNS, cardiovascular, or GI systems. What is used to treat thyroid storm?

supportive care; PTU (prophythiouracil); potassium iodine, sodium iodine, or Lugol solution; beta-blockade for peripheral effects; dexamethasone to prevent peripheral conversion of T4 to T3

Carbon monoxide should be considered when multiple individuals present from the same location with similar symptoms, known exposure to CO-producing equipment, and beginning of dormant heating units. Treatment???

supportive care; supplemental oxygen; HBO in certain populations.

brief loss of consciousness w/ rapid recovery

syncope

patent ductus arteriosus Sx (PE)

systolic murmur (newborn) continuous "machinery" murmur best heard below L clavicle widened pulse pressure w/ palpable bounding pulses

definitive tx for Torsades

this is a polymorphic Vtach which needs to be tx'd w/ a pacer

approach to sinus tachycardia

this is a scary arrhythmia b/c it cannot be shocked (if you shock em' you will kill em'), so you need to think about why they have it, which can be problematic. this could be d/t a PE (most common presenting sign of a PE is sinus tach), volume depletion, hemorrhage to name a few.

Briefly define the pathophys of an elevation in cardiac biomarkers

this is indicative of infarction (i wonder about ischemia)

how is acetaminophen poisoning diagnosed?

this is my guess but likely based on the HPI. also check LFTs in which case the AST may be over 1000 (that part is fact)

tx for peritonsilar abscess

typically can tx w/ abx (clindamycin or amoxicillin); severe cases may require drainainge

utility of Canadian head CT rules

use this to assess whether or not head imaging is indicated for head trauma

All pts with testicular trauma should be imaged how?

Color doppler ultrasound

What is anaphylaxis?

Multiorgan system involvement that occurs within seconds to hours after exposure to a known allergen that may lead to CV collapse

What are causes of junctional rhythms?

Myocarditis CAD Dig toxicity Hyperkalemia

pneumonia is a potential complication of influenza (those w/ the flu may get pneumonia shortly after). what is the most common etiological agent in this case?

Staph aureus

What is referral criteria for transfer to a burn center?

Second degree with >15% of BSA (or >10% for high risk patients) Third degree with >5% BSA Concomitant trauma Comorbid conditions Burns on face, hands, feet, genitals, or major joints Circumferential chest or extremity burn Electrical, chemical, inhalation burn Severe drug reaction (treated just like burns)

Tx of Anterior nose bleed

-Direct presure -Vasoconstrictors -Cauterization - Packing

Brown-Sequard presentation

-Ipsilateral loss of proprioception and motor function - Contralateral loss of pain and temperature sensation

Hallmarks of Shock

-Tachypnea - Tachycardia - Narrowing pulse pressure

Surgical management of hemothorax is indicated when?

1-1.5 L of blood loss OR > 200 ml/hr x 2-4 hrs

first thing that needs to be assessed in any patient, especially true of mental impairment (i.e. stroke)

ABCs

Signs and Symptoms of Pneumothorax

- Respiratory distress/ tachypnea - Hypoxia -Hypotension - Distended neck veins - Absent breath sounds on affected side

Risk Factors for a pneumothorax

- Smoking - Pulmonary disease - Age (20-40) - Tall, thin, male - Family hx - trauma

Cerebellotonsillar herniation

Cerebellar tonsils herniate through the foramen magnum - Pts deteriorate quickly to flaccid paralysis and death

What are systemic signs/symptoms of Erysipelas?

Chills Fever HA Joint pain

Kehr sign

Left shoulder pain caused by blood in the peritoneal cavity.

What is a common cause of right side heart failure?

Left sided heart failure

Where is Cellulitis most commonly found?

Legs Face Trunk Arm

Clinical findings of Zoster?

Painful eruption in a dermatomal pattern - often at T10. Trigeminal eruptions can include the tip of the nose (Hutchinson sign) - can risk corneal involvement

Cervical and Lumbar Radiculopathies S1 - motor weakness? decreased reflex? decreased sensation?

S1 motor - gastroc-soleus - flexion/plantar flexion reflex - Achilles tendon sensation - lateral foot

Orthopnea

SOB when lying down, relieved by sitting up and sleeping with pillows

How do you treat EM?

Self-limiting disease Supportive care: Antihistamines, topical steroids, prednisone Control Herpes outbreaks with Acyclovir

____is hematogenous spread of bacteria to synovial membrane lining the joint. Common organisms are S aureus, Neisseria gonorrhea, gram-negative bacilli, parvovirus B19, Hep B, Mumps

Septic arthritis

What are some drugs that cause SJS?

Sulfonamides Penicillins Phenytoin Phenobarbital Carbamazepine Valproic acid Allopurinol Corticosteroids

How do you treat pilonidal disease?

Surgical drainage Follicle removal with unroofing of sinus tracts

Aortic Aneurysm Signs and Symptoms

Symptoms: *Syncope and chest pain*, Sudden onset of abdominal, back, or flank pain, hematuria, scrotal mass, femoral neuropathy Signs: *Pulsatile mass*, ecchymosis, abdominal bruits, tenderness, distal extremity ischemia

Aortic Dissection Signs and Symptoms

Symptoms: Abrupt onset, Chest or back pain, *Tearing or Ripping* which propagates, *Neuroo deficits* Signs: Asymmetric BP, shock vs HTN, New murmur, Tamponade, may mimic AMI and CVA

What is the initial treatment for unstable tachycardia/PSVT?

Synchronized cardioversion

What is the key principle of treating an unstable arrhythmia in a patient with a pulse?

Synchronized cardioversion

Tx of A flutter with instability?

Synchronized cardioversion with 50J

How do you treat Erysipelas?

Systemic antibiotics: PCN, Erythromycin, Celphalosporins, Azithromycin

How do you treat Bullous Pemphigoid?

Treat fluid and electolyte disturbances Oral steroids Dapsone Tetracycline

acyanotic cardiac defect that results in pulmonary venous hypertension

coarctation (narrowing) of the aorta

What is the tx for RF?

penicillin, cephalexin, or azithromycin, and treat all household contacts with + throat cultures

what is Cullen's sign and what it is indicative of?

periumbillical ecchymoses, indicative of hemoperitoneum

two cardiac arrhythmias that are d/t ischemia until proven otherwise

polymorphic Vtach, Vfib

What is the tx for cauda equina syndrome?

emergent surgical decompression

physiology of exudative pleural effusion

increased pleural membrane permeability or impaired lymphatic drainage exudative effusions are protein rish MMC: pneumonia- when adjacent tissue becomes inflamed second MMC: malignant effusions

define cardiogenic pulmonary edema

increased pulmonary capillary wedge pressure secondary to ineffective filling or pumping of the heart. could be d/t CHF, valvular dz (aortic, mitral), arrhythmias, myocarditis

Restrictive cardiomyopathy: often caused by a ____process, or post-radiation or post open-heart surgery. What is the most common first symptom?

--infiltrative process - amyloidosis, sarcoidosis, and hemochromatosis -- changes in myocardium --most common first symptom is exertion intolerance and fluid retention, signs of right heart failure

What are the adjunctive tx for fibrinolysis or PCI?

1. antiplatelets (ASA, clopidogrel) 2. anticoagulants (UFH, LMWH, DTI, direct factor Xai)

What is the pharmacologic therapy for heart failure?

1. diuretics for fluid retention 2. ACEi 3. vasodilators (hydralazine & nitrates) 4. BB for LV dysfunction 5. digitalis to increase cardiac contractility

_______is the abnormal dilation of large conducting airways - due to congenital abnormalities (cystic fibrosis) or an acquired process (alplha-1-antitrypsin deficiency).

Brochiectasis

What is the criteria for the PERC SCORE????? HADCLOTS This is used to rule out the need for further imaging.

H - hormone/estrogen use A - age>50 D - dvt/pe history C - coughing up blood L - leg swelling disparity O - O2 sat<95 T - tachy>100 S - surgery/recent trauma

What are the pre hospital treatments for ACS?

MONA; morphine, oxygen, nitroglycerin (0.4mg SL x3 prn), aspirin (325mg)

________can lead to cardiac tamponade if it is large. Friction rub noted if secondary to pericarditis.

Pleural effusion.

What virus causes most cases of bronchiolitis? What are the hallmarks signs of this type of infection?

RSV - respiratory syncytial virus; tachypnea, tachycardia, fever, hypoxia

Endocarditis (or infection of the endothelial surface of the heart) is most commonly caused by what bacteria?

Strep viridans, Staph aureus, and Enterococcus

What are the criteria for diagnosing ARDS - acute respiratory distress syndrome? Mortality 40-60%

a. ratio of PaO2/FiO2 < or = 200 b. detection of bilateral pulmonary infiltrates on CXR c. pulmonary wedge pressure < or = 18mmHg or no clinical sign of elevated left atrial pressure

Traumatic rupture of thoracic aorta: If high clinical probability of aortic injury, ____should be done. Unstable patients with suspected aortic rupture should have a bedside ___. All other stable patients may undergo ____of the chest for evaluation of aortic rupture.

aortography; TEE (transesophageal echocardiogram); CT angiogram

How long should Plavix/Clopidogrel be used for bare metal or drug eluting stents?

bare metal - 30d-12m drug eluting - >/=12mon

In ____, auscultation of the chest reveals fine rales or audible wheezing and a prolonged expiratory phase secondary to air trapping. CXR: hyperinflation, air trapping, peribronchial cuffing/thickening.

bronchiolitis

What does the lab testing show in an asthmatic patient?

high WBC with eosinophilia Sputum shows --- Curschmann's spirals (mucous casts of small airways) Charcot-Leyden crystals

What is the most common cause of pleural effusions?

left sided heart failure (increased hydrostatic pressure in microcirculation) ALSO - 1. decreased oncotic pressure - hypoalbuminemia 2. decreased pressure in pleural space - collapsed lung 3. increased permeability - in pneumonia 4. impaired lymphatic drainage - in malignancy 5. movement of fluid from peritoneal space - in ascites

Patients with pericardial tamponade present with Beck's triad. What is that?

muffled heart sounds distended neck veins hypotension

What is the agent of choice for BP lowering for patients with hypertensive encephalopathy, intracranial bleeding, and heart failure? Use with what for dissecting aneurysm?

nitroprusside; propranolol -- clonidine can also be used but sedation is common

What are some drugs associated with Torsades de pointes?

tricyclic antidepressants, erythromycin, ketoconazole, haloperidol, cisapride, disopyramide, pentamidine, sotalol, class I anti-arrhythmics

Zone III

Angle of the mandible to base of the skull

What should always be checked first in a trauma victim?

Airway

What are 1st line tx for Chlamydia?

Azithromycin 1gm PO x 1 OR Doxycycline 100mg PO BID x 7 days

What is happening in mitral regurgitation (MR)?

Backflow of blood in to LA (LA will get bigger) LV will get bigger to try and compensate, eventually it will fail. Very common.

What is the Rule of nines in burn injuries?

Anterior torso - 18% Posterior torso - 18% Arms - 9% each Legs - 18% each Head - 9% Genitals 1%

Tx of A.fib with RVR in an unstable patient?

Synchronized cardioversion with 100-200J

T/F In a renal injury, the degree of hematuria is not indicative of severity of injury

TRUE

What is considered a major burn in adults? Peds?

25% adults 15% peds

Cardiogenic shock

-Failure of the heart to pump blood to the rest of the body

Le Fort III Fx

-Craniofacial dysjunction - Entire face shifts when you tug on teeth - Airway COMPROMISE IS COMMON - Nasogastric and nasotracheal tubes are contraindicated

Signs and symptoms of Upper urinary tract trauma

-Flank or abdominal pain - N/V - Lower rib fx - Any penetrating wounds whose trajectory crosses the paravertebral gutter

Chronic subdural hematoma

-May or may not present with history of head injury - See in elderly and alcoholics - HYPODENSE lesion on CT scan

3 Components of Glasgow coma scale?

1.)Eye opening 2.) Verbal Response 3.) Motor Response

Who is most affected by pilonidal disease?

M>F Ages 15-30

name the most common narrow complex tachycardia

afib

Fusiform cerebral aneurysms are elongated dilations of large arteries usually associated with atherosclerosis. Where do they typically develop?

basilar artery

CT is the imaging modality of choice for comatose patients. if the CT is negative, what should you suspect is the cause of the stroke (assuming hypoglycemia and opiate or benzo overdose has been ruled out)

basilar artery thrombosis; confirm w/ MRI

DKA and HHS allow for hyperglycemia. consequently, one component of tx is the administration of (low dose) insulin, but only if the serum potassium level is high enough so not as to cause hypokalemia. what is the concern of hypokalemia in these patients?

cardiac arrhythmias

ventricular septal defect Tx

close spontaneously (small defect) surgical repair ~ 4-6 mos (larger defects)

what is the most feared complication of DKA

coma

abx to use for supraglottitis mgmt

combination of a third generation cephalosporin (ceftriaxone, cefotaxime) and MRSA coverage (vancomycin or clindamycin)

best way to clean an open wound

copious irrigation with saline under pressure (I say this becuase this was a question and answer on an EOR exam, I observed this in the ED wherein they used a syringe)

first step in the mgmt of DKA or HHS

fluid replacement w/ isotonic saline

utility of glucocorticoids and bronchodilators (racemic epinephrine) in mgmt of acute supraglottitis according to uptoday

none. the ADRs of glucocorticoids for this condition may outweigh the benefits, no indication for epi (dunno why)

add hypoglycemia to the ddx of altered MS

noted

ST Elevations in V1, V2

posterior wall MI, posterior descending artery

name two feared conditions of a drug overdose

seizures, coma

gram positive cocci in chains that is commonly associated with post-flu pneumonia

streptococcus pneumoniae

With a blowout fracture, on plain xray, a ____sign may be seen from orbital fat herniating into the maxillary sinus or the ____sign from bone fragments in the sinus.

tear drop sign; open bomb-bay door sign

this is a oncologicla emergency and thus you should put it on your ddx for any chemo patient

tumor lysis syndrome

What are some common allergens that cause allergic CD?

Antibiotics Metal salts Dyes Plants Oils Nickel Latex

Stable patients in A.fib for >48 hours should be...

Anticoagulated with heparin or lovenox before cardioversion

Unequal blood pressure in arms can indicate?

Aortic Disruption

What are causes of cardiogenic syncope?

Arrhythmias (V tach, SVT, sick sinus) PE Aortic stenosis Congenital abnormalities Carotid sinus hypersensitivity Heart blocks Aortic dissection

What is the main goal of therapy for fluid resuscitation in burn patients?

0.5 to 1.0 mL/kg/hour of urine output

Atrial fibrillation - regularly irregular - the most common sustained arrhythmia in adults - what three treatments are used?

1. rate control w BB, CCB, or digoxin 2. Anticoagulation w heparin & warfarin 3. rhythm control w amiodarone or cardioversion

What are three common causes of EM?

1.Drugs (sulfonamides, phenytoin, barbiturates, PCN, allopurinol) 2. Infections (HSV, Mycoplasma) 3. Idiopathic: 50% of cases

Epistaxis Posterior

10%, Elderly, coagulopathy *Sphenopalatine artery* Posterior packing, ENT consult, admit, antibiotics

Normal Intracranial pressure

10-15 mmHg for adults

How do you manage acute burns?

100% O2 with pulse Ox Early intubation with any signs of inhalation injury Determine tetanus status Fluid resusictation for all burns >20% Foley NG/OG tube Prevent hypothermia Silvadene

What is the blood pressure in which tPA is excluded? What can be given to reduce blood pressure below this number so that tPA can be administered?

185/110; labetalol, nicardipine

generally speaking, phosphate supplementation is not recommended in DKA or HHS patients; the exception is if the phosphate drops below a certain value... what is that value?

1mg/dl aka 0.32mmol/L

When is a CT scan indicated for Rib fx?

1st and last 3 rib involement

Diagnostic studies for amenorrhea?

1st line: Beta hCG, TSH, Prolactin Other: FSH, LH, Estrogen, Testosterone Genetic testing

What is considered an adequate trial of a COC for endometriosis?

3-6 months taken continuously

How long can it take to pass a kidney stone?

4-6 weeks, so you may see a patient in the ED but can discharge them

Packing for anterior nose bleed should be left in for how long?

48 hours and prophylactic ABX should be started

What are classic signs/symptoms of acute arterial occlusion?

6 P's: Pain Paralysis Pallor Paresthesia Poikilothermia (cold) Pulselessness

The presence of pulses may help to approximate the systolic blood pressure. Radial pulse present means ~ ____mmHg Femoral pulse present means ~ ____mmHg Carotid pulse present means ~ ____mmHg.

80, 70, 60

Diagnostic studies for AAA?

Abdominal US is study of choice!

______presents with sudden onset of eye pain, blurred vision, h/a, n/v and they may report a "halo around lights".

Acute angle closure glaucoma - an increase in intraocular pressure and the potential for optic nerve injury

Ddx of acute MI?

Acute aortic dissection PE Tension pneumothorax Esophageal rupture GI cause: esophagitis Pulm: Pleuritis, PNA Chest wall pain Psych: Anxiety, depression

When do you administer an antiarrhythmic drug with V.fib?

Administer Amiodarone after 2 cylces of CPR and defibrillation (Lidocaine is 2nd line)

___should NEVER be used in the treatment cocaine associated chest pain.

Beta Blockers

Hangman's fracture

Bilateral fracture of pedicles of C2 due to hyperextension - Unstable

Blood and nervous supply of bladder?

Blood: Internal Iliac artery Nerve: Lumbar and sacral plexus

Common Fractures: ____is fx of neck of fourth or fifth metacarpal, striking w clenched fist. ____is fx of distal radius with dorsal displacement, FOOSh ____is fx of distal radius with volar displacement, fall on back of hand. ____is fx/dislocation of atlas and axis, due to extreme hyperextension during abrupt deceleration.

Boxer's fracture, Colles fracture, Smith fracture, Hangman's fracture

PE signs of ACS?

Brady or tachy Hypotensive or hypertensive JVD Soft heart sounds Transient murmur of MR S4 gallop Possible bibasilar rales Low grade fever Pericardial friction rubs

Foreign body aspiration: S/S ____aspiration presents with cough, decreased air entry, dyspnea, and wheezing ____aspiration presents with cough, cyanosis, dyspnea, and stridor

Bronchial; Laryngotracheal

What is Pericardial effusion?

Build up of fluid between the pericardium and heart. Produces a restrictive pressure on the heart. Similar pathophysiology to pleural effusion of the lungs.

A patient in VF cardiac arrest has failed to respond to defibrillation & epinephrine 1 mg IV. You give the nurse an order to administer epinephrine every 3 minutes during the code. Which of the following dose regimens is recommended? a. epinephrine 1 mg, 3 mg, 5 mg, and 7 mg (escalating regimen) b. epinephrine 0.2 mg/kg per dose (high-dose regimen) c. epinephrine 1 mg IV push, repeated every 3-5 minutes d. epinephrine 1 mg IV push, followed in 3 minutes by vasopressin 40 U IV

C

What drug-dose combination is recommended as the first line medication to give to a patient in asystole? a. epinephrine 3 mg IV b. atropine 3 mg IV c. epinephrine 1 mg of a 1:10 000 solution IV d. atropine 0.5 mg IV

C

A 31-year-old man who works for a moving company presents to the ED because he thought he was having a heart attack. He does not smoke, and jogs 3 days a week. His father died of a heart attack in his sixties. He describes a gradual onset of chest pain that is worse with activity and resolves when he is at rest. His HR is 68 beats per minute, BP is 120/70 mm Hg, and RR is 14 breaths per minute. On examination, his lungs are clear and there is no cardiac murmur. You palpate tenderness over the left sternal border at the third and fourth rib. An ECG reveals sinus rhythm at a rate of 65. A chest radiograph shows no infiltrates or pneumothorax. Which of the following is the most appropriate next step in management? a. Administer aspirin and send for a troponin. b. Administer aspirin, clopidogrel, and heparin and admit for acute coronary syndrome (ACS). c. Administer ibuprofen and reassure the patient that he is not having a heart attack. d. Inject corticosteroid into the costochondral joint to reduce inflammation. e. Observe the patient for 6 hours.

C. The patient has costochondritis, an inflammatory process of the costochondral or costosternal joints that causes localized pain and tenderness. Any of the seven costochondral junctions may be affected, and more than one site is affected in 90% of cases. The second to fifth costochondral junctions are most commonly involved. In contrast to myocardial ischemia or infarction, costochondritis is a benign cause of chest pain, often with an insidious onset, and is an important consideration in the differential diagnosis for chest pain. Of note, 5% to 7% of patients with cardiac ischemia also have chest-wall tenderness. The onset is often insidious. Chest-wall pain with a history of repeated minor trauma or unaccustomed activity (eg, painting, moving furniture) is common. The goal of therapy is to reduce inflammation. NSAIDs are typically prescribed

Cervical and Lumbar Radiculopathies C7 - motor weakness? decreased reflex? decreased sensation?

C7 motor - triceps, finger extensors reflex - triceps sensation - index, long fingers, dorsum of hand

Diagnostic studies for orthostatic hypotension?

CBC BMP EKG Orthostatic vitals Tilt test if there are no findings

what is the one major ddx for ARDS

CHF. differentiate based on history; ARDS typically preceded by some major form of illness, namely sepsis, or some major insult such as nearly drowning. they look clinically similar--crackles on auscultation, CXR demonstrating pulmonary edema. get a BNP and ECHO, both of which will or may be normal in CHF and is definitely normal in ARDS

Panic disorder is frequently encountered in the ED and the s/s mimic those of other illnesses. What tests should be done for the diagnosis of exclusion?

CK, troponins, D-dimer, EKG, radiographic studies

imaging modality of choice for a thoracic aneurysm

CT (note that that's not echo)

How does ventilation differ with cardiac arrest vs respiratory arrest?

Cardiac arrest: Give 1 ventilation every 6-8 seconds (8-10 breaths per min) Respiratory arrest: Give 1 ventilation every 5-6 seconds (10-12 breaths per min)

This type of brain herniation causes pinpoint pupils and increased muscle tone and my progress to hyperventilation and decorticate posturing

Central Transtentorial herniation

What are clinical features of Scarlet fever?

Classic symptoms: SANDPAPER rash in groin, axilla with DESQUAMATION (peeling) after 3-4 days. Strep symptoms: Pharyngitis, fever Bright red tongue with a "STRAWBERRY" appearance

What is the "non-classical" appearance of endometriosis?

Clear vesicles White spots Yellow spots Nodules "Pockets" - circular folds Normal tissue - may need microscope to see

Diagnosis of abruptio placentae?

Clinical diagnosis

Dx of Chlamydia?

Clinical diagnosis Gram stain Complement fixation test of immunofluorescence, ELISA, DNA probes

How do you diagnose VVC?

Clinical symptoms Normal pH Whiff test - negative KOH - positive Wet prep - budding yeast forms or mycelia are seen

Clinical features of lice?

Concentrate on waist, shoulders, axillae, and neck Bites produce red spots that progress to papules and wheals Intensely pruritic White ova adhere to hair shaft

What is endometriosis?

Condition where there is endometrial tissue found outside the endometrial cavity

What are two ways to surgically treat endometriosis?

Conservative or definitive treatment (hysterectomy). Conservative surgery involves lysing adhesions and removing endometriosis to restore the anatomy though use of laparoscopy.

Head injuries Treatments

Consider transfer to traumacenter Neurosurgical consult Hyperventilation controversial, Airway protection for GCS <8 or rapid decline Mannitol/Hypertonic saline for ICP Burr hole Correct platelets, coagulopathy Avoid hypotension/hypoxia

Injury to the spinothalamic tract causes?

Contralateral pain and temperature sensation loss 2 levels below the lesion

In a Tension Pneumothorax the trachea deviates to which side?

Contralateral side

____is enlargement or dysfunction of the right ventricle due to pulmonary hypertension.

Cor pulmonale - may see prominent P waves in leads II, III, and aVF (anterior leads)

_____refers to a superficial disruption of the surface of the cornea. Tx: update ____vaccine!, topical anesthetics, cycloplegic drops, irrigation, topical aminoglycoside or fluoroquinolone. What should not be given to take home?

Corneal abrasion; tetanus; topical anesthetics (bc its repetitive use impairs healing)

What is the definitive diagnosis for angina?

Coronary angiogram

What is the definitive diagnostic procedure for ACS?

Coronary angiography

What tracts are located anteriorly within the spinal cord?

Corticospinal and spinothalamic tracts

________triad for finding cerebral perfusion pressure is CPP=MAP-ICP. Brain ischemia results when pressure is less than 40mmHg.

Cushing triad

Tx for PACs?

D/c precipitating drugs Treat underlying disorders Metoprolol if there are significant symptoms

MAP

DBP + (SBP - DBP)

How do you treat cellulitis?

Depends on severity: PCN Cephalosporins Augmentin If MRSA - use TMP-SMX (Clinda or Doxy if allergic to sulfa)

Management of DUB?

Depends on underlying cause If there is severe bleeding can do iron therapy, volume replacement, IV or oral high dose estrogens Progestin trial OCPs Cyclic protestins (in young patients) D&C Refractory cases may require endometrial ablation or hysterectomy

Clinical features of anaphlaxis?

Dermatologic: pruritis, flushing, urticaria, erythema multiforme, angioedema Respiratory: SOB, wheezing, cough, stridor, rhinorrhea Cardiovascular: Dysrhythmias, collapse, arrest GI: Cramping, N/V GU: Urgency, cramping Eye: Tearing, pruritis, redness

Corticospinal tract

Descending motor tract - Damage to it results in ipsilateral muscle weakness, spasticity and paralysis

Classification of arrhythmias?

Disorders of impulse formation or automaticity Abnormalities of conduction, reentry, and triggered activity

ventricular septal defect Dx

ECHO is diagnostic EKG: LVH or BVH with tall R in V5, V6 and deep Q wave

hypertrophic cardiomyopathy Dx

ECHO w/ ventricular septal thickening > 15 mm

how do you distinguish systolic from diastolic heart failure?

ECHO. diastolic is aka HF w/ preserved EF

indication for an Automated Implanted Cardio Defibrilator (AICD) in heart failure patients

EF of less than 35%

What populations present with atypical chest pain or no chest pain when having an AMI?

Elderly Diabetics Women Patient with psych d/o

_____is a condition defined pathologically as dilation of the air space distal to the terminal brochiole with destruction of the interalveolar septa. "pink puffer" CXR - hyperinflated with flat diaphragms

Emphysema

Virchow's triad consists of venous stasis, endothelial damage, and hypercoagulable state; name some hypercoagulable states

Factor V Leiden, Protein C and S deficiencies, antiphospholipid syndrome, pregnancy, CA

Most common injury related to trauma?

Fall

How do you treat Impetigo?

For Non-bullous: 2% Mupirocin and oral antibiotics For Bullous: Oral abx (can return to school after being on abx for 24 hours)

When is PCI indicated?

For STEMI within 90 minutes of symptoms

Severe TBI

GCS 3-8

What do you do if the LMP is unknown?

Get an U/S

A therapeutic trial of what may help symptoms of endometriosis? What is the downside of this?

GnRH agonist (Depo-Lupron) which inhibits the pituitary release of FSH/LH. Can only use for 6 months because it decreases bone mineral density and makes you feel like you're in menopause (hot flashes, decreased bone mineral density)

What organisms cause acute epiglotitis?

H. flu, Strep pneumo or Strep pyogenes, Staph aureus

What does the murmur of AS sound like?

HARSH, SYSTOLIC, crecendo-decrecendo heard best at URSB and radiates to carotid and LSB. Heard with patient sitting and leaning forward.

What is a murmur?

Hearing blood move past a valve in a way it's not supposed to due to increased blood flow, stenosis, or regurgitation.

___refers to blood in the anterior chamber of the eye -- management includes keeping the head elevated, limiting eye movements, prescribing analgesics, and avoiding use of anticoagulants or antiplatelet medications. Results from torn blood vessels within the iris and ciliary body.

Hyphema

When would you want to admit a patient with PID?

Inability to exclude surgical emergency Pregnancy Does not respond to oral antibiotics Unable to tolerate oral regimen Severe illness Tubo-ovarian abscess HIV infection Immunocompromised

Brown recluse spider bite

Initial bite is painless Vemon is necrotizing - reactions range from mild urticaria to full thickness necrosis. Lesion is a sinking macule, pale gray in color, slightly eroded in center, with halo of tender inflammation and hemorrhage Lesion extends to muscle

Goal of tx with hypertensive emergency

Initial goal is to reduce MAP by no more than 25% within minutes to 1 hour with further lowering over next 2-6 hours

Ellis III fracture

Involves the enamel, dentin and the pulp - Urgent dental follow-up - Rx ABX and oral analgesia

Which bacteria in Pneumona is common in diabetes, alcoholism, and nosocomial infections?

Klebsiella

What would you find on an EKG with CHF?

LA hypertrophy Arrhythmias (can be any rhythm) LV Hypertrophy

name the two most common etiological agents of bacterial meningitis occurring in college students

Niesseria meningitides, Streptococcus pneumonia

Diagnostic studies for PROM?

Nitrazine paper Fern test Amnisure US - checks amniotic fluid index

What meds should a patient go home with after ACS?

Nitroglycerin BB ACEi ASA/Clopidogrel anticoagulant (up to 8days for LMWH) aldosterone agonist statin LIFESTYLE CHANGES

What is orthostatic hypotension?

Occurs when there is a sudden change in posture after being supine for a prolonged period and is associated with inadequate compensatory increases in heart rate and peripheral vascular resistance

What are features of a PAC on EKG?

P wave appears sooner than next expected sinus beat P wave has a different shape P wave may or may not be conducted through the AV node

Premature beats

PACs PJCs PVCs

What should be on your Ddx with endometriosis?

PID Ovarian neoplasms Uterine myomas Ectopic pregnancy Adhesions

when evaluating any chest pain pt, you need to rule out the big red flag conditions. mnemonic for them is PACE PM... name each of these letters

Pneumothorax Aortic aneurysm Cardiac tamponade Esoapheal rupture Pulmonary embolism Myocardial infarction

What are risk factors for an EP?

Previous EP Hx of PID Previous abdominal or tubal surgery IUD use Assisted reproduction

What are signs/symptoms of Vulvovaginal candidiasis (VVC)?

Pruritis Vaginal soreness Dysparunia External dysuria Vaginal discharge - resembles cottage cheese, no odor

____is an early systolic opening ejection click followed by a systolic ejection murmur which radiates to the base.

Pulmonic Stenosis

give the proper definition of each: QRS (on an EKG)

Q is the first downward deflection which precedes an R which is the first upward deflection (if there are two upward deflections, the smaller is called an r or r') and S is the first downward deflection after the R wave

Clinical features of arrhythmias?

Ranges from: Asymptomatic Hemodynamic instability Shock Death

What are clinical features of German Measles?

Rash that starts on the face and spreads to the rest of the body (less intense than measles) Low grade fever Postauricular and occipital adenopathy Aching joints, especially among young women. FORCHEIMER spots (fleeting small, red spots on the soft palate) - can also be seen with measles and scarlet fever.

What are risk factors for VVC?

Recent antibiotic use Pregnancy Diabetes Tight, non-breathable clothing Chronic steroid use

What is the "classical" appearance of endometriosis?

Red spots Dark blue Dark black Dark brown "Powder burn" Cystic

What are characteristics of first degree burn?

Red, blanches, painful No blister Minimal tissue destruction Most common cause: Sunburn Heals in 7-10 days No scarring

Tx for 2nd degree burns?

Remove blisters Apply Silvadene and dressing Analgesics

How do you treat contact dermatitis?

Remove offending agent Wet dressings soaked with Burrow's solution (aluminum acetate in water) Topical corticosteroids (Class I or II) Do not pop blisters If severe: systemic corticosteroids with a 2 week taper starting at 60mg Supportive care: mild soaps, oatmeal baths, antihistamines

If EKG is negative for STEMI but patient is symptomatic what is the next step?

Repeat EKG Get cardiac biomarkers

Tx for AA?

Replace with graft, either endovascular or open surgical repair (5 year survival is >60%)

What are pressure ulcers (decubitus)?

Result from impaired blood supply caused by localized pressure. Sacrum and hip are most commonly affected.

Treatment of PAD?

Risk factor modification (smoking cessation, diet modification, exercise, etc) BB ACE-I Statins ASA or Plavix Cilstazol (Pletal) Revascularization

What are some of the PE findings for infective endocarditis?

Roth spots - small white spots on retina surrounded by hemorrhage Osler nodes - small tender lesions on fat pads of fingers and toes Janeway lesions - painless, reddish, macular lesions on hands or feet

defintion of severe HTN

SBP of at least 180 or a diastolic of at least 110

which class of antidepressant is venlafaxine

SNRI

What are general signs and symptoms of CHF?

SOB and fatigue Orthopnea Edema Nocturia Hypotension/narrow pulse pressure Diaphoresis Pulmonary congestion JVD Tachycardia and tachypnea

It is vital to make sure the symptoms of the first concussion have subsided because of the gloomy possibility of ________ ie. when the brain swells catastrophically - patient either dies or is left severely disabled.

Second impact syndrome

What are diagnostic studies for an EP?

Serum hCG - will be low for GA (normally doubles q48 hrs) TVUS - IUP should be seen when hCG levels are between 1500-2000

What are follow up requirements for tx of EP?

Serum hCG levels should be done or pelvic exam to exclude any remaining evidence of pregnancy

What are clinical features of a ruptured EP?

Severe abdominal/shoulder pain due to peritonitis Tachycardia Syncope Orthostatic hypotension

What are the s/s of labyrinthitis and vestibular neuritis?

Severe, disabling vertigo for 24-48 hours followed by weeks of imbalance -Vomiting -Pts think they are dying

In ____ seizures, consciousness is maintained. In ____, there is a loss of consciousness, indicating the spreading or generalizing to both cortical hemispheres.

Simple partial; Complex partial

I had traditionally associated S1Q3T3 on an EKG as being pathonomonic for a PE, but this applies to a right heart strain in general. what's the relationship between a PE and right heart strain

The clot in the pulmonary vasculature cuases blood to back up on the right side of circulation thus causing right heart strain

Secondary survey

The patient is evaluated from head to toe & the indicated studies (eg, radiographs, laboratory tests, invasive diagnostic procedures) are obtained.

What is the initial treatment for a symptomatic bradyarrhythmia?

Transcutaneous cardiac pacing! Atropine or positive chronotropic (epinephrine, dopamine)

Aortic dissections - Stanford type A vs. Stanford type B. What's the difference and which is treated surgically?

Type A - involves ascending aorta - surgical tx!!! Type B - involves descending aorta - BP control

How does acute MI pain differ from angina pain?

Typically it is more severe and there are associated symptoms (SOB, diaphoresis, N/V, etc)

Tx for unstable patient with V.tach who is pulseless?

Unsynchronized cardioversion starting at 100J

How do you diagnose Rubella?

Usually a clinical diagnosis Can do paired sera - get a blood sample now and then in a week. This is usually done in immunocompromised patients.

Black widow spider bite

Venom is neurotoxic Initial bite feels like pinprick and within minutes to hours progresses to severe muscle cramping, generalized abdominal, leg, back pain/spasms

72yo female presents w/ unilateral lower extremity edema, similar to a DVT in the past. She recently was on a long bus trip; homan's sign negative. Homan's sign only has good positive predictive value as it is negative in 50% of the cases, but it is probably uneconomical to do a venous duplex US of all patients w/ a similar presentation. what tool should be used to assess the probability that a patient has a DVT

Well's criteria or PERC score (I say this b/c I know that it exists, but I forget to utilize it)

What is external aburption?

When blood escapes from the uterus and vaginal bleeding occurs

___ causes carditis in 50-80% of pts; most common in 6-15 yo

acute rheumatic fever

Dilated Cardiomyopathy: most common cause is ____. Others?

alcohol; may also be idiopathic, myocarditis, or drugs (doxorubicin) -- 1 in 3 cases of heart failure are caused by dilated cardiomyopathy

tx for bacterial sinusitis

amox-clav (not just amox)

The classic CXR for _____demonstrates a widened mediastinum, hilar adenopathy, and the absence of a focal infiltrate.

anthrax

spontanous pneumothorax can occur in those that are tall (particularly 20yo men that smoke) or it can result from a lung dz; name a few of these lung dz's

asthma, COPD, cystic fibrosis (know this one for the boards), HIV, TB, Marfan's syndrome (know this one for the boards)

hypertrophic cardiomyopathy Sx

asymptomatic systolic ejection murmur at the LLSB and/or apex (less when supine or squatting)

ddx for hemoptysis

bronchitis (i think this is the most common cause), blood thinners, CA (bronchiogenic, larynx), pulmonary embolus, TB (oh crap; i saw a Honduran guy in the ED that mentioned his hemoptysis and I didn't really think anyting of it...)

Diving: The most common form of barotrauma occurs during _____ and is middle ear "squeeze" -- caused by inability to equalize pressure causing TM bleeding or rupture. Barotrauma during ____ is due to expansion of gas in body cavities.

descent, ascent

Grey-Turner sign

ecchymoses of the flanks associated with fulminant hemorrhagic pancreatitis; very poor prognostic sign

a patient presenting w/ respiratory distress and drooling is which condition until proven otherwise?

epiglotitis

t/f chest pain that is rapidly relieved by NTG is more likely ischemic.

false. it was previously believed that it was but this is now a misnomer. NTG also relaxes the esopageal sphincter.

most common presenting complaint for MI in elderly

fatigue... not chest pain

define typical chest pain (typical for angina)

left sided chest pain that comes on w/ activity and is relieved (not necessarily resolved) by rest and NTG

describe the ECG findings of atrial tachycardia

multiple p waves followed by a QRS complex... this applies to all atrial tachyarrthymias, differentiated based on the rate

What nerve injury is associated with a femoral shaft or lateral tibial plateau fracture?

peroneal nerve

pt SOB, CXR ordered demonstrated wedge-shaped infart; this term is pathonomonic for which condition?

pulmonary embolism

CHF - Systolic dysfunction means a problem with the ____. What drug is contraindicated?

pump; CCB!

in evaluating a patient with chest pain, what is the utility in asking about radiation of pain?

radiation (typically down one or both arms or up neck) is suggestive of ACS

Second Impact syndrome

rapid swelling and herniation of the brain after second head injury that occurs before the symptoms of a previous head injury have resolved

two broad classifications for retinal detachment

rhegamatogenous (break in retina), nonrhegmatogenous

virchow's triad

some form of venous stasis (post surgery can't walk around, 16 hours on a long flight), endothelial injury (smoking, surgery, IV cath), hypercoagulability (acquired d/t OCP or HRT, CA, congenital such as Factor V Leiden)

name one characteristic of fungal rashes

tend to cause some sort of scaling

tell me everything that you know about non-anginal, atypical and typical angina

these definitions hinge on three ssx: 1. left sided chest pain 2. brought about w/ activity 3. relieved (not necessarily resolved) by rest and NTG presence of all three is typical angina; 2/3 is atypical angina; less than 2 is non-anginal

pt presenting w/ CP is asked if they have a hx of migraines and/or Raynaud's... why?

these two things are associated w/ Prinzmetal angina

patients w/ hemophilia A don't form clots well. which clotting study/lab should be ordered for these patients? will it be elevated or decreaesd?

they'll have a prolonged aPTT

what are the breath sounds associated w/ asthma

wheezing

A 19-year-old woman presents with bilateral lower-abdominal pain, fever, nausea, vomiting, and general malaise. Her last menstrual period was 5 days ago. Vitals are HR 98 beats per minute, BP 110/65 mm Hg, RR 18 breaths per minute, and temperature of 102.7°F. Pelvic examination demonstrates exquisite cervical motion tenderness and right adnexal tenderness. Laboratory reports are notable for a WBC 15,000/μL, an ESR of 95 mm/h, and a negative urine β-human chorionic gonadotropin (β-hCG). Transvaginal ultrasound demonstrates a right complex mass with cystic and solid components. Which of the following is the most appropriate next step in management? a. Ceftriaxone IM plus a 14-day course of oral doxycycline and follow-up in the gynecology clinic. b. Oral ofloxacin plus metronidazole and follow-up in the gynecology clinic. c. Analgesics for a ruptured ovarian cyst and follow-up in the gynecology clinic. d. Admission for IV antibiotics and possible laparoscopic drainage. e. Admission for emergent medical or surgical treatment of an ectopic pregnancy.

*D* This patient has a tubo-ovarian abscess (TOA), a common and potentially fatal complication of pelvic inflammatory disease (PID). While PID can usually be treated with outpatient antibiotics, the Center for Disease Control (CDC) lists several guidelines for inpatient admission, including: (1) cases in which surgical emergencies (eg, appendicitis) cannot be excluded; (2) pregnancy; (3) failure of outpatient therapy; (4) inability to tolerate oral intake; (5) severe illness, including nausea, vomiting, or high fever; and (6) TOA. This patient has several of these factors and should, therefore, be admitted for further management. IV antibiotics are curative in 60% to 80% of cases and must be selected to cover N gonorrhoeae and Chlamydia trachomatis, the most common pathogens responsible for PID, as well as bacteroides, the most common cause of TOA. Surgical drainage or salpingectomy and oophorectomy may be required in resistant cases.

Extraperitoneal perforation of bladder

- "Flame shaped" contrast density lateral to bladder

3 causes of Hypovolemic shock

- Hemorrhage - Dehydration - Third Spacing

What is the Parkland formula for fluid resuscitation in burn victims?

4mL/kg x %BSA burned. First half given over first 8 hours, second half over next 16 hours.

Basilar skull fracture

A fracture involving the temporal, occipital, ethmoid and/ or sphenoid bones - Associated with tearing of the dura and CSF leaks - raccoon eyes, Battle sign

What are SE of Danazol?

Acne worsening Voice deepening Weight gain Edema Adverse lipid profile

What is Erythema Multiforme (EM)?

An acute inflammatory disease which is usually induced by drug hypersensitivity or preceding infection.

What is Impetigo?

An acute, contagious superficial skin infection of the skin.

Management of PID?

Antibiotics Antipyretics Analgesics Bed rest Removal of IUD if present Partners should be evaluated and treated

Tx of A.fib long term?

Anticoagulants (Warfarin, Dabigatran, etc)

What is preop management of arterial occlusion?

Anticoagulate with IV heparin Arteriogram

What are you hearing in S4?

Atrial contraction ("gallop")

_____is released from cardiac ventricles in response to increased wall tension.

BNP - B-type natriuretic peptide

tx for PCP pneumonia

Bactrim IV and steroids (don't forget those steroids)

Diagnostic labs for DUB?

Beta-hCG to rule out pregnancy CBC LFTs Iron studies PT/PTT Documentation of ovulation Thyroid panel Serum progesterone Prolactin Serum FSH

Subarachnoid hemorrhage

Bleeding into subarachnoid space between the arachnoid and the pia matter - Commonly spontaneous and atraumatic, often the result of rupture of an aneurysm or arteriovenous malformation - Pt will c/o worst headache of their life

What cervical spine vertebrae innervate the diaphragm?

C3 through C5

Cervical and Lumbar Radiculopathies C5 - motor weakness? decreased reflex? decreased sensation?

C5 motor - deltoids/biceps reflex - biceps sensation - deltoids

Diagnostic studies for pericarditis?

CBC - WBC elevated EKG - diffuse ST elevation Echo, dopper, CT, MRI may be helpful

Trauma: Ecchymosis over lower abdomen from lap belt is associated with a lumbar spine fracture ie. ____fracture. Ecchymosis over flanks or _____sign or umbilicis ie. ____sign may represent retroperitoneal hemorrhage.

Chance fracture, Grey Turner's sign, Cullen sign

Diagnostic studies for Varicella-zoster?

Clinical diagnosis

hyper or hypoglycemia w/ altered mental status in a diabetic patient is what condition until proven otherwise?

DKA or HHS

What are some "other" possible causes of chronic pelvic pain?

Depression Somatization disorder Celiac disease Neurologic dysfunction Shingles Prophyria Familial Mediterranean Fever

Aortic disruption most commonly involves?

Descending aorta

_____is due to deficiency of or resistance to vasopression/anti-diuretic hormone. S/S are intense thirst, polyuria, dehydration. Treatment is with what?

Diabetes Insipidus; desmopressin acetate (and possibly HCTZ)

What kind of murmurs are always pathologic?

Diastolic

atrial septal defect Dx

ECHO is diagnostic

What do you give after 3 cycles of CPR/defibrillation if patient still doesn't have a pulse?

Epi or vasopressin

Tetanus is mandated in all burn patients except for when?

Except those immunized within the past 12 months

Non-pharm therapy for CHF?

Exercise Low Na Diet Smoking and alcohol cessation Stress reduction Daily weight monitoring Cardiac rehab

What is the most useful and cost effective noninvasive test for angina?

Exercise stress testing

What are clinical features of left sided heart failure?

Exertional pulmonary vascular congestion Exertional dyspnea + cough Fatigue Orthopnea Paroxysmal nocturnal dyspnea Basilar rales Gallops Exercise intolerance

What clotting factors are Vitamin K deficient?

Factors II, VII, IX, or X

Le fort Fracture

Fracture of the maxilla. (There are different types of Le Fort fractures; they are dictated "Le Fort 1, 2, or 3" and are transcribed as Le Fort I, Le Fort II, and Le Fort III.)

What are causes of cervicitis?

Gonorrhea Chlamydia Herpes simplex HPV

Concussion grading - American Academy of Neurology: ____=confusion but no LOC for <15 minutes. ____=confusion but no LOC for >15 minutes. ____=LOC either brief or prolonged.

Grade 1, Grade 2, Grade 3

How do you diagnose BV?

Gram stain - gold standard Whiff test - positive KOH - negative

Clinical features of cardiogenic shock?

Hypotension Increase in JVP secondary to poor emptying of heart and systolic failure

If a patient with a DVT has active internal bleeding, uncontrolled HTN, CNS tumor, recent trauma or surgery, or recurrent DVT despite anticoagulation, what alternative treatment should be considered?

IVC filter

Management for ovarian cysts?

If <8cm, follow for one or two cycles in premenopausal women If large or persistent, laparoscopy is warranted

What does a progestin trial tell you?

If bleeding stops, anovulatory cycles are confirmed

Epinephrine indications? MOA?

Indicated in cardiac arrest. Also for profound bradycardia and hypotension as a drip. MOA: Vasopressor. Increases heart rate, contractility, and peripheral vascular resistance.

Symptoms of PVD?

Intermittent claudication Rest pain ED Sensorimotor impairment Tissue loss

How do you treat 1st degree burn?

Keep clean +/-neosporin Analgesics Hydration

What is the pathophysiology of burns?

Leads to direct injury of skin and then inflammatory response Loss of barrier leads to fluids out and bacteria in There can be pulmonary damage which leads to inhalation of CO and other particulate matter

What side diaphragm injury is most common?

Left (Sits up higher)

Disease of which coronary artery results in the highest rate of mortality?

Left main coronary artery (LAD+circumflex)

What are indications for CABG?

Left main disease 2+ vessel disease Unstable angina refractory to medical management/PTCA Postinfarct angina Coronary artery rupture, dissection, or thrombosis after PTCA

Most common type of skull fracture?

Linear skull fractures

____hypertension is potentially life threatening - HTN plus rentinopathy, cardiovascular/renal compromise, or encephalopathy.

Malignant

Most common bone fractured in an orbital wall fx?

Maxillary bone

What are the 6 childhood exanthems (rash-causing diseases)?

Measles (Rubeola) Scarlet fever German measles (Rubella) Filatov-Dukes disease Fifth disease (Erythema Infectiosum) Roseola

What is CK-MB useful for?

Measure when reevaluating for reinfarction because Troponin remains elevated for 5-7 days

Typical tx for EP?

Methotrexate vs surgical (laparoscopy)

What is 1st line therapy for Trich?

Metronidazole 2g PO x1

______presents with a mid-systolic click heard best at apex.

Mitral valve prolapse

What are types of ovarian cysts?

Most are functional (90%): Follicular, Corpus luteum (Luteal cyst), theca lutein cysts Malignant

What are cardiac PE findings with CHF?

Parasternal lift Enlarged or displaced PMI Diminished first heart sound S3 gallop S4 gallop in diastolic failure

What is thrombophebitis/DVT?

Partial or complete occlusion of a vein plus inflammatory changes

Who is most affected by EM?

Patients 20-40 yo Males>Females

Who does Bullous Pemphigoid usually present?

Patients >60

WHen would mechanical ventilation be required for asthma exac?

Peak flow <25%

Cullen sign

Periumbilical discoloration indicative of peritoneal bleeding

What is torsades de pointes?

Polymorphic V tach in which the QRS complex twists around the baseline. May occur spontaneously or when the patient has hypokalemia or hypomagnesemia or following meds that prolong the QT

How does primary and secondary dysmenorrhea change with age?

Primary - decreases with age Secondary - increases with age

ergonovine challenge is used for what condition?

Prinzmetal angina

What is the hallmark of vasovagal syncope?

Prodrome of dizziness, nausea, pallor, diaphoresis, and diminished vision while standing

what is a more common EKG change that occurs in kids (considered to be abnormal, but a more common abnormality)

QTc prolongation

What is the path of blood through the heart?

RA, Tricuspid, RV, Pulmonic valve, Pulmonary artery, Lungs, Pulmonary veins, LA, Bicuspid (Mitral) Valve, LV, Aortic valve, Aorta

Diagnostic procedure of choice for bladder injuries?

Retrograde Cystogram

What is Premature rupture of membranes (PROM)?

Rupture of amniotic membranes before the onset of labor at or beyond 37 weeks of gestation. Occurs in 8% of pregnancies. 90% of women will go in to labor within 24 hours of PROM. Preterm PROM occurs before 37 weeks.

pathophysiology of sick sinus syndrome

SA node has completely random firing resulting in periods of tachycardia and periods of bradycardia

Death in the third peak is typically due to?

Sepsis or multiple organ dysfunction

____lesions have a depression in the center, spread from the face/arms to trunk/legs, and are all at the same stage.

Small pox

What women with what conditions should not be given OCPs?

Smokers HTN DM Vascular disease Breast Cx Liver disease Focal headaches

ED rash cocktail

Solumedrol Benedryl Zantac

How is spinal shock distinguished from neurogenic shock?

Spinal shock causes loss of all neurologic function below the level of the lesion - Neurogenic shock is characterized by autonomic dysfunction

Second most commonly injured organ?

Spleen

A Kehr's sign is usually indicative of?

Splenic injury

How do you treat SJS?

Stop offending drug Supportive care: Replace fluids and electrolytes, high caloric supplementation Cool Burrow solution (aluminum acetate) Prevent sepsis

How do you manage fetal distress?

Stop pitocin (if applicable) Change maternal position Administer O2 Measure fetal scalp pH

What is a rare complication of Measles?

Subacute sclerosing panencephalitis (SSP) - fatal encephalitis that occurs years after initial infection

____hematoma is an aneurysm presenting with a thunderclap headache, stiff neck, and delirium.

Subarachnoid

Treatment for thrombophlebitis?

Superficial thrombophlebitis is treated with bed rest, local heat, elevation of extremity, NSAIDs Antibiotics Surgical intervention if serious

Treatment for Erythema Infectiosum?

Symptomatic tx

T/F Suspected spinal cord injuries require MRI

TRUE

bradycardia is concerning b/c it can decompensate into another arrhythmia... what is that arrhythmia?

Torsades

What causes Trichomoniasis?

Trichomonas - a flagellated protozoan

_____is blowing and musical best heard along left sternal border.

Tricuspid regurgitation

alcoholic presents to ED w/ abdominal pain. after HPI and PE you're suspicious of cirrhosis. What's the go-to imaging to find out?

US (I say this b/c I'm quick to pull the trigger for MRI, typically need a lower imaging form prior to higher level)

How do you diagnose Trich?

Wet mount - will see motile flagellates Pap smear

What are clinical features of a spider bite?

Will feel pain 3 hours after bite and systemic symptoms begin 4-6 hours after bite.

Methanol is broken down into formic acid which may cause blindness. Etylene glycol causes profound metabolic acidosis and renal dysfunction. Treatment of both is centered around what?

antagonism of alcohol dehydrogenase w either ethanol or 4-MP and by dialysis

what is a pneumothorax

collpased lung

In Guillain-Barre, what is the typical direction of weakness on the body?

legs then ascending pattern up the body ascending symmetric weakness or paralysis and loss of deep tendon reflexes - CSF, high protein and normal cell count

opiates are one cause of a coma. when a patient presents to the ED in a coma, you may not know the cause and drug overdose is a leading cause. what are some of the features on the patient that suggest narcotic overdose?

respiratory depression, constricted pupils (need to consider alternatively use of miotic drugs, argyll robertson pupils d/t syphilis, pontine hemorrhage, organophosphates from insecticides)

define the difference between SIRS and sepsis

sepsis is SIRS w/ a known source (i.e. an infection)

what differentiates severe sepsis from septic shock?

severe sepsis is responsive to fluid, septic shock is not

SIRS criteria

temperature (less than 36, greater than 38), WBC (less than 4K or greater than 12K), HR>90, RR>20 this needs to be combined w/ clinical judgment

tell me everything that you know about hypertrophic cardiomyopathy

tends to occur in young patients (late teens) and is a sudden cause of death. alternatively, may cause SOB d/t obstructed LV outflow tract, which sounds like aortic stenosis (thus aortic stenosis on a young patient is hypertrophic cardiomyopathy until proven otherwise). diagnosis made w/ ECHO, tx w/ BB to increase preload

cyanotic lesion w/ ductal-dependent pulmonary blood flow; right to left shunt

tetralogy of fallot

utility of abx in pneumothorax

there is none. don't use abx. i say this because I was unsure as to whether or not they were indicated, they're not.

Types of Distributive shock

- Anaphylactic shock - Septic shock - Spinal Shock

Ellis II fracture

- Exposed area should be covered with calcium hydroxide paste, glass ionomer, or a strip of adhesive barrier - Involves dentin as well as the enamel - Require dental involvement within 24 hours

Areas Scanned in a FAST scan

- Hepatorenal recess (Morrison's pouch) - Perisplenic space - Pericardium - Pelvis

Class 1 hemorrhagic shock

- Loss of 15% of blood volume - Minimal tachycardia - No measureable change in BP, Pulse pressure, and RR - Body can compensate well

Management of Class I and II hemorrhagic shock

- NL saline, lactated ringers (Volume expander)

Miller blade

- Straight blade goes right over epiglottis and pushes it out of the way manually

Tx of Cardiac Tamponade

- Thoracotomy is TOC - Pericardiocentesis can be done to provide immediate decompression

What is an acute MI?

A result of prolonged myocardial ischemia, usually as a result of a thrombus formation on a disrupted or eroded atherosclerotic plaque. 1/5th of patients will die from V.fib

Altered Mental Status

AEIOU: Alcohol, Endocrine/Electrolytes, Infection, Oxygen/Opiates, Uremia TIPS: Trauma/Temperature/Toxins, Insulin, Psychiatric/Porphyria, Stroke/Schock/SAH

What is German Measles?

AKA "RUBELLA" Symptoms last 2-3 days Half of patients with Rubella are asymptomatic

How do you distinguish AS from hypertrophic obstructive cardiomyopathy?

AS decreases with Valsalva Hypertrophic cardiomyopathy increases with Valsalva (decreases with squatting and hand grip)

What is the MC cause of 3rd trimester bleeding?

Abruptio placentae

Anterior or posterior dislocation more common in mandibular dislocation?

Anterior

Diagnostic studies for thoracic AA?

Aortography CT/MRI are preferred over US

Labs for CHF?

BNP BMP LFT CBC

Hypertension

Blood Pressure >140/90 Common in the ED, 33% of HTN in ED is situational, Treatment in asymptomatic patients is controversial

How do you treat urticaria?

Cause should be eliminated H1 antihistamine administered (Diphenhydramine, Hydroxyzine, fexofenadine, cetirizine) H2 antihistamine (famotidine or ranitidine) if not responsive to H1. Give EpiPen if there is concern for anaphylaxis

Non operative spinal stabilization may be achieved by?

Cervical orthoses

What is the pathway of ascending infection that leads to PID?

Cervicitis-->Endometritis-->Salpingitis/oophoritis/tuboovarian abscess--->Peritonitis

Fixed Drug Eruption

Drug rxn occuring in solitary erythematous patch, plaque, erosion, or bullae Upon rechallenge still breaks out at same site

Diagnostic studies and findings with cardiac tamponade

Echo CXR - water bottle heart shape EKG - electrical alternans

What is the classic patient to have an AA?

Elderly male smoker with CAD, emphysema, and renal impairment

What are women who don't menstruate in the presence of estrogen production at risk for?

Endometrial cancer - lining keeps building up which can lead to atypical cells

What is adenomyositis?

Extension of endometrial glands into the uterine musculature

T/F magnitude of hematuria correlates with magnitude of injury

FALSE

if there is ONE major risk factor for an aortic dissection, it is this

HTN (not HLD)

When should aspirin or clopidogrel be used in a patient with a STEMI?

Immediately

Anterior cord damage results in?

Impairment and loss of motor function, pain and temperature sensation below the level of the lesion

What is cervicitis?

Inflammed, friable cervix that is quick to bleed

What is secondary dysmenorrhea?

Painful menstruation caused by an identifiable clinical condition, usually in the uterus or pelvis. Usually affects women >25 yo

Risk factors for DVT?

Surgical procedures (notably hip replacement) Prolonged bed rest Lower extremity trauma Oral contraceptives/HRT Inherited states (Factor V Leiden, protein C, protein S, antithrombin deficiencies) Elderly Obesity Travel Multiparity IBD Lupus

T/F Pulsus Paradoxus is seen in severe cardiac tamponade

TRUE

What is preload? afterload?

What is preload? afterload? Preload: AKA end-diastolic volume (EDV). LV wall stress before contraction. Afterload: AKA end-systolic volume (ESV) Force opposing/stretching muscle after contraction begins. Clinically reflected by SBP.

what is a tension pneumothorax

air can enter the pleural space but cannot exit

What nerve injury is associated with a sacral fracture?

cauda equina

a murmur consistent w/ aortic stenosis in a young patient (i.e. 20) is d/t which condition until proven otherwise?

hypertrophic cardiomyopathy

Cardiology

is fun!

ventricular septal defect Sx

loud harsh holosystolic murmur heard best at LSB note: large defects may lead to CHF, pulmonary infections, and delayed growth (failure to thrive)

how to tx a outpatient, healthy person with community acquired pneumonia

macrolide or doxy

time is of the essence in evaluating a patient suspected of having a stroke... why?

need to determine whether or not thrombolytics are apporpriate; they're useless after something like 4 hrs of onset of ssx

why must heparin be given before warfarin

otherwise risk developing warfarin skin necrosis; also makes em' more coagulable briefly

all diastolic, holosystolic, late systolic, or continuous murmurs should be considered what

pathologic

How is osteomyelitis diagnosed? Lab findings? What is the treatment for osteomyelitis?

plain x-ray, WBC is normal, possibly anemia of chronic disease, elevated sed rate, IV antibiotics or antifungals for 4-6 weeks

major ddx for cough and fever (3)

pneumonia, bronchitis, abscess

Treatment for ____ which presents with fever, tachycardia, and CVA tenderness is with Ampicillin & an aminoglycoside for 2 weeks.

pyelonephritis

What nerve injury is associated with an humeral shaft fracture?

radial

when evaluating patients suspected of having had a stroke (or actively having) it is a good idea to inquire about extremity pain... why?

that is atypical for a stroke and suggests an alternate diagnosis

four causes of intrinsic renal failure

vascular (dissection, thrombosis, emboli), glomerular (glomerulonephritis), interstitial (acute interstitial nephritis), tubular (acute tubular necrosis which is the most common form)

Mallory-Weiss tears occur after a bout of retching -- from bleeding that involves tears of the underlying _____.

venous or arterial plexus

Pharyngitis Etiology: Rhinorrhea suggests ___etiology. Pharyngeal exudates suggests ___etiology. Vesicles or ulcers suggests ___etiology. Conjunctival congestion suggests ____.

viral; bacteria (Strep or Epstein Barr); herpes simplex; adenovirus

T/F Massive hemothorax requires immediate tx without delay for imaging

TRUE (Tube thoracostomy)

HR range of atrial tachycardia

125-250bpm

What percent of patients with unstable angina will have a normal EKG?

25%

CDC guidelines for PPD result interpretation: 1. __mm or greater positive when a. HIV positive or who have HIV risk factors; b. recent close contact w TB; c. CXR w evidence of healed TB infex 2. __mm or greater positive when a. born in high prevalence area b. IV drug user C. low income population d. nursing home resident e. <age4 f. DM, malignancy, corticosteroid use, immunosuppression, renal failure. 3. __mm or greater is positive in all other cases.

5mm; 10mm; 15mm

How do you interpret BNP levels?

<100 is unlikely to be CHF 100-500 is may be CHF 500 is likely to be CHF

Grade IV liver laceration

>10 cm deep

How do you classify HF (3)?

Anatomically (R vs L) Physiologically (Systolic vs diastolic) Functionally (How symptomatic is patient)

Ddx with chest pain?

Angina pectoris Unstable angina Prinzmetal angina Acute MI Aortic dissection Pericarditis Acute pericardial tamponade PE MSK cause (costochondritis) GI cause (PUD, gastritis) Pulm cause (PNA)

Vascular injury to the neck can be imaged via?

Angiography or CT angiography

Diagnostic studies for arterial occlusion?

Arteriogram (A-gram) EKG Doppler

What are some signs and symptoms of mitral stenosis?

Asymptomatic Mild DOE Eventually A. fib Rare due to routine PCN use Mostly seen in females

What are clinical features of a thoracic AA?

Asymptomatic Substernal, back, neck pain Dyspnea Stridor Cough Dysphagia Hoarseness Symptoms of SVC syndrome

What are clinical features of ovarian cysts?

Asymptomatic masses Painful mass Menstrual delay Hemorrhage because of rupture

Arrhythmia Types

Atrial Fibrillation Atrial Flutter PSVT Heart Block PVC, PAC, MAT Ventricular Fibrillation Ventricular Tachycardia WPW

How do you treat third degree burns?

Early excision of eschar STSG autograft (split thickness skin graft)

PE Diagnosis and Treatment

Dx: Well's Criteria (risk factors), Ddimer, Doppler US, PERC Rule, CTA, VQ Scan Tx: Heparin (quick on/off) vs. LMWH (easier to give) Thrombolytics (Saddle PE, admit to ICU) Embolectomy

minimal workup for all pts presenting w/ chest pain

CBC w/ diff, CXR, BMP, troponin, EKG

Tx of MAT?

CCBs Mg sulfate Replete K levels

What drugs are used for rate control of A.fib?

CCBs (Diltiazem, Verapamil) Amiodarone (if EF<40%) Dig Dronedarone BB (if no heart failure)

What is the most frequent cause of hospitalization in the elderly?

CHF - 10% of patients have it after age 80

_____refers to the condition of excess mucus production and productive cough occurring for at least 3 months a year for 2 consecutive years. "blue bloater", course rhonchi

Chronic Bronchitis

Functional Classification of Heart Failure: ___-No cardiac symptoms with ordinary activity. ___-Cardiac symptoms w MARKED activity but asymptomatic at rest ___-Cardiac symptoms w MILD activity but asymptomatic at rest ___-Cardiac symptoms at rest.

Class I, Class II, Class III, Class IV

T/F Loss of consciousness is a defining characteristic of a concussion

FALSE

When is an AMI most likely to present?

Early morning because of surge of catecholamines

What is the definitive method for diagnosing heart murmurs?

Echo (particularly transesophageal) Cardiac cath

Prevention for DVT?

Elevation of foot of bed Leg exercises Compression hose SCDs Anticoagulation if high risk (enoxaparin or unfractionated heparin followed by warfarin) <---preferred method

Most common cause of Hypovolemic shock?

Hemorrhage

Which Hepatitis are transmitted by fecal-oral route? parenteral/sexual route?

Hep A&E - fecal-oral Hep B,C,D - parenteral/sexual

What is the pathophysiology of urticaria?

IgE mediated. Caused by release of histamines, bradykinen, kallikrein, and other vasoactive substances from mast cells and basophils in the skin. Causes small blood vessels to leak and results in intradermal edema.

What is first line therapy for treatment of dysmenorrhea? What is the MOA?

NSAIDs - they inhibit prostaglandin synthesis and decrease the volume of menstrual flow.

What are US findings of a non-viable pregnancy?

Inappropriate development and growth Poorly formed or unformed fetal pole Fetal demise

What are two types of contact dermatitis?

Irritant - caused by a chemical (ex: detergents) Allergic - caused by an allergen (ex: poison ivy)

Cervical and Lumbar Radiculopathies L4 - motor weakness? decreased reflex? decreased sensation?

L4 motor - anterior tibialis reflex - patellar tendon sensation - shin

Describe the pathophysiology of left sided heart failure

LV can't pump blood so fluid backs up in to the lungs and causes pulmonary symptoms

What are PE findings for an imperforate hymen?

Normal breast developement Labs: none Tx: Surgically open

What are PE findings for Androgen insensitivity? Labs? Tx?

Normal breast development Labs: High testosterone Tx: Remove testes, start estrogen

______cysts only cause pain when they leak contents causing tissue irritation or mechanical pressure on adjacent organs. What is the diagnostic imaging modality of choice? What patient requires emergent gynecological surgery?

Ovarian; pelvic/transvaginal ultrasound; patients with hemoperitonium and/or hypotension

PE EKG

S in lead 1, Q in lead 3, and inverted T in lead 3 are characteristic findings

T/F Hypotenison is associated with Increased morbidity and mortality in trauma pts

TRUE

What causes infective Endocarditis?

Strep viridans Staph aureus Enterococci

What is the best test when you have a stable patient with no EKG changes and no cardiac biomarker elevation?

Stress test

What type of hemorrhage is commonly spontaneous and atraumatic?

Subarachnoid hemorrhage

What is endometriosis associated with?

Subclinical peritoneal inflammation - with every period you get increased peritoneal fluid, increase of fluid white cells, and inflammatory cytokines

T/F Nasogastric and nasotracheal tubes are contraindicated in Le fort III fractures

TRUE

What is classic anginal chest pain?

Substernal/epigastric anterior chest pain Dull, fullness, pressure, tightness, crushing, squeezing Radiates to arm, neck, back, jaw

What are the s/s of menieres disease?

Sudden unilateral SNHL, roaring tinnitus, and vertigo for hours -Ear fullness

T/F Pts with CSF leaks are at an increased risk for meningitis and must be started on broad spectrum ABX

TRUE

T/F Pulmonary Contusions and Hematomas increase Risk for ARDS

TRUE

T/F TBI can alter Cerebral perfusion pressure (CPP)

TRUE

Aortic disruption

Traumatic aortic rupture caused by Rapid deceleration or shearing forces

What are the two signs of PE on CXR?

Westermark's sign - area of decreased pulmonary vascularity with a cutoff sign Hampton's hump - shadow or density in contact with one or more pleural space corresponding to lung segment involved

in evaluating a patient with chest pain, what is the utility in asking about the character of the pain in regards to when it was at its worst and how it is now?

a crescendo pattern is more suggestive of ACS

OPQRST is good for evaluating chest pain, although severity is pretty much useless (but is more suggestive of aortic dissection if severe). be sure to add size distribution of pain... why?

a small area of involvement, such as the size of a coin, is more suggestive of ACS whereas a larger area is less suggestive

chest pain that radiates between the scapulae is what condition until proven otherwise?

aortic dissection... check the vitals

MCC of lung abscess s/s CXR Tx

aspiration polymicrobial (anaerobic) fever, weightloss, fetid/bloody sputum halitosis CXR: cavitation, air-fluid level Tx: clindamycin

what is the presence or absence of cremaster reflex good for

assessing presence/absence of lumbar spine issues (saw this done on a PE of a pt w/ back pain)

how long must chest pain persist for it to be considered unstable (at least I think unstable... the one with the worse prognosis)

at least 30 minutes, some will say more than 20

What will a CXR show for a pleural effusion show?

blunting of costophrenic margins, can see free fluid in LLD position

pts presenting w/ ssx consistent w/ epiglotitis require artificial airway placement. discuss the indications for an endotracheal versus tracheotomy

endotracheal intubation is preferred, if that can't be accomplished do a tracheostomy (ostomy, not otomy)

Which of the below patients should receive Adenosine for initial treatment of their condition? a. 44 y/o alert female with PSVT with rate of 170/minute b. 65 y/o male complaining of chest pain, with ECG showing atrial tachycardia with rate of 140/minute c. 60 year old female with dizziness, chest pain, BP of 88/60 with ECG showing atrial fibrillation with ventricular response of 115/minute d. 70 y/o male with palpitations, agitation and ECG wthat shows wide complex tachycardia with rate of 200/minute

A

What are clinical features of Measles?

AKA Rubeola 3 C's: Cough, Coryza (cold-like symptoms), Conjunctivitis 4 day fevers KOPLIK's SPOTS- oral lesions that appear before the rash Macular-papular rash in the hairline that spreads down over 3 days

What are 1st line antibiotics for treatment of Gonorrhea?

Ceftriaxone (IM) + Azithromycin (PO) x 1 OR Doxycycline 100mg PO BID x 7 days

What are the stages of endometriosis?

I - Presentation of 2-3 superficial implants II - Appearance of one or more implants that occur within deeper layers of the tissue III - Many deep implants with small endometriomas on one or both ovaries. May also have filmy adhesions IV - Deep implants, large endometriomas, dense adhesions

What is aortic dissection?

Separation of the walls of the thoracic aorta from an intimal tear and disease of the tunica media. A false lumen is created and a "reentry" tear may occur, resulting in a double barrel aorta

Whiplash

Soft tissues surrounding the cervical spine get overly-stretched or damaged as a result of a whipping hyperflexion/extension action. Most often happens in car accidents & falls

Subdural Hematoma

- Collection of blood between the dura mater and the arachnoid layer - Sudden acceleration-deceleration causes shearing of bridging veins - Alcoholics and elderly are more susceptible

physical exam findings of pancreatitis

minimally some epigastric pain and decreased or absent bowel sounds... put this in the ddx for ileus and obstruction

An 18-year-old man presents to the ED after getting stabbed in his abdomen. His HR is 140 beats per minute and BP is 90/40 mm Hg. He is yelling that he is in pain. Two large-bore IVs are inserted into his antecubital fossa and fluids are running wide open. After 2 L of fluids, his BP does not improve. Which of the following is the most common organ injured in stab wounds? a. Liver b. Small bowel c. Stomach d. Colon e. Spleen

*C* Since most people are right handed and hold the offending instrument in their right hand, the LUQ is most commonly injured in stab wounds. However, the liver occupies the most space in the abdomen and therefore is the most common organ injured.

What is an inevitable abortion?

+Vaginal bleeding Cervix is open Products of conception have not passed, but no way to save pregnancy

What is an incomplete abortion?

+Vaginal bleeding Cervix is open There is partial passage of products of conception

Nexus Criteria

- Absence of tenderness in posterior midline - No evidence of intoxication - Normal mental status - Absence of a neurological deficit - No painful distracting injuries

Decorticate posturing

- Indicated intracranial injury above level of midbrain

When would a CT be indicated in pts with a mild TBI?

- Intoxicated pts - Pts who cannot be reliably observed after release - Pts on anticoagulants - Pts > 65 y/o

Class II Hemorrhagic shock

- Lose up to 1500 mls of blood - Tachycardia and Tachypnea - Decreased Pulse pressure

Tx of Pulmonary contusion/ hematoma

- Mechanical ventilation - High frequency oscillation - IVF

What are the relative contraindications to thrombolytic therapy?

1. uncontrolled hypertension 2. previous CVA or intracranial pathology 3. noncompressible vascular punctures 4. prolonged CPR >10 minutes 5. pregnancy 6. active peptic ulcer disease 7. recent trauma within 2 weeks 8. major surgery within 3 weeks 9. recent internal bleeding within 2-4 weeks 10. current anticoagulation with INR 2-3 11. known bleeding diathesis 12. history of chronic severe hypertension 13. prior streptokinase allergic reaction (don't give streptokinase)

How do you distinguish AS from MR?

AS radiates to the carotids

Where do AA occur? Which is more common?

Abdominal (90%) Thoracic (10%)

What patterns on fetal monitoring indicated fetal distress?

Absence of baseline variability Bradycardia (FHR<110) Recurrent variable or late decelerations

What is a complete abortion?

+Vaginal bleeding Cervix is open All products of conception have passed

What are the EKG findings of a STEMI?

1 MM ST segment elevation in two contiguous leads

cardiac involvement needs to be put into the ddx for abdominal pain. why?

AMI can present w/ abdominal pain.

Chest pain DDx

AMI, Angina, Aortic Dissection, PE, Community Acquired Pneumonia, Pneumothorax,Pleurisy, Pericarditis, GERD, Esophageal Spasm, Malignancy, Musculoskeletal, EDA, Shingles

What are the s/s of myxedema coma?

AMS -Hypothermia -Hypotension, bradycardia, hyponatremia, hypoglycemia, hypoventilation -Edema of hands and face -Thickened nares, swollen lips, or enlarged tongue

Shock Work up

*History, history, History* CBC, CMP, UA, Trop, Coag Panel, hCG, ABG, Lactate CXR EKG Cultures

CVA Workup and DDX

*Hypoglycemia*-CHO CBC, INR/aPTT, BMP EKG,Trop Stat Head CT Carotid US ECHO

Central cord syndrome presents with?

-Greater motor weakness in the upper extremities than in the lower extremities

target BP goal (for MI patients?) w/ comorbidities of DM, HF, renal insufficiency)

130/80mmHg

Stage 1 Hypertension is defined as greater than ____. Stage 2 Hypertension is defined as greater than ____.

140/90; 160/100

Viral Exanthems 4th disease or Filatove Dukes disease is not considered a separate disease anymore. Thought to be a milder, variant form of scarlet fever. ___presents w flu-like illness and later lacey rash on arms & legs, "slapped cheeks". ___presents with high fever then defervescence then maculopapular rash; HHV6

5th disease or erythema infectiosum; 6th disease or roseola

What is Stanford A and B?

A - Involves ascending aorta B - involves descending aorta

Tx for symptomatic junctional rhythm?

Atropine .5mg q 5min till you reach 2mg

SVT

Attempt Vagal Maneuvers Adenosine (6, 12, 12) Cardioversion

What is the most common arrhythmia?

A fib

pt presented to ED after dog bite, aside from swelling asymptomatic. he needs rabies prophylaxis and some abx coverage. name the abx that should be given for him (bites on arms)

Augmentin... it's good for staph but not MRSA but low suspicion of MRSA here. Clindamycin technically works but coverage for staph is getting worse and it tears up the GI tract (C. diff)

What is Stevens-Johnson Syndrome (SJS)?

A vesiculobullous disease of the skin, mouth, eyes, and genitalia. Ulcerative stomatitis leads to hemorrhagic crusting. <10% skin loss

What is the MOA of Danazol?

A weak androgen - it inhibits gonoadotropin release at the hypothalamus so there is no FSH/LH surge mid-cycle. This prevents growth of endometrial tissue.

What are conditions other than an MI that troponins can be elevated in?

Acute and chronic HF Cardiomyopathy Pericarditis, myocarditis LVH Severe HTN Arrhythmias (A.fib, SVT) Sepsis Stroke Strenuous exercise

What are chronic complications that occur from PID?

Adhesions which can lead to: Ectopic pregnancy Infertility Chronic pelvic pain

What is Cellulitis?

An acute, spreading infection of dermal and subcutaneous tissues

A widened mediastinum of CXR is indicative of?

Aortic Disruption/ Rupture

____is a disease in which the bone marrow and stem cells that reside there are damaged. This causes a deficiency of RBC, WBC, and platelets. CBC shows pancytopenia. What hemoglobin requires small volumes (3-5ml/kg) over 3-4 hours to be infused? What can happen if given too rapidly? What is their best chance for survival?

Aplastic anemia; </-6g/dL; heart failure; bone marrow transplant

What are the most common causes of 2ndary amenorrhea with normal estrogen?

Asherman syndrome PCOS

Who is Brugada syndrome most often seen in?

Asian men

What is the anti-inflammatory used for RF?

Aspirin is DOC for anti-inflammatory, even in kids

What is multifocal atrial tachycardia (MAT)?

At least 3 different sites of atrial ectopy. Most often found in elderly patients with decompensated COPD

Causes of AA? Which is most common?

Atherosclerosis <---MC cause Genetic Giant cell arteritis Vasculitis Trauma Marfan syndrome Ehlers-Danlos syndrome

BATTLE CAMP is a mnemonic for hemoptysis... spell out each letter

B=bronchiectasis or bronchitis A=aspergillous or autoimmune T=TB T=Tumor (pulmonary) L=lung abscess E=Embolus as in pulmonary embolism (has to be pretty huge) C=cystic fibrosis, coagulopathy A=AVN or diffuse alveolar hemorrhage M=Mitral valve issue particularly stenosis P=pneumonia which is the most common cause according to onlinemeded

What is the first line therapy for chronic angina?

BB

The black widow and brown recluse spider produce severe reactions. Treatment is ice to prevent venom spread, local wound care, and abx for severe necrosis. Which spider has a antivenom that can be given for acute symptoms.

Black widow antivenom

Subdural Hematoma

Bridging Veins *Cresent* Venous bleed

What are symptoms of PAD?

Claudication which is relieved by rest <--- 1st symptom Ischemia Pain with exercise Late disease: skin ulceration, gangrene, loss of limb

barking cough in a kid is characteristic of which infection?

Croup

minimally three conditions/classes of medicatinos that may suggest pancreatitis for epigastric pain

DM, alcohol, glucocorticoids

DVT Diagnosis

Duplex US (if negative, follow up US in 3-5 days) Impedance Plethysmography Venography MRI DDimer Well's Criteria

What factors increase incidence of arrhythmias?

Electrolyte imbalances Hormonal imbalances Hypoxia Drug effects MI

Treatment of chronic venous insufficiency?

Elevation of legs Avoidance of extended sitting/standing Compression hose Stasis ulcers: wet compresses, hydrocortisone cream, skin grafting

How do you manage PROM after 37 weeks?

Expectant management if patient is stable If delivery needs to be expedited, induction of labor with prostaglandin cervical gel or oxytocin is indicated

What are 2 common types of abruption? Which is more common? Which is more severe?

External abruption - more common Concealed abruption - more severe

T/F Succinylcholine is appropriate to use in a burn pt

FALSE (causes Hyperkalemia in an already hyperkalemic Pt)

T/F Mild TBI results in a predominantly structural insult

FALSE, It is a predominantly metabolic insult

Diagnostic modality of choice for "tripod fracture"

Facial CT scan

What are the first and second line agents for seizure -- after establishing patent airway and reversing any metabolic causes?

First-line --- benzos Second-line --- phenytoin/fosphenytoin

What is Brugada syndrome?

Genetic disorder that causes syncope, v.fib, and sudden death often during sleep Characterized by RBBB, J point elevation, saddle shaped or sloped ST segment

What causes pilonidal disease?

Hair Sitting Friction Tight clothes Obesity Local tumor Poor hygiene

Which Hepatitis must you have in order to get Hepatitis D?

Hep B

Aortic aneurysm workup and treatment

High suspicion(non-urgent, urgent, emergent) US vs CT, 2 large bore IV's, Type and Cross 6-10 units PRBC, BP control, don't over resuscitate Surgical Consult- 3.5-4 cm needs consult and further work up

What is the EKG findings in hypocalcemia and hypercalcemia?

Hypocalcemia -- prolongation of QT interval; inverted T waves Hypercalcemia -- shortening of QT interval; prolongation of PR interval

Clinical features of Chlamydia?

In males - watery discharge which is less painful than Gonorrhea. In females - asymptomatic, cervicitis, salpingitis, PID

Aortic Dissection

Intimal tear with leaking blood into media and longitudinal cleavage from the adventitia Risk factors: *HTN*, connective tissue disease (Marfan's), pregnancy, smoking, aortic valve abnormalities M>F, 50-70 years old, 33% mortality if untreated

____occurs in infants and children 2month - 6 years. Currant jelly stools, inconsolable crying, drawing up legs. What can be used both as a diagnostic and therapeutic maneuver?

Intussusception; air enemas

What is an exudative pleural effusion caused by?

LOCAL ↑capillary and membrane permeability, usually infectious

Most commonly injured organ?

Liver

Appendicitis

Luminal obstruction-mucous accumulation-increased pressure-lymphatic obstruction- perforation Most common in 2nd and 3rd decades Classic: *Periumbilical pain, dull, migration to RLQ, sharp*, anorexia, vomiting, fever

Torsades medical tx

Mag (I think this is sufficient to get them back into bradycardia) followed by a beta agonist such as isoproterenol, or in my experience atropine (if i recall correctly)

How do we treat Torsades de pointes?

Magnesium

Tx of Tooth subluxation

Manipulate into proper position and splint where indicated

Can you diagnose CHF by itself?

No - always an underlying pathology

What is sinus tachycardia?

Normal sinus P waves and PR interval with atrial rate between 100-160.

Clinical features of A. flutter?

Originates from small area within the atria. Regular atrial rate between 250-350 Saw tooth waves

Tx of lice?

Permethrin on body Pyrethrin with piperonyl butoxide in hair Fine-tooth comb to remove dead lice and nits Wash all clothing, bedding, etc

What is a corpus luteal cyst?

Produced each cycle following ovulation from the remanants of the follicular cyst. It produces progesterone to maintain a pregnancy. If the patient does not get pregnant, progesterone levels decrease and withdrawal bleeding occurs. It is NORMAL in pregnancy but can cause pain.

What is shingles?

Reactivation of the varcella virus that has been dormant in ganglionic satellite cells.

Secondary Spinal cord injury

Result of vascular, chemical, and inflammatory process that follows primary injury

Procedure of choice to diagnose all suspected urethral injuries?

Retrograde Urethrogram (RUG)

What are the most common causative agent of Cellulitis?

Staph aureus Strep

What are risk factors for abruptio placentae?

Trauma <---#1 HTN Smoking Decreased folic acid Cocaine use Alcohol use (>14 drinks/week) Uterine anomalies High parity Previous abruption AMA

What is the rhythm that most people die from?

Ventricular fibrillation

How do you diagnose contact dermatitis?

Usually clinical diagnosis Patch test (for allergic CD only)

What are common causative organisms of PID?

Usually polymicrobial: N. gonorrhoeae C. trachomatis

Clinical features of cardiogenic syncope?

Usually sudden without prodrome

What is the minimum diagnostic criteria for PID?

Uterine tenderness or Adnexal tenderness or Cervical motion tenderness

____arrhythmia most frequently results from lower energy AC exposures whereas ____is more common after exposure to higher energy DC.

Ventricular fibrillation; asystole

Aortic Dissection workup and Treatment

Work up: CXR, CTA (stable pt), TEE (unstable), Pain control, Control shear forces if hypertensive (esmolol, labetolol, 60 bpm goal HR)-beta blocker plus nitroprusside, Fluids for hypotension, Prepare PRBC, Consult surgeon. Need to Type and Cross 10 units of PRBC

briefly, what is N-acetyl cysteine

acetaminophen toxicity antidote

What are the treatments for bradyarrhythmias?

atropine, pacing, or epinephrine/dopamine

beta blockers decrease 3 things: HR, BP, name the third

afterload

what types of bacteria are most commonly associated with cavitations

anaerobes aerbic gram-negative bacilli and staph aureus

With acute sinusitis, patient presents with sinus pressure or pain over infected sinus. Do not treat with antibiotics until how much time has passed? What is the drug of choice first line?

at least 7 days; Augmentin 1000/62.5 for 7-10 days.

I had traditinoally associated SIRS and Sepsis as a post-op compliation. although it can be d/t that, name some other conditions that can cause it.

autoimmune disorder, pancreatitis, burns, vasculitis, surgery complications, adrenal insufficiency, pulmonary embolism, compilcated aortic aneurysm, cardiac tamponade, anaphylaxis, drug overdose

In a patient with possible placenta previa, why should vaginal examination not be performed?

because digital examination could trigger a hemorrhage

acute rheumatic fever Tx

benzathine penicillin salicylates bed rest

Achalasia shows a classic ______ deformity on barium swallow. Treatment of achalasia can be done with a muscle relaxant like ____ or endoscopic injection of _______ toxin.

birds beak; nifedipine; botulinum

In a young patient with stroke symptoms and possibly recent neck trauma, what should be considered?

carotid dissection

name the labs that must be promptly ordered in patient suspected of having a stroke

coagulation studies (PT, PTT b/c you need to evaluate whether or not they can be given tPA), rapid glucose (their "stroke" may be d/t hypoglycemia)

name some causes of QT prolongation

congenital, acquired (typically medications, especially atypical antipsychotics, some antiarrhythmics), magnesium depletion, hypokalemia (alcoholics, diarrhea)

iniitla tx for Torsades

despite what another slide may say, it seems like shocking is the answer

name three causes of pre renal failure

hypovolemic (most common), cardiogenic, distributive

normal QT interval in males and females

males less than 0.43sec females less than 0.45 sec consider it prolonged if it is greater than 0.02 seconds added onto each of these (i.e. greater than 0.45 in males, greater than 0.47 in females)

in my opinion, pretty much any kid presenting w/ respiratory distress gets an xray... why?

many conditions have a similar presentation and this may help to differentiate; minimally may demonstrate thumbprint sign pathonomonic for epiglotitis, may show steeple sign pathnomonic for croup, may demonstrate foreign body indicative of foreign body aspiration (most commonly seen in RLL), may demonstrate pneumothorax

severe HTN needs to be assessed for end organ damage as this is what distinguishes an urgency from an emergency. what are some of the featuers to check for in regards to end organ damage?

need to consdier the heart, brain, liver, kidneys (some sources also say lungs) HTN is a major risk factor for an aortic dissection, especially when this elevated so check for mediastinal widening on CXR check for encephalopathy as told by altered MS, HA, visual changes, seizures, vomiting Acute pulmonary edema may occur (especially in hx of trauma, sepsis) as told by SOB, orthopnea, hemoptysis, S3 or S4, JVD, hepatosplenomegaly, CP/pressure ACS AKI as told by decreased urine output/hematuria, back pain, swelling of lower extremities

acyanotic cardiac defect that results in increased pulmonary blood flow; left-to-right shunts; connection btw aorta and pulmonary artery

patent ductus arteriosus

indications for a chemical stress test opposed to exercise stress test

patient unable to exercise (could be d/t weight, arthritis, whatever) or they have preexisting EKG abnormalities (dysrhtymia, pacemaker, etc)

what are some things that you can expect to find on an EKG of a pt w/ pericarditis

peaked T waves (also hyperkalemia), PR elevation/depression in aVR

left heart failure ssx

primarily pulmonary--crackles heard on exam, orthopnea, PND, DOE

Acute _____failure is defined by an increased creatinine x3 and a decrease in GFR by >75% or urine output <0.3 mL/kg/h for 24 hours or anuria for 12 hours.

renal

SIRS, sepsis, severe sepsis, septic shock, multi organ dysfunction syndrome are on a continuum. define what differentiates each fo these

sepsis is SIRS w/ a known source of infection, severe sepsis is sepsis w/ end organ damage of one organ and is responsive to fluids, septic shock is severe sepsis that is not responsive to fluids, MODS is above w/ failing organs (hypotension that cannot be resolved)

define sepsis, severe sepsis and septic shock

sepsis is two or more of the SIRS criteria plus infection; severe sepsis is sepsis plus organ damage or hypoperfusion (kidney failure, CNS changes, DIC, ARDS); septic shock is severe sepsis plus hypotension despite adequate fluid resuscitation (less than 60 systolic without pressors)

beriberi is aka

thiamine deficiency

Which of two most common volvulus's can be treated with rectal tube insertion?

sigmoid (cecal must be treated with surgery)

Four factors for ____potential are given the greatest weight: 1. depression or hopelessness 2. rational thinking loss 3. organized or serious attempt 4. stated future attempt

suicide SADPERSON sex age depression or hopelessness previous attempts/psych admissions excessive alcohol or drug use rational thinking loss separated, divorced, widowed organized or serious attempt no social support

pt presents to ED w/ complaint of left eye pain that started when she woke up, had contact lenses in overnight. pt is unable to open eyes, has been crying all day d/t pain, reports clear discharge. which exam(s) must be performed on this patient?

visual acuity (not actually practical in her case, but i say this to emphasize that it must be documented on each patient presenting w/ eye pain). also do a slit lamp. HPI of contact lenses in overnight suggests keratitis

____ is the most common syncope type in adolescents

vasovagal

Tx of Flail chest

- Maintain ventilatory support - Supplemental O2 - Pain control

HR of afib

>350bpm

Management of ACS?

MONA + Plavix IVF Strict bed rest with telemetry Serial EKGs Pulse ox PCI for STEMI

Modality of choice for suspected spinal cord lesions?

MRI

History to collect in Trauma setting?

"AMPLE" - Allergies - Medications - Past medical history - Last meal and last tetanus - Events preceding the injury

Testicular torsion occurs when the testis twists on the spermatic cord, causing venous outflow obstruction and eventual arterial occlusion. What is the most common deformity that predisposes a patient? What is generally the first imaging study? What is the definitive treatment?

"bell clapper" deformity; color Doppler ultrasound; surgical exploration

What does the AP and lateral neck radiographs show in patients with croup?

"steeple" sign - child may also have a mild leukocytosis -- Remember, PE shows a bark-like cough

How do you determine severity of burns?

% of skin burned Depth

presentation of a peritonsilar abscess

(kid) w/ sore throat (worst sore throat that they've ever had), hot potato voice, difficulty breathing; physical exam reveals deviated uvula, large tonsil

A 32-year-old woman presents to the ED with a persistent fever of 101°F over the last 3 days. The patient states that she used to work as a convenience store clerk but was fired 2 weeks ago. Since then, she has been using drugs intravenously daily. Cardiac examination reveals a heart murmur. Her abdomen is soft and nontender with an enlarged spleen. Chest radiograph reveals multiple patchy infiltrates in both lung fields. Laboratory results reveal white blood cells (WBC) 14,000/μL with 91% neutrophils, hematocrit 33%, and platelets 250/μL. An ECG reveals sinus rhythm with first-degree heart block. Which of the following is the most appropriate next step in management? a. Obtain four sets of blood cultures, order a transthoracic echocardiogram (TTE) and start antibiotic treatment. b. Order a monospot test and recommend that the patient refrain from vigorous activities for 1 month. c. Administer a nonsteroidal anti-inflammatory drug (NSAID) and inform the patient she has pericarditis. d. Administer isoniazid (INH) and report the patient to the Department of Health. e. Order a Lyme antibody and begin antibiotic therapy.

*A* - to R/O endocarditis which has high incidence in IVDA

How are SCIWORA pts evaluated?

MRI and neurosurgical evaluation

Le Fort I fx

- Maxillary fracture that extends to the nasal aperature - airway complications are uncommon - Maxillofacial surgery should be consulted with the ED

Which is one of the most sensitive tests to quantify the extent of an infarction?

MRI with gadolinium

Management of Class III and IV hemorrhagic shock

- Packed RBC

Tx of Rib Fx

- Pain Control - Incentive Spirometry - Positioning, Oscillation - NO immobilizers/binders

Evidence of urethral injury

- Pelvic crush injury - Blood at meatus - Distended bladder - No urine output from catheter

Management of Small PTX

- Supportive care - Supplemental O2 - Monitor - Discharge if no progression after 6-8 hours

Zone 1

- Suprasternal notch and clavicle border to the inferior cricoid

What imaging studies can be done for amenorrhea?

MRI/CT of hypothalamus and pituitary or pelvis Pelvic US or TVUS

Number one cause of trauma associated deaths?

MVAs

Tx of Tension pneumothorax

- Tube thoracostomy - Requires definitive Chest tube placement

Rapid Sequence Intubation

- is a medical procedure involving a prompt induction of general anesthesia and subsequent intubation of the trachea. RSI is typically used in an emergency setting or for patients in the operating room.

What test can be done to detect PROM (premature rupture of membranes)?

--nitrazine test detects pH of the fluid - alkaline if amniotic fluid --fern test detects crystallization of salts in the amniotic fluid

T/F The trauma pt who presents with significant head trauma, unconscious, with focal neurological deficit, or spinal tenderness has a cervical spine injury until proven otherwise

TRUE

T/F Type and Screen should always be ordered in a trauma scenario

TRUE

T/F a vaginal exam is indicated in all females who have pelvic fractures

TRUE

T/F disruption of 2 of the cervical spine columns causes instability of the cervical spine and danger to the spinal cord

TRUE

How do you treat VVC?

1 day, 3 day, or 7-14 day treatment duration. 1 day: Butoconazole cream Fluconazole 150mg PO - preferred Ticonazole cream

Etiology of Bacterial Meningitis in which age groups: 1. Strep pneumo; N Meningitis; H Flu 2. Group B Strep; E Coli; Listeria monocytogenes 3. Strep pneumo; Listeria monocytogenes

1. Ages 1 month - 50 years 2. neonates 3. >50 years or alcoholics

HR range for paroxysmal tachycardia

150-250bpm

What are the AV blocks?

1st degree: prolonged PR 2nd degree (Wenchkebach): longer, longer longer PR then dropped beat 2nd degree, type II: intermittent nonconducted atrial beats 3rd degree: complete dissociation of atria and ventricles. They both beat randomly and are unsynchronized

What is the yearly risk of an emoblic event with A.fib?

5%. Lifetime risk is 25%

some notes about heparin...

if you're also giving warfarin (such as in a heparin-warfarin bridge), always give heparin first (giving warfarin first may cause hypercoagulability). always watch out for HIT (if they get HIT once they should never get heparin again)

For synchronized cardioversion, use ____Joules for paroxysmal supraventricular tachycardia and _____Joules for a fib and VT.

50J, 100J

A minimum CPP of ____ mmHg is requires to perfuse the brain

60 mmHg

how to calculate the target HR for a stress test

80% maximum for age

What percent of spontaneous abortions occur in the 1st trimester? What percent of this is associated with a chromosomal abnormality?

80%; 80%

Mean arterial pressure (MAP) should be maintained above?

90 mm Hg

Epistaxis Anterior

90%, Trauma, dry air, URI, infection *Kiesselbach's Plexus* Look for source, Cautery, Gelfoam/Surgicel, Packing, ENT follow up

Cholecystitis

95% Caused by Cholelithiasis 5Fs- * Fat, Forty, Female, Fertile, and Fair* Sudden RUQ pain, radiates to back, associated with fatty meals, N/V, *Arrest of inspiration on palpation RUQ-Murphy's sign*

What burns qualify for the Parkland formula?

>20% BSA burned (2nd and 3rd degree burns only)

What drug is indicated next for a patient who remains in VF cardiac arrest after defibrillation & epinephrine 1 mg IV? a. amiodarone 300 mg IV b. Lidocaine 1 to 1.5 mg/kg IV push c. Procainamide 50 mg/min, up to a total dose of 17 mg/kg d. Magnesium 1 to 2 g, appropriately diluted, IV push

A

Which of the following is indicated in the treatment of a patient with hypotension and a Mobitz type 2 heart block? a. Atropine b. Synchronized cardioversion c. Transcutaneous pacing d. Epinephrine 1 mg IV

A

How do you manage a hemodynamically unstable patient with vaginal bleeding?

ABCs 2 large bore IVs Labs: CBC, CMP, T&C, Pregnancy test, PT/PTT, UA Cardiac monitor/pulse ox Crystalloid Fluids Blood Control hemorrhaging GYN consult

What are the drugs of choice for V.Tach in a stable patient? What is the preferred order?

ALP= Amiodarone, Lidocaine, Procainamide

What are characteristics of irritant contact dermatitis?

Affected area is sharply marginated and confined to area of contact. Signs: Erythema, vesicle, erosion, crust, scale Could occur within a few hours or over months

_____presents with abrupt onset of fatigue, malaise, bone pain, sweats, bleeding, and easy bruising. PE reveals pallor, petechiae, and ecchymoses.

Acute leukemia

The most common cause of ____is Strep pneumo, H flu, M cat and classic treatment is high dose amoxicillin - 80-90mg/kg/day

AOM - acute otitis media

What should be on your Ddx for musculoskeletal pelvic pain?

Abdominal wall myofascial pain - will have trigger points Chronic back pain Fibromyalgia Lower back pain

What is Dysfunctional uterine bleeding (DUB)?

Abnormal uterine bleeding in the absence of an anatomic lesion. Usually caused by a problem with the hypothalamic-pituitary-ovarian hormonal axis. Most commonly occurs after menarche and in perimenopause.

What are complications of abruptio placentae?

Abruption can lead to activating the extrinsic clotting mechanism which can cause DIC Compromised to placental blood flow Hemorrhage Renal failure Coagulation failure Death

How is Hyperosmolar Hyperglycemic State different from DKA?

Absence of ketones, BS is usually over 1000

What is secondary amenorrhea?

Absence of menses for 3 months in a woman who has previously menstruated. If a woman has an irregular period, then it must be 6 months

Complete spinal cord lesion

Absence of motor or sensory function below the level of the lesion

Pustular Drug Eruption

Acne like eruption with no comedones-can occur on arms/legs Generalized eruption - febrile, leukocytosis Caused by drugs

Torsed Gonad Female

Adnexal tenderness to palpation and possibly mass Must exclude ectopic pregnancy Pain control Prompt OB/GYN consult Ultrasound Laparotomy

What patient factors do you need to consider when treating endometriosis?

Age Extent of disease Symptoms Fertility

Hypertension Drug of Choice for: angina diabetes hyperlipidemia CHF Previous MI Chronic Renal Failure Asthma, COPD

Angina - BB, CCB Diabetes - ACEi & CCB, avoid diuretics Hyperlipidemia - ACEi & CCB, avoid diuretics/BB CHF - diuretics & ACEi, avoid CCB/BB Previous MI - BB/ACEi Chronic renal failure - diuretics, CCB Asthma - diuretics & CCB, avoid BB

What is unstable angina?

Angina that is either new in onset or differs from a patient's typical stable angina pattern

_____remains the gold standard for diagnosis of intestinal ischemia and may provide a route for infusion of vasodilatory drugs such as papaverine to help restore blood flow to the threatened segment of bowel.

Angiography

Tx options for MR?

Annuloplasty <---preferred tx Valve replacement - done when there is severe deformity or degeneration of valve

____occurs when the anterior aspect of the spinal cord is injured. Paralysis and loss of pain/temp sensation distal to the injury. Position, touch, and vibration sense is usually preserved. Prognosis poor.

Anterior cord syndrome

_____is a diastolic murmur heard along left sternal border. Austin-Flint murmur. What kind of pulse?

Aortic regurgitation; "water hammer" pulses

____is a decrescendo-crescendo rough, systolic murmur. Heard best at base of heart and radiates to neck. What drug is contraindicated?

Aortic stenosis; ACEi

___ulcers usually have punched out appearance on lateral foot. No hair. ____ulcers usually have edema, contact dermatitis, and irregularly shaped sores.

Arterial; Venous

mnemonic for reading CXR---ABCDEGHI

Assessment of quality Bone and soft tissues Cardiac Diaphragms Effusion

What are clinical features of an abdominal AA (AAA)?

Asymptomatic Pulsating abdominal mass Abdominal or back pain

What is an Austin Flint murmur? Quincke sign? "Water hammer" pulse?

Austin flint murmur - Reverberation of regurgitant flow Quincke sign - capillary pulsations of uvula "Water hammer" pulse - increased pulse pressure palpated over peripheral arteries Musset sign - head bobbing with each heart beat

Clay-Shoveler's fracture

Avulsion of the end of the spinous process of a lower cervical vertebrae - Stable Fracture

Where does Scabies usually distribute?

Axilla, umbilicus, groin, penis, instep of foot, web spaces, fingers, and toes In infants face and scalp can be involved

bacterial vs viral meningitis spinal fluid analysis

Bacterial: (*all high but glucose low*) opening pressure > 300 mm WBC count >1000/mm3 Glucose < 40 mg/dL Protein >200 mg/dL Viral: (*all low but glucose high*) opening pressure < 300 mm WBC count < 1000/mm3 Glucose > 40 mg/dL Protein < 200 mg/dL

More common fractures: ____is fracture/dilocation of thumb at the carpometacarpal joint. ____is Y-shaped three part fracture of thumb at carpometacarpal joint. ____is injury to the ulnar collateral ligament of thumb MCP joint w or wo fracture. ____is an oblique fracture through the base of the radial styloid. ____is an intra-articular fracture with a displaced radial articular fragment.

Bennett, Rolando, Gamekeeper's, Chauffeaur, Barton's

What should be on your Ddx for urologic pelvic pain?

Bladder malignancy Interstitial cystitis - will have irritative voiding symptoms - UTI symptoms with negative culture. Chronic UTI Urolithiasis

#1 goal in the OR

Bleeding control

____occurs when a lateral half of the spinal cord is injured at a specific level. Ipsilateral motor paralysis and contralateral sensory loss. Prognosis good.

Brown-Sequard syndrome

____is autoimmune blistering disease that primarily affects the elderly, IgG, biopsy shows subepidermal bulla with infiltration of eosinophils. Treatment?

Bullous pemphigoid; tx - steroids and possibly minocycline

A _____may develop after acute MI, PE, aortic stenosis and is due to a conduction delay in the right or left bundles.

Bundle branch block

What are fascicular blocks?

Bundle branch blocks

When using vasopressin on a patient who remains in persistent VF arrest, which of the following guidelines for use of vasopressin is true? a. Give vasopressin 40 U every 3 to 5 minutes b. Give vasopressin for better vasoconstriction and ß-adrenergic; stimulation than provided by epinephrine c. Give vasopressin as an alternative to epinephrine in the first or second dose of a vasopressor agent as treatment for ventricular fibrillation d. give vasopressin as the first-line pressor agent for clinical shock caused by hypovolemia

C

Which of the below are the proper sequence of events indicated for the performance of CPR and the operation of an AED. a. Send someone to call 911, attach AED electrode pads, open the airway, turn on the AED, provide, 2 breaths, check for a pulse b. Wait for the AED and barrier device to arrive, open the airway, provide 2 breaths, check for a pulse, if no pulse attach AED electrode pads, follow AED prompts c. Check scene safety, check the patient for responsiveness, send someone to call 911, check for a pulse, if no pulse start compressions until the AED arrives then attach the AED, follow AED prompts d. Provide 2 breaths, check for a pulse, if no pulse perform chest compressions for 1 minute, call for the AED, when the AED arrives attach electrode pads

C

tx for Prinzmetal angina

CCBs

What can exacerbate sick sinus syndrome?

CCBs BB Dig Sympatholytic agents Antiarrhythmics Aerosol propellant abuse

ED management of CHF exacerbation

CPAP/BiPAP Frequent re-evaluation, cardiac monitoring Nitrates are 1st line therapy Diuretics should be reserved for volume overloaded patients Nitroprusside if there is severe HTN

____presents as acute, painless, monocular vision loss in addition to risk factors for the disease ie. HTN, a fib, atherosclerosis, collagen vascular disease etc.

CRAO - central retinal artery occlusion

What imaging modality is a very accurate means to differentiate ischemic from hemorrhagic stroke? What is the imaging gold standard for stroke? In general, ischemic strokes are not visible on noncontrast CT of the head until at least ____ hours after brain infarction.

CT - noncontrast; CT angiography; 6 hours

What are the imaging modalities of choice for the detection of intraabdominal injury after trauma?

CT abdomen or ultrasound

utility of imaging in suspected case of peritonsilar abscess

CT only if you absolutely must confirm the diagnosis; this tends to occur in kids and you don't want to give them CA, this is also expensive. it can be difficult to differentiate tonsilitis from peritonsilar abscess and CT (or US--less common) can be done to differentiate the two

Imaging modality of choice to rule out significant cervical spine injury when suspicion is high

CT scan

Most specific scan for injury in blunt abdominal trauma?

CT scan

Primary diagnostic modality for observation of non-surgical abdominal trauma?

CT scan

____ is the procedure of choice for imaging pts with microscopic hematuria, history of shock, or sudden deceleration injury

CT scan

Modality of choice to image kidney in a trauma situation?

CT with IV contrast

some of the goals of therapy for the tx of sepsis syndrome are CVP, urinary output, MAP, central venous saturation... what are the values for each of these and what do they indicate?

CVP 10-12, urinary output of at least 0.5cc/kg/hr, MAP at least 65 to correct hypotension, central venous sat of at least 70% achieved w/ lots of fluid, BS abx (narrow if you know the infx), control infx sites (remove central lines), pressors, steroids

Dx of Rib Fx

CXR

Diagnostic tests for CHF? Which is the best test?

CXR EKG Echo <---best

Diagnostic tests for aortic dissection?

CXR - widened mediastinum, pleural effusion TEE CTA Aortography - gold standard (but not often done)

What causes Vulvovaginal candidiasis?

Candida albicans Not an STI!

_____presents with distended neck vein, indistinct heart sounds, narrow pulse pressure, and pulsus paradoxus. Tx: pericardiocentesis and treat underlying cause.

Cardiac tamponade

_____is due to reduction in volume of lumbar spinal canal, causing compression and paralysis - presents w leg weakness, urinary and anal sphincter control.

Cauda Equina syndrome

What is a transudative pleural effusion cause by?

Caused by SYSTEMIC hydrostatic or oncotic pressure imbalances, usually CHF

CVA

Cerebral ischemia/infaction Occlusive vs. Hemorrhagic HTN, atherosclerosis, DM, lipids, smoking, family hx, estrogen, trauma, Arteriovenous Malformations, aneurysms, tumors TIA(symptoms resolve in 24 hours)- 33% have CVA within 5 years

What are common signs of pregnancy? (9)

Chadwick sign Increased basal body temp Skin changes (melasma, linea nigra) +Preg test Hegar sign (softening of uterus) Uterine growth Fetal heart tones Palpation of fetus U/S of fetus

What are some pathologic changes of the heart that cause CHF?

Changes of myocardial contractility Structural integrity of the valves Preload or afterload of ventricle Changes in heart rate

What are characteristics of third degree burns?

Charred, pearly white Hallmark: Painless because of destroyed skin layer and nerve endings Most common cause: Grease, steam, flame, high voltage

Clostridium and Bacteroides fragilis are the typical pathogens for _________. Bacteroides fragilis and E Coli are the typical pathogens for ______. What are the imaging modalities of choice for cholecystitis and appendicitis?

Cholecystitis; appendicitis; Chole - ultrasound, then HIDA; Appendicitis - CT scan

What is Levine sign?

Clenched fist and teeth to describe classic anginal pain

What is Pelvic inflammatory disease (PID)?

Clinical syndrome associated with ascending spread of microorganisms from the vagina or cervix to the endometrium, fallopian tubes, ovaries, and contiguous structures.

What is Bacterial Vaginosis?

Common infection and recurrent cause of abnormal vaginal discharge in women of childbearing age. Caused by overgrowth of Gardnerella and other anaerobes. Not an STI.

Management of abruptio placentae?

DELIVERY of fetus and placenta is definitive tx (however depends on status of fetus and degree of separation) Blood type, cross-match, coag studies Insertion of large bore IV C-section is preferred route of delivery

76yo female presents w/ complaint of SOB. upon further questioning, you decided that it is DOE. physical exam reveals bilateral 2+ pitting edema of lower extremities, hepatosplenomegaly, JVD. what is your leading diagnosis? what will the work up entail? the tx?

DOE should make you think of HF; this can occur in left or right, but other ssx of JVD, peripheral edema, hepatosplenomegaly suggest right heart failure. Now the workup progresses to an ECHO to determine if it is systolic, diastolic or both; this will also give an ejection fraction value, which if less than 35% requires an AICD. Everyone with heart failure gets minimally ACEI and BB, most will also get a diuretic (typically initially furosemide, may also add spironolactone). workup will prolly also include a BNP, EKG, CBC w/ diff Every patient w/ HF also requires a fluid and Na restricted diet (no more than 2L/day fluid, no more than 2g/day Na)

The FAST exam is trauma has replaced what technique for examining hemodynamically unstable patients?

DPL - diagnostic peritoneal lavage

Advantage of DPL over FAST scan?

DPL definitively identifies blood, while FAST scan cannot discriminate between blood and fluid

What is the difference between dysfunctional uterine bleeding (DUB) and abnormal uterine bleeding (AUB)?

DUB has no anatomic lesion whereas AUB is "bleeding for a reason"

the Well's criteria can be used to predict whether or not a person has a PE, much like the PERC score; what is the Well's criteria

DVT sx (unilateral leg swelling, erythma, warmth) gets 3 pts, leading dx PE gets 3 pts, tachycardia (>100bpm) gets 1.5 pts, immobilized for more than 3 days (or?) surgery w/in past 4 weeks gets 1.5 pts, hemoptysis gets 1 pt, CA gets 1 pt PE likely if they have more than 4, unlikely if less than 4 pts

What is the clinical significance of an arrhythmia?

Depends how much it impairs cardiac output or how likely it is to deteriorate in to a more serious disturbance

Diffuse axonal injury

Disruption of the axonal fibers in the white matter and brainstem - Blurring of the grey and white matter margins, punctate hemorrhages and cerebral edema are seen

What are causes of secondary amenorrhea besides pregnancy?

Drug use Stress Significant weight change Excessive exercise

How is bacterial vaginosis diagnosed? Treatment?

Dx: fishy odor, increased vaginal discharge, elevated pH, and clue cells Tx: Metronidazole

A 21-year-old woman presents to the ED complaining of lightheadedness. Her symptoms appeared 45 minutes ago. She has no other symptoms and is not on any medications. She has a medical history of mitral valve prolapse. Her HR is 170 beats per minute and BP is 105/55 mm Hg. Physical examination is unremarkable. After administering the appropriate medication, her HR slows down and her symptoms resolve. You repeat a 12-lead ECG that shows a rate of 89 beats per minute with a regular rhythm. The PR interval measures 100 msec and there is a slurred upstroke of the QRS complex. Based on this information, which of the following is the most likely diagnosis? a. Ventricular tachycardia b. Atrial flutter with 3:1 block c. Atrial fibrillation d. Lown-Ganong-Levine (LGL) syndrome e. Wolff-Parkinson-White (WPW) syndrome

E. WPW syndrome is caused by an accessory electrical pathway (ie, Bundle of Kent) between the atria and ventricles. The primary significance of WPW syndrome is that it predisposes the individual to the development of reentry tachycardias. The classic ECG findings include a short PR interval (< 120 msec), widened QRS interval (> 100 msec), and a delta wave (slurred upstroke at the beginning of the QRS).

What are differentiating factors between EM and SJS or TEN?

EM: Occurs in young males Frequent recurrences Less fever Milder mucosal lesions No association with a collagen vascular disease, HIV, or cancer

SJS is thought to be a severe variant of ___. TEN is thought to be a severe variant of ___.

EM; SJS

GLASCOW COMA SCALE

EYE OPENING 4 - spontaneous 3 - open to speech 2 - open to pain 1 - no response VERBAL 5 - alert and oriented 4 - disoriented conversation 3 - inappropriate words 2 - nonsensical sounds 1 - no response MOTOR 6 - spontaneous 5 - localizes pain 4 - withdraws to pain 3 - decorticate posturing 2 - decerebrate posturing 1 - no movement

Decelerations of fetal heart monitoring: ____decelerations begin and end at the same time as the contraction. ____decelerations occur at any time and drop more than early or late decelerations. Result of umbilical cord compression. ____decelerations begin at peak of a contraction and slowly return to baseline after the contraction is complete. Uteroplacental compromise.

Early; Variable; Late

Diagnostic studies for endocarditis?

Echo - key! Helps identify valve involved CXR EKG

How do you treat endocarditis?

Empiric antibiotic tx (Gentamicin with Ceftriaxone OR Vancomycin) THEN adjust therapy once blood cultures come back Valve replacement if infection does not resolve

____is infection of the dermis with lymphatic involvement. More superficial that cellulitis; typically more raised and demarcated. Possible red, painful streaks of lymphangitis. Tx: Pen V, Amox, Azith, Clarith

Erysipelas - St. Anthony's fire

How do you tell the difference between Erysipelas and Cellulitis?

Erysipelas is superficial, raised, well-defined borders Cellulitis is deep, flat, and has poorly defined margins

Desquamation disease

Erythema multiforme minor (EM) Erythema multiforme major (Includes SJS, TEN)

learning scenario: 2yo female presents w/ abdominal pain, has a hx of GERD that hasn't been well managed despite multiple different PPIs and H2 blockers. Pediatric gastroenterologist recently tried to change one out for another in hopes of controlling reflux, now pt has abdominal pain that is persisting despite medical tx (abnormal for pt). denies other ssx including n/v/d/c, fevers, chills.

I had tunnel vision and thought this was due to changing medical mgmt recently. don't forget about ddx, in this case minimally includes appendicitis, intussusception, Meckel's diverticulum. CBC would raise suspicion for appendicitis, US more telling and could be done to move over to colon and look for intussusception (currant jelly stool is just a board question, not applicable to real life)

Preferred sedation med in rapid sequence intubation?

Etomidate (Doesn't drop BP)

How do you manage PPROM?

Expectant management if mother and baby are stable. Mother should be hospitalized and be on strict bed rest Betamethasone should be given if <34 weeks Abx to prevent infection NST and BPP daily to monitor fetal well being Amniocentesis to check for lung maturity Delivery if there is maternal or fetal distress

heart murmurs are graded on a scale of I-VI, such as II/VI systolic decrescendo murmur heard at the ULSB... describe how each of these grades is established

I--heard, but super faint such that you're not entirely positive that it's there II-faint but it's definitely there (immediately identifiable) III--moderately loud IV--loud and associated w/ a palpable thrill V--very loud but can't be heard w/o a stethoscope VI--loudest and can be heard w/o a stethescope these earlier stages are fairly subjective

What should not be used during an acute asthma exac.?

ICS

Initial tx for ACS?

IV access + telemetry + MONA: Morphine Oxygen Nitro ASA

What are risk factors for infective endocarditis? Which organisms are responsible?

IV drug user (staph aureus is most common cause) Prosthetic valve (staph aureus, Gram neg, fungi) Regurgitant cardiac defect which provides a nidus for development of vegetation Infection from procedure (bacteremia)

Pericarditis work-up and Treatment

IV, monitor, pain control EKG CXR CBC, BMP, ESR, Trop(4-6 hours after onset of chest pain), +/- blood cultures Tx: Anti-inflammatory agent,Cardiology consult

Cholecystitis Treatment

IV, pain control, NPO DDx: Pancreatitis, hepatitis, PUD, renal lithiasis, AMI, PE CBC, CMP, lipase, UA Ultra Sound Outpatient surgical consult for most cholelithiasis Admission for cholecystitis- medicine vs surgery

kawasaki disease Tx

IVIG high dose aspirin steroids (if IVIG unresponsive)

____or "honey colored crust" are contagious, superficial skin infections caused by Staph Aureus. Tx: Mupirocin ointment or Bactroban cream. Severe infection, use dicloxacillin. Keep clean.

Impetigo

What is indicated when EF is <35

Implantable defibrillator

What is Ectopic pregnancy (EP)?

Implantation of a pregnancy anywhere but the endometrium

What is peripheral vascular disease (PVD)?

Spectrum of disease of peripheral vessels. Includes PAD, varicose veins, thrombophlebitis, DVT, and chronic venous insufficiency.

How do you diagnose Bullous Pemphigoid?

Punch biopsy half blister/half normal skin. Send one to pathology and the other for immunofluorescence. You will see linear IgG deposits along basement membrane.

____is most common in first 2 months of life, first born males. Presents with projectile vomiting and "olive-shaped" mass. U/S to image. String sign on upper GI contrast study. What is the definitive treatment?

Pyloric Stenosis; laporoscopic pyloromyotomy

_____often originates at a site of dental infection and is a cellulitis of the floor of the mouth and neck originating in the submandibular space.

Ludwig's angina

medical tx for wide complex Vtach

amiodarone... this works for pretty much any Vtach

Flexion teardrop fracture

Extreme flexion causes a fracture of the anteroinferior portion of the vertebral body. - Highly unstable because they are associated with disruption of the ligamentous structure at the level of injury

T/F? genotype positive-phenotype negative for HCM is a disqualificaiton criteria for most competitive sports

F no evidence yet, but clinical diagnosis of HCM is a disqualification criteria

What causes Aortic Stenosis (AS)?

Genetic - due to a unicuspid or bicisupid valve CAD - due to a calcified/degenerative valve. MOST COMMON VALVULAR DISEASE in the US.

The most common pathogens of epiglottitis are Strep pyrogenes, Strep pneumo, and Staph aureus. Which vaccine has dropped the incidence of epiglottitis? Class lateral neck radiograph finding?

HIB; "thumb print" sign

diagnosis of pneumothorax

HPI may or may not be contributory as a spontaneous pneumothorax may not have a remarkable hx. suspicion is raised based on oxygen sat (good positive predictive value, not negative), or sinus tachycardia. order a CXR which will typically be sufficient; if it is negative but clinical suspicion remains give em' a CT

The ________principle means that as preload increases, the ventricle is stretched during diastole filling and the ejection fraction is increased.

Frank-Starling principle

What are clinical features of EM?

Frequent recurrences Lesions begin as macules and become papular. Then vesicles and bullae form in the center of he papules. Targetoid IRIS lesions are characteristic. Typically occurs on extremities MUCOSAL LESIONS that are painful and erode.

The Cerebrum lobes and functions: ____=voluntary motor function, verbal communication, personality, and judgement ____=general sensory functions ____=hearing and smell ____=visual information and story visual memories Broca is motor speech area. Wernicke is comprehending spoken or WRITTEN language.

Frontal Lobe; Parietal Lobe; Temporal Lobe; Occipital Lobe

____seizures (ie. abnormal excessive cortical neuron activity) are thought to originate from both cortical hemispheres resulting in loss consciousness and tonic-clonic muscle contractions.

Generalized seizures

What is the first thing you should in a patient with a suspected pregnancy?

Get a urine pregnancy test (hCG) - accurate after a woman has missed her period

Diagnostic studies for Gonorrhea?

Gram stain of urethral discharge with culture

Never do a pelvic exam in which patients?

Gravid female >20 weeks with bleeding

What is Urticaria?

Group of disorders that have many causes. Most common causes include food or drug allergies, heat/cold, stress, infection.

_____is most likely due to an immune-mediated mechanism --- due to lymphocytic infiltration and macrophage-mediated demyelination and axonal degeneration. Campylobacter, mononucleosis, CMB, herpes, mycoplasma

Guillain-Barre

What are clinical features of PROM?

Gush or persistent leakage of fluid from vagina or vaginal discharge PE shows pooling of fluid

What is the PERC score? HADCLOTS

H - hormone/estrogen use A - age>50 D - dvt/pe history C - coughing blood L - leg swelling disparity O - O2 sat < 95% T - tachy>100 S - surgery/recent trauma

chlorthalidone is aka

HCTZ

Syncope

HEAD, HEART, VESSLS (mnemonic) HEAD- Hypoglycemia/hypoxia, Epilepsy, Anxiety, Dysfunction of brain stem HEART- Heart attack, Embolism of pulmonary artery, Aortic obstruction, Rhythm disturbance, Tachycardia (vtach) VESSLS- Vasovagal, Ectopic pregnancy, Situational, Carotid Sinus sensitivity, Low SVR (hypotension), Subclavial steal *Check blood glucose*

What is the principal risk factors for EM?

HSV infection!! Previous history of EM

What is malignant HTN?

HTN with signs of acute end-organ damage 220/140

What are souces of emboli for arterial occlusions?

Heart (Afib) Aneurysms Atheromatous plaque

What is Congestive heart failure (CHF)?

Heart failure is a condition that occurs when the heart cannot pump or fill with enough blood due to pathologic changes in the heart. The heart must work harder to deliver blood to the body.

What is the Modified Duke Criteria?

Helps establish the diagnosis of infective endocarditis. Patients must have either 2 major, 1 major and 3 minor, or 5 minor criteria to be diagnosed

DVT Treatment

Heparin or LMWH Thrombolysis (role unclear) Inpatient vs home care IVC filter (failed heparin or LMWH, contraindication to anticoagulation, breakthrough DVT)

Which Hepatitis can become chronic?

Hepatitis B, C, D

What are clinical features of TEN?

High fever +Nikolsky sign (epidermis detaches with rubbing) Diffuse erythema Necrotic epidermis Sheetlike loss of epidermis High mortality

What is the most sensitive clinical sign of angina on an EKG?

Horizontal or down sloping ST segment depression

correction

Howell-Jolly bodies, not Ab

What is the treatment for hypocalcemia and hypercalcemia?

Hypocalcemia -- oral Ca for asymptomatic pt; IV Ca for symptomatic patients Hypercalcemia -- IV isotonic saline and IV furosemide

Should Clinical Indications

Hyptension,Cyanosis, Altered mental status, end organ dysfunction, lactate elevation

What is Debakey I, II, III?

I - involves ascending and descending aorta II - involves ascending aorta only III - Involves descending aorta only

What are the four stages of decubitus ulcers?

I - non-blanching erythema II - Necrosis, superficial or partial thickness involving the epidermis and dermis III - Deep necrosis with full-thickness skin loss IV - Full-thickness ulceration with extensive damage and necrosis of muscle, bone, and supporting structures

How do you manage spontaneous abortion?

If pregnancy is early the patient can be managed expectantly with careful f/u, hCG titers, and TVUS to make sure abortion proceeds to completion Misoprostol (Cytotec) D&C - to ensure complete emptying of uterus Immunoglobulin should be given to Rh negative women If abortion is septic - evacuation of uterus, medical support, and antibiotics

Tx of cardiac tamponade?

If there is hemodynamic compromise, a pericardiocentesis is necessary to relieve fluid accumulation. NSAIDs - if due to pericarditis Abx - if infectious

Drug Induced Urticaria

IgE mediated-after sensitiziation occurs 7-14 days, if previously sensitized can occur in minutes Immune complex mediated - 7-10 days to react, up to 28 days NSAIDs - After drug administration 20-30 min

What is intermittent claudication?

Impairment in walking, or pain, discomfort or tiredness in the legs that occurs during walking and is relieved by rest. The perceived level of pain from claudication can be mild to extremely severe. Claudication is most common in the calves but it can also affect the feet, thighs, hips, buttocks, or arms. MC caused by PAD.

What is a junctional rhythm?

Impulses are generated by the AV node and go retrogradely in to the atria Occurs in patients with sinus brady, SA node exit block, or AV block There is no P wave (masked by QRS) or inverted P wave BPM is 40-60

When is defibrillation indicated?

In a pulseless patient with a shockable rhythm (like V.fib)

What is Erysipelas?

Infection involving the superficial dermal lymphatics

Which EKG leads exhibit changes with an inferior MI? Posterior? Anteroseptal? Anterior? Anterolateral?

Inferior - II, III, aVF Posterior - V1, V2 Anteroseptal - V1, V2 Anterior - V1-V3 Anterolateral - V4-6

ST Elevations in II, III, aVF

Inferior MI; right coronary artery

Zone II

Inferior cricoid cartilage to the angle of the mandible

Pericarditis

Inflammation of pericardium Cause: Idiopathic, viral, malignant, Dressler's S/Sx: Sharp, *Restrosternal, Improved with sitting up and leaning forward, Friction Rub* Common presentations: young, chest pain, no risk factors, after viral infection Dressler's common after procedure for MI

What is Pericarditis?

Inflammation of the pericardium which is usually idiopathic or viral (90%). Other causes include bacterial, autoimmune, connective tissue disease, neoplasm, radiation, chemo, drug toxicity, cardiac surgery, or myxedema. MC in men and <50 yo.

Coup injury

Injury directly below the point of impact

Ellis I fracture

Involves enamel only - No acute intervention is necessary

Presentation of Mandibular fracture

Jaw pain, malocclusion, numbness of the lower lips, and often obvious deformity

acute rheumatic fever Sx (diagnostic criteria)

Jones Criteria (diagnosis) Major: polyarthritis carditis subcutaneous nodules erythema marginatum syndeham chorea Minor: fever arthralgia elevated acute phase reactants (ESR, CRP) EKG showing heart block (prolonged PR interval)

More common fractures ____is a transverse fracture of diaphyseal region of base of fifth metatarsal. ____is a fracture/dislocation of tarsometatarsal joint. ____is a stress fracture of the metatarsal. ___is fracture of anterolateral aspect of the distal tibia in adolescents before complete closure of the epiphysis.

Jones, Lisfranc, March, Tillaux

What would you find on a CXR with CHF?

Kerley B lines Pulmonary edema Cardiomegaly Pulmonary effusions Venous dilations

What EKG finding is highly suspicious for STEMI?

LBBB

Where is mitral stenosis best heard?

LLD position at APEX with full exhalation. No radiation.

Acronym for tx of CHF exacerbation

LMNOP Lasiks (furosemide) Morphine (vasodilator) Nitrates (vasodilator) Oxygen Position

EKG is ordered for patients suspected of having Lyme disease... why?

Lyme dz can cause carditis, namely a third degree heart block

S4 gallop is pathologic and can indicate what? (3)

LV diastolic dysfunction LV hypertrophy Acute MI

What is surgical tx of EP?

Laparoscopy is standard Laparotomy if patient is unstable or has known adhesions

What are clinical features of Bullous Pemphigoid?

Large TENSE bullae Negative Nikolsky sign Bleeding erosions may occur Axillae, thigh, groin, and abdomen are commonly affected. Mucous membrane lesions are less severe and painful than Pemphigus Vulgaris.

Arrhythmia Considerations

Large and heterogeneous group of conditions in which there is abnormal electrical activity in the heart. Benign vs. Malignant, Asymptomatic vs. Symptomatic (palpitations, dizziness, weakness, chest pain, dyspnea) Fast or slow, Stable or unstable Multifactorial

With knee ligamentous injuries -- which ligament is not usually treated with surgery?

MCL - medial collateral ligament

What would you find in an Echo with CHF?

Low EF (if systolic dysfunction) Hypertrophy Valve abnormality Wall abnormalities

Cysts in postmenopausal women are ______ until proven otherwise

Malignant

What is Long QT syndrome?

May be congenital or acquired. Associated with recurrent syncope, QT interval that is .5-.7 seconds long, ventricular arrhythmias, and sudden death

Clinical features of Gonorrhea?

Men - burning on urination, milky discharge which progresses to yellow, creamy discharge and urethral pain is more pronounced Women - Asymptomatic, dysuria, frequency, urgency, purulent urethral discharge. Vaginitis and cervicitis are common.

Exanthematous Drug Reactions

Mimics measles like viral exanthem-symmetric brightly erythematous macules and papules, discrete and confluent Systemic involvement is low "Morbilliform or maculopapular drug reaction MC type of drug reaction!!

What 3 factors does cardiac output depend on?

Preload Contractility Afterload

What is Abruptio placentae?

Premature separation of a normally implanted placenta after the 20th week but before birth

Scabies

Mite Sarcoptes scabiei Burrow into skin and deposit eggs which mature in 10-14 days Highly contagious because infested humans do not manifest sign or sx for 3-4 weeks SEVERE ITCH, worse at night

______presents with an opening snap in early diastole. Soft, low-pitched, diastolic rumble heard best at the apex in the left decubitus position, palpable right ventricular heave.

Mitral stenosis

What valve is affected with Rheumatic heart disease?

Mitral valve - causes stenosis

More common fractures ____is an ulna shaft fx with proximal radius dislocation. ____is a radial fracture w a distal ulna dislocation. MUGR ____is a isolated ulna fracture. ____is fx of proximal third of fibular associated w rupture of distal tibiofibular syndesmosis.

Monteggia, Galeazzi, Nightstick, Maisonneuve

Clinical features of ACS?

Nontraumatic severe chest pain which is prolonged >30min even at rest Crushing retrosternal pain Heaviness/tightness Unexplained indigestion Epigastric pain Diaphoresis, dyspnea, nausea, vomiting, weakness, anxiety, restlessness, light-headedness, syncope

___is inflammation of the testes often from Coxsackie B or mumps virus. Fever, chills, nausea, lower abdominal pain. Tx: supportive

Orchiitis

What is the MC site of arterial atherosclerotic occlusion in the lower extremities?

Occlusion of the superficial femoral artery (SFA) in Hunter's canal

What is the most common cause of an EP?

Occlusion of the tubes due to adhesions

What are some causes of tachycardia?

Occurs in response to one of three categories of stimuli: 1. Physiologic (pain, exertion, stress) 2. Pharm (sympathomimetics, caffeine, bronchodilators) 3. Pathologic (fever, dehydration, hypoxia, anemia, hyperthyroidism, PE, heart failure)

Incomplete spinal cord lesion

Occurs when motor, sensory, or both functions are present below the level of the lesion

_______refers to inflammation at any point along the optic nerve and presents with acute vision loss, with a particular reduction in color vision. Optic disc appear swollen. Tx: steroids

Optic Neuritis

Golden hour

Optimum limit of time between the moment of injury and definitive care at a hospital

What are lice?

Organism - Pediculosis humanus and Pthirius pubis-1-3mm long flattened lice with 2 pairs of legs and claws

What are the most common ovarian growths?

Ovarian cysts

How do you treat Scarlet fever?

PCN VK or Amoxicillin

What are other causes of DUB?

PCOS Exogenous obesity Adrenal hyperplasia

What PE and EKG changes are seen with dilated cardiomyopathy?

PE: S3, JVD, crackles - possible mitral regurg EKG: nonspecific ST and T wave changes, LBBB

Ventricular arrhythmias

PVCs V tach Toursades Long QT syndrome Brugada syndrome V fib

Clinical features of dysmenorrhea?

Painful cramping in pelvic area/lower abdomen beginning before or at the onset of menses and lasting for 1-3 days PE, labs, imaging is normal for primary dysmenorrhea N/V/D Fatigue

____is defined as hypertension (>140/90) in pregnancy associated with proteinuria and nondependent edema. Eclampsia are seizures in a pt with pre-elampsia. What is given for seizure treatment? What is the first line treatment for blood pressure control?

Preelampsia; Magnesium Sulfate; Hydralazine

What is the most common cause of secondary amenorrhea?

Pregnancy

How do you treat decubitus ulcers?

Prevention! Repositioning Massaging prone areas Frequent monitoring Minimize friction Moist sterile gauze if ulcer develops Surgical debridement

What is Toxic Epidermal Necrolysis (TEN)?

Progression of SJS to full thickness skin detachment. >30% skin loss

Clinical features of chronic venous insufficiency?

Progressive edema followed by skin changes Itching, dull pain with standing, ulceration Shiny, thin skin with pigment and sub-Q changes Stasis ulcer just above the ankle

Long term pharm therapy for CHF?

Vasodilators: ACEI (to counteract compensatory vessel contriction) Diuretics : Lasix, Bumex (to get rid of excess Na) Inotropic agents : Digoxin (to increase contractility of the heart to improve CO) Beta blockers: to improve EF, reduce LV dilation, reduce dysrhythmias

Subacute subdural hematoma

Pts may not have symptoms for days after the injury - CT scan shows ISODENSE lesions

What are these criteria for? At Presentation: Age>55 WBC>16000 Glucose>200 LDH>350 AST>250 At 48 hours: Fall in HCT>10% Increase in BUN>5mg/dL Calcium<8mg/dL Arterial PO2<60mmHg Base deficit>4mg/L Fluid deficit>6L

Ransom criteria for the prediction of mortality in pancreatitis.

How does Cellulitis present?

Red, hot, edematous, shiny, tender plaque with irregular borders. Center may be nodular and there may be purulent discharge.

What is supraventricular tachycardia (SVT)?

Regular, rapid rhythm that arises from impulse reentry or ectopic pacemaker above the bifurcation of the bundle of His. BPM is >160

What are important elements of the history you should ask for a patient presenting with vaginal bleeding?

Reproductive history - menarche, LMP, dysmenorrhea Sexual history - contraception, sexual partners, h/o STIs History of trauma? Retained FB? Meds PMHx - coagulopathies, PCOS, thyroid disease Associated symptoms - GI, urinary, musculoskeletal

Primary Spinal cord injury

Result of mechanical disruption, transection or distraction of neural elements

What are clinical features of cord prolapse?

Ropelike, soft, elongated mass on speculum exam or bimanual. This is an OB emergency!

Cranial Nerves

S - I -olfactory - smell S - II - optic - vision M - III - oculomotor - EOM M - IV - trochlear - EOM B - V - trigeminal - facial sensation M - VI - abducens - EOM B - VII - facial - frown, puff checks S - VIII - vestibulocochlear (acoustic) - hear B - IX - glossopharyngeal - tongue B - X - vagus - say ahh M - XI - spinal accessory - turn head against resistance M - XII - hypoglossal

What is sinus bradycardia?

SA node rate is <60 BPM usually in response to one of three categories of stimuli: 1. Physiologic (vagal tone) 2. Pharm (CCBs, BBs, Dig) 3. Pathologic (AMI, increased ICP, carotid sinus hypersensitivity, hypothyroidism, sick sinus syndrome)

target blood pressure for transfusion in a hemorrhagic patient

SPB>90

What is Chlamydia?

STD caused by a large group of obligate intracellular parasites. MC cause of nongonococcal urethritis. Leading cause of infertility

Clinical features of aortic rupture?

Same as dissection + hypotension/shock

Status Epilepticus

Seizure lasting more than 30 minutes or repetitive symptoms without lucid interval Tx: Benzo's (ativan), Fosphenytoin (faster than phenytoin), Barbiturate vs Propofol (intubated), Consider Toxin/Eclampsia if still refractory, Neuro ICU with EEG

What are diagnostic studies for a spontaneous abortion?

Serial hCG titers, serum progesterone, or serial U/S to confirm a viable pregnancy

What is the criteria for treatment of an EP with MTX?

Serum hCG titer of <5000 Ectopic mass of <3.5cm on ultrasound No hx of blood disorders, PUD, or pulmonary disease Thrombocytes >100,000 Normal renal and hepatic function Hemodynamically stable pt Reliable pt who will follow up

Atlanto-occipital dislocation

Skull displacement from atlas, either anteriorly, superiorly, or posteriorly; vertebral artery may be damaged and dens of axis may be pushed against medulla oblongata (Internal decapitation)

What are some common chemicals that cause irritant CD?

Soaps, cleaners, solvents, detergents

First priority in multiple trauma

Spinal Immobilization (Along with assessment of airway)

_____angina is brought on by activity/exercise. ____angina may show transient ST changes and inverted T waves. ____is pain mainly occurring at rest due to vasospasm of coronary arteries.

Stable, unstable, prinzmetal/variant (responds well to CCB or nitrates, BB may exacerbate vasospasm)

What causes impetigo?

Staph aureus (50-70%) Strep

Osteomyelitis is infection of the bone that may occur from a variety of methods. What are the most common organisms?

Staph aureus, E Coli, Pseudomonas aeruginosa, Salmonella (common in sickle cell)

What is Virchow's triad?

Stasis Injury Hypercoagulable state

What is the tx for labyrinthitis and vestibular neuritis?

Steroids/PT

TEN and SJS typically evolve after exposure to certain drugs or infections - like sulfonamide, anticonvulsants, herpes virus). If less than 10% of body is affected, it is aka ____. If greater than 30% of the body is affected, it is aka ____.

Steven Johnsons Syndrome; Toxic epidermal necrosis

workup on a pt w/ severe HTN to assess end organ damage

need to check out the heart, kidnyes, liver, brain, lungs CMP--specifically BUN/Cr, LFTs UA--specifically looking for hematuria, proteinuria Head CT in suspected brain involvement--altered MS, severe HA, focal deficits, visual disturbances CXR for those w/ CP/back pain/SOB--look for mediastinal widening indicative of dissecting aorta, flast pulmonary edema; follow up w/ CT angiography for suspected aortic dissection

T/F CT scan is very sensitive and specific for liver lacerations

TRUE

T/F Eye lid lacerations involving the lid margin. the nasolacrimal system, and the tarsal plate or levator muscle MUST be repaired by opthalmology

TRUE

T/F Gastrograffin should be used in an esophagogram instead of Barium if trauma is suspected

TRUE

T/F Most bladder ruptures are extaperitoneal

TRUE

T/F Most penile fractures need operative repair usually for the hematomas

TRUE

T/F Nasal fracture is a clinical diagnosis

TRUE

T/F Nasogastric and nasotracheal tubes are contraindicated in Le Fort III fractures

TRUE

T/F Nasogastric tube placement is contraindicated in trauma pts with suspected basilar skull fracture

TRUE

T/F Penetrating trauma pts should get ABX

TRUE

T/F Post menopausal females are at a higher risk for sternal Fx

TRUE

T/F Shaken baby syndrome is a cause of diffuse axonal injury

TRUE

What is abortion?

Termination, whether miscarriage or therapeutic, before 20 weeks of gestation. Spontaneous abortion is miscarriage. Occurs in 15-20% of clinically recognized pregnancies.

How is pericarditis managed?

Treat underlying cause -NSAIDs, may need steroids -Pericardiocentesis if tamponade or large effusion present

How do you treat secondary dysmenorrhea?

Treat underlying condition Remove IUD if indicated Symptomatic tx (NSAIDs, heat, etc) Hysteroscopy, D&C, Laparoscopy (can both diagnose and treat)

Tx of acquired long QT syndrome?

Treatment of electrolyte abnormalities Discontinuation of drugs that prolong the QT

The types of dermatitis are atopic, contact, nummular eczematous, perioral, seborrheic, and stasis.

Treatment, depending on cause, is usually centered on removing allergen and skin hydration.

What are diagnostic studies for preterm labor/delivery?

US - used to determine length of cervix (normal is 4cm) Exam of cervicovaginal secretions for fetal fibronectin (glycoprotein) Vaginal cultures UA

Diagnostic studies for placenta previa?

US is study of choice - often diagnosed before 20 weeks on US Blood type, cross-match, coag studies with large bore IV if patient is unstable

What is the classic presentation of an EP?

Unilateral adenexal pain Amenorrhea or spotting Tenderness or mass on pelvic exam Dizziness/syncope GI distress

What is the cause of endometriosis?

Unknown - may be immunologic, genetic, from retrograde menstruation (most widely accepted cause).

What two acute coronary syndromes are treated the same?

Unstable and NSTEMI

This type of brain herniation leads to downward gaze with absence of vertical eye movements and pinpoint pupils

Upward transtentorial herniation

Hypertension Urgency & Emergency

Urgency: Diastolic BP >120, Systolic >140, No end organ damage Emergency: *End organ Damage*, CNS, CV, Renal, AMI *Increased ICP, HTN, Wide PP, bradycardia,irregular respirations* Pulmonary Edema, Aortic dissection, Eclampsia

NEXUS criteria

Validated decision tools that may be used to determine whether cervical spine radiographs are warranted in the trauma pt. - Very sensitive but not specific

What is the definitive treatment for heart murmurs?

Valve replacement

name two abx effective against MRSA (at least in regards to airway issues); hint: linezolid is not one

Vancomycin, clindamycin

How are vascular causes of claudication differentiated from nonvascular causes or arthritis?

Vascular claudication appears after a specific distance and resolves after a specific time of rest while standing

What are 3 causes of syncope?

Vasodepressor (Vasovagal) Orthostatic (postural) hypotension Cardiogenic

If the patient meets what criteria, a PE is unlikely

age < 50, HR < 100, SpO2 ≥ 95%, no hemoptysis, no estrogens, no prior h/o DVT or PE, no unilateral leg swelling, no surgery or hospitalization in past 4 weeks

Epiglottitis MC b/w ages 2 and 6. What is the treatment of epiglottitis?

airway management w intubation or tracheostomy; oxygen; hydration; antibiotics (Ceftriaxone or Cefotaxime) ; steroids

tell me everything you know about cardiac tamponade

aka pericardial tamponade. caused by a collection of fluid into the pericardial sac, typically blood. often has a hx of trauma particularly a stab wound. pt will probably have SOB but look for Beck's triad--JVD, muffled heart sounds, hypotension (rarely get all three simultaneously so keep a high index of suspicion in the hx of a stab wound). verify w/ an echo, tx w/ pericardiocentesis (i think that's the technical word, aka pericardial evacuation)

name two causes of pancreatitis

alcoholism, gall bladder (alcoholism is more popular, i say this b/c i saw a pt that reported pancreatitis w/o a hx of alcohol consumption and he was perplexed about this but it didn't register with me that gall bladder dz could cause pancreatitis)

Salicylates overdose causes a respiratory ____ and metabolic ____. Treatment of choice is ____along with activated charcoal, whole-bowel irrigation, and urinary alkalinization.

alkalosis; acidosis; hemodialysis; hemodialysis can also be used in methanol overdose

what is a common atypical presentation of pneumonia that may occur at the extremes of age

altered mental status. i also associate this w/ a UTI, therefore it seems safe to say that altered MS can be a presentation of any bacterial infection, not just UTI

this drug is used to control pretty much all tachyarrhythmias

amiodarone

i interviewed a patient in the ED presenting w/ CP. the interview got really long, had a hx of DM, HTN, HLD and was taking "multiple medications." pt reported that he didn't take all of his medications yesterday and I left it at that. where did I go wrong?

ask about specifics regardless of time. need to know which medications he didn't take yesterday--his DM, his HTN, which? specifically which ones of that?

name three common complications of rib fracture

atelectasis, pneumonia, pneuomothorax. a rib fracture itself isn't problematic, but it tends to make patients not want to breath sufficiently deep and they consequently develop these things which can be fatal. also consider to add atelectasis and PNA as compliciations to any patients that isn't breathing deeply

what type of patient may have a physiologic S3

athletes pregnant females healthy young people

acyanotic cardiac defect that results in increased pulmonary blood flow; left-to-right shunts

atrial septal defect

name three atrial tachyarrhythmias

atrial tachycardia, atrial fibrillation, atrial flutter

class of drugs known to cause QT prolongation

atypical antipsychotics

hypertrophic cardiomyopathy etiology

autosomal dominant w/ incomplete penetrance

approach to hemoptysis

first you need to decipher is this is a pulmonary etiology versus gastric etiology. pulmonary is diagnosed generally w/ radiology (start w/ an Xray, may need a CT), gastric requires EGD. Ddx includes BATTLE CAMP B=bronchiectasis or bronchitis A=aspergillous or autoimmune T= TB T=Tumor namely pulmonary L=Lung abscess E=Embolus namely pulmonary, air, placental C=Cystic fibrosis or coagulopathy (are they on blood thinners?) A=AVN M=Mitral valve issue particularly stenosis P=pneumonia

COPD pateints are at risk for developing a pneumothorax. what is the tx for this pneumothorax?

chest tube insertion. this goes to say that it is not more oxygen, it is not supportive care, it is chest tube insertion

Anthrax and Yersinia pestis plague are treated with ___or___.

fluoroquinolones or tetracyclines

steeple sign is pathonomonic for which condition

croup

Contrecoup injury

damage to the brain on the side opposite the point of a blow as a result of the brain's hitting the skull

Hypotonic hyponatremia is divided into three categories ie. hypovolemic, isovolemic, or hypervolemic. Give examples of each type.

hypovolemic -- GI losses, excessive sweating isovolemic -- SIADH hypervolemic -- CHF, cirrhosis, renal failure

three general classifications for cardiomyopathy

dilated, restrictive, hypertrophic

opiates are known to cause itching, consequently a certain drug can be given when Rx'ing opiates... name that drug

diphenhydramine (Benadryl), but need to consider that this will be very sedating as you're giving a sedating antihistamine in conjunction w/ an opiate which is in and of itself sedating. itching does not apply to Fentanyl

be specific when asking ROS questions... today I learned that this applies to "vision changes." don't merely ask about "vision changes" but elaborate on what you mean. what are some better things to inquire about in regards to vision changes

diplopia, blurry vision

pharmacological agent to induce heart stress in a patient that requires a stress test but can't do it via exercise (contraindications such as OA, diabetic foot ulcer)

dobutamine

there are two forms of chemical stress tests... what are they?

dobutamine echo (positive inotrope; positive if decreased cardiac wall movement), dipyridamole-thalium (aka scintigraphy, positive if decreased uptake)

In pericardial tamponade, the ECG may show ____as the heart swings within the accumulated pericardial fluid. What diagnostic tool helps identify pericardial tamponade?

electrical alternans; FAST exam

What is the treatment for tension pneumo?

emergent needle thoracostomy (2nd intercostal space) to convert it to a simple pneumothorax, followed by chest tube insertion (5th intercostal space) for definitive treatment

pulmonary edema is considered an emergency

i say this b/c i thought it was bad news bears, but not so much of an emergency

Patients with anaphylaxis should first have a patent airway. What medications are involved in treatment?

epinephrine; steroids to prevent late-phase reactions, and H1/H2 blockers ie. Benedryl and Ranitidine

Vasopressin and octreotide, vasodilators, are used in severe cases of ________to constrict dilated vessels.

esophageal variceal bleeding

approach to rabies bite

find out what the animal was as not all animals carry rabies; not sure on which do and do not, I think only mammals do minus rabbits. raccoons definitely do. being in a room with a bat is sufficient enough to merit a vaccine and immunoglobulin. rabies vaccine (Rabavert) given at first contact (day zero), 3, 7, 14 days; immunoglobulin also given on first contact but not the remaining three visits for the vaccine doses. ask about ssx (minimally aerophobia, hydrophobia, ascending paralysis). consider that the patient may develop cellulitis, some providers may choose to start abx (I saw utilization of Rocephin and Flagyl)

name three causes of pancreatitis

gall stones, alcohol, medication induced (i say this because I knew that gallstone and alcoholic exist, medication induced is the emphasis here)

a piece of advice about gathering HPI

gather all relevant info including pertinent positives and negatives. Think of worst case scenario ddx when interviewing in rule in/out relevant ssx. Try to confine this all to a one trip visit, so think about the workup that you're going to do and whether or not the patient has a contraindication to anything (i.e. metal for MRI, recent use of blood thinners for tPA). One thing that I kept forgetting to ask is about recent hospital admissions, as this will dictate course of abx use

LDL level is a risk factor for angina... specifically what value? What is it if they also have DM?

general population is less than 100, if they have DM then <70

how is acetaminophen toxicity treated?

give em' the antidote--N acetyl cysteine. if it has been w/in the past 4 hours also give em' some activated charcoal to soak up any tablets that are still floating around in the stomach

airway management is of supreme importance in the management of epiglottitis in kids. given this, what is the utility of an artificial airway

give it to all, not just those w/ a severe obstruction (according to uptodate)

How should fluids be replaced in a patient with DKA?

give several L of NS, switching to ½ NS once hyponatremia corrects, and adding dextrose once serum glucose ↓ to 200 Replace K+ deficits: initial hyperkalemia will rapidly become hypokalemia once insulin is started; may need to supplement K before starting insulin!

utility of xray in patients w/ respiratory symptoms thought to be due to foreign body aspiration

good to confirm the diagnosis but cannot be used to exclude it

Malignant hypertension bp? What is the rule of thumb for lowering?

greater than 220/140; 10% in first hour and 15% for the next 3-12 hours, to normal over next 2 days

What laboratory value is depressed in all hemolytic disorders? What test is absolutely critical to evaluating hemolytic anemia?

haptoglobin (normal plasma protein that binds and clears hemoglobin released into plasma); peripheral blood smear or The Coombs test

number one cause of death in the US

heart attack

general indication for defibrillator

heart failure patients w/ a low EF

Baseline fetal _____ is between 110 and 160 beats/minute.

heart rate

what role does vascularity play when deciding whether or not to Rx abx for a wound?

highly vascular areas (i.e. head) don't need abx as desperately as less vascular areas do; also puncture wounds require abx

what is the rule or relationship regarding DKA and ketones?

if they don't have ketones, it's not DKA (could be HHS). best place to check for ketones is in serum or in urine

antihypertensive tx of choice for eclampsia and preeclampsia

hydralazine target 160/100 second line: labetalol delivery is the only definitive tx for preeclampsia

this should be on the ddx for any CKD patient (that is, patient comes in with some complaints and you notice that they have a hx of CKD... you should be thinking of this condition)

hyperkalemia; manifested as peaked T waves on ECG, can also cause arrhythmias if i recall correctly

name two reversible causes of coma

hypoglycemia, opiate overdose

What will BMP show for DKA?

hyponatremia (intracellular shift), hyperkalemia (shift out of cells)

Patients with adrenal crisis should be treated with IV glucose, saline, and hydrocortisone. What are the laboratory findings in adrenal insufficiency with sodium, potassium, and glucose?

hyponatremia; hyperkalemia; and hypoglycemia

How do patients with tension pneumothorax present?

hypotension, distended neck veins, absent breath sounds over affected hemithorax, tracheal deviation

WHat causes menieres disease?

increased endolymphatic fluid

What is the treatment for septic arthritis?

joint drainage, systemic antibiotics for 3-4 weeks.

mainstay tx for DKA

insulin and fluids; probably don't need to worry about the specifics but may need to add K and glucose to fluids

What three things causing potassium to shift into cells? What are the EKG findings associated with hypokalemia?

insulin; alkalosis; and B-adrenergic agonists EKG: U waves, T-wave flattening or inversion, ST-segment depression

coarctation of the aorta Sx

irritability failure to thrive difficulty feeding headaches leg claudication CHF shock decreased femoral pulses (neonates) upper extremity hypertension nonspecific ejection murmur at apex

saw pt in ED in respiratory distress, ABGs ordered demonstrated metabolic acidosis. I had forgotten what the next step is... what is the utility of the anion gap in the assessment of metabolic acidosis

need to determine if a metabolic acidosis is anion gap positive or normal... increased anion gap (and i can't recall what that is) has a cause found in the mnemonic MUDPILES M-methanol U-uremia (secondary to CKD) D-DKA P-propylene glycol I-infection, iron, INH, inborn errors of metabolism L-lactic acidosis E-ethylene glycol S-salicylates normal is other stuff for which i know of no handy mnemonic so forget about it

yo presents w/ abdominal pain similar to prior episode of kidney stone. suspected kidney stone, what's the workup and tx

labs include CBC w/ diff, CMP, lipase. imaging includes non-contrast abdominopelvic CT (this is giong to be more telling; can do an US in younger patients in whom you want to avoid radiation exposure, order a pregnancy test prior to radiation exposure in pts of child bearing years--not this pt). symptomatic relief w/ opioids (1 mg dilaudid) or NSAIDS (15mg ketorolac for this patient d/t age, otherwise could use a higher dose such as 30). kidney stones also tend to cause nausea, give her some zofran. Generally tx as outpatient w/ pain relief and hydration until stone passes; stone larger than 5mm are less likely to pass and thus consider urology referral, refer if it doesn't pass in 4-6wks.

Pneumonia CXR findings and associated bacteria: 1. Strep Pneumo, H Flu, and Klebsiella have ____consolidations. 2. Staph Aureus, Legionella, and Mycoplasma appear ___. 3. Pneumocystitis jiroveci is common in AIDS patients and appears _____.

lobar; patchy; diffuse interstitial and alveolar infiltrates

approach to sore throat

need to figure out if this is d/t an infectious process thus inquire aobut constitutional ssx and how long this has been going on (more than a week more suggestive of bacterial) need to consider other causes such as trauma, neoplasm be sure to inquire about respiratory issues (stridor is indicative of upper respiratory issue)

Antibiotic treatment for Pertussis is a ____or Bactrim as a alternative.

macrolide - erythromycin, azithromycin, clarithromycin -- avoid erythromycin in infants <1 due to infantile hypertrophic pyloric stenosis

foreign body aspiration tends to be difficult to diagnose. what are the implications of this?

maintain a high index of suspicion for someone presenting w/ a respiratory issue

ABCs of emergency med

make sure every patient has a patent airway, is capable of adequately breathing, has adequate circulation... this can generally be established from the general survey

What is the treatment for mild and more severe bronchiolitis/RSV?

mild - albuterol, racemic epinephrine more severe - add IV hydration, ribavirin, RSV-IG prophylaxis Steroids not beneficial

____ is the most common valvular residual lesions of acute rheumatic carditis

mitral regurgitation

What is the classic arrhythmia associated with COPD?

multifocal atrial tachycardia

most common ADRs of statins

muscle pains, may cause rhabdomyolysis; this may be related to a vitamin D deficiency

while on the ED rotation, I noticed that I was consistently finding tympanny in the upper quadrants of the abdomen... what's the take away from this?

my physical exam skills need some work. the same tympanny I heard on them I hear on me, which tells me that it's not truly tympanic

With croup, protect airway first! What does less severe treatment of croup involve?

nebulized saline, racemic epinephrine, and steroids if patient has stridor at rest.

approach to patient w/ suspected stroke (i.e. facial droop)... be specific, no broad blanket terms b/c you know that you don't know this

need to assess ABCs first.

approach to hemoptysis

need to figure out if this is hemoptysis versus hematemesis. source can generally be determined based on HPI and imaging--xray typically sufficient for hemoptysis, endoscopy for hematemesis mnemonic for etiology: B--bronchitis, bronchiectasis A--Autoimmune, aspergillosis T--TB T--Tumor L--Lung Abscess E--Embolism (has to be a big one) C--Cystic fibrosis A--AVM (arterio venous malformation--I saw one of these on a guy's scrotum, they tend to cause bleeding wherever they occur and they can occur pretty much anywhere) M--Mitral valve P--Pneumonia

low dose IV insulin should be given to patients in DKA or HHS only if serum potassium is above a certain value... what's taht value?

needs to be at least 3.3

Tramadol decreases the seizure threshold... what are the implications of this?

needs to be avoided in seizure pts

what's the relationship between DVT and nephrotic syndrome?

nephrotic syndrome is a hypercoagulable state and therefore patients are more prone to forming a DVT

What is the hallmark s/s of heat stroke?

neurologic dysfunction - frequently associated with cerebral edema

remember to rule out red flag condtiions in patients. this applies to HA. what are the red flag conditions of a pt presenting w/ a HA?

new onset HA in an older patient w/o a hx of HA temporal arteritis neoplasm aneurysm leading to SAH angle closure glaucoma

Macular degeneration can be non-exudative (dry) or exudative (wet). There is no way to prevent which type? What is the treatment for the other type if neovascularization is present?

non-exudative (dry); laser photocoagulation

remember to include CA in the ddx for pain

noted. recall that the vertebrate is the most common site for mets

in evaluating a patient with chest pain, what is the utility in asking about the severity of the pain?

nothing, it's useless according to studies

pt population and timing of Prinzmetal angina

occurs in relatively young patients (<50), especially females, in the morning

name a major potential complication of COPD

pneumothorax. this is the most common complication of COPD. major risk factors for pneumothorax are male sex, smoker, mitral valve prolapse, Marfan's.

although complications of sinusitis are pretty rare, they do occur. what is a major one to consider

orbital cellulitis

Periorbital and orbital cellulitis: reports of visual changes and ocular pain is more indicative of _____. What is the imaging modality of choice? What is the treatment?

orbital cellulitis; orbital CT; 2nd or 3rd generation cephalosporin + vancomycin

pts presenting w/ chest pain suspicious for anginal require an EKG w/in first 10 minutes of presentation. what do you do if that one comes back negative?

order another 10 minutes down the road; up to 40% are negative the first go around despite an MI, therefore need a second

What are signs/symptoms of BV?

pH>4.5 Fishy discharge which is gray or frothy No vulvitis or vaginitis Wet mount will show Clue cells

tx for rib fracture

pain control (opiates). the goal is to get them to breathe sufficiently deep so they don't develop atelectasis, PNA.

when evaluating a patient with chest pain, what is the utility in asking a patient about how long the pain has persisted?

pain that lasted only for a few seconds is seldom ischemic, that which lasts more than 30 minutes suggests either AMI or non-cardiac, that which is in between these two may suggest ischemia

pt presents to the ED w/ acute onset abdominal pain, he's lying still on the bed and looks toxic. what's the first condition that you think of?

peritonitis... these patients look sick and lie still as movement brings about pain

Because ___is capable of causing hypersalivation, AV block, asystole, and seizures, its use is limited to severely ill patients whose diagnosis of anticholinergic intoxication is certain.

physostigmine

an unconscious patient w/ no known trigger may be an overdose patient. what physical exam finding may suggest opioid intoxication and what drug should be given to reverse this?

pinpoint pupils. give em' some narcan

three broad classifications of acute renal failure

pre renal, intrinsic, post renal

32yo female presents w/ what seems to be a kidney stone. you want to do a non-contrast abdominal CT to be sure... what lab must you order prior to this CT?

pregnancy test

Placenta ____ typically have bright red painLESS vaginal bleeding without fetal distress. Placenta ___ present with dark, painFUL, vaginal bleeding; abdominal pain; uterine hypertonicity; and tenderness. Fetal distress often present.

previa; abruption

all about ectopic pregnancy according to Tintinalli (textbook)

problematic d/t the fact that this is a leading cause of maternal death particularly in the first trimester. risk factors are anything that perturb the fallopian tubes (previous ectopic pregnancy, assisted reproduction techniques, PID, prior tubal surgery, IUD use, in utero diethylstillbesterol exposure). classic triad of symptoms includes pain (abdominal/pelvic?), vaginal bleeding or spotting in a woman w/ amenorrhea (this triad is more common in spontaneous or threatened abortion than in ectopic pregnancy). consider ectopic pregnancy in any woman presenting w/ abdominal/pelvic pain in any woman w/ unexplained ssx of hypovolemia (ectopic pregnancy is often diagnosed incidentally via US). may find adnexal tenderness, cervicitis (think gonorrhoea, chlamydia). Diagnosis typically made by US, other options include laparoscopy, surgery. contrary to my previously held belief, Bhcg levels cannot be used to predict an ectopic pregnancy from an intrauterine. the principle goal is to ensure that IUP is present (this was in bold), typically via US. If there is a clinical suspicion, do a transvaginal US regardless of what the Bhcg level is (ectopic can be present even if Bhcg is very low). surgery is required for cases that can only be diagnosed via laparoscopy. medical tx can be performed w/ MTX

how is bronchitis diagnosed

productive cough w/ an unremarkable xray

in any patient presenting w/ chest pain, what is the utility in asking about radiation?

radiation can suggest ACS (albeit not too strongly in and of itself)

tx for foreign body aspiration

remove it. in my readings it has been via bronchoscopy (rigid bronchoscopy or something to that effect) and they have left it at that. this goes to say that I do not think abx are required.

What is the progression of CO2 in an asthmatic or COPD patient?

respiratory alkalosis (loss of CO2) and hypoxia THEN respiratory acidosis which indicates impending respiratory failure

is severe sepsis responsive to fluid or unresponsive fluid?

responsive

Foreign body aspiration: 1. MC location of obstruction? 2. TX: ____for <1 year old; ____for >1 year old. 3. MC causes of obstruction?

right main stem bronchus; back blows for <1 year; Heimlich for >1 year; nuts, peanuts, hot dogs

what is the goal in evaluating chest pain patients

rule in/out red flag conditions (PACEPM... pneumothorax, aortic dissection, cardiac tamponade, esophageal rupture, pulmonary embolism, MI), differentiate cardiac origin from others (particularly GI), differentiate stable angina (CP brought on by exertion relieved by rest) from unstable angina (CP brought on by exertion not relieved by rest)

approach to chest pain

rule out the high acuity stuff (i.e. STEMI) so order an EKG if the EKG shows STEMI, they go straight to the cath lab. if they don't havea STEMI move to biomarkers. if biomarkers (troponin) is positive they have an NSTEMI and go straigtht to cath lab,; if negative they have UA which gets a stress test which if positive requires cath lab

tx for dilated cardiomyopathy

same as that of HF (i think this may even be a type of HF)... ACEI, BB, diuresis

Where is the needle decompression performed for a tension pneumothorax?

second intercostal space, mid-clavicular line

which abx should be used to tx epiglotitis

second or third generation cephalosporins such as cefuroxime, ceftriaxone

anytime I see hypotension, I'm inclined to think of volume depletion... name two other things that I need to consider as they are potentially deadly, one more acutely than the other

sepsis (so look for a temperature of less than 36 or greater than 38, WBC less than 4K or greater than 12K, HR greater than 90), cardiac tamponade (check for Beck's triad---hypotension, muffled heart sounds, JVD; usually accompanied by a hx of trauma)

approach to severe HTN

severe HTN is that which has a SBP of at least 180 or a DBP of at least 110. pts w/ such as blood pressure need to be assessed for end organ damage as this is the feature that distinguishes the scenario from a HTN urgency from emergency

when assessing pain for severity, it is useless to ask about a scale of 1-10 without a reference. what are some good references to use?

severe pain occurs in kidney stones, delivery of a baby, bony fracture (i had kidney stone in mind, i keep forgetting how severe bone pain is so this also holds utility when inquiring about trauma and suspected fracture)

in evaluating a patient for chest pain, it is important to rule in/out ACS. what is the utility in asking about that quality of the chest pain (more specifically, what is the utility in knowing if the chest pain is sharp versus dull)?

sharp pain is less suggestive of ACS, dull pain is more suggestive of ACS

In _____, acute painful episodes due to acute vaso-occlusion from clusters of sickled red cells may occur spontaneously or be provoked. How is the pain crisis treated?

sickle cell crisis; treat the underlying cause (ie. abx if infection suspected; hydrate well - give oxygen if hypoxic)

there is one cardiac arrhythmia which should never be given electricity b/c if you do it could kill them... name that one arrhythmia

sinus tachycardia.... so before you go to shock em', be sure that they don't have a P wave followed by a QRS

Pericarditis is inflammation of the pericardium. How is pain improved? What is the PE finding?

sitting and leaning forward; ST elevation in all precordial leads, normal cardiac enzymes

pericarditis chest pain is typically relieved by positional changes... which position?

sitting up and leaning foward... commit to memory that it is this position in particular and not general positional changes... i say this b/c i interviewed a 36yo F w/ CP in the ED and inquired about positional changes but failed to commit which position in particular

vagal maneuvers have what impact on the heart?

slow it down... the vagal nerve slows down the SA node when activated, hence the utility of vagal maneuvers for slowing down the heart rate

What three things most commonly cause DIC ("death is coming") ie. disseminated intravascular coagulation?

snake bite, pregnancy, and sepsis

staph is gram (what) and what is its shape

staphylococcus aureus is a gram positive cocci in clusters note: staph aureus pneumonia is classically associated with post-flu bacterial pneumonia

tell me everything you know about Prinzmetal angina

typically a female presenting w/ CP w/ no identifiable cause, has been having these episodes, often has a hx of HA (both Prinzmetal and HA can be d/t vasospasm). cardiac markers likely negative, do an ergonavine challenge test, tx w/ CCBs

medical term that means that a condition is essentially psychological (today I saw a pt in the ED w/ an extensive hx of abdominal issues secondary to idiopathic neurological gastroparesis; had numerous visits to the ED, no PCP, poor followup, hx of anxiety and depression. there was no identifiable organic nature to her HPI... what's the diagnosis)

supratentorial (at least this is the term that floats around in the ED, may be the same as somatic conversion?)

The most common intracranial aneurysm is a saccular or berry aneurysm and located where? Treated with surgical clipping

the circle of Willis or its major branches

tx for stable angina patients

they get discharged w/ daily low dose ASA, BB (propranolol) according to Paul Bolin; also get statin of LDL is greater than 100

community acquired pneumonia is treated w/ broad spectrum abx until culture results are back. name the class of med(s)

third generation cephalosporin (ceftriaxone) and a macrolide (azithromycin). alternatively, can go w/ a fluoroquinolone (moxifloxacin)

everything about how to place a chest tube

this goes in around the 4th or 5th intercostal space, midaxillary line; i've only ever seen em' placed in the right side which suggests to me that the right lung is more commonly affected in the case of a pneumothorax. gotta use the sterile technique, so clean the target area, drape, gown up and be sure to wear a face shield b/c this procedure will have airborne blood. inject copious local anesthetic (one doc said that you can't possibly use too much), go pretty deep b/c you want to get to pleural space, make a small incision (~1.5") around the fifth intercostal space going through epidermis, through the subcutaneous fat, down to through fascia then use finger to go the whole way through into the pleural space, dig around in there to clear the way for the tube. place the chest tube directing it towards patient's head until it stops (like to elicit an "ouch"). before placing the tube, you want to place it directly above the rib (the intercostal nerves run below the rib). tie tube down w/ sutures, hook it up to the wound vac, take an Xray (chest tube has radiopaque substance)

a learning scenario: 17mo male w/ hx of asthma presents to ED w/ very labored breathing; tachypnic w/ use of abdominal muscles. Intermittent crying before passing out, oxygen sat 78%. Respiratory therapy was consulted, started racemic epinephrine (no wheezes heard therefore didn't do albuterol b/c that only works on lower airway--always figure out if respiratory issue is upper or lower airway). cbc demonstrated leukocytosis leukocytosis of 34, upper respiratory xray (ordered for suspicion of croup, alternatively epiglottitis) revealed RML PNA. pt was afebrile

this goes to say that WBC level may have some correlation w/ the severity of PNA and may thus impact presentation. the fact that this kid was afebrile raises concern for sepsis (commit to memory that criteria for sepsis is high or low temp). also ordered venous blood gas which revealed metabolic acidosis (low pH, low bicarb) which is kind of bad news bears as it can result in coma and death.

fun fact about DOE

this is a sign of minimally heart failure and occurs w/ left or right

two indications for CABG

three vessel dz, left main artery dz

definitive tx for cardiomyopathy

transplant, but this is unrealistic. point being that this is a terminal condition

approach to chest injury

try to get a feel for the force of impact. need to consider the lungs--pneumothorax, contusion, hemothorax, and the heart--pericardial effusion, and the great vessels--aortic dissection (most common in high speed MVC)

physical exam findings of pneumothorax

unilateral chest pain and dyspnea, decreased tactile fremitus, hyperresonance on percussion, diminished breath sounds

DVT presentation

unilateral leg swelling, red hot and tender leg (not all are red in my experience); confirm w/ US, tx w/ anticoagulation

Spinal cord injuries - the following injuries are _____: Teardrop fracture, bilateral facet dislocation, atlantooccipital dislocation, atlantoaxial dislocation, hangman's fracture, burst fraction, and Jefferson fracture.

unstable or potentially unstable

Pertussis: Catarrhal stage - 1-2 weeks - resembles a ____ Paroxysmal stage - 2-4 weeks - _____ Convalescent stage - 1-2 weeks - ______. Nasal swab culture for Bordetella pertussis is diagnostic.

upper respiratory infection; whooping cough; cough disappears

Patients with unruptured ovarian cysts less than 5cm can be treated how?

usually can be treated w NSAIDs and will involute in 2-3 menstrual cycles

tx for a stable narrow complex tachycardia

vagal maneuver, if that fails do adenosine (6mg, if that fails follow w/ 12mg)

Virchow's Triad for DVT?

venous stasis, endothelial injury, hypercoagulable state

Laryngitis is most commonly due to ____agents although bacterial causes are Strep pyrogenes or M cat. Treatment is supportive with voice rest, warm saline gargles, and increased humidity.

viral

first thing that you need to determine when you are called to a room d/t heart arrhythmia

whether or not the patient is stable (which probably is true for all scenarios); altered MS, hypotension, cardiac symptoms of ischemia (anginal CP) if pt is unstable get a 12 lead ECG and first check if it's a narrow or wide QRS, regular or irregular (most common irregular is afib)

pt complains of blood coming out of mouth (i.e. hemoptysis). what's the first thing that you need to consider

whether this is truly hemptysis i.e. coughing up blood, rather than throwing up blood

Prinzmetal angina may cause ST elevation but negative cardiac enzymes... do these patients get cath'd?

yes. ST elevation is an absolute indication for cardiac cath; may do an ergonovine challenge test during cath

considering that opiate overdose is a common reversible cause of coma, is it safe to give this to all patients presenting w/ a coma

yes. ideally these patients will have miosis, respiratory failure, bradycardia, but these are not always present. it is safe to give an opiate antagonist to all comatose patients, this cannot be said of flumazenil

A 64-year-old woman with a history of depression and hypertension was found down by her husband and brought in by the paramedics. Her husband says that she has recently been depressed and expressed thoughts of suicide. She usually takes fluoxetine for depression and atenolol for hypertension. On arrival, the patient is obtunded, but responds to pain and is maintaining her airway. Her temperature is 98.1°F, BP is 70/40 mm Hg, HR is 42 beats per minute, RR is 12 breaths per minute, and oxygen saturation is 94% on room air. On examination, her pupils are 3 mm and reactive bilaterally. Lungs are clear to auscultation. Heart is bradycardic, but regular, with no murmurs, rubs, or gallops. Extremities have no edema. An ECG shows first-degree AV block at 42 beats per minute, but no ST/T wave changes. Blood sugar is 112 mg/dL. What is the most specific treatment for this patient's ingestion? a. Fluid bolus b. Atropine c. Glucagon d. Epinephrine e. Cardiac pacing

*C* Toxic ingestion must always be considered, especially in suicidal patients. The patient regularly takes atenolol for hypertension and may have overdosed on this occasion. β-Blocker toxicity classically causes bradycardia and hypotension. Antidotes for β-blocker toxicity, such as glucagon, should be given to this patient immediately. Glucagon is thought to work through a separate receptor that is not blocked by β-adrenergic antagonists, ultimately enhancing inotropy and chronotropy. Other medications that may be useful are phosphodiesterase inhibitors, which block cAMP breakdown and maintain intracellular calcium levels. High-dose insulin is a promising experimental treatment for β-blocker toxicity. Ultimately, the patient may require the other treatment options, but glucagon should be the first-line therapy.

An 87-year-old woman with a history of dementia, arthritis, and hypertension presents to the ED for abdominal pain. Her caretaker reports that she is having mid-epigastric pain and had one episode of nonbloody, nonbilious vomiting prior to arrival. The patient is oriented to name only. Temperature is 99.8°F, HR is 110 beats per minute, BP is 80/44 mm Hg, RR is 16 breaths per minute, and oxygen saturation is 96% on room air. On examination, the abdomen is soft, nontender, with no masses, rebound or guarding. Stool is brown and guaiac negative. You place two IV lines and begin fluid resuscitation. You send her blood to the laboratory and order a radiograph of her chest that is shows free air under the diaphragm. Which of the following is the most appropriate next step in management? a. Start IV antibiotics b. Order a CT scan of her abdomen c. Call the surgery service d. Place a central venous line e. Discharge home with Maalox

*C*Abdominal pain in the elderly can be challenging for many reasons, including poor histories and deviation from classic presentations of diseases. However, abdominal pain in patients over 65 must be taken seriously since 25% to 44% require surgical intervention and more than 50% require admission to the hospital. This CXR reveals free air under the right diaphragm, likely from a perforated viscous. This is a surgical emergency and the surgical service should be contacted.

A 25-year-old G3P1011 presents to the ED with a 6-hour history of worsening lower abdominal pain, mostly in the RLQ. She also noticed some vaginal spotting this morning. She is nauseated, but did not vomit. Her last menstrual period was 2 months ago, but her cycles are irregular. She is sexually active and has a history of pelvic inflammatory disease. Her BP is 120/75 mm Hg, HR is 95 beats per minute, temperature is 99.2°F, and RR is 16 breaths per minute. Her abdomen is tender in the RLQ. Pelvic examination reveals right adnexal tenderness. Her WBC count is slightly elevated and her β-hCG is positive. After establishing IV access, which of the following is the most appropriate next step in management? a. Call the OR to prepare for laparoscopy b. Order an emergent CT scan of the abdomen c. Perform a transvaginal ultrasound d. Order a urinalysis e. Swab her cervix and treat for gonorrhea and Chlamydia

*C.* Any woman with abdominal pain, vaginal bleeding, and a positive pregnancy test needs to be ruled out for an ectopic pregnancy. Her vital signs are stable so that she can undergo a transvaginal ultrasound. This is used to document an intrauterine pregnancy and the health of the fetus. If no intrauterine pregnancy is observed, the suspicion for an ectopic pregnancy increases.

A 48-year-old man presents to the ED with ethanol intoxication. His BP is 150/70 mm Hg, HR is 95 beats per minute, temperature is 97.9°F, RR is 14 breaths per minute, and oxygen saturation is 93% on room air. The patient is somnolent and snoring loudly with occasional gasps for air. On examination, the patient's gag reflex is intact, his lungs are clear to auscultation, heart is without murmurs, and abdomen is soft and nontender. He is rousable to stimulation. A head CT is negative for intracranial injury. His ethanol level is 270 mg/dL. Which of the following actions is most appropriate to assist the patient with respirations? a. Nasal airway b. Oral airway c. Bag-valve-mask ventilation d. Laryngeal mask airway e. Tracheoesophageal airway

*A* A nasal airway is made of a pliable material that allows it to be placed into the nostril of a somnolent patient with an intact gag reflex. The nasal airway is an excellent device that can be placed in a patient who may have decreased pharyngeal muscle tone and an obstructing soft palate and tongue. It allows air to bypass such obstructions. The patient in the vignette is intoxicated and appears to have episodes of transient obstruction.

A 55-year-old man is brought to the ED by his family. They state that he has been vomiting large amounts of bright red blood. The patient is an alcoholic with cirrhotic liver disease and a history of portal hypertension and esophageal varices. His vitals on arrival are HR 110 beats per minute, BP 80/55 mm Hg, RR 22 breaths per minute, and yemperature 99°F. The patient appears pale and is in moderate distress. Which of the following is an inappropriate option in the initial management of a hypotensive patient with a history of known esophageal varices presenting with hematemesis? a. Sengstaken-Blakemore tube placement b. Two large-bore IV lines and volume repletion with crystalloid solutions c. Nasogastric (NG) lavage d. IV octreotide e. Gastrointestinal (GI) consult

*A* Acute GI bleeding develops in less than one-third of patients with portal hypertension and varices. With upper GI bleeds, the initial step is assessment of the hemodynamic status. Hypotension with or without tachycardia identifies a high-risk patient with severe acute bleeding. This patient requires immediate treatment. If initial resuscitative efforts fail or if a patient remains hypotensive, more aggressive measures may be required, including consideration of Sengstaken-Blakemore tube placement to physically tamponade the bleeding source, but this is not part of the initial management and has been associated with adverse reactions.

A 35-year-old man presents to the ED complaining of a headache over the previous 4 weeks. He was assaulted with a bat 4 weeks ago and was admitted to the hospital for observation in the setting of a small traumatic subdural hematoma. Repeat noncontrast CT scan of the head 2 weeks ago was normal with resolution of the hematoma. He states he has headaches several times each day. They last from 5 minutes to several hours. They are sometimes band-like, other times they are localized to the site where he was struck. They can be pulsating or constant and are associated with sensitivity to sound. A head CT scan today is normal. Which of the following is the most likely diagnosis? a. Postconcussive syndrome b. Posttraumatic hydrocephalus c. Subdural hygroma d. Cluster headache e. Posttraumatic stress disorder

*A* After head trauma, 30% to 90% of patients complain of headache during their convalescence. Postconcussive headaches are notable for their variability in frequency, location, and associated symptoms. They are often exacerbated by physical activity or changes in position and may be clinically difficult to distinguish from other headache syndromes. In patients with preexisting migraines, increased frequency of their normal migraine syndrome is often noted. Most patients have resolution of their headaches after 4 weeks. In 20% of patients, their postconcussive headache persists for longer than a year. Headache may be one feature of a larger postconcussive syndrome including nervous system instability. This may include fragmentation of sleep, emotional lability, inability to tolerate crowds, restlessness, inability to concentrate, and anxiety.

A 40-year-old woman presents to the ED complaining of fever and 1 day of increasingly severe pain in her RUQ. She denies nausea or vomiting and has no history of fatty food intolerance. The patient returned from a trip to Mexico 6 months ago. About 2 weeks ago she experienced intermittent diarrhea with blood-streaked mucus. Her BP is 130/80 mm Hg, HR is 107 beats per minute, temperature is 102°F, and RR is 17 breaths per minute. Physical examination reveals decreased breath sounds over the right lung base. Abdominal examination shows tenderness to percussion over the RUQ and normal active bowel sounds. There is no Murphy sign. Her WBC is 20,500/μL. Chest radiograph reveals a small right-pleural effusion. Which of the following is the most likely diagnosis? a. Amebic abscess b. Cholecystitis c. Cryptosporidium d. Enterobiasis e. Pyogenic abscess

*A* Amebic abscesses are common in countries with tropical and subtropical climates and areas with poor sanitation. Entamoeba histolytica causes an intestinal infection, and the liver is seeded via the portal system. The clinical presentation includes abdominal tenderness in the RUQ, leukocytosis, and fever. Diagnosis is supported by identifying a pathogenic protozoan in the stool. Management consists of supportive care and administering metronidazole. If medical therapy is unsuccessful, percutaneous catheter drainage is required.

An undomiciled 49-year-old man presents to the ED with altered mental status. His BP is 149/75 mm Hg, HR is 93 beats per minute, temperature is 97.5°F, RR is 18 breaths per minute, and O2 saturation is 99% on room air. Physical examination reveals an unkempt man with the odor of "alcohol" on his breath. His head is atraumatic and pupils are 4 mm, equal, and reactive. The neck is supple. Cardiovascular, pulmonary, and abdominal examinations are unremarkable. There is no extremity edema and his pulses are 2+ and symmetric. Neurologically, he withdraws all four extremities to deep stimuli. ECG is sinus rhythm. Laboratory results reveal: Sodium 141 mEq/L, Arterial blood pH 7.26, Potassium 3.5 mEq/L, Lactate 1.7 mEq/L, Chloride 101 mEq/L, Ethanol level undetectable, Bicarbonate 14mEq/L, Measured serum osmolarity 352 mOsm/L, BUN 15 mg/dL, Calculated serum osmolarity 292 mOsm/kg, Creatinine 0.7 mg/dL, Glucose 89 mg/dL, Urinalysis: no blood, ketones, or protein. Which of the following statements below best describes the laboratory findings? a. Anion gap metabolic acidosis and osmol gap b. Anion gap metabolic acidosis without osmol gap c. Nonanion gap metabolic acidosis and osmol gap d. Nonanion gap metabolic acidosis without osmol gap e. Metabolic alkalosis with secondary acidosis

*A* An anion gap is the difference between unmeasured anions (eg, proteins, organic acids) and unmeasured cations (eg, potassium, calcium, magnesium). The anion gap can be calculated from the formula: Anion gap = [Na+] − [HCO3 + Cl−]

30-year-old man presents to the ED complaining of sudden onset of abdominal bloating and back pain lasting for 2 days. The pain woke him up from sleep 2 nights ago. It radiates from his back to his abdomen and down toward his scrotum. He is in severe pain and is vomiting. His temperature is 101.2°F and HR is 107 beats per minute. A CT scan reveals a 9-mm obstructing stone of the left ureter with hydronephrosis. Urinalysis is positive for 2+ blood, 2+ leukocytes, 2+ nitrites, 40 to 50 WBCs, and many bacteria. You administer pain medicine, antiemetics, and antibiotics. Which of the following is the most appropriate next step in management? a. Admit for IV antibiotics and possible surgical removal of stone. b. Observe in ED for another 6 hours to see if stone passes. c. Discharge with antibiotics and pain medicine. d. Discharge patient with instructions to consume large amounts of water. e. Discharge patient with antibiotics, pain medicine, and instructions to drink large amounts of water and cranberry juice.

*A* An obstructing stone with an overlying infection is an impending urologic emergency. Bacteria in an obstructed collecting system can cause abscess formation, renal destruction, and severe systemic toxicity. The patient requires admission for IV antibiotics and removal and drainage of the stone. In addition to obstruction with infection, other indications for admission include persistent pain, persistent nausea and vomiting, urinary extravasation, and hypercalcemic crisis.

A 37-year-old woman with a history of migraines presents to the ED complaining of crampy lower abdominal pain for 3 days. Workup reveals an intrauterine pregnancy and early prenatal care is arranged with obstetrics as an outpatient. You are concerned because her headaches are controlled with a significant number of medications. She uses medications for both abortive therapy and for prophylaxis. Which of the following classes of medications do you advise she discontinue while pregnant? a. Anticonvulsants b. β-Blockers c. Triptans d. Acetaminophen e. Antiemetics

*A* Anticonvulsant medications are sometimes used as prophylaxis for migraines. Phenytoin, valproic acid, phenobarbital, and topiramate are among the antiepileptics commonly used to prevent migraines. Phenytoin causes fetal hydantoin syndrome, a constellation of birth defects, including growth retardation, cleft palates, hand deformities, and structural cardiac defects. Valproic acid has similar effects with the addition of neural tube defects. Phenobarbital, declining in its use for migraine prevention, causes cardiac defects, facial clefts, and urinary tract abnormalities. Early data on topiramate reveals teratogenicity in animal models.

An 81-year-old woman presents to the ED after tripping over the sidewalk curb and landing on her chin causing a hyperextension of her neck. She was placed in a cervical collar by paramedics. On examination, she has no sensorimotor function of her upper extremities. She cannot wiggle her toes, has 1/5 motor function of her quadriceps, and only patchy lower extremity sensation. Rectal examination reveals decreased rectal tone. Which of the following is the most likely diagnosis? a. Central cord syndrome b. Anterior cord syndrome c. Brown-Séquard syndrome d. Transverse myelitis e. Exacerbation of Parkinson disease

*A* Central cord syndrome is often seen in patients with degenerative arthritis of the cervical vertebrae, whose necks are subjected to forced hyperextension. Typically, it is seen in a forward fall onto the face in an elderly person. This causes the ligamentum flavum to buckle into the spinal cord, resulting in a contusion to the central portion of the cord. This injury affects the central gray matter and the most central portions of the pyramidal and spinothalamic tracts. Patients often have greater neurologic deficits in the upper extremities, compared to the lower extremities, since nerve fibers that innervate distal structures are located in the periphery of the spinal cord. In addition, patients with central cord syndrome usually have decreased rectal sphincter tone and patchy, unpredictable sensory deficits.

A 17-year-old boy is found unconscious in a swimming pool. He is brought into the ED by paramedics already intubated. In the ED, the patient is unresponsive with spontaneous abdominal breathing at a rate of 16 breaths per minute, BP of 80/50 mm Hg, and HR of 49 beats per minute. In addition to hypoxemia, what condition must be considered earliest in the management of this patient? a. Cervical spine injury b. Electrolyte imbalance c. Metabolic acidosis d. Severe atelectasis e. Toxic ingestion

*A* Diving injuries must always be suspected in near-drowning patients. This patient presents with abdominal breathing and spontaneous respirations. This pattern provides an important clue to a cervical spine injury. The diaphragm is innervated by the phrenic nerve, which originates from the spinal cord at the C3-C4 level, whereas the intercostal muscles of the rib cage are supplied by nerves that originate in the thoracic spine. Therefore, abdominal breathing in the absence of thoracic breathing indicates an injury below C4. His bradycardia in the presence of hypotension is suspicious for neurogenic hypotension, which is caused by loss of vasomotor tone and lack of reflex tachycardia from the disruption of autonomic ganglia. However, this is a diagnosis of exclusion and should only be made once all other forms of shock are ruled out. It is important to maintain c-spine immobilization to prevent further progression of an injury.

A 51-year-old man is brought to the emergency department (ED) by emergency medical services (EMS) with a blood pressure (BP) of 90/60 mm Hg, heart rate (HR) of 110 beats per minute, respiratory rate (RR) of 18 breaths per minute, and oxygen saturation of 97% on room air. The patient tells you that he has a history of bleeding ulcers. On examination, his abdomen is tender in the epigastric area. He is guaiac positive, with black stool. He has a bout of hematemesis and you notice that his BP is now 80/50 mm Hg, HR is 114 beats per minute, as he is slowly starting to drift off. Which of the following is the most appropriate next step in therapy? a. Assess airway, establish two large-bore intravenous (IV) lines, cross-match for two units of blood, administer 1 to 2 L of normal saline, and schedule an emergent endoscopy. b, Assess airway, establish two large-bore IVs, cross-match for two units of blood, and administer a proton pump inhibitor. c. Place two large-bore IVs, cross-match for two units of blood, administer 1 to 2 L of normal saline, and schedule an emergent endoscopy. d. Intubate the patient, establish two large-bore IVs, cross-match for two units of blood, administer 1 to 2 L of normal saline, and schedule an emergent endoscopy. e. Intubate the patient, establish two large-bore IVs, cross-match for two units of blood, and administer a proton pump inhibitor.

*A* Emergency medicine always starts with an assessment of the patient's airway. For patients suspected of having a significant GI bleed, two large-bore IV lines need to be established rapidly. Treating an undifferentiated upper GI bleed is like treating a gunshot wound to the abdomen—you should expect the worst. Immediate volume resuscitation should begin with 1 to 2 L of normal saline. If there is no improvement in the BP of a hypotensive patient, then blood should be administered. Sending a cross-match early is advisable since it can take up to an hour to retrieve. Use type O, Rh-negative (if female) or type O, Rh-positive (if male) if type-specific blood is not ready. It is crucial to remember that the initial hematocrit of the patient is a poor indicator of the severity of acute bleeding because it takes 24 to 72 hours to equilibrate.

A 20-year-old man was found on the ground next to his car after it hit a tree on the side of the road. Bystanders state that the man got out of his car after the collision but collapsed within a few minutes. Paramedics subsequently found the man unconscious on the side of the road. In the ED, his BP is 175/90 mm Hg, HR is 65 beats per minute, temperature is 99.2°F, RR is 12 breaths per minute, and oxygen saturation is 97% on room air. Physical examination reveals a right-sided fixed and dilated pupil. A head CT shows convex shaped collection of blood. Which of the following is the most likely diagnosis? a. Epidural hematoma b. Subdural hematoma c. Subarachnoid hemorrhage (SAH) d. Intracerebral hematoma e. Cerebral contusion

*A* Epidural hematomas are the result of blood collecting in the potential space between the skull and the dura mater. Most epidural hematomas result from blunt trauma to the temporal or temporoparietal area with an associated skull fracture and middle meningeal artery disruption. The classic history of an epidural hematoma is a lucent period following immediate loss of consciousness after significant blunt head trauma. However, this clinical pattern occurs in a minority of cases. Most patients either never lose consciousness or never regain consciousness after the injury. On CT scan, epidural hematomas appear lenticular or biconvex (football shaped), typically in the temporal region.

A 67-year-old woman presents to the ED with a painful facial rash that has been worsening over the past 2 days. On physical examination, she has a deeply erythematous, shiny area of warm and tender skin on her left face with a sharply-demarcated and indurated border. There is minimal edema. Vitals are HR 88 beats per minute, BP 125/70 mm Hg, RR 16 breaths per minute, and temperature 101°F. Which of the following is the most appropriate next step in management? a. IV antibiotics and hospital admission b. Oral cephalosporin and outpatient follow-up c. IV acyclovir and Tzanck smear d. Systemic steroids and laboratory testing for rheumatoid factor e. Dermatology consult and biopsy of the rash

*A* Erysipelas is an acute superficial cellulitis of the dermis, lymphatics, and subcutaneous tissue. It ischaracterized by a sharply demarcated border surrounding skin that is raised, deeply erythematous, indurated, and painful, and is associated with nephrotic syndrome, postoperative wounds, and small breaks in the skin. Erysipelas is more superficial than cellulitis and is more likely to occur in the young and in the elderly, but the distinction between the two is often subtle and therapeutically irrelevant. Treatment of erysipelas and facial cellulitis requires hospital admission and parenteral antibiotics. Treatment is aimed at the predominant organism, group A Streptococcus, but Staphylococcus and other Streptococcus species are also found. An immediate ophthalmologic consult should be obtained if there is any orbital or periorbital involvement.

A 43-year-old man presents to the ED complaining of right hand pain. The individual states that 3 days ago he accidentally punctured his index finger with a thumb tac. Subsequently, the finger became red and began to swell, and he had pain with movement of the finger. His tetanus status is up to date. You suspect the patient's diagnosis is tenosynovitis. Which of the following statements regarding Kanavel four cardinal signs of flexor tenosynovitis is correct? a. The infected finger is held in slight flexion at rest b. Asymmetric swelling of the finger c. Tenderness along the extensor tendon sheath d. Painless with passive range of motion e. Purplish hue at the distal tip

*A* Flexor tenosynovitis typically results from a puncture wound. The causative agents are usually Staphylococcus aureus or Streptococcus. The diagnosis is based on the presence of Kanavel four cardinal signs of flexor tendon synovitis. These include the finger held in slight flexion at rest, (b) symmetric swelling or sausage digit of the finger, (c) tenderness along the flexor tendon sheath, and (d) pain with passive range of motion of the finger. Answer choice (e) is not one of Kanavel signs.

A 19-year-old man is brought to the ED by EMS after he was found lying on the floor at a dance club. EMS states that the patient seemed unconscious at the dance club but as soon as they transferred him onto the gurney he became combative. Upon arrival in the ED, his BP is 120/65 mm Hg, HR is 75 beats per minute, temperature is 98.9°F, RR is 12 breaths per minute, and oxygen saturation is 98% on room air. On physical examination, his pupils are midsized, equal, and reactive to light. His skin is warm and dry. Lung, cardiac, and abdominal examination are unremarkable. As you walk away from the bedside, you hear the monitor alarm signaling zero respirations and the oxygen saturation starts to drop. You perform a sternal rub and the patient sits up in bed and starts yelling at you. As you leave him for the second time, you hear the monitor alarm again signal zero respirations. You administer naloxone, but there is no change in his condition. Which of the following is most likely the substance ingested by this patient? a. γ-Hydroxybutyrate (GHB) b. Diazepam c. Cocaine d. Phencyclidine (PCP) e. Heroin

*A* GHB is a natural neurotransmitter that induces sleep. GHB has been sold as a muscle builder (release of growth hormone), a diet aid, and a sleep aid. Patients with GHB overdose generally have a decreased level of consciousness. In contrast to other sedative/ hypnotic overdoses, the level of consciousness tends to fluctuate quickly between agitation and depression. A distinctive feature of GHB intoxication is respiratory depression with apnea, interrupted by periods of agitation and combativeness, especially following attempts at intubation.

A 33-year-old man, who was drinking heavily at a bar, presents to the ED after getting into a fight. A bystander tells paramedics that the patient was punched and kicked multiple times and sustained multiple blows to his head with a stool. In the ED, his BP is 150/75 mm Hg, HR is 90 beats per minute, RR is 13 breaths per minute, and oxygen saturation is 100% on non-rebreather. On examination, he opens his eyes to pain and his pupils are equal and reactive. There is a laceration on the right side of his head. He withdraws his arm to pain but otherwise doesn't move. You ask him questions, but he just moans. Which of the following is the most appropriate next step in management? a. Prepare for intubation b. Suture repair of head laceration c. Administer mannitol d. Bilateral burr holes e. Neurosurgical intervention

*A* Head injury severity is assessed on the mechanism of injury and on the initial neurologic examination. Although the GCS is currently used in multiple settings, it was initially developed for the clinical evaluation of hemodynamically stable, adequately oxygenated trauma patients with isolated head trauma. A score of 14 to 15 is associated with minor head injury, 9 to 13 indicates moderate, and 8 or less is associated with severe head injury. His GCS score is 8 (2 points for eye opening to pain, 2 points for mumbling speech, 4 points for withdrawing from pain). He is classified with a severe head injury. The overall mortality of severe head injury is almost 40%. It is recommended to intubate patients with a GCS score of 8 or less for airway protection. These patients are at risk for increased ICP and herniation, which can lead to rapid respiratory decline. All patients with severe traumatic brain injury require an emergent CT scan and should be admitted to the intensive care unit in a hospital with neurosurgical capabilities.

A 68-year-old man presents to the ED complaining of a daily headache for almost a month. He describes the headache as being dull, difficult to localize, most intense in the morning, and abating in the early afternoon. He also noticed progressive weakness of his right upper and lower extremity. Which of the following headache syndromes are the signs and symptoms most consistent with? a. Headache caused by a mass lesion b. Cluster headache c. Tension-type headache d. Headache from intracranial hypertension e. Waking or morning migraine

*A* Headaches caused by a mass lesion are classically described as worse in the morning, associated with nausea and vomiting, and worse with position. Rarely do patients present with focal neurologic symptoms. When they do, imaging is a necessary adjunct prior to leaving the ED. If a mass lesion is part of the differential diagnosis, LP should be deferred until neuroimaging has been performed because of the risk of herniation.

A 29-year-old woman presents to the ED complaining of double vision for 3 days. She states that she has been feeling very tired lately, particularly at the end of the day, when even her eyelids feel heavy. She feels better in the morning and after lunch when she is able to rest for an hour. Her BP is 132/75 mm Hg, HR is 70 beats per minute, temperature is 98.4°F, and RR is 12 breaths per minute. On examination you find ptosis and proximal muscle weakness. What is the most appropriate diagnostic test to perform? a. Edrophonium test b. Serologic testing for antibodies to acetylcholine receptors c. Head computed tomography (CT) scan d. Electrolyte panel e. Lumbar puncture

*A* High clinical suspicion in this case is for myasthenia gravis, an autoimmune condition in which acetylcholine receptor antibodies block acetylcholine binding and prevent normal neuromuscular conduction. The disease typically affects young women and older men and presents with generalized weakness worsening with repetitive muscle use that is usually relieved with rest. Ptosis and diplopia are usually present. The edrophonium test is used to help diagnose myasthenia gravis. It involves administering edrophonium, a short-acting anticholinesterase, which prevents acetylcholine breakdown. With the increased acetylcholine levels at the neuromuscular junction, the patient experiences a subjective and objective improvement of symptoms by preventing rapid breakdown of acetylcholine at the myoneural junction

You receive notification from EMS that they are bringing in a 17-year old male who was found unconscious by a police officer. The police officer at the scene states that he snuck up on a group of kids that he thought were using drugs. Two of them got away and one just fell to the ground seconds after standing up. Lying on the ground next to the adolescent were plastic bags. The emergency medical technician states that the patient was in ventricular fibrillation. He was shocked in the field and is now in a sinus rhythm. The EMT also administered IV dextrose, thiamine, and naloxone without any change in mental status. Which of the following substances was the patient most likely abusing? a. Butane b. Ethanol c. Heroin d. Cocaine e. PCP

*A* Hydrocarbons (HC) are a diverse group of organic compounds that contain hydrogen and carbon. Some common products containing HCs are household polishes, glues, paint remover, and industrial solvents. Acute HC toxicity usually affects three main target organs: the lungs, CNS, and heart. The lungs are most commonly affected by aspiration of ingested HCs. Pulmonary toxicity is associated with cough, crackles, bronchospasm, pulmonary edema, and pneumonitis on chest radiograph. Certain HCs (eg, toluene, benzene, gasoline, butane, chlorinated HCs) can have sedative/opioid-like effect and cause euphoria, disinhibition, confusion, and obtundation. HCs can also cause sudden cardiac death, particularly after sudden physical activity after intentional inhalation. It is thought that the HCs produce myocardial sensitization of endogenous and exogenous catecholamines, which precipitates ventricular dysrhythmias and myocardial dysfunction. One scenario is the solvent-abusing person. EMS workers often describe an individual who has used inhaled solvents, performed some type of physical activity, and then suddenly collapsed. In the scenario above, the patient inhaling butane was approached by a police officer and tried to run away. This sudden exertion most likely led to a cardiac dysrhythmia. Paraphernalia is often found at the scene, including plastic bags used for "bagging" (pouring HCs in a bag, then deeply inhaling) or a HC-soaked cloth used for "huffing" (inhaling through a saturated cloth). Other paraphernalia include gasoline containers, multiple butane lighters, and spray paint cans.

A 20-year-old man presents to the ED with fever and severe right lower quadrant (RLQ) pain for 1 day. Prior to this episode, he reports 2 months of crampy abdominal pain, generalized malaise, a 10-lb weight loss, and occasional bloody diarrhea. On examination, his HR is 115 beats per minute, BP is 125/70 mm Hg, RR is 18 breaths per minute, and temperature is 100.8°F. His only significant past medical history is recurrent perirectal abscesses. On physical examination, the patient appears uncomfortable and has a tender mass in the RLQ, without guarding or rebound. Rectal examination is positive for trace heme-positive stool. An abdominal computed tomography (CT) scan reveals no peri-appendiceal fat stranding. There is inflammation of the distal ileum and several areas of the colon. There are no rectal inflammatory changes. Which of the following is the most likely diagnosis? a. Crohn disease (CD) b. Ulcerative colitis (UC) c. Appendicitis d. Pseudomembranous enterocolitis e. Diverticulitis

*A* IBD is a chronic inflammatory disease of the GI tract. There are two major types: CD and UC. CD can involve any part of the GI tract, from mouth to anus, and is characterized by segmental involvement. The distal ileum is involved in the majority of cases; therefore, acute presentations can mimic appendicitis. CD spares the rectum in 50% of cases. There is a bimodal age distribution, with the first peak occurring in patients 15 to 22 years of age, and a second in patients 55 to 60 years of age. Definitive diagnosis is by upper GI series, air-contrast barium enema, and colonoscopy. Segmental involvement of the colon with rectal sparing is the most characteristic feature. Other findings on colonoscopy include involvement of all bowel wall layers, skip lesions (ie, interspersed normal and diseased bowel), aphthous ulcers, and cobblestone appearance from submucosal thickening interspersed with mucosal ulceration. Extraintestinal manifestations are seen in 25% to 30% of patients with CD.

A 55-year-old woman presents to the ED complaining of 1 day of a left-sided headache that is associated with scalp and ear pain. She describes the pain as gradual in onset, dull, and constant. She describes a week of constitutional symptoms prior to the onset of her headache syndrome including joint pain, tenderness of the muscles of her lower extremities, and fatigue. She is afebrile with no nuchal symptoms, photophobia, or phonophobia. Physical examination reveals a tender scalp and a thickened, painful temporal artery. Which of the following is the most appropriate next step in management? a. Initiate corticosteroid therapy b. LP to rule out subarachnoid hemorrhage (SAH) c. Injection of lidocaine at the base of the occiput d. Initiate antibiotic therapy e. Send an ESR

*A* In a case of suspected TA, initiation of corticosteroid therapy is indicated emergently to prevent irreversible complications. Loss of vision is known to occur and prompt initiation of corticosteroid therapy decreases this possibility. TA, also referred to as giant cell arteritis, is a granulomatous inflammation of the proximal great vessels and its carotid bifurcations. It has an overlapping clinical syndrome with polymyalgia rheumatica.

A 31-year-old man is riding his mountain bicycle down a steep hill when he hits a rock in the path and is thrown off the bicycle. In the ED, the individual complains only of left arm pain. Your primary survey is unremarkable. After administering medication for pain control, you send the patient for a radiograph of his arm. Which of the following injuries is consistent with this patient's radiograph showinga proximal fracture of the ulnar shaft and a radial head dislocation? a. Monteggia fracture b. Galeazzi fracture c. Nightstick fracture d. Colles fracture e. Smith fracture

*A* Monteggia fractures are of the proximal one-third of the ulnar shaft combined with a radial head dislocation. This injury commonly occurs from either a direct blow to the posterior aspect of the ulna or a fall on the outstretched hand with the forearm in forced pronation. This fracture is associated with an injury to the radial nerve. It is important to always look for an associated fracture or dislocation when one is noted in a forearm bone. A Galeazzi fracture (b) is a fracture of the distal radial shaft associated with a distal radioulnar dislocation at the distal radioulnar joint (DRUJ). This fracture is often confused with Monteggia fracture. *A way to remember the difference is to recall that Monteggia ends in an "a" and in this fracture, the ulna (also ends in "a") is fractured.*

A 37-year-old woman is brought into the ED by her friend who states that the patient swallowed approximately 50 capsules of 325-mg acetaminophen 6 hours ago in an attempted suicide. The patient states she feels nauseated and vomits while you take her history. Her BP is 100/75 mm Hg, HR is 97 beats per minute, temperature is 98.9°F, RR is 18 breaths per minute, and oxygen saturation is 99% on room air. Examination is unremarkable except for mild epigastric tenderness. Which of the following is the correct antidote for APAP overdose? a. NAC b. Physostigmine c. Flumazenil d. Naloxone e. Digibind

*A* NAC is the cornerstone of therapy for the potentially lethal acetaminophen overdose. NAC acts as a glutathione precursor to reduce NAPQI (N-acetyl-p-benzoquinoneimine), the toxic metabolite of acetaminophen. It can be administered orally or intravenously. NAC is most effective if administered within 8 hours of the ingestion; however, it may still be of benefit if given more than 24 hours after an acute acetaminophen overdose.

A 20-year-old man presents with several weeks of painful rectal bleeding. He denies fever, nausea, or vomiting. He is sexually active with women only and usually uses condoms. He denies any history of CD, UC, or malignancy. He states that the pain is most severe during and immediately after defecating. Bleeding is bright red and only enough to stain the toilet paper. Which of the following is the most common etiology of painful rectal bleeding? a. External hemorrhoid b. Anal fissure c. Anorectal tumor d. Internal hemorrhoid e. Venereal proctitis

*A* Pain and bleeding are common complaints associated with anorectal disorders. A good history and a thorough physical examination, including a digital rectal examination and anoscopy should be performed whenever feasible. Anal fissures (ie, fissures in ano) result from linear tears of the anal canal at or just inferior to the dentate line and extend along the anal canal to the anal verge. This area has a rich supply of somatic sensory never fibers. Consequently, anal fissures are exquisitely painful and represent the most common cause of painful rectal bleeding in the first year of life and in adults. They are usually produced by the passage of a large, hard stool but may also occur with severe diarrhea.

Paramedics bring a 44-year-old man to the ED. He was found in the middle of the street after being struck by a car. His systolic BP is 70 mm Hg; a diastolic BP cannot be obtained. The heart rate is 125 beats per minute, and oxygen saturation is 89% on room air. The patient's eyes are closed. You ask the patient his name and he doesn't respond. There is no response when you ask him to move his limbs. You notice that his left foot is severely deformed and there is a large laceration to his right arm. Which of the followingis the most appropriate next step in management? a. Prepare for emergent orotracheal intubation. b. Begin aggressive fluid resuscitation and administer morphine for pain. c. Apply a tourniquet just above his left foot and begin fluid resuscitation. d. Apply pressure to the laceration, splint the left foot, and order a radiograph. e. Administer packed RBCs and bring him to the CT scanner for a pan-scan.

*A* Patients often present to the ED with life-threatening conditions that require rapid and simultaneous evaluation and treatment. The fundamentals of emergency medicine begin with the ABCs. Airway assessment and management have priority over all other aspects of resuscitation in the critically ill or injured patient.

A 47-year-old man with a history of hypertension presents to the ED complaining of continuous left-sided chest pain that began while snorting cocaine 1 hour ago. The patient states he never experienced chest pain in the past when using cocaine. His BP is 170/90 mm Hg, HR is 101 beats per minute, RR is 18 breaths per minute, and oxygen saturation is 98% on room air. The patient states that the only medication he takes is alprazolam to "calm his nerves." Which of the following medications is contraindicated in this patient? a. Metoprolol b. Diltiazem c. Aspirin d. Lorazepam e. Nitroglycerin

*A* Patients with chest pain in the setting of cocaine use should be evaluated for possible myocardial ischemia. Patients suspected of ACS should be managed accordingly with oxygen, nitrates, morphine, aspirin, and benzodiazepines; however, β-adrenergic antagonist therapy is absolutely contraindicated. If β-adrenergic receptors are antagonized, α-adrenergic receptors are left unopposed and available for increased stimulation by cocaine. This may worsen into coronary and peripheral vasoconstriction, hypertension, and possibly ischemia. Therefore, benzodiazepines, which decrease central sympathetic outflow, are the cornerstone in treatment to relieve cocaine-related chest pain.

A 55-year-old woman presents to the ED stating that her nose has been bleeding profusely for the last 3 hours. After 25 minutes of bilateral pressure on her nasal septum, there is still profuse bleeding. You place anterior nasal packing bilaterally but bleeding still persists. The patient is starting to get anxious. Her BP is 110/70 mm Hg, HR is 80 beats per minute, RR is 18 breaths per minute, and oxygen saturation is 98%. Laboratory results reveal a white blood cell (WBC) count of 9000, hematocrit (HCT) 34%, platelets of 225,000, and international normalized ratio (INR) 1.1. Under direct visualization, you note the bleeding originating from the posterior aspect of her septum. Which of the following is the most appropriate management? a. Place posterior nasal packing, start antibiotics, and admit the patient to a monitored hospital bed. b. Place the patient supine and wait for spontaneous resolution of the bleeding. c. Keep pressure on her nasal septum and administer fresh frozen plasma and platelets. d. Place posterior nasal packing, and discharge the patient home with follow-up in 24 hours. e. Apply silver nitrate to the nasal mucosa until the bleeding stops.

*A* Posterior epistaxis is identified when posterior bleeding occurs with a properly placed anterior nasal packing. Posterior packing is mandated using either a commercially available balloon or a standard Foley catheter inserted into the posterior nares and inflated with water. Patients with posterior nasal packs should be admitted to a monitored bed. In addition to cardiac dysrhythmias, myocardial infarctions, cerebrovascular accidents, and aspiration have been reported in these patients. Antibiotics are often started to prevent sinusitis and toxic shock syndrome from obstruction of the nasal packing.

A 52-year-old woman is brought to the ED by her husband for altered mental status for 1 day. The patient has hypertension and diabetes but has not been taking her medications for the last 5 days since she lost her insurance and could not afford her prescriptions. Her BP is 168/91 mm Hg, HR is 125 beats per minute, temperature is 99.8°F, and RR is 18 breaths per minute. Her fingerstick glucose is 900 mg/dL. There is glucose in her urine, but no ketones. Which of the following is the most appropriate next step in management? a. Administer IV fluids and insulin b. Obtain head CT scan c. Obtain ECG d. Obtain chest radiograph and urine culture e. Administer broad coverage antibiotics

*A* Profound hyperglycemia, absence of ketonuria, and diabetes medication noncompliance should raise your suspicion for nonketotic hyperosmolar crisis (NKHC) in this patient. This condition is a syndrome of hyperglycemia without ketoacidosis as small amounts of insulin protect against adipose tissue metabolism. This syndrome is more common in type 2 diabetics. Causes of NKHC are similar to those of DKA and include diabetes medication noncompliance, infection, stroke, and myocardial infarction (MI). Patients are profoundly dehydrated because of osmotic diuresis. The mainstay of NKHC therapy consists of replacing fluid losses. Electrolyte deficiencies should be replaced and insulin administered. Fluid deficit in NKHC is significant and needs to be slowly corrected, as rapid correction may lead to cerebral edema. Insulin requirements in NKHC are usually less than in DKA.

A 64-year-old man presents to the ED complaining of knee pain since yesterday. He denies trauma or similar presentation in the past. On examination, you note an erythematous, tender, and swollen knee. Radiographs do not reveal a fracture, but do show calcium deposits. Arthrocentesis demonstrates 20,000/μL white blood cells (WBCs) with a predominance of neutrophils, a negative Gram stain, and rhomboid shaped crystals that are positively birefringent under polarized light. Which of the following is the most likely diagnosis? a. Pseudogout b. Gout c. Septic joint d. Rheumatoid arthritis e. Osteoarthritis

*A* Pseudogout is the most common cause of acute monoarticular arthritis in the elderly. It affects woman and men equally, primarily after their sixth decade of life. It is caused by the deposition of calcium pyrophosphate crystals. The knee is the most commonly involved joint, followed by the wrist and ankle. The synovial fluid reveals rhomboid-shaped crystals that are weakly positive birefringent under polarized light. Treatment is generally supportive with NSAIDs.

A 24-year-old competitive ice-skater is practicing for the Olympic trials when she falls during one of her jumps and lands on her outstretched hand. She is brought to the ED complaining of wrist pain. On examination, you note tenderness at the anatomic snuffbox and pain with axial loading of the thumb. You suspect she has a scaphoid fracture. In what part of a fractured scaphoid is the incidence of avascular necrosis highest? a. Proximal scaphoid. b. Waist (middle third) of the scaphoid. c. Distal scaphoid. d. Tubercle of the scaphoid. e. The scaphoid is not at risk for avascular necrosis.

*A* The blood supply to the scaphoid normally penetrates the cortex at the distal aspect of the bone. Therefore, there is no direct blood supply to the proximal portion of the bone, which predisposes this fragment to avascular necrosis and delayed union. The more proximal the fracture is in scaphoid injuries, the greater the likelihood of developing vascular necrosis.

A 22-year-old soccer player presents to the ED complaining of right knee pain and swelling. He states that earlier in the day he was in a soccer match and was running for the ball, but stopped abruptly and tried to run in a new direction. Immediately thereafter, he felt intense pain in his knee with instant swelling. Which of the following is the most commonly injured major ligament of the knee? a. Anterior cruciate ligament (ACL) b. Posterior cruciate ligament (PCL) c. Medial collateral ligament (MCL) d. Lateral collateral ligament (LCL) e. Patella ligament

*A* The cruciate ligaments are two internal bands extending from the tibia to the femur, one anteriorly and the other posteriorly. They control anteroposterior and rotary stability of the knee and prevent hyperextension. The ACL is the most frequently injured ligament in the knee. It has a rich blood supply which accounts for the high incidence of hemarthrosis when the ligament is injured. A history that includes a pop or snap at the time of injury (eg, during a sudden turn in direction while playing sports) suggests a rupture of the ACL until proven otherwise, particularly when associated with the rapid development of a knee effusion.

A 25-year-old man is brought to the ED by emergency medical service (EMS) accompanied by his girlfriend who reports that the patient had a seizure 30 minutes ago and is still confused. The girlfriend reports that the patient is a known epileptic who has been doing well on his latest medication regimen. The exact seizure medications are unknown. On arrival to the ED, the patient develops continuous clonic movements of his upper and lower extremities. The patient's vital signs are BP of 162/85 mm Hg, HR of 110 beats per minute, and pulse oximetry of 91% on room air. Capillary glucose level is 95 mg/dL. Which of the following is the most appropriate next step in management? a. Place the patient in a lateral decubitus position. b. Administer lorazepam. c. Administer phenytoin. d. Perform rapid sequence intubation on the patient. e. Look up the patient's medical records and administer his current antiepileptic regimen.

*A* The initial approach to a seizing patient should involve protecting the patient from injury. Seizing patients should be immediately placed in a lateral decubitus position to prevent aspiration of gastric contents. Other initial measures are oxygen administration, pulse oxymetry, glucose level determination, and an IV line.

A 43-year-old warehouse worker is helping to stock large refrigerator boxes onto the forklift. One of the boxes starts to fall off the shelf and he tries to catch it before it falls. In doing so, his hand is hyperextended at the wrist and causes immediate pain. In the ED, you note limitation of the normal motion of the wrist with palpable fullness on the volar aspect. The patient also complains of tingling in the distribution of the median nerve. Which of the following injuries is most frequently associated with the development of acute carpal tunnel syndrome? a. Lunate dislocation b. Perilunate dislocation c. Scapholunate dislocation d. Capitate dislocation e. Scaphoid fracture

*A* The median nerve runs through the carpal tunnel between the flexor carpi radialis and the palmaris longus. It provides sensation to the palmar aspect of the radial three and one-half fingers as well as the dorsal aspect of the tips of the index and middle fingers and the radial half of the ring finger. Lunate dislocations are usually caused by hyperextension injuries. The lunate is usually displaced volarly or dorsally. The median nerve may be compressed in the carpal tunnel by the lunate, and the patient may display signs of acute carpal tunnel syndrome.

A 67-year-old woman with a history of hypertension and congestive heart failure presents with "burning" epigastric pain that began 2 hours after eating a meal. She states that she has had similar pain over the past several weeks, and has been taking antacids and a medication that her primary-care physician had prescribed with moderate relief. The pain has occurred with increasing frequency and now awakens her from sleep. She states she came to the ED today because the pain was not relieved with her usual medications. She denies nausea, vomiting, diarrhea, or fever. She also denies hematemesis, black stool, or bright red blood per rectum. On physical examination, she is tender at the epigastrium, with an otherwise normal abdominal, pulmonary, and heart examination. Stool guaiac tests positive for occult blood. Which of the following is the most common serious complication of peptic ulcer disease? a. GI hemorrhage b. GI perforation c. GI penetration d. Gastric outlet obstruction e. Pernicious anemia

*A* The most serious complications of PUD include hemorrhage, perforation, penetration, and gastric outlet obstruction. Hemorrhage, which occurs in 15 of patients, is the most common complication.

A 32-year-old dental hygienist presents to the ED with a painful lesion at the distal aspect of her right index finger. The individual states that she had a low-grade fever and malaise over the last week and subsequently developed pain and burning of the infected digit. Within the next week, she noted erythema, edema, and the development of small grouped vesicles on an erythematous base as depicted in the image below. Which of the following should be avoided when managing this condition? a. Performing an incision and drainage to facilitate healing and avoid bacterial superinfection. b. Splinting the finger and recommending elevation and analgesics. c. Prescribing an antiviral agent, such as acyclovir. d. Applying a dry dressing over the lesions to prevent transmission. e. Prescribing an antibiotic if there is evidence of bacterial superinfection.

*A* The patient has a herpetic whitlow, a viral infection of the distal finger. It is caused by the herpes simplex virus type I or II. This condition typically occurs in health-care providers with exposure to oral secretions, and in patients with coexistent herpes infections. It generally presents with a prodrome period of fever and malaise. Subsequently, there is localized burning, itching and pain that precede the development of the classic clear herpetic vesicles. Typically, only one finger is involved. The diagnosis is usually made clinically, but if doubt remains, or if the presentation is atypical, it can be confirmed with a Tzanck smear or viral culture. When managing this condition, it is important to note that surgical drainage is contraindicated. It can result in secondary infection and delayed healing.

A 61-year-old woman with metastatic breast cancer presents to the ED with chest pain, cough, and shortness of breath. She states these symptoms began 1 week ago and progressively worsened. She denies fever or chills. On examination, you notice jugular venous distension. Her BP is 105/70 mm Hg and HR is 98 beats per minute. A chest radiograph is shown below. Which of the following ECG finding is associated with this presentation? a. Low-voltage complexes b. High-voltage complexes c. ST-segment depression d. ST-segment elevation e. T-wave inversion

*A* The patient presents with a pericardial effusion probably secondary to her metastatic breast cancer. Pericardial effusion is often asymptomatic but with accumulating fluid can cause chest pain, shortness of breath, cough, and fever. Ultimately, it can lead to cardiac tamponade, which develops in up to 10% of all cancer patients. The ECG classically shows low-voltage complexes and, rarely, electricaln alternans. Treatment of nontraumatic pericardial effusion and tamponade is pericardiocentesis to remove the fluid.

A 29-year-old tall, thin man presents to the ED after feeling short of breath for 2 days. In the ED, he is in no acute distress. His BP is 115/70 mm Hg, HR is 81 beats per minute, RR is 16 breaths per minute, and oxygen saturation is 98% on room air. Cardiac, lung, and abdominal examinations are normal. An ECG reveals sinus rhythm at a rate of 79. A chest radiograph shows a small right-sided (less than 10% of the hemithorax) spontaneous pneumothorax. A repeat chest x-ray 6 hours later reveals a decreased pneumothorax. Which of the following is the most appropriate next step in management? a. Discharge the patient with follow-up in 24 hours b. Perform needle decompression in the second intercostal space, midclavicular line c. Insert a 20F chest tube into right hemithorax d. Observe for another 6 hours e. Admit for pleurodesis

*A* The patient presents with a primary spontaneous pneumothorax (PTX), which occurs in individuals without clinically apparent lung disease. In contrast, secondary spontaneous pneumothorax occurs in individuals with underlying lung disease, especially chronic obstructive pulmonary disease (COPD). For otherwise healthy, young patients with a small primary spontaneous PTX (less than 20% of the hemithorax), observation alone may be appropriate. The intrinsic reabsorption rate is approximately 1% to 2% a day, and accelerated with the administration of 100% oxygen. Many physicians observe these patients for 6 hours and then repeat the chest x-ray. If the repeat chest x-ray shows no increase in the size of the PTX, the patient can be discharged with follow-up in 24 hours. Air travel and underwater diving (changes in atmospheric pressure) must be avoided until the PTX completely resolves.

A 53-year-old man presents to your ED stating he has had an excruciating right-sided headache since leaving the movie theater. He states that the headache is unilateral, severe, and associated with nausea and vomiting. His vision is blurry and notes seeing halos around objects. He denies trauma or a history of headaches in the past. Physical examination reveals right conjunctival injection and a pupil that reacts only marginally. Which examination is likely to yield the correct diagnosis? a. Measurement of intraocular pressure b. Funduscopic examination c. Fluorescein examination d. LP with cell count e. Visual acuity testing

*A* The patient presents with acute angle closure glaucoma which results from obstruction of aqueous outflow of the anterior chamber of the eye with a resulting rise in intraocular pressure. It is the result of a shallow anterior chamber or a chamber distorted by the development of a cataract. Classically, it occurs when a patient leaves a prolonged dimly lit situation. When the iris becomes mid-dilated, it maximally obstructs the trabecular meshwork occluding aqueous humor flow. Intraocular pressures may rise from normal (10-21 mm Hg) to levels as high as 50 to 100. Visual acuity is usually decreased in the affected eye as a result of corneal edema. Treatment is aimed at lowering intraocular pressure with acetazolamide, ophthalmic β-blockers, prostaglandin analogues, and pilocarpine to induce miosis. Ophthalmologic consultation and follow-up is indicated. Patients may present complaining of headache, nausea, and vomiting, but will often endorse that the symptoms began with acute eye pain.

A 28-year-old man who just finished a 7-day course of antibiotics for pharyngitis presents to the ED with progressive difficulty swallowing. His BP is 130/65 mm Hg, HR is 95 beats per minute, temperature is 100.1°F, RR is 16 breaths per minute, and oxygen saturation is 99%. On examination, the patient is in no acute distress but has a fluctuant mass on the right side of the soft palate with deviation of the uvula. Which of the following is the most appropriate next step in management? a. Needle aspiration, antibiotics, and follow-up in 24 hours b. Pain control, observation for 6 hours c. Admission for incision and drainage in the OR d. Antibiotics and follow-up in 24 hours e. CT scan, antibiotics, and follow-up in 24 hours

*A* The patient's presentation is typical for a peritonsillar abscess. Signs and symptoms include a sore throat, muffled voice, trismus, fluctuant mass, deviation of the uvula, odynophagia, and drooling. Many of these patients have a history of being recently treated for strep throat. The abscess is usually unilateral and in the superior pole of the tonsil. Airway patency must be assessed because of the obstructing potential of an abscess. Treatment includes either needle aspiration or incision and drainage of the abscess, in addition to antibiotic treatment. Some studies demonstrate the safety and cost-effectiveness of needle aspiration over incision and drainage.

The following is a radiograph showing The most common findings are curvilinear calcification of the aortic wall or a paravertebral soft tissue mass. of a 72-year-old man who presented to the ED complaining of gradually worsening back pain that he describes as constant and dull. He denies nausea, vomiting, diarrhea, and hematuria. Which of the following is an important predisposing factor for the development of the condition seen in this individual? a. Atherosclerosis b. Hyperparathyroidism c. Ethanol abuse d. Prostate cancer e. Hernia

*A* The radiograph demonstrates an abdominal aortic aneurysm. On the lateral radiograph, it is the circular structure just anterior to the vertebral column; on the AP view, it is the circular structure overlying the lower vertebrae, most prominently on the patient's left side. Signs of AAA large enough to cause symptoms are seen on plain radiographs approximately 66% to 75% of the time. The most common findings are curvilinear calcification of the aortic wall or a paravertebral soft tissue mass. Rarely, with longstanding aneurysms, erosion of one or more vertebral bodies may be seen. Atherosclerosis, age > 60, smoking, and family history are all important predisposing factors for the development of AAA.

A 25-year-old man presents to the ED complaining of dull periumbilical pain that migrated to his RLQ over the last hour. He states that he has no appetite and vomited twice. His BP is 125/75 mm Hg, HR is 87 beats per minute, temperature is 100.6°F, and RR is 16 breaths per minute. Laboratory results reveal WBC 11,000/μL, hematocrit 48%, platelets 170/μL. On physical examination, the patient complains of pain when you flex his knee with internal rotation at his hip. What is the name of this sign? a. Obturator b. Psoas c. Rovsing d. McBurney e. Murphy

*A* The test is the Obturator sign, in which the patient is supine with the right thigh flexed; passive internal or external rotation of the hip eliciting pain is a positive test for appendicitis. The pain is attributed to an inflamed appendix that is irritated by stretching the obturator internus muscle.

A 55-year-old man presents to the ED 6 hours after ingesting two bottles of his baby aspirin. He complains of nausea, vomiting, dizziness, and tinnitus. His temperature is 100.3°F, BP is 140/80 mm Hg, HR is 105 beats per minute, RR is 31 breaths per minute, and oxygen saturation is 99% on room air. Arterial blood gas on room air reveals a pH of 7.52, PCO2 10 mm Hg, and PO2 129 mm Hg. The blood salicylate level returns at 45 mg/dL. Which of the following is the most appropriate next step in management? a. Administer activated charcoal, begin IV hydration, and administer sodium bicarbonate. b. Administer activated charcoal, begin IV hydration, and intubate the patient for respiratory failure. c. Administer activated charcoal, begin IV hydration, and administer NAC. d. Arrange for immediate hemodialysis. e. Observe the patient overnight to allow the body to metabolize the salicylates.

*A* The treatment of salicylate toxicity has three objectives: (1) prevent further salicylate absorption, (2) correct fluid deficits and acid-base abnormalities, and (3) reduce tissue salicylate concentrations by increasing excretion. Activated charcoal should be administered as soon as possible to reduce salicylate absorption. Dehydration occurs early in salicylate intoxication and should be treated with IV hydration. Urine alkalization should be considered in patients with salicylate levels greater than 35 mg/dL. This is performed by administering IV sodium bicarbonate. Because salicylic acid is a weak acid, it is ionized in an alkaline environment and gets "trapped," limiting the amount that crosses the blood-brain barrier and increasing urinary excretion.

A 56-year-old man is brought in from the homeless shelter for strange, irrational behavior and unsteady gait for 1 day. A worker at the shelter reports that the patient is a frequent abuser of alcohol. On examination, the patient is alert but oriented to name only and is unable to give full history. He does not appear clinically intoxicated. You note horizontal nystagmus and ataxia. What is the most likely diagnosis? a. Wernicke encephalopathy b. Korsakoff syndrome c. Normal pressure hydrocephalus d. Central vertigo e. Alcohol withdrawal

*A* This patient exhibits the classic triad of Wernicke encephalopathy (WE): confusion, ataxia, and ophthalmoplegia. WE is a result of thiamine deficiency leading to decreased glucose metabolism and neuronal destruction, primarily in the cerebellum, hypothalamus, vestibular system, and memory. It is typically found in chronic alcoholics caused by nutritional deficiency, but can also occur in other malnutrition states, pregnancy, persistent vomiting, or dialysis. WE can mimic acute stroke symptoms and can lead to permanent nystagmus and ataxia. It carries 10% to 20% mortality if untreated.

A 32-year-old gravida 1, para 1 who gave birth by normal vaginal delivery at 38-weeks gestation 2 days ago presents to the ED complaining of bilateral hand swelling and severe headache that started 2 hours ago. Her BP is 187/110 mm Hg, HR is 85 beats per minute, temperature is 97.5°F, and RR is 15 breaths per minute. Urinalysis reveals 3+ protein. As you are examining the patient, she proceeds to have a generalized tonic-clonic seizure. Which of the following is the most appropriate next step in management? a. Administer magnesium sulfate IV. b. Administer labetalol to reduce her BP and morphine sulfate to address her headache. c. Administer sumatriptan and place the patient into a dark quiet room. d. Administer a loading dose of phenytoin, order a head CT scan, and call for a neurology consult. e. Administer diazepam and normal saline IV.

*A* This patient has postpartum eclampsia, which needs to be managed with magnesium sulfate and admission to the obstetrical service. Preeclampsia is defined as new-onset hypertension (> 140/90 mm Hg) and proteinuria (1 g/L in random specimen or > 3 g/L over 24 hours). Some clinicians also use generalized edema as a requirement. Preeclampsia is most common in the third trimester. Eclampsia occurs with the development of seizures or coma in a patient with preeclampsia. A preeclamptic woman may worsen after delivery and develop late postpartum eclampsia, which usually occurs in the first 24 to 48 hours postpartum but may present several weeks after delivery. Management of eclamptic seizures in the ED involves administering magnesium sulfate, which is believed to act as a membrane stabilizer and vasodilator, reducing cerebral ischemia. Although magnesium sulfate is not a direct antihypertensive, the hypertension associated with eclampsia is often controlled adequately by treating the seizure.

A 35-year-old woman with systemic lupus erythematosus (SLE) is brought to the ED by her brother after he found her febrile and confused. Physical examination reveals fever, tachycardia, a waxing and waning mental status, petechiae over her oral mucosa, pallor, and mildly heme-positive stool. Her urinalysis is positive for blood, red cell casts, and proteinuria. Laboratory results reveal a blood urea nitrogen (BUN) of 40 mg/dL, and a creatinine of 2 mg/dL. Her bilirubin is elevated (unconjugated > conjugated) and her international normalized ratio (INR) is 0.98. Her complete blood count reveals WBC 12,000/μL, hematocrit 29%, and platelet count 17,000/μL with schistocytes on the peripheral smear. Which of the following is the most appropriate next step in management? a. Admit to the intensive care unit (ICU) for plasmapheresis and close monitoring for acute bleeds. b. Admit to the ICU for platelet transfusion and monitoring for acute bleeds. c. Begin corticosteroids, transfuse platelets, and call surgery for immediate splenectomy. d. Admit to the ICU for dialysis and close monitoring for acute bleeds. e. Perform a noncontrast head computed tomography (CT), followed by a lumbar puncture (LP) for analysis of cerebrospinal fluid.

*A* This patient has thrombotic thrombocytopenic purpura (TTP), caused by increased platelet destruction. In TTP, platelet-fibrin thrombi deposit in vessel and cause injury to RBCs and platelets, resulting in microangiopathy hemolytic anemia and thrombocytopenia. Patients tend to be females who are 10 to 45 years of age. Risk factors include pregnancy, autoimmune disorders (eg, systemic lupus erythematosus [SLE]), infection, allogenic bone marrow transplantation, malignancy, and certain medications. The pentad can be remembered with the mnemonic FAT RN: fever, hemolytic anemia, thrombocytopenia, renal failure, and neurologic change (waxing and waning mental status). Treatment includes daily plasmapheresis until platelet count normalizes. RBCs may be transfused in patients ymptomatic from anemia (eg, tachycardia, hypoxia, orthostatic hypotension). All patients with TTP should be admitted to an ICU for close monitoring of acute bleeds.

A 19-year-old man was struck by a motor vehicle while crossing the street. In the ED, he is awake, alert, and oriented, but complaining of severe right-leg pain. His temperature is 98.9°F, BP is 85/50 mm Hg, HR is 125 beats per minute, and RR is 24 breaths per minute. You confirm that his airway is patent, breath sounds are equal bilaterally, and his abdomen is soft and nontender. His right leg is shorter than his left leg, slightly angulated, and swollen in his anterior thigh area. There is no open wound. Which of the following is the most likely diagnosis? a. Hypovolemic shock b. Neurogenic shock c. Cardiogenic shock d. Anaphylactic shock e. Septic shock

*A* This patient is in hypovolemic shock secondary to blood loss from a femoral fracture. Hypovolemic shock occurs when there is inadequate volume in the circulatory system, resulting in poor oxygen delivery to the tissues. Hemorrhage, GI losses, burns, and environmental exposures can all be responsible for hypovolemic shock. In trauma, hemorrhage is the most common cause of hypovolemic shock. This patient fractured his femur, disrupting the nearby vascular supply, resulting in significant blood collection in the soft tissue.

A 63-year-old man presents to the ED complaining of headache, vomiting, and "not being able to think straight" for 1 day. The patient states that he has hypertension and diabetes but ran out of his medications in the last week. His BP is 245/138 mm Hg, HR is 90 beats per minute, temperature is 98.7°F, and his RR is 14 breaths per minute. Fingerstick glucose is 178 mg/dL. On examination the patient appears slightly confused and oriented to name and place only. The neurologic examination is significant for papilledema. Which of the following is the most ppropriate next step in management? a. Nitroprusside IV b. Magnesium sulfate IV c. Metoprolol by mouth d. Hydrochlorothiazide by mouth e. Obtain head CT

*A* This patient presents with headache and altered mental status in the setting of severe hypertension leading to the diagnosis of hypertensive encephalopathy. A hypertensive emergency is defined by severe hypertension with evidence of organ dysfunction. The BP needs to be aggressively, but carefully lowered to prevent cerebral bleeding and progression to coma and death. Patients can also present with seizures, focal neurological deficits, visual acuity changes, or coma. Patients with hypertensive encephalopathy are managed with short-acting titratable IV antihypertensive medications such as IV nitroprusside or labetalol. The BP should not be significantly lowered since it can result in brain hypoperfusion and infarction. Typically, the mean arterial pressure (MAP) is lowered by 20% to 25% in the first hour of treatment.

A 21-year-old college student is brought by her roommate to the emergency department (ED). The roommate states that earlier in the day the patient complained of a severe headache, stiff neck, and photophobia. On their way to the ED, the roommate states that the patient was confused. Her vital signs are blood pressure (BP) 110/80 mm Hg, heart rate (HR) 110 beats per minutes, respiration rate (RR) 16 breaths per minute, and temperature 102°F. What is the next step in the management of this patient? a. Start empiric antibiotics, noncontrast head computed tomography (CT) prior to performing lumbar puncture (LP). b. Order a noncontrast head CT and start antibiotics once the results are back. c. Give 1 g of acetaminophen, start fluid hydration, and perform an LP. d. Perform an LP and start antibiotics once the results are back. e. Order a noncontrast head CT, perform an LP, then start antibiotics

*A* This patient presents with symptoms consistent with meningitis. Antibiotics are administered empirically as diagnostic workup proceeds. The best choice in this patient is ceftriaxone, which has good CNS penetration. In order to avoid transtentorial herniation in this patient with a neurologic deficit (confusion), a noncontrast head CT should be performed prior to LP. It is controversial whether or not a head CT needs to be performed prior to all LPs. However, if there is papilledema or a neurologic deficit, then head CT is mandatory.

A 30-year-old woman presents to the ED with fever, headache, a "sunburn-like" rash, and confusion. A friend states that the patient has complained of nausea, vomiting, diarrhea, and a sore throat over the past few days. Her last menstrual period began 4 days ago. Vital signs are HR 110 beats per minute, BP 80/45 mm Hg, RR of 18 breaths per minute, and temperature of 103°F. On physical examination, you note an ill-appearing female with a diffuse blanching erythroderma. Her neck is supple without signs of meningeal irritation. You note a fine desquamation of her skin, especially over the hands and feet, and hyperemia of her oropharyngeal, conjunctival, and vaginal mucus membranes. Laboratory results reveal a creatine phosphokinase (CPK) of 5000, WBC 15,000/μL, platelets of 90,000/μL, BUN 40 mg/dL, creatinine 2 mg/dL, and elevated liver enzymes. Which of the followin is the most likely causative organism? a. Staphylococcus aureus b. Rickettsia rickettsii c. Streptococcus pyogenes d. Neisseria meningitidis e. Neisseria gonorrhoeae

*A* This patient suffers from toxic shock syndrome (TSS), a severe, life-threatening syndrome characterized by high fever, diffuse macular erythroderma, profound hypotension, desquamation, and multisystem involvement (including vomiting or diarrhea, severe myalgias, mucus membrane hyperemia, renal or hepatic dysfunction, decreased platelets, and disorientation). TSS can rapidly progress to multisystem dysfunction and shock An exotoxin produced by S aureus is the presumed cause in menstrual-related TSS (MRTSS) and two endotoxins have been implicated in non-menstrualrelated TSS (NMRTSS).. TSS should be considered in any unexplained febrile illness associated with erythroderma, hypotension, and diffuse organ pathology. Patients with MRTSS usually present between the third and fifth day of menses. In severe cases, headache is the most common complaint. The rash is a diffuse, blanching erythroderma, often described as a painless "sunburn" that fades within 3 days and is followed by desquamation, especially of the palms and soles. For severe cases, treatment includes aggressive IV fluid resuscitation, IV oxacillin or cefazolin, and hospital admission in a monitored setting.

A 56-year-old man presents to the ED complaining of intermittent lightheadedness and nausea throughout the day. He believes it started after eating leftover shrimp salad in the morning. On further questioning, he reports that during the lightheadedness episodes the room is spinning around him and the episodes are triggered by turning his head to the right. He denies hearing loss, tinnitus or other associated symptoms. His BP is 137/85 mm Hg, HR is 67 beats per minute, temperature is 98.5°F, and RR is 14 breaths per minute. The patient reproduces the symptoms by turning his head to the right. Which of the following is the most likely diagnosis? a. Benign positional vertigo (BPV) b. Food poisoning c. Meniere disease d. Labyrinthitis e. TIA

*A*BPV is a transient positional vertigo associated with nystagmus. The problem occurs secondary to the creation and movement of canaliths (free-moving densities) in the semicircular canals of the inner ear with a particular head movement. Neurologic deficits are absent in BPV. Note that horizontal, vertical, or rotary nystagmus can occur in BPV. It is important to pay special attention to a patient with vertical nystagmus because it may be associated with a brainstem or cerebellum lesion. BPV is treated with the Epley maneuver (a series of head and body turns that reposition the canalith), antiemetics, and antihistamines. Key differences between peripheral and central vertigo are seen in the chart.

A 45-year-old woman presents to the ED immediately after landing at the airport from a transatlantic flight. She states that a few moments after landing she felt short of breath and felt pain in her chest when she took a deep breath. Her only medications are oral contraceptive pills and levothyroxine. She is a social drinker and smokes cigarettes occasionally. Her BP is 130/75 mm Hg, HR is 98 beats per minute, temperature is 98.9°F, RR is 20 breaths per minute, and oxygen saturation is 97% on room air. You send her for a duplex ultrasound of her legs, which is positive for deep vein thrombosis. What is the most appropriate management for this patient? a. Place patient on a monitor, provide supplemental oxygen, and administer unfractionated heparin. b. Place patient on a monitor, order a chest computed tomography (CT) scan to confirm a pulmonary embolism (PE), and then administer unfractionated heparin. c. Place patient on a monitor and administer aspirin. d. Instruct the patient to walk around the ED so that she remains mobile and does not exacerbate thrombus formation. e. Place the patient on a monitor, provide supplemental oxygen, and administer warfarin.

*A*The patient has a confirmed venous thrombosis and has symptoms consistent with a pulmonary thromboembolism. Data now show that almost every deep venous thrombosis (DVT) embolizes to some extent. The presence of a PE in this patient can be presumed by a confirmed DVT with pulmonary symptoms. All patients need to be on a monitor and should receive supplemental oxygen despite normal oxygen saturation. Oxygen acts as a pulmonary vasodilator. Heparin is the first-line therapy in this patient and should be administered promptly. Failure to achieve a therapeutic partial thromboplastin time (PTT) value within the first 24 hours leads to a 23% incidence of new embolism

A 22-year-old man presents to the ED complaining of dysuria for 3 days. He states that he has never had this feeling before. He is currently sexually active and uses a condom most of the time. He denies hematuria but notes a yellowish discharge from his urethra. His BP is 120/75 mm Hg, HR is 60 beats per minute, and temperature is 98.9°F. You send a clean catch urinalysis to the laboratory that returns positive for leukocyte esterase and 15 white blood cells per high power field (WBCs/hpf). Which of the following is the most appropriate next step in management? a. Send a urethral swab for culture and administer 125-mg ceftriaxone intramuscularly and 1-g azithromycin orally. b. Send urine for culture and administer SMX/TMP orally. c. Discharge the patient with strict instructions to return if his symptoms worsen. d. Order a CT scan to evaluate for a kidney stone. e. Have him follow-up immediately with a urologist to evaluate for testicular cancer.

*A.* Dysuria in young men is almost always because of urethritis, which is commonly caused by a sexually transmitted disease. Urethritis is classically divided into gonococcal (GU) and nongonococcal (NGU) types. GU is caused by N gonorrhoeae, while the major pathogen in NGU is C trachomatis. Nearly all men with GU have purulent urethral discharge. NGU may be asymptomatic or with a yellow, mucopurulent discharge. It was demonstrated in multiple studies that the two pathogens coexist in men with urethritis up to 50% of the time. Therefore, antibiotics should be directed at eliminating both organisms. A third-generation cephalosporin or ciprofloxacin as a one-time dose is used to treat GU. Common antibiotics used to treat NGU include azithromycin as a single dose and doxycycline or erythromycin for 7 days.

A 40-year-old man with a history of untreated HIV for 8 years comes into the ED complaining of cough, fever, and malaise for 3 days. He is tachypneic and diaphoretic. Chest radiograph reveals bilateral infiltrates. Arterial blood gas (ABG) analysis is significant for a PaO2 of 62 on room air. His chest radiograph is seen below. Which of the following is the most appropriate initial management? a. Corticosteroid treatment prior to antibiotic therapy b. Immediate treatment with IV trimethoprim/sulfamethoxazole (TMP/SMX) c. Administer antibiotics after a rapid sputum Gram stain is obtained d. Nebulizer treatment e. Racemic epinephrine

*A.* In a patient with untreated HIV and bilateral infiltrates on chest x-ray, PCP must be considered, in addition to community-acquired pneumonia. Corticosteroids are shown to be beneficial as adjunctive therapy in patients with moderate to severe PCP. They (1) limit oxygen deterioration, (2) decrease mortality and respiratory failure, and (3) accelerate recovery. Therapy should be initiated to all children and adult patients with a PO2 < 70 mm Hg or an A-a gradient > 35 mm Hg. It is important to initiate steroid therapy prior to starting antibiotics to avoid worsening hypoxia that is secondary to the inflammatory reaction caused by dying organisms.

A 68-year-old woman with recently diagnosed uterine cancer is brought to the ED by her daughter. The patient complains of acute onset right-sided chest pain that is sharp in character and worse with inspiration. Her BP is 135/85 mm Hg, HR 107 beats per minute, RR 20 breaths per minute, and oxygen saturation 97% on room air. Physical examination reveals a swollen and tender right calf. ECG is sinus tachycardia. Which of the following is the most appropriate next step in management? a. Start heparin therapy prior to diagnostic study b. Administer thrombolytics c. Order a ventilation-perfusion scan d. Order a CT angiogram e. Order a D-dimer

*A.* In the absence of a contraindication and a pretest probability that exceeds 50%, empiric heparin therapy should be administered. The patient has a high pretest probability for a PE with the following suggestive features: malignancy, tachycardia, tachypnea, suspected DVT, pleuritic chest pain, and dyspnea. Although heparin has no intrinsic fibrinolytic effect, it has an immediate effect on thrombin inhibition, thus preventing extension of the PE.

A 71-year-old man is playing cards with some friends when he starts to feel a pain in the left side of his chest. His fingers in the left hand become numb and he feels short of breath. His wife calls the ambulance and he is brought to the hospital. In the ED, an ECG is performed. Which of the following best describes the order of ECG changes seen in an MI? a. Hyperacute T wave, ST-segment elevation, Q wave b. Q wave, ST-segment elevation, hyperacute T wave c. Hyperacute T wave, Q wave, ST-segment elevation d. ST-segment elevation, Q wave, hyperacute T wave e. ST-segment elevation, hyperacute T wave, Q wave

*A.* The earliest ECG finding resulting from an AMI is the hyperacute T wave, which may appear minutes after the interruption of blood flow. The hyperacute T wave, which is short-lived, evolves to progressive elevation of ST segments. In general, Q waves represent established myocardial necrosis and usually develop within 8 to 12 hours after a ST-elevation MI, though they may be noted as early as 1 to 2 hours after the onset of complete coronary occlusion.

A 51-year-old man describes 1 week of gradually worsening scrotal pain and dysuria. He is sexually active with his wife. His temperature is 100.1°F, HR 81 beats per minute, BP 140/75 mm Hg, and oxygen saturation is 99% on room air. On physical examination, his scrotal skin is warm and erythematous. A cremasteric reflex is present. The posterior left testicle is swollen and tender to touch. Color Doppler ultrasonography demonstrates increased testicular blood flow. Urinalysis is positive for leukocyte esterase. What is the most likely diagnosis? a. Epididymitis b. Testicular torsion c. UTI d. Testicular tumor e. Varicocele

*A.* The patient has epididymitis. It is often difficult to distinguish epididymitis from testicular torsion and the clinician should always rule out torsion first if the diagnosis is in doubt. It is the most common misdiagnosis for testicular torsion. Epididymitis is generally a disease of adult men. The causative organism in men over 35 years old is E coli, while C trachomatis and Neisseria gonorrhoeae predominate in men less than 35 years old. The older patient may have a history of gonococcal urethritis (GU) tract manipulation or a history of prostatitis. The onset is usually gradual and urinary tract symptoms may precede the pain.

A 55-year-old man presents to the ED with worsening weakness, muscle cramps, and paresthesias. His past medical history is significant for hypertension and diabetes. He smokes one pack of cigarettes per day. On examination, the patient is alert and oriented and diffusely weak. An ECG shows a wide QRS complex, peaked T waves, and no P waves. Which of the following is the most important next step in management? a. Administer calcium gluconate. b. Administer insulin and dextrose. c. Administer aspirin and call the catheterization laboratory. d. Order an emergent head CT scan and get a neurology consult. e. Collect a sample of his urine to test for ketones.

*A.* The patient has life-threatening hyperkalemia. His ECG shows a wide QRS complex, peaked T waves, and no P waves. At any moment the patient's rhythm can go into ventricular fibrillation or asystole. There are many sxs of hyperkalemia that are often difficult to discern from those of the primary condition that precipitated the hyperkalemia. Patients may begin with lethargy and weakness and progress to paralysis and areflexia. If there are no ECG abnormalities in a patient with hyperkalemia, treatment can start with potassium binding resins (eg, Kayexalate). However, this patient requires immediate administration of calcium because he has an unstable cardiac rhythm. Calcium (gluconateor chloride) antagonizes the effects of potassium in the myocardium and briefly stabilizes the cardiac membrane.

While eating dinner, a 55-year-old man suddenly feels a piece of steak "get stuck" in his stomach. In the ED, he complains of dysphagia, is drooling, and occasionally retches. On examination, his BP is 130/80 mm Hg, HR is 75 beats per minute, RR is 15 breaths per minute, and oxygen saturation is 99% on room air. He appears in no respiratory distress. Chest x-ray is negative for air under the diaphragm. Which of the following is the most appropriate next step in management? a. Administer 1-mg glucagon intravenously while arranging for endoscopy. b. Administer a meat tenderizer such as papain to soften the food bolus. c. Administer 10-mL syrup of ipecac to induce vomiting and dislodge the food bolus. d. Perform the Heimlich maneuver until the food dislodges. e. Call surgery consult to prepare for laparotomy.

*A.* The patient most likely has a partial or complete obstruction in his lower esophagus secondary to the steak he ate. This usually occurs near the gastroesophageal junction. Administration of glucagon may cause enough relaxation of the esophageal smooth muscle to allow passage of the bolus in approximately 50% of patients. Its relaxant effect is limited to smooth muscle and therefore can only be used for impactions in the lower esophagus. If glucagon does not work, definitive management is with endoscopy.

A 58-year-old woman is brought to the emergency department (ED) by emergency medical service (EMS) after slipping on a patch of ice while walking to work and hitting her head on the cement pavement. Bystanders acknowledged that the patient was unconscious for approximately 1 minute. On arrival, her vital signs are: blood pressure (BP) 155/75 mm Hg, heart rate (HR) 89 beats per minute, respiratory rate (RR) 18 breaths per minute, and pulse oxygenation 98% on room air. She has a 5-cm laceration to the back of her head that is actively bleeding. You ask the patient what happened but she cannot remember. You inform her that she is in the hospital as a result of a fall. Over the next 10 minutes she asks you repeatedly what happened and where she is. You do not find any focal neurologic deficits. As you bring the patient to the CT scanner she vomits once. CT results show a normal brain scan. Which of the following is the most likely diagnosis? a. Cerebral concussion b. Diffuse axonal injury c. Cerebral contusion d. Posttraumatic epilepsy e. Trauma-induced Alzheimer disease

*A.* The patient sustained a cerebral concussion. This is caused by a head injury leading to a brief loss of neurologic function. These individuals are often amnestic to the event and frequently ask the same questions over and over again (perseverations). Headache with or without vomiting is generally present; however, there are no focal neurologic findings on examination. Loss of consciousness results from impairment of the reticular activating system. Patients show rapid clinical improvement. CT scan is normal.

A 60-year-old man is brought to the ED complaining of generalized crampy abdominal pain that occurs in waves. He has been vomiting intermittently over the last 6 hours. His BP is 150/75 mm Hg, HR is 90 beats per minute, temperature is 99.8°F, and his RR is 16 breaths per minute. On abdominal examination you notice an old midline scar the length of his abdomen that he states was from surgery after a gunshot wound as a teenager. The abdomen is distended with hyperactive bowel sounds and mild tenderness without rebound. An abdominal plain film confirms your diagnosis. Which of the following is the most appropriate next step in management? a. Begin fluid resuscitation, bowel decompression with a nasogastric tube, and request a surgical consult. b. Begin fluid resuscitation, administer broad-spectrum antibiotics, and admit the patient to the medical service. c. Begin fluid resuscitation, give the patient stool softener, and administer a rectal enema. d. Begin fluid resuscitation, administer broad-spectrum antibiotics, and observe the patient for 24 hours. e. Order an abdominal ultrasound, administer antiemetics, and provide pain relief

*A.* The patient's clinical picture is consistent with a SBO. Fluid resuscitation is important because of the inability of the distended bowel to absorb fluid and electrolytes at a normal rate. Compounded with vomiting, fluid loses can lead to hypovolemia and shock. Nasogastric suction provides enteral decompression by removing accumulated gas and fluid proximal to the obstruction. A surgical consult is necessary because definitive treatment may require taking the patient to the OR to relieve the obstruction. An old surgical adage states "Never let the sun set or rise on a bowel obstruction." Broad-spectrum antibiotics (b and d) are appropriate when surgery is planned or when there is suspicion for vascular compromise or bowel perforation.

A 58-year-old man is brought to the ED for a syncopal episode at dinner. His wife states that he was well until she found him suddenly slumping in the chair and losing consciousness for a minute. The patient recalls having some chest discomfort and shortness of breath prior to the episode. His rhythm strip, shows prolonged PR interval and random dropped beats Which of the following best describes these findings? a. Mobitz type I b. Mobitz type II c. First-degree atrioventricular (AV) block d. Atrial flutter with premature ventricular contractions (PVCs) e. Sinus bradycardia

*A.* The rhythm strip shows seconddegree AV block type II or Mobitz type II. Mobitz II presents with a prolonged PR interval (PR > 0.2 seconds) and random dropped beats (ie, P wave without QRS complex). The PR intervals are always the same duration. The block is below the level of the AV node, generally the His-Purkinje system. This heart block reflects serious cardiac pathology and may be seen with an anterior wall MI, which is the case with this patient

A 49-year-old woman presents to the ED with difficulty breathing after a morning jog. Her initial vitals include a HR of 60 beats per minute, a BP of 120/55 mm Hg, and an RR of 20 breaths per minute with an oxygen saturation of 94% on room air. Upon physical examination, the patient appears to be in mild distress with audible wheezing. She is able to speak in partial sentences and states that she occasionally uses an inhaler. Given this patient's history and physical examination, which of the following measures should be taken next? a. Peak expiratory flow b. Chest radiograph c. β-Natriuretic peptide level d. Rectal temperature e. ABG

*A.* This patient is suffering from an exercise-induced asthma exacerbation. Various triggers can cause bronchospasm. These include dust, various perfumes, underlying upper respiratory infections, cigarette smoke, menstrual flow, and various medications including aspirin. The medical intervention in this patient should include a β2-agonist nebulized solution, corticosteroids, and oxygen administration. The most useful measure to track the patient's progress with each treatment is a peak expiratory flow, and should be a part of the initial assessment and monitoring. The forced expiratory volume in 1 second from a maximal inspiration (FEV1) may be used in the ED setting. More commonly, peak flow meters are used which measure the peak expiratory flow rate in liters per second starting with fully inflated lungs. Both of these measurements require full patient cooperation whose values should be the average of three consecutive forced expirations. Normalized value ranges are determined by the patient's age, sex, and height.

A 63-year-old man is brought to the ED by EMS complaining of severe abdominal pain that began suddenly 6 hours ago. His BP is 145/75 mm Hg and HR is 105 beats per minute and irregular. On examination, you note mild abdominal distention and diffuse abdominal tenderness without guarding. Stool is heme positive. Laboratory results reveal WBC 12,500/μL, hematocrit 48%, and lactate 4.2 U/L. ECG shows atrial fibrillation at a rate of 110. A CT scan shows thickening of bowel wall Which of the following is the most likely diagnosis? a. Abdominal aortic aneurysm b. Mesenteric ischemia c. Diverticulitis d. SBO e. Crohn disease

*B* Acute mesenteric ischemia is caused by the lack of perfusion to the bowel. It primarily affects patients over the age of 50 years, particularly those with significant cardiovascular or systemic disease. The etiology is either from arterial occlusion (eg, embolization from the heart); venous thrombosis that is associated with a hypercoagulable state; or nonocclusive, because of reduced cardiac output (eg, congestive heart failure [CHF], recent myocardial infraction [MI], hypovolemia). Most patients present with abdominal pain that is initially dull and diffuse. In this early state, patients frequently complain of severe pain, but have minimal tenderness on examination (ie, the characteristic "pain out of proportion to examination"). As infarction develops, abdominal distension and peritoneal signs develop. Sudden onset of pain suggests arterial vascular occlusion by emboli. This may occur with patients who present in atrial fibrillation. Insidious onset suggests venous thrombosis or nonocclusive infarction. Radiographs may reveal dilated loops of bowel, air in the bowel wall ("pneumatosis intestinalis"), and thickening of the bowel wall, as seen in the CT scan. Management involves fluid resuscitation, antibiotics, and surgical intervention.

A 27-year-old man is seen in the ED for a leak around a surgical G-tube that was placed 2 weeks ago and has been used for enteral feeding for 1 week. Inspection reveals the tube is pulled out from the stoma, but is still in the cutaneous tissue. The abdomen is soft and nondistended and there are no signs of skin infection. Which of the following is the most appropriate next step in management? a. Insert a Foley catheter into the tract and aspirate. If gastric contents are aspirated the tube can be used for feeding. b. Insert a Foley catheter into the tract, instill water-soluble contrast, and obtain an abdominal radiograph prior to using for feeding. c. Remove the tube and admit the patient for observation. d. Remove the tube and immediately obtain a CT scan of the abdomen. e. Return to the OR for closure of gastrotomy and placement of a new tube.

*B* Although there are no studies stating how long it takes for a tract to mature, tracts that are 7 to 10 days old probably will remain open long enough to allow replacement. Insertion of a new tube should be performed with water-soluble lubricant. If resistance is met, the attempt should be aborted. After replacing the tube, 20- to 30-mL bolus of water soluble contrast material should be instilled into the tube, and a supine abdominal radiograph should demonstrate rugae of the stomach. If there is any question of improper placement, immediate consultation should be obtained.

A 41-year-old man, the restrained driver in a high-speed motor vehicle collision, is brought to the ED by EMS. The patient is breathing without difficulty with bilateral and equal breaths sounds. He has strong pulses peripherally indicating a BP of at least 90 mm Hg. The HR is 121 beats per minute. His Glasgow Coma Scale (GCS) is 14. A secondary survey reveals chest wall bruising. You suspect a cardiac injury. Which of the following locations most commonly involve cardiac contusions? a. Right atrium b. Right ventricle c. Left atrium d. Left ventricle e. Septum

*B* Blunt cardiac injuries usually result from high-speed vehicular collisions in which the chest wall strikes the steering wheel. Although they are all associated with potentially fatal complications, they should be viewed clinically as a continuous spectrum of myocardial damage: concussion (no permanent cell damage), contusion (permanent cell damage), infarction (cell death), tamponade (bleeding into the pericardium), and rupture (exsanguination). The mechanism of injury in a cardiac contusion involves a high-speed deceleration, which causes the heart to move forward, forcibly striking the sternum. In addition, the direct force of hitting an object (eg, the steering wheel) also can damage the heart. The right ventricle is the most commonly injured because it is the most anterior aspect of the heart and is closest to the sternum.

A 45-year-old man is brought into the ED after a head-on motor vehicle collision. His BP is 85/45 mm Hg and HR is 130 beats per minute. He is speaking coherently. His breath sounds are equal bilaterally. After 2 L of fluid resuscitation, his BP is 80/40 mm Hg. A FAST examination reveals fluid in Morison pouch. Which of the following organs is most likely to be injured in blunt abdominal trauma? a. Liver b. Spleen c. Kidney d. Small bowel e. Bladder

*B* Blunt trauma is the most common mechanism of injury seen in the United States. The forces exerted on the abdomen put all of the organs at risk for injury. Motor vehicle collisions with another vehicle or with pedestrians are the major causes of blunt abdominal trauma. The spleen is the organ most often injured, and in approximately 66% of these cases, it is the only damaged intraperitoneal organ.

A 41-year-old office worker presents to the ED with pain in her right wrist and fingers that is associated with a tingling sensation. The pain occasionally awakens her from sleep and improves when the hand is shaken. She recalls falling on her wrist 2 years ago while she was ice-skating. Her symptoms are reproducible when her wrist is held in flexion for 60 seconds. Which of the following nerves is affected in patients with this syndrome? a. Ulnar b. Median c. Axillary d. Radial e. Musculocutaneous

*B* Carpal tunnel syndrome is a neuropathy of the median nerve that occurs as a result of compression of the nerve within the carpal tunnel of the wrist. Symptoms include paresthesias and pain in the distribution of the median nerve; the volar thumb, second, third, and half of the fourth digit. Symptoms are usually worse after strenuous activities and at night. Pain improves with shaking of the hand or holding it in a dependent position. A sensitive test used to diagnose carpal tunnel is Phalen test. Have the patient hold the affected wrist in hyperflexion for 60 seconds. The test is positive if paresthesias or numbness develops in the median nerve distribution

A 67-year-old woman is brought to the ED after being struck by a cyclist while crossing the street. On arrival to the ED, her eyes remain closed to stimuli, she makes no verbal sounds, and withdraws only to painful stimuli. You assign her a GCS of 6. Her BP is 175/90 mm Hg and HR is 75 beats per minute. As you open her eye lids, you notice that her right pupil is 8 mm and nonreactive and her left is 4 mm and minimally reactive. Which of the following is the most common manifestation of increasing intracranial pressure (ICP) causing brain herniation? a. Change in level of consciousness b. Ipsilateral pupillary dilation c. Contralateral pupillary dilation d. Significantly elevated BP e. Hemiparesis

*B* Cerebral herniation occurs when increased ICP overwhelms the natural compensatory capacities of the central nervous system (CNS). Increased ICP may be the result of post traumatic brain swelling, edema formation, traumatic mass lesion expansion, or any combination of the three. When increasing ICP cannot be controlled, the intracranial contents will shift and herniate through the cranial foramen. Herniation can occur within minutes or up to days after a traumatic brain injury. Once the signs of herniation are present, mortality approaches 100% without rapid reversal or temporizing measures. Uncal herniation is the most common clinically significant form of traumatic herniation and is often associated with traumatic extracranial bleeding. The classic signs and symptoms are caused by compression of the ipsilateral uncus of the temporal lobe. This causes compression of cranial nerve III leading to anisocoria, ptosis, impaired extraocular movements, and a sluggish pupillary light reflex. As herniation progresses, compression of the ipsilateral oculomotor nerve eventually causes ipsilateral pupillary dilation and nonreactivity.

22-year-old man is brought to the ED after sustaining a single gunshot wound (GSW) to his right thigh. On arrival, his HR is 105 beats per minute and BP is 115/75 mm Hg. You note a large hematoma of his medial thigh. The patient complains of numbness in his right foot. On extremity examination, the right foot is pale and you cannot palpate a distal pulse but can locate the dorsalis pedis by Doppler. In addition, the patient cannot move the foot. Which of the following is the most appropriate next step in management? a. Angiography b. Exploration and repair in the OR c. Fasciotomy to treat compartment syndrome d. Wound exploration e. CT scan of the right extremity

*B* Clinical manifestations of penetrating arterial injury of the extremity are generally divided into "hard signs" and "soft signs." Hard signs include pulsatile bleeding, expanding hematoma, palpable thrill or audible bruit, and evidence of distal ischemia (eg, pain, pallor, pulselessness, paralysis, paresthesia, poikilothermia). Soft signs include diminished ankle-brachial indices, asymmetrically absent or weak distal pulse, history of moderate hemorrhage or wound close to a major artery, and a peripheral nerve deficit. Emergent surgery is generally necessary when there are hard signs of vascular injury. Although the management of penetrating extremity injury is evolving, whenever there is evidence of distal ischemia, the patient should be taken to the OR for exploration and repair. When severe ischemia is present, the repair must be completed within 6 to 8 hours to prevent irreversible muscle ischemia and loss of limb function. In the presence of hard signs without evidence of ischemia, some surgeons may prefer to first perform angiography to better define the injury.

A 19-year-old man is brought into the ED by paramedics with a stab wound to the right lower abdomen. The medics applied a pressure dressing and started an IV line en route to the hospital. On arrival, the patient has no complaints and wants to leave. His temperature is 98.4°F, BP is 130/95 mm Hg, HR is 111 beats per minute, RR is 20 breaths per minute, and oxygen saturation is 98% on room air. He is alert and oriented to person, place, and time. His abdomen is soft and nontender, with normal bowel sounds. He has a 2-cm stab wound with visible subcutaneous fat in his right lower quadrant (RLQ). You initiate the focused abdominal sonogram for trauma (FAST) examination. Which type of fluid should you start for his initial resuscitation? a. 7% Sodium chloride b. 0.9% Sodium chloride c. 10% Albumin d. Type and cross-matched blood e. Type-specific blood

*B* Crystalloids, such as normal saline (0.9% sodium chloride) or lactated ringers are the preferred resuscitation fluid in the United States. The patient is currently stable; however, he is tachycardic and has suffered an injury with the potential for significant morbidity. The FAST examination will help illustrate the extent of this patient's injury. In the meantime, the patient should be started on isotonic crystalloid solution. There is currently no convincing evidence that either solution—normal saline or Ringers lactate—is superior to the other.

A 19-year-old woman presents to the ED with abdominal pain, nausea, vomiting, diarrhea, and hematemesis after ingesting an unknown substance in a suicide attempt. Which of the following antidotes are correctly paired? a. Organophosphate—Physostigmine b. Iron overdose—Deferoxamine c. Aspirin overdose—NAC d. Benzodiazepine overdose—Narcan e. Anticholinergic overdose—Fomepizole

*B* Deferoxamine is a specific chelator of ferric iron (Fe3+). It binds with iron to form a water-soluble compound, ferrioxamine, which can be excreted by the kidneys. Deferoxamine has a half-life of 1 hour, so continuous infusion is the preferred method of administration. The patient's clinical presentation is consistent with acute iron poisoning. Initial presentation reflects the corrosive effects of iron on the gut and includes nausea, vomiting, diarrhea, and sometimes GI bleeding. Patients with severe overdose may present with shock or coma.

A 27-year-old woman is brought to the ED by her husband after having a first-time seizure at home. She has no past medical history, and had no complications while delivering her newborn vaginally 1 week prior to presentation. In the ED, her BP is 178/95 mm Hg, HR is 97 beats per minute, temperature is 99.1°F, and RR is 18 breaths per minute. On examination, she has mild edema of her hands and feet. The seizure stopped spontaneously, but the patient is postictal appearing and cannot answer your questions. Which of the following is the most appropriate diagnostic test? a. Complete blood count b. Head CT c. LP d. Urinalysis e. ECG

*B* Eclampsia develops after the 20th week of gestation and is considered a complication of severe preeclampsia. The progression from severe preeclampsia to seizures and coma is thought to be a result of hypertensive encephalopathy, vasogenic edema-associated cortical ischemia, edema, or hemorrhage. Therefore, a head CT should be quickly obtained. Eclampsia is a clinical diagnosis with patients having seizures without evidence of CNS, metabolic, or any other seizure etiology. Most patients have systolic BP higher than 160 mm Hg or diastolic BP higher than 110 mm Hg and proteinuria; however, eclampsia can occur with minimally elevated BP—or elevated, relative to baseline—and without proteinuria. Evidence of end-organ damage prior to development of seizures is common. Symptoms include altered mental status, headache, visual disturbances, and abdominal pain; signs include hemolysis, impaired liver function with elevated liver enzyme levels, low platelets (HELLP), hemoconcentration, proteinuria, oliguria, pulmonary edema, generalized peripheral edema, microangiopathic hemolytic anemia, and fetal growth retardation. Eclampsia is most common during the antepartum period, yet 20% to 25% of cases ccur during the postpartum period. Although postpartum eclampsia may occur as long after birth as 3 weeks postpartum, most cases (98%) occur on the first postpartum day. Eclampsia is not the cause of seizures that occur during the first trimester or well into the postpartum period. These seizures are suggestive of CNS pathology.

A 31-year-old man is brought to the ED by EMS who state that the man was found lying on the floor of his garage. He is rousable in the ED, speaks with slurred speech, and vomits. His BP is 140/85 mm Hg, HR is 94 beats per minute, temperature is 98.8°F, RR is 17 breaths per minute, and oxygen saturation is 99% on room air. You place an IV line, draw blood, and start a liter of normal saline running through the line. Laboratory results reveal serum sodium 139 mEq/L, potassium 3.5 mEq/L, chloride 101 mEq/L, bicarbonate 14 mEq/L, BUN 15 mg/dL, creatinine 1 mg/dL, glucose 105 mg/dL, arterial blood pH 7.27, COHb 4%, and lactate 2.8 mEq/L. Urinalysis shows: 1+ protein, trace ketones, WBC 4/hpf, red blood cell (RBC) 2 to 3/hpf, and multiple envelope-shaped and needle-shaped crystals. Which of the following conditions would best explain his metabolic acidosis? a. Carbon monoxide (CO) poisoning b. Ethylene glycol poisoning c. Diabetic ketoacidosis d. Lactic acidosis e. Isopropyl alcohol poisoning

*B* Ethylene glycol is a colorless, odorless, slightly sweet-tasting liquid that is found in antifreeze. Ingestions of antifreeze are either accidental, suicidal, or in substitute of ethanol. Ethylene glycol is metabolized to glycolic acid, which results in a profound anion gap metabolic acidosis (Na) − ([Cl] + [HCO3]); (139 − [101 + 14]) = 24. Glycolic acid is subsequently metabolized to oxalic acid, which combines with calcium to form calcium oxalate crystals, which then precipitate in renal tubules, brain, and other tissues. The finding of crystalluria is considered the hallmark of ethylene glycol ingestion; however, its absence does not rule out the diagnosis. Another useful test in the ED involves examining freshly voided urine for fluorescence with a Wood lamp. Sodium fluorescein is added to antifreeze to aid in the detection of radiator leaks. Ingestion of ethylene glycol is associated with neurologic, cardiopulmonary, and renal abnormalities.

A 26-year-old woman, who was found lying on the floor of her apartment next to an unlabeled empty pill bottle, is brought into the ED. Her HR is 117 beats per minute, BP is 95/65 mm Hg, RR is 14 breaths per minute, and oxygen saturation is 97% on 2-L nasal cannula. On examination, the patient appears obtunded, her pupils are 3 mm and reactive. Her oropharynx is dry and there is no gag reflex to pharyngeal stimulation. Her neck is supple. The heart is tachycardic without murmurs, the lungs are clear to auscultation, and the abdomen is soft. There is normal rectal tone and brown stool that is heme negative. Her skin is cool and moist with no signs of needle tracks. Neurologically, she is unresponsive, but withdraws all extremities to deep palpation. Finger stick blood glucose is 85 mg/dL. Her ECG reveals sinus tachycardia at 119 with a QRS complex of 120 msec and a terminal R wave in lead aVR. Which of the following is the most appropriate next step in management? a. Orotracheal intubation, administer activated charcoal through orogastric tube, and IV Narcan b. Orotracheal intubation, administer activated charcoal through orogastric tube, and IV sodium bicarbonate c. Orotracheal intubation, administer activated charcoal through orogastric tube, and IV NAC d. Orotracheal intubation, administer syrup of ipecac through orogastric tube, and IV sodium bicarbonate e. Induce vomiting prior to intubation to lower the risk of aspiration then administer IV sodium bicarbonate

*B* For young patients with altered mental status, toxic ingestion should be high on the differential. This clinical scenario is most consistent with toxic ingestion of a TCA. Treatment of all toxic ingestions should begin with assessment of airway, breathing, and circulation. As a result this patient's obtunded mental status and loss of gag reflex, orotracheal intubation is indicated for airway protection. Subsequently, activated charcoal can be administered. Because of the anticholinergic effects of TCAs, absorption is prolonged and GI motility is delayed leading to greater toxicity. Therefore, an additional dose of charcoal should be administered several hours later. In an obtunded patient, it is important to first secure an airway prior to administering charcoal to prevent aspiration in the event of vomiting.

A 40-year-old diabetic man presents to the ED with severe perineal pain and fever as high as 103°F. Physical examination demonstrates crepitus over the medial thigh and widespread discoloration with sharp demarcation over the scrotum. The scrotum is warm and markedly edematous. His pain appears out of proportion to the physical examination. Which of the following is the most likely diagnosis? a. Epididymitis b. Fournier gangrene c. Scrotal edema d. Paraphimosis e. Testicular torsion

*B* Fournier gangrene is a polymicrobial necrotizing fasciitis of the perineal subcutaneous tissue and male genitalia that originates from the skin, urethra, or rectum. It usually begins as a simple abscess or benign infection that quickly spreads, especially in an immunocompromised patient, often a diabetic male. If not promptly diagnosed, it can lead to end-artery thrombosis in the subcutaneous tissue and widespread necrosis. Treatment includes aggressive fluid resuscitation; broad-spectrum antibiotics to cover gram-positive, gramnegative, and anaerobic bacteria; wide surgical debridement; and, possibly, hyperbaric oxygen therapy.

A 40-year-old man who is an employee of the hospital is brought to the ED actively seizing. A coworker states that the patient has a known seizure disorder and currently takes phenytoin for the disorder. He also tells you that the patient has been under stress recently and may not have taken his last few doses of medication. You call for the nurse to place a face mask with 100% oxygen and gain IV access. You then ask for a medication to be drawn up. Which is the most appropriate initial medication you should administer in this actively seizing patient? a. Phenytoin b. Diazepam c. Phenobarbital d. Valproic acid e. Lithium

*B* Generally, the first-line pharmacologic treatment in an actively seizing patient is a parental benzodiazepine, such as diazepam (Valium), lorazepam (Ativan), or midazolam (Versed). Benzodiazepines are effective in terminating ictal activity in 75% to 90% of patients. Diazepam can be administered intravenously, intramuscularly, or down an endotracheal tube. Lorazepam and midazolam can be given intravenously or intramuscularly. All three have similar efficacy in terminating seizures.

A 40-year-old man with a known history of ethanol abuse states that 2 hours ago he ingested two bottles of extra-strength Tylenol. The patient has no medical complaints except for some nausea. At 4 hours post ingestion you send blood to the laboratory to measure the serum acetaminophen concentration. The level returns and falls above the treatment line when you plot it on the APAP nomogram. You administer activated charcoal and decide to start IV NAC. Which of the following is a known adverse effect of IV NAC administration? a. Hepatic failure b. Anaphylactoid reaction c. Hypertensive crisis d. Confusion e. Change in urine color

*B* IV NAC has been responsible for anaphylactoid reactions, including rash, bronchospasm, hypotension, and death. These complications are dose and concentration dependent and are prevented by slow administration of dilute NAC. Some other side effects include GI disorders, tachycardia, and chest tightness.

A 34-year-old man is brought to the ED after being shot in the right side of his chest. The patient is awake and speaking. Breath sounds are diminished on the right. There is no bony crepitus or tracheal deviation. His BP is 95/65 mm Hg, HR is 121 beats per minute, and RR is 23 breaths per minute. Supine chest radiograph reveals a hazy appearance over the entire right lung field. You place a 36F chest tube into the right thoracic cavity and note 1200 cc of blood in the chest tube drainage system. Which of the following is an indication for thoracotomy? a. 500 cc of initial chest tube drainage of blood b. 1200 cc of initial chest tube drainage of blood c. Persistent bleeding from the chest tube at a rate of 50 cc/h d. Chest radiograph with greater than 50% lung field white out e. Evidence of a PTX

*B* Indications for thoracotomy include: • Initial chest tube drainage of 1000 to 1500 cc of blood • 200 cc/h of persistent drainage • Patient remains hypotensive despite adequate blood replacement, and other sites of blood loss have been ruled out. • Patient decompensates after initial response to resuscitation. • Increasing hemothorax seen on chest x-ray studies.

A 23-year-old woman who is an elementary school teacher is brought to the ED after syncopizing in her classroom while teaching. Prior to passing out, she describes feeling lightheaded and dizzy and next remembers being in the ambulance. There was no evidence of seizure activity. She hasm no medical problems and does not take any medications. Her father died of a "heart problem" at 32 years of age. She does not smoke or use drugs. BP is 120/70 mm Hg, pulse rate is 71 beats per minute, RR is 14 breaths per minute, and oxygen saturation is 100% on room air. Her physical examination and laboratory results are all normal. A rhythm strip is shows prolonged QT interval. Which of the following is the most likely diagnosis? a. Wolff-Parkinson-White syndrome b. Long QT syndrome c. Lown-Ganong-Levine syndrome d. Complete heart block e. Atrial flutter

*B* Long QT syndrome (LQTS) is a congenital disorder characterized by a prolongation of the QT interval on ECG and a propensity to ventricular tachydysrhythmias, which may lead to syncope, cardiac arrest, or sudden death in otherwise healthy individuals. The QT interval on the ECG, measured from the beginning of the QRS complex to the end of the T wave, represents the duration of activation and recovery of the ventricular myocardium. In general, HR corrected QTc values above 440 msec are considered abnormal. LQTS has been recognized as the Romano-Ward syndrome (ie, familial occurrence with autosomal dominant inheritance, QT prolongation, and ventricular tachydysrhythmias) or the Jervell and Lang-Nielsen syndrome (ie, familial occurrence with autosomal recessive inheritance, congenital deafness, QT prolongation, and ventricular arrhythmias). Patients with LQTS usually are diagnosed after a cardiac event (eg, syncope, cardiac arrest) already has occurred. In some situations, LQTS is diagnosed after sudden death in family members.

A 46-year-old alcoholic man presents to the ED with cough, fever, and rigors for 2 days. He describes his sputum as rust-colored. His vital signs are: temperature 101.1°F, HR 94, BP 125/75 mm Hg, RR 20, and pulse oxygen of 97% on room air. Auscultation reveals crackles in the left-lower lobe. Chest radiograph is significant for a left-lower lobar infiltrate. Which of the following organism is the most common cause of community acquired bacterial pneumonia? a. Haemophilus influenza b. Streptococcus pneumoniae c. Klebsiella pneumonia d. Group A streptococci e. Pseudomonas aeruginosa

*B* Most pneumonia is the result of a single species of bacteria. Streptococcus pneumoniae is the most common cause of community-acquired pneumonia. It is also the most common cause of bacterial pneumonia in HIV-infected patients. Patients may appear ill and exhibit tachypnea. Fever and chills are usually present. Auscultation of the lungs reveals decreased breath sounds over the infiltrate and the radiograph shows a lobar pattern.

A 17-year-old man presents to the ED after getting hit in the right eye with a tennis ball during a tennis match. On arrival to the ED, you note periorbital swelling and ecchymosis. The patient's visual acuity is 20/20. When you are testing his extraocular muscles, you note that his right eye cannot look superiorly but his left eye can. He also describes pain in his right eye when attempting to look upward. Which of the following is the most likely diagnosis? a. Zygomatic arch fracture b. Orbital floor fracture c. Retrobulbar hematoma d. Ruptured globe e. Mandible fracture with entrapment of the pterygoid

*B* Orbital floor fractures typically occur when a blunt object with a radius of curvature less than 5 cm (eg, often a fist or ball smaller than a softball) strikes the orbit. The blunt force causes an increase in intraorbital pressure causing a fracture along the weakest part of the orbit, usually the inferior or sometimes medial wall. Patients usually complain of pain that is greatest with upward eye movement. They may have impaired ocular motility or diplopia if the inferior rectus muscle becomes entrapped. They may also present with infraorbital hypoesthesia because of compression of the infraorbital nerve. Generally, the patient has normal visual acuity unless there is an associated ocular injury. A classic radiographic finding is the "teardrop sign," which represents herniated orbital fat and muscle in the roof of the maxillary sinus. There may also be an air-fluid level in the maxillary sinus as a result of bleeding into it. Patients usually do well and recover completely in mild to moderate fractures.

A 34-year-old woman presents to the ED after ingesting an unknown quantity of her antidepressant pills. EMS workers found an empty bottle of amitriptyline on her apartment floor. She is awake but appears delirious. Her BP is 130/65 mm Hg, HR is 101 beats per minute, temperature is 99.1°F, RR is 16 breaths per minute, and oxygen saturation is 100% on room air. On examination, her pupils are 7 mm and reactive. Her face is flushed and mucous membranes are dry. Her lungs are clear and heart is without murmurs. The abdomen is soft, nontender, and with decreased bowel sounds. She is moving all four extremities. ECG reveals sinus rhythm at a rate of 99 and QRS just under 100 msec. In a tricyclic antidepressant (TCA) overdose, which of the following is responsible for her mydriasis, dry mucous membranes, and delirium? a. Sodium channel blockade b. Muscarinic receptor blockade c. Inhibition of serotonin and norepinephrine reuptake d. Histamine receptor blockade e. α-Receptor blockade

*B* Overdose of TCA results in toxicity by a number of different mechanisms. The anticholinergic properties of TCAs result in the toxidrome "blind as a bat (mydriasis), red as a beet (flushed skin), hot as a hare (hyperthermia secondary to lack of sweating), dry as a bone (dry mucous membranes), mad as a hatter (mental status changes)." This is secondary to muscarinic receptor blockade. The cardinal signs of TCA overdose include ventricular dysrhythmias, hypotension, and decreased mental status. Sodium bicarbonate is a potentially lifesaving intervention in TCA overdose because an alkaline pH combined with a sodium load increases conductance through cardiac fast sodium channels and prevents ventricular dysrhythmias as evidenced by narrowing of the QRS complex on an ECG.

A 54-year-old man with a history of hepatitis C, alcohol abuse, and cirrhotic ascites presents with increasing abdominal girth and abdominal pain. He complains of increasing difficulty breathing, especially when lying down, caused by worsening ascites. On physical examination, the patient is cachectic and appears older than his stated age. He has a diffusely tender abdomen and tense ascites. The liver is palpable 4 cm below the costal margin. Vitals include a BP of 110/65 mm Hg, HR of 110 beats per minute, RR of 22 breaths per minute, and temperature of 102°F. Which of the followingis the most common organism seen in spontaneous bacterial peritonitis? a. Pseudomonas aeruginosa b. Enterococcus c. Streptococcus pneumoniae d. Enterobacteriaceae e. Streptococcus viridans

*B* Spontaneous bacterial peritonitis (SBP) is the most common complication of cirrhotic ascites and should be suspected in all patients with a history of cirrhosis who present with fever, abdominal pain or tenderness, worsening ascites, or encephalopathy. Paracentesis should be performed to confirm the diagnosis. Ascitic fluid should be tested for glucose, total protein, lactate dehydrogenase (LDH), Gram stain, and WBC count. Culture results are often negative; culture results' yield may be improved, however, by placing 10 mL of ascitic fluid in a blood culture bottle. Total WBC count > 1000/μL or neutrophil count > 250/μL is diagnostic for SBP. Gram-negative Enterobacteriaceae (eg, E coli, Klebsiella) account for 63% of all cases. Empiric treatment for suspected SBP includes third-generation cephalosporins, ticarcillin clavulanic acid, piperacillin-tazobactam, or ampicillin-sulbactam.

You are working in the ED on a Sunday afternoon when four people present with acute onset vomiting and crampy abdominal pain. They were all at the same picnic and ate most of the same foods. The vomiting began approximately 4 hours into the picnic. They deny having any diarrhea. You believe they may have "food poisoning" so you place IV lines, administer IV fluids, and observe. Over the next few hours, the patients begin to improve, the vomiting stops and their abdominal pain resolves. Which of the following is the most likely cause of their presentation? a. Scombroid fish poisoning b. Staphylococcal food poisoning c. Clostridium perfringens food poisoning d. Campylobacter e. Salmonellosis

*B* Staphylococcal food poisoning is caused by an enterotoxin-forming strain of Staphylococcus organisms in the food before ingestion. Most protein-rich foods support the growth of staphylococci, particularly ham, eggs, custard, mayonnaise, and potato salad. The illness has an abrupt onset, beginning 1 to 6 hours after ingestion of the contaminated food. Cramping and abdominal pain, with violent and frequent retching and vomiting are the predominant symptoms. Diarrhea is variable; it is usually mild, and occasionally absent. Although often aggressive in onset, staphylococcal food poisoning is short lived and usually subsides in 6 to 8 hours, rarely lasting more than 24 hours. The patient is often recovering when first seen by a physician. The short incubation period and multiple cases in people eating the same meal are highly suggestive of this disease.

A 74-year-old woman is brought to the ED by EMS for altered mental status. Her BP is 138/72 mm Hg, HR is 91 beats per minute, RR is 17 breaths per minute, and temperature is 100.9°F. A head CT is normal. LP results revealed the following: WBC 1020/mL with 90% polymorphonuclear cells, Glucose 21 mg/dL, Protein 225 g/L. Which of the following is the most likely diagnosis? a. TB meningitis b. Bacterial meningitis c. Viral meningitis d. Fungal meningitis e. Encephalitis

*B* The CSF analysis in bacterial meningitis typically shows an elevated WBC count with predominant polymorphonuclear leukocytes. Protein is elevated and glucose is low. A Gram stain may show bacteria. The most specific marker for the diagnosis is a positive culture. Tests that evaluate the presence of antigen in the CSF (eg, latex agglutination studies) are particularly useful in the diagnosis of partially treated bacterial meningitis.

A 43-year-old homeless man presents to the ED with fever and nuchal rigidity. His mental status is depressed but his neurologic examination is otherwise nonfocal. Noncontrast head CT is normal. You obtain an LP for diagnostic purposes and initiate empiric antibiotic treatment for bacterial meningitis. The result of the CSF analysis is complete after 1 hour. The protein and glucose are within normal range but the WBC count consists of 220 mononuclear cells. The Gram stain is negative. The patient was recently purified protein derivative (PPD) negative and had a normal chest x-ray. In addition to the treatment already initiated, what is the next most appropriate step in this patient's management? a. Empiric treatment with isoniazid b. Empiric treatment with antiviral medications for herpes virus c. Empiric treatment with antifungals d. Antibiotic coverage for Bartonella sp. e. Addition of steroids to the antibiotic regimen

*B* The CSF analysis in this patient is consistent with a viral or atypical cause of meningitis. Although patients may have a mononuclear predominance and still have bacterial meningitis, viral causes should be considered. CSF should be sent for PCR analysis and empiric treatment initiated for herpes encephalitis. The mortality of meningoencephalitis caused by herpes simplex virus (HSV) is exceptionally high if untreated.

A 75-year-old man presents to the ED with a depressed level of consciousness. His wife is at the bedside and states he was stacking heavy boxes when he complained of a sudden intense headache. He subsequently sat down on the couch and progressively lost consciousness. She states that he had a headache the previous week that was also sudden but not as intense. He had gone to visit his primary-care physician who sent him to have a CT scan of the brain, which was normal. Over the course of the past week, he complained of intermittent pulsating headaches for which he took sumatriptan. In the ED, you intubate the patient and obtain the noncontrast head CT shwoing blood in the subarachnoid space. The scan is most consistent with which diagnosis? a. Meningoencephalitis b. SAH c. Normal pressure hydrocephalus d. Epidural hematoma e. Subdural hematoma

*B* The CT depicts subarachnoid arachnoid blood. This patient may have had a sentinel bleed, a small SAH, the previous week. Noncontrast CT misses a small percentage of SAH and therefore, in cases of high suspicion, an LP must be obtained to exclude the diagnosis.

A 63-year-old man who lives in a homeless shelter presents to the ED complaining of headache with photophobia for 6 hours. Upon arrival to the ED, the triage nurse places him in an isolation room. The triage note states that the patient was alert and conversant during the nursing interview. You enter the isolation room and attempt to speak to the patient but he is lethargic and combative. You note that his temperature is 103°F. He is unwilling to move his neck and winces when you attempt to check his pupillary reflexes with a penlight. The nurse informs you that laboratory analyses are delayed this evening because of staffing issues. Which of the following is the most appropriate next step in management? a. Diagnostic LP b. Initiation of IV antibiotic therapy c. A loading dose of IV corticosteroids d. Aggressive antipyretic therapy e. Sedation of the patient and noncontrast head CT

*B* The clinical presentation is consistent with meningitis. The patient's mental status has declined between initial nursing assessment and the physician's interview. Delay of antibiotic therapy in order to first confirm the diagnosis with CSF analysis may lead to increased mortality.

A 73-year-old man is seen in the ED for abdominal pain, nausea, and vomiting. His symptoms have progressively worsened over the past 2 to 3 days. The pain is diffuse and comes in waves. He denies fever or chills, but has a history of constipation. He reports no flatus for 24 hours. Physical examination is notable for diffuse tenderness and voluntary guarding. There is no rebound tenderness. An abdominal radiograph shows distended loops of small bowel and the upright film shows multiple air-fluid levels in a stepladder appearance. Which of the following is the most likely diagnosis? a. Constipation b. SBO c. Cholelithiasis d. Large bowel obstruction e. Inflammatory bowel disease

*B* The clinical scenario and radiograph are consistent with SBO. Patients usually present with diffuse, crampy abdominal pain that is often episodic in nature. Typically, the patient reports no recent bowel movements or flatus passage. The most common causes of SBO are adhesions and hernias. All patients with suspected SBO should have flat and upright abdominal radiographs performed. Flat plate abdominal films can show distended loops of small bowel and the upright film can show multiple air-fluid levels in a stepladder appearance.

A 65-year-old man with a history of occasional painless rectal bleeding presents with 2 to 3 days of constant, dull RLQ pain. He also complains of fever, nausea, and decreased appetite. He had a colonoscopy 2 years ago that was significant for sigmoid and cecal diverticula, but was otherwise normal. On physical examination he has RLQ tenderness with rebound and guarding. His vitals include HR of 95 beats per minute, BP of 130/85 mm Hg, and temperature of 101.3°F. The abdominal CT demonstrates the presence of sigmoid and cecal diverticula, inflammation of pericolic fat, thickening of the bowel wall, and a fluid-filled appendix. Which of the following is the most appropriate next step in management? a. Discharge the patient with broad-spectrum oral antibiotics and surgical follow-up. b. Begin IV hydration and broad-spectrum antibiotics, keep the patient npo (nothing by mouth), and admit the patient to the hospital. c. Begin IV antibiotics and call a surgical consult for an emergent operative procedure. d. Arrange for an emergent barium enema to confirm the diagnosis. e. Begin sulfasalazine 3 to 4 g/d along with IV steroid therapy.

*B* The diagnosis of diverticulitis is made by abdominal CT, which may demonstrate inflammation of pericolic fat, bowel wall thickening, the presence of diverticula, or peridiverticular abscess. The treatment of diverticulitis includes IV hydration, bowel rest, and broad-spectrum antibiotics to cover both aerobic and anaerobic bacteria. These typically include a combination of metronidazole and ciprofloxacin or levofloxacin. Well-appearing patients can be treated as outpatients with oral antibiotics and close follow-up, but patients with fever, signs of localized peritonitis or obstruction, and those who have failed outpatient therapy must be admitted to the hospital.

A 23-year-old woman presents to the ED with RLQ pain for the last 1 to 2 days. The pain is associated with nausea, vomiting, diarrhea, anorexia, and a fever of 100.9°F. She also reports dysuria. The patient returned 1 month ago from a trip to Mexico. She is sexually active with one partner but does not use contraception. She denies vaginal bleeding or discharge. Her last menstrual period was approximately 1 month ago. She has a history of pyelonephritis. Based on the principles of emergency medicine, what are the three priority considerations in the diagnosis of this patient? a. Perihepatitis, gastroenteritis, cystitis b. Ectopic pregnancy, appendicitis, pyelonephritis c. Pelvic inflammatory disease (PID), gastroenteritis, cystitis d. Ectopic pregnancy, PID, menstrual cramps e. Gastroenteritis, amebic dysentery, menstrual cramps

*B* The emergency physician (EP) approaches a problem by considering the most serious disease consistent with the patient complaint and working to exclude it. Thinking of the worst first is a reversal from the sequence of patient management in many other specialties. Once the EP rules out the life-threatening processes, more benign processes may be considered. This principle is particularly important when placed in the context of the patient population seen in the ED. Most of the patients are new to the EP; many are intoxicated or are brought to the ED by others. This leads to an array of fragmented histories, masked physical findings, and high emotional levels. In this setting, it is even more important for the EP to maintain a high level of suspicion for serious diseases.

A 41-year-old man who was a restrained front seat passenger in a highspeed motor vehicle collision is brought to the ED by the paramedics. His vital signs are BP 90/50 mm Hg, HR 125 beats per minute, and RR 20 breaths per minute. On examination, he is alert and answers your questions. His breath sounds are equal bilaterally and chest wall is without contusion or bony crepitus. The abdomen is soft and nondistended. His pelvis is unstable. A FAST examination and DPL are negative for intraperitoneal fluid. Initial radiographs reveal a normal chest film and an open book pelvic fracture. Despite infusion of 2 L of lactated ringer solution, his BP is now 80/40 mm Hg. Which of the following is the most appropriate next step in management? a. Immediate exploratory laparotomy b. Pelvic angiography with embolization of the pelvic vessels c. CT scan d. Pericardiocentesis e. Retrograde urethrogram

*B* The evaluation of patients with pelvic fractures begins with the primary trauma survey (ABCs). Fractured pelvic bones bleed briskly and can lacerate surrounding soft tissues and disrupt their extensive arterial and venous networks. Hemorrhage is a common cause of death in patients with pelvic injuries. Bleeding in the retroperitoneum can accommodate up to 4 L of blood. Most pelvic bleeding is from the fractures and low-pressure sacral venous plexus. In the ED, a stabilizer can be applied to the pelvis to help control the hemorrhage. Once an abdominal source of bleeding is ruled out as a source of hypotension, the patient should undergo pelvic angiography with embolization of bleeding vessels. The patient should also be resuscitated with packed RBCs until the bleeding is controlled.

A 49-year-old man is brought to the ED by EMS stating that he vomited approximately three cups of blood over the last 2 hours. He also complains of epigastric pain. While examining the patient, he has another episode of hematemesis. You decide to place a NG tube. You insert the tube, confirm its placement, and attach it to suction. You retrieve 200 mL of coffee-ground blood. What is the most common etiology of an upper GI bleed? a. Varices b. Peptic ulcer c. Gastric erosions d. Mallory-Weiss tear e. Esophagitis

*B* The most common cause of upper GI bleeds in adults is peptic ulcer disease accounting for approximately 45% of the cases. Hematemesis is the presentation in approximately 50% of patients with upper GI bleeds. The appearance of coffee-grounds in the stomach is caused by the conversion of hemoglobin to hematin or other pigments by hydrochloric acid in the stomach.

A 22-year-old college football player presents to the ED with knee pain. He states that during the football game today, he received a direct blow to the lateral aspect of his knee. Radiograph of the knee is negative for fracture, but shows an effusion. On examination, you confirm the hemarthrosis. As you examine his knee ligaments, you note joint instability with valgus stress in 30 degrees of flexion. The rest of your examination is unremarkable. This patient's injury is most consistent with? a. Rupture of the LCL b. Rupture of the MCL c. Rupture of the ACL d. Rupture of the PCL e. Tear of the medial meniscus

*B* The most common mechanism of injury resulting in ligamentous injury is a valgus stress with an external rotary component on the flexed knee. This is common in skiing and football injuries. The MCL is commonly injured. On stability testing of the knee, patients with MCL tears usually show instability with a valgus stress with the knee in 30 degrees of flexion. Ligamentous tears are also associated with hemarthrosis.

An 82-year-old nursing home patient presents to the ED in septic shock. Her BP is 75/40 mm Hg, HR is 117 beats per minute, temperature is 96.5°F, RR is 29 breaths per minute, and oxygen saturation is 87% on room air. As you perform laryngoscopy to intubate the patient, you easily visualize the vocal cords and subsequently pass the orotracheal tube through the vocal cords. You place the colorimetric end-tidal carbon dioxide device over the tube and get appropriate color change. There are equal, bilateral breath sounds on auscultation and you observe chest wall motion with ventilation. Which of the following is the most reliable method for verifying proper ET tube placement? a. Chest radiograph b. Visualization of the ET tube passing through the vocal cords c. Observation of chest wall motion with ventilation d. Hearing equal, bilateral breath sounds on auscultation e. End-tidal carbon dioxide color change

*B* The most serious complication of ET intubation is unrecognized esophageal intubation with resultant hypoxic brain injury. Esophageal placement is not always obvious. The best assurance that the tube is placed into the trachea is to see it pass through the vocal cords.

A 40-year-old woman is brought to the ED by the paramedics complaining of bilateral foot weakness and numbness that started a few hours ago and is progressively worsening. She denies similar episodes in the past. On the review of systems, she describes having abdominal cramps with nausea, vomiting, and diarrhea 2 weeks ago that resolved after 2 to 3 days. Her BP is 124/67 mm Hg, HR is 68 beats per minute, temperature is 98.8°F, and RR is 12 breaths per minute. On examination, you elicit 2/5 strength, decreased sensation, and loss of deep tendon reflexes in the lower extremities below the hips. Which of the following is the most likely diagnosis? a. Hypokalemic periodic paralysis b. Guillain-Barré syndrome c. Peripheral vascular disease d. Tetanus e. Brain abscess

*B* The patient has a progressive ascending peripheral neuropathy, also known as Guillain-Barré syndrome. Patients can usually remember a preceding viral illness, usually gastroenteritis. Deep tendon reflexes are typically absent.

An unconscious 51-year-old woman is brought to the ED by EMS. A friend states that the patient was complaining of feeling weak. She vomited and subsequently "blacked out" in the ambulance. The friend states that the patient has no medical problems and takes no medications. She also states that the patient smokes cigarettes and uses cocaine, and that they were snorting cocaine together prior to her blacking out. The patient's BP is 195/80 mm Hg, HR is 50 beats per minute, temperature is 98.6°F, and RR is 7 breaths per minute. What is the eponym associated with her vital signs? a. Cushing syndrome b. Cushing reflex c. Cullen sign d. Charcot triad e. Chvostek sign

*B* The patient has a triad of hypertension, bradycardia, and respiratory depression, which is called Cushing reflex. This is observed in one-third of patients with a potentially lethal increase in ICP. Increased ICP may result from traumatic brain injury or, as in this patient's case, from hemorrhagic stroke and subsequent brain edema. Tobacco and cocaine use are known risk factors for hemorrhagic stroke. Increasing ICP can result in cerebral herniation, which has a mortality rate close to 100%. For any chance of survival, it must be rapidly controlled by intubation, elevation of the head of the bed, hyperventilation, mannitol, and definitive neurosurgical intervention.

An 18-year-old girl is brought to the ED from a party for agitation and attacking her boyfriend with a knife. Her boyfriend admits that she had several liquor shots and used intranasal cocaine at the party prior to becoming agitated, paranoid, and attacking him. Her BP is 145/80 mm Hg, HR is 126 beats per minute, temperature is 100.8°F, and RR is 20 breaths per minute. The patient is agitated, screaming, and resisting examination. What is the next best step in the management of this patient? a. IV β-blocker b. IV benzodiazepine c. Acetaminophen d. Lithium e. Drug abuse specialist consult

*B* The patient has cocaine induced autonomic and CNS hyperactivity causing agitation, paranoia, and a hyperadrenergic state. More severe CNS manifestations of cocaine poisoning include hyperthermia, intracranial hemorrhage, seizures, spinal cord infarctions, and acute dystonic reactions. The CNS effects of cocaine are managed with benzodiazepines, which decrease sympathetic tone and prevent hyperthermia and seizures.

A 59-year-old man is brought into the ED accompanied by his son who states that his father is acting irritable and occasionally confused. The son states that his father has a history of hepatitis from a transfusion he received many years ago. Over the past 5 years, his liver function slowly deteriorated. His vital signs include BP of 145/80 mm Hg, a HR of 78 beats per minute, RR of 16 breaths per minute, oxygen saturation of 98%, and temperature of 98°F. Laboratory results are all within normal limits, except for an ammonia level that is significantly elevated. Which of the following is the best therapy? a. Vancomycin and gentamycin b. Lactulose and neomycin c. Ampicillin and gentamycin d. Levofloxacin e. Ciprofloxacin

*B* The patient has hepatic encephalopathy, which is a clinical state of disordered cerebral function occurring secondary to acute or chronic liver disease. Laboratory tests may be normal in patients, but the serum ammonia level is usually elevated. Lactulose and neomycin represent the main therapeutic agents. Lactulose is a poorly absorbed sugar metabolized by colonic bacteria that traps ammonia and helps excrete it in the stool. Neomycin is a poorly absorbed aminoglycoside that is believed to act by reducing colonic bacteria that are responsible for producing ammonia.

A 57-year-old man with a past medical history of hypertension and migraines presents to the ED complaining of a headache that started 2 days ago. He states the headache began suddenly with peak intensity while he was defecating. The pain is continuous particularly in the occipital region and is associated with mild nuchal rigidity and mild photophobia. He denies having a recent fever. A noncontrast head CT is obtained and is normal. Which of the following is the most appropriate next step in management? a. Administer metoclopramide for nausea relief and ketorolac for pain control b. LP c. Empiric treatment for meningitis with IV antibiotics d. IV mannitol to lower intracranial pressure (ICP) e. Angiography to evaluate for an aneurysm

*B* The patient presents with a clinical history that is consistent with a SAH. Brain CT without contrast is the procedure of choice for diagnosing SAH and should be done in any individual with a new onset of a severe or persistent headache. It has a sensitivity of 95% for detecting SAH. If the CT is negative, an LP should be performed because some patients with SAH have a normal CT scan. A yellow supernatant liquid (xanthochromia), obtained by centrifuging a bloody CSF sample, can help distinguish SAH from a traumatic tap. If the diagnosis is still in question, an angiography may be required

A 51-year-old woman presents to the ED after 5 consecutive days of crushing substernal chest pressure that woke her up from sleep in the morning. The pain resolves spontaneously after 20 to 30 minutes. She is an avid rock climber and jogs 5 miles daily. She has never smoked cigarettes and has no family history of coronary disease. In the ED, she experiences another episode of chest pain. An ECG reveals ST-segment elevations and cardiac biomarkers are negative. The pain is relieved with sublingual nitroglycerin. She is admitted to the hospital and diagnostic testing reveals minimal coronary atherosclerotic disease. Which of the following is the most appropriate medication to treat this patient's condition? a. Aspirin b. Calcium channel blocker (CCB) c. β-Blocker d. H2-Blocker e. Antidepressant

*B* The patient's clinical presentation is consistent with Prinzmetal or variant angina. This condition is caused by focal coronary artery vasospasm. It occurs at rest and exhibits a circadian pattern, with most episodes occurring in the early hours of the morning. The pain commonly is severe. Distinguishing unstable angina related to coronary atherosclerosis from variant angina may be difficult and requires special investigations, including coronary angiography. Patients may also exhibit ST elevations on their ECGs. Nitrates and CCBs are the mainstays of medical therapy for variant angina. Nitroglycerin effectively treats episodes of angina and myocardial ischemia within minutes of administration, and the long-acting nitrate preparations reduce the frequency of recurrent events. CCBs effectively prevent coronary vasospasm and variant angina, and they should be administered in lieu of β-blockers.

78-year-old man presents to triage of your ED complaining of gradual onset of headache over the course of the day. The headache is present almost everyday and wonders if it is related to the unusually cold temperatures this winter. He describes the headache as bounding and constant. You notice that his face is very ruddy in appearance. He is afebrile but looks rather lethargic and seems somewhat short of breath. He is afebrile and saturating 100% on pulse oximetry. One of your coworkers informs you that the patient's wife is in another part of the emergency room with a similar presentation. Two more ambulances arrive, one with a patient complaining of a similar headache and another with a patient who is obtunded. All live in the same building. What is the next most appropriate step in the management of these patients? a. Place the patient in respiratory isolation for presumed Neisseria meningitidis infection. b. Draw a blood gas and send it for cooximetry. c. Start antibiotic therapy and perform an LP. d. Treatment for migraine with triptans or IV antiemetics. e. Transfer of all three patients to the nearest hyperbaric facility.

*B* The patients are experiencing symptoms of carbon monoxide (CO) poisoning. CO is colorless and odorless. Patients often present with mild nonspecific symptoms including headache, malaise, and fatigue. Severe toxicity manifests as neurologic and cardiac toxicity. Severe cases may manifest as disseminated intravascular coagulation, circulatory shock, multiorgan failure, ischemic cardiac disease, renal failure, or noncardiogenic pulmonary edema. Although there is decreased blood oxygen content, patients will not exhibit cyanosis as there is not enough deoxyhemoglobin present to cause it. Common sources of CO include fossil fuel—burning engines, fumes from coal- or gas-burning stoves, and smoke from accidental fires. CO intoxication is more prevalent during the winter when potentially faulty heating systems are in use or when patients attempt to supplement their home heat using their oven. Initial therapy is aimed at increasing arterial oxygen content by providing supplemental oxygen. Mild intoxication can be managed with supplementary oxygen alone. Elevated carboxyhemoglobin levels require treatment with hyperbaric oxygen.

A 22-year-old college volleyball player presents to the ED complaining of left shoulder pain that began while attempting a serve during a volleyball match. She states that this has happened to her before. On examination, the left shoulder looks "squared-off." She complains of severe pain when she tries to adduct or internally rotate the shoulder. A radiograph is shows anterior dislocation of the left shoulder. What is the most common fracture associated with the patient's diagnosis? a. Bankart fracture b. Hill-Sachs deformity c. Clavicular fracture d. Coronoid fracture e. Scapular fracture

*B* The radiograph confirms an anterior dislocation of the left shoulder. Patient's typically present in severe pain with the dislocated arm held in slight abduction and external rotation by the opposite extremity. The patient leans away from the injured side and cannot adduct or internally rotate the shoulder without severe pain. Associated fractures may occur in up to 50% of anterior dislocations. The most common of these is a compression fracture of the humeral head, known as a Hill-Sachs deformity.

A 24-year-old man presents to the ED complaining of right wrist pain that began after he slipped and fell and landed on his outstretched hand. You examine the hand and wrist, and note no abnormalities except for snuffbox tenderness. A radiograph does not reveal a fracture. Which of the following is the most appropriate next step in management? a. Place an ACE wrap around the hand and wrist until the pain resolves. b. Immobilize the wrist in a thumb spica splint and have the patient follow-up with an orthopedist for repeat radiographs in 10 to 14 days. c. Continue to ice the wrist for 24 to 48 hours. d. Order a CT scan to evaluate for an occult fracture. e. Place the wrist and arm in a cast for 6 weeks.

*B* The scaphoid is the most commonly fractured carpal bone. It is typically seen in patients in their twenties to thirties after a FOOSH. Classically, physical examination reveals tenderness in the anatomic snuffbox, the space between the extensor pollicis longus and the extensor pollicis brevis. On radiography, however, up to 15% of scaphoid fractures are not detected. As the necrotic bone at the fracture site is resorbed, the fracture line often becomes apparent on radiographs at 10 to 14 days after injury. Therefore, patients with snuffbox tenderness and an initial radiograph should be splinted in a thumb spica splint and asked to return for repeat radiographs in 10 to 14 days.

A 71-year-old man is found lying on the ground one story below the balcony of his apartment. Paramedics bring the patient into the ED. He is cool to touch with a core body temperature of 96°F. His HR is 119 beats per minute and BP is 90/70 mm Hg. His eyes are closed but they open when you call his name. His limbs move to stimuli and he answers your questions but is confused. On examination, you note clear fluid dripping from his left ear canal and an area of ecchymosis around the mastoid bone. Which of the following is the most likely diagnosis? a. LeFort fracture b. Basilar skull fracture c. Otitis interna d. Otitis externa e. Tripod fracture

*B* The skull base comprises the floors of the anterior, middle, and posterior cranial fossae. Fractures in this region typically do not have localized symptoms. However, indirect signs of injury may include visible evidence of bleeding from the fracture into surrounding soft tissue. Ecchymosis around the mastoid bone is often described as Battle sign and periorbital ecchymosis is often described as "raccoon eyes." The most common basilar skull fracture involves the petrous portion of the temporal bone, the external auditory canal, and the tympanic membrane. It is commonly associated with a torn dura leading to cerebrospinal fluid (CSF) otorrhea or rhinorrhea. Other signs and symptoms of a basilar skull fracture include hemotympanum (eg, blood in the tympanic cavity of the middle ear), vertigo, decreased hearing or deafness, and seventh nerve palsy. Periorbital and mastoid ecchymosis develop gradually over hours after an injury and are often absent in the ED. If clear or pink fluid is seen from the nose or ear and a CSF leak is suspected, the fluid can be placed on filter paper and a "halo" or double ring may appear. This is a simple but nonsensitive test to confirm a CSF leak. Evidence of open communication, such as a CSF leak, mandates neurosurgical consultation and admission.

A 47-year-old man is brought to the ED by EMS after being found wandering in the street mumbling. His BP is 150/75 mm Hg, HR is 110 beats per minute, temperature is 100.5°F, RR is 16 breaths per minute, oxygen saturation is 99% on room air, and fingerstick glucose is 98 mg/dL. On examination, the patient is confused with mumbling speech. His pupils are dilated and face is flushed. His mucous membranes and skin are dry. Which of the following toxic syndromes is this patient exhibiting? a. Sympathomimetic syndrome b. Anticholinergic syndrome c. Cholinergic syndrome d. Opioid syndrome e. Ethanol syndrome

*B* The term toxidrome refers to a constellation of physical findings that can provide important clues in a toxic ingestion. This is particularly useful in patients that cannot provide an adequate history. The anticholinergic syndrome typically presents with delirium, mumbling speech, tachycardia, elevated temperature, flushed face, dry mucous membranes and skin, dilated pupils, and hypoactive bowel sounds. The anticholinergic syndrome can be remembered by the phrase "blind as a bat (mydriasis), red as a beet (flushed skin), hot as a hare (hyperthermia secondary to lack of sweating), dry as a bone (dry mucous membranes), and mad as a hatter (mental status changes)."

A 22-year-old man is brought to the ED 20 minutes after a head-on motor vehicle collision in which he was the unrestrained driver. On arrival, he is alert and coherent but appears short of breath. His HR is 117 beats per minute, BP is 80/60 mm Hg, and oxygen saturation is 97% on a nonrebreather. Examination reveals bruising over the central portion of his chest. His neck veins are not distended. Breath sounds are present on the left, but absent on the right. Following administration of 2 L of lactated ringer solution, his systolic BP remains at 80 mm Hg. Which of the following is the most appropriate next step in management? a. Sedate, paralyze, and intubate b. Perform a needle thoracostomy c. Perform a DPL d. Perform a FAST examination e. Perform a pericardiocentesis

*B* The treatment of a tension PTX involves immediate reduction in the intrapleural pressure on the affected side of the thoracic cavity. The simplest and quickest way to establish this is by inserting a 14-gauge catheter into the thoracic cavity in the second intercostal space in the midclavicular line. After this procedure, a chest tube should be inserted as definitive management. Needle thoracostomy is necessary when a patient's vital signs are unstable; otherwise, direct insertion of a chest tube is adequate for suspicion of a hemo- or pneumothorax. A tension PTX is a life-threatening emergency caused by air entering the pleural space that is not able to escape secondary to the creation of a one way valve. This increased pressure causes the ipsilateral lung to collapse, shifting the mediastinum away from the injured lung, compromising vena caval blood return to the heart. The severely altered preload results in reduced stroke volume, increased cardiac output, and hypotension.

A 29-year-old man is 10 ft up on a ladder, painting his barn ceiling when he loses his balance and falls off the ladder, landing on both of his feet and then falling to the ground. He is brought to the ED by emergency medical services (EMS). On examination there are no obvious deformities, but you note swelling, tenderness, and ecchymosis of the right hindfoot. He has full range of motion at the ankle, knee, and hip. There is mild tenderness over his lumbar spine. Hs neurovascular status is normal. Radiographs confirm the diagnosis. Which of the following statements regarding this type of injury is true? a. This injury is the second most commonly fractured tarsal bone. b. 10% are associated with compression fractures of the dorsolumbar spine. c. 90% are bilateral. d. The mechanism of injury is usually secondary to rotational force at the subtalar joint. e. These fractures require operative repair to avoid gait defects

*B* This individual sustained a calcaneal fracture of the right foot. The calcaneus is the most commonly fractured tarsal bone (a); the talus is the second most commonly injured tarsal bone. A calcaneal fracture is usually caused by a compression injury (d), such as a fall from a height with the patient landing on his or her feet. The examination reveals swelling, tenderness, and ecchymosis of the hindfoot and the inability to bear weight on the fracture. 10% are bilateral (c) and 10% are associated with compression fractures of the dorsolumbar spine. Therefore, it is important to examine the patient's entire spine. Treatment varies depending on the extent of injury. In general, nondisplaced or minor extra-articular fractures only require supportive care with immobilization in a posterior splint and follow-up with an orthopedic surgeon

A 45-year-old man is brought to the ED by his coworkers after collapsing to the floor while at work. A coworker states that the patient mistakenly took several tablets of his oral diabetic medications a few hours ago. The patient is unresponsive and diaphoretic. His BP is 142/78 mm Hg, HR is 115 beats per minute, temperature is 98.9°F, and RR is 12 breaths per minute. A bedside glucose reads 42 mg/dL. Which of the following is the most appropriate management of this patient? a. Administer IV dextrose, obtain a repeat fingerstick glucose every hour and if normal after 6 hours discharge the patient home. b. Administer IV dextrose and continue monitoring his blood sugar for at least 24 hours. c. Administer IV fluids and insulin. d. Administer IV fluids. e. Administer activated charcoal and IV fluids.

*B* This is a case of hypoglycemia as a result of oral hypoglycemic medication overdose. Diabetic oral agents that cause hypoglycemia work by increasing pancreatic insulin secretion. This group includes sulfonylureas (glyburide, glipizide) and nonsulfonylurea secretagogues (repaglinide, nateglinide). Other common causes of hypoglycemia are insulin overdose, alcohol abuse (inhibition of gluconeogenesis) and sepsis. The presentation of a hypoglycemic patient generally involves signs and symptoms of CNS dysfunction owing to the release of counterregulatory hormones secondary to the unavailability of glucose. Symptoms include anxiety, diaphoresis, palpitations, and confusion. Don't be fooled by improved blood glucose levels after dextrose administration in overdose with oral hypoglycemic agents. Hypoglycemia can last more than 24 hours because of long-lasting pancreatic effects and will recur after dextrose infusion. Patients need to be observed in the hospital with frequent bedside glucose checks. They can be placed on a dextrose drip. Octreotide, an inhibitor of insulin release can also be administered.

You are caring for a 54-year-old woman with a history of schizophrenia and coronary artery disease who presents to the ED for chest pain. Her vital signs are within normal limits and her ECG is normal sinus rhythm with nonspecific ST/T wave changes. Her first troponin is sent to the laboratory and you are planning to admit her to the hospital for a complete acute coronary syndrome (ACS) evaluation. She receives aspirin and nitroglycerin and her chest pain resolves. A few minutes later, the nurse alerts you that the patient has become unconscious. You go to the bedside and find the patient awake and alert. You review the rhythm strip showing Torsades. What is your next step in management? a. Observe patient b. Magnesium sulfate IV c. Lidocaine IV d. Transvenous pacemaker e. Isoproterenol IV

*B* This patient had a run of torsades de pointes, an atypical VT where the QRS axis swings from positive to negative within a single ECG lead. This dysrhythmia is frequently seen in patients with significant heart disease who have a prolonged QT interval. There are many possible causes of prolonged QT; however, common etiologies include drugs (eg, antidysrhythmics, psychotropics), electrolyte abnormalities, and coronary heart disease. This patient was likely on a phenothiazine for her schizophrenia leading to prolonged QT syndrome and an episode of torsades de pointes. Administration of magnesium sulfate has been shown to decrease runs of torsades.

A 62-year-old man with a history of hypertension presents to the ED with severe constant mid-epigastric pain for the past hour. Over the last several months, he has had intermittent pain shortly after eating, but never this severe. He states he now has generalized abdominal pain that began suddenly about 15 minutes ago. He has no history of trauma, has never had surgery, and takes no medications. His vitals include a HR of 115 beats per minute lying supine, increasing to 135 when sitting up, BP of 170/105 mm Hg supine, falling to 145/85 mm Hg when sitting up. He appears pale. His abdomen is rigid and diffusely tender with guarding and rebound. Bowel sounds are absent and stool hemoccult is positive. The white blood cell (WBC) count is 8500/μL, hemoglobin 8.5 mg/dL, hematocrit 27%, and platelets 255/μL. Which of the following is the most likely diagnosis? a. Boerhaave syndrome b. Perforated gastric ulcer c. Abdominal aortic aneurysm (AAA) d. Inflammatory bowel disease (IBD) e. Diverticulosis

*B* This patient had an untreated gastric ulcer that just perforated. The history of epigastric pain related to eating points to a gastric ulcer, whereas pain 2 to 3 hours after eating is more likely caused by a duodenal ulcer. The sudden onset of generalized abdominal pain associated with a rigid abdomen is concerning for a perforated viscus, in this case, a perforated gastric ulcer. This is a surgical emergency. An abdominal and upright chest radiograph can be performed quickly to look for free air, which will be seen under the diaphragm on the chest radiograph. This is useful for the majority of perforations, which are anterior, but may miss posterior perforations because the posterior duodenum is retroperitoneal. The treatment includes IV hydration, antibiotics, and immediate surgical correction.

Paramedics bring in a 54-year-old man who was found down in his apartment by his wife. He is successfully intubated in the field and paramedics are currently performing cardiopulmonary resuscitation (CPR). He is transferred to an ED gurney and quickly attached to the cardiac monitors. You ask the paramedics to hold CPR and assess the patient and the rhythm strip. The monitor shows sinus bradycardia, but no pulses are palpable. On examination you appreciate bilateral breath sounds with mechanical ventilation, a soft abdomen, no rashes, and a left arm AV graft. In addition to CPR with epinephrine and atropine every 3 to 5 minutes, which intervention should be performed next? a. Administer 1 ampule of sodium bicarbonate b. Administer 1 ampule of calcium gluconate c. Administer 1 ampule of D50 (dextrose) d. Place left-sided chest tube e. Perform pericardiocentesis

*B* This patient has cardiac electrical activity (sinus bradycardia), but no detectable pulses. He is therefore in a state of pulseless electrical activity (PEA) and management should be directed by the AHA PEA algorithm. Patients in PEA should be treated with CPR, epinephrine every 3 to 5 minutes, and atropine every 3 to 5 minutes (if PEA rate is less than 60 per minute), but a search for an underlying etiology with targeted interventions should still be performed.

A 50-year-old man with a history of hypertension presents to the ED with severe left-sided chest pain for 1 hour. The pain radiates down his left arm and he feels nauseated. His temperature is 98.3°F, BP is 160/92 mm Hg, HR is 92 beats per minute, RR is 16 breaths per minute, and oxygen saturation is 98% on room air. The physical examination is normal. His ECG shows ST-segment elevations in leads II, III, and aVF. You administer aspirin, nitroglycerin, and morphine sulfate and wait for his laboratory results. The nurse calls you over 10 minutes later and tells you that the patient's BP dropped to 60/30 mm Hg with a HR of 100 beats per minute. Which of the following most likely explains his hypotension? a. Medication-related adverse reaction b. Cardiogenic shock c. Papillary muscle rupture d. Free wall rupture e. Rupture of the interventricular septum

*B* This patient is in cardiogenic shock from a right ventricular infarction. The ECG shows STsegment elevation in leads II, III, and aVF indicating the patient is having an inferior wall MI. Approximately 30% of inferior MIs involve the RV. A right-sided ECG would confirm the diagnosis. RV infarction results in reduced right ventricular end-systolic pressure, ultimately leading to decreased cardiac output. Patients with RV infarctions are therefore dependent on preload to maintain cardiac output. In this case, nitroglycerin, a powerful preload reducer, pushed this patient into cardiogenic shock. Patients with RV infarction and hypotension should be treated with IV normal saline to support preload. Dobutamine may also be necessary to support the BP. Furthermore, this patient is having an acute ST-segment elevation MI and may be a candidate for percutaneous coronary intervention or fibrinolytics.

A 25-year-old man fell off his surfboard and landed on rocks. He was pulled from the water by lifeguards and brought to the ED in full cervical and spinal immobilization. He is alert and oriented to person, place, and time. He is complaining of weakness in all of his extremities. His temperature is 98.4°F, BP is 85/50 mm Hg, HR is 60 beats per minute, RR is 20 breaths per minute, and oxygen saturation is 98% on room air. On examination, he has no external signs of head injury. His heart is bradycardic without murmurs. The lungs are clear to auscultation and the abdomen is soft and nontender. He has grossly normal peripheral sensation, but no motor strength in all four extremities. Which of the following is the most likely diagnosis? a. Hypovolemic shock b. Neurogenic shock c. Cardiogenic shock d. Anaphylactic shock e. Septic shock

*B* This patient is in neurogenic shock. He suffered an acute cervical spine injury after his fall onto rocks and has hypotension and bradycardia. The pathophysiology behind neurogenic shock is still under investigation but it's thought to be partially caused by disrupted sympathetic outflow tracts and unopposed vagal tone. Note that all other forms of shock attempt to compensate for hypotension with tachycardia. Neurogenic shock lacks sympathetic innervation; therefore, bradycardia results. Given that this is a trauma patient, all other sources for hypotension must be ruled out. He should be treated with cervical spine immobilization and IV fluids. Pressors may be needed if hypotension does not respond to fluids or fluid overload becomes a concern.

A 54-year-old undomiciled woman presents to the ED with severe cough, general malaise, and subjective fevers for the last week. She also describes coughing up "chicken livers" during this time and reports that her symptoms are getting progressively worse. Her initial vitals include a HR of 100 beats per minute, a BP of 145/66 mm Hg, temperature of 99.9°F and an RR of 16 breaths per minute with an oxygen saturation of 95% on room air. She states that she has a history of alcohol abuse, but denies taking any medications or illicit drugs. A chest radiograph shows a lobar pneumonia. Given this patient's clinical presentation, which of the following is this patient at most risk for contracting? a. Streptococcus pneumoniae b. Klebsiella pneumoniae c. Mycoplasma pneumoniae d. Legionella pneumophila e. Haemophilus influenzae

*B* Undomiciled, alcoholic patients are at particular risk for contracting K pneumoniae. Classically, it presents with a productive cough with currant jelly sputum, fever, general malaise, and an overall toxic appearance. A dense lobar infiltrate with a bulging fissure appearance on a chest radiograph is often described.

You are a passenger aboard an airplane and a 78-year-old woman is complaining of chest pain and difficulty breathing. You are the only medical professional available and volunteer to help. Fortunately, the aircraft is well-equipped with basic medical equipment, as well as with advanced cardiac life support (ACLS) medications and a cardiac monitor. On examination, the passenger's BP is 75/40 mm Hg, HR is 180 beats per minute, and RR is 24 breaths per minute. On examination, the patient is in obvious distress, but able to answer basic questions. Her heart is tachycardic, regular, and without murmurs, rubs, or gallops. Physical examination is remarkable for a bounding carotid pulse. You attach the cardiac monitor and see a regular rhythm at 180 beats per minute with wide QRS complexes and no obvious P waves. After asking the pilot to make an emergency landing what do you do next? a. Amiodarone IV b. Synchronized cardioversion c. Verapamil IV d. Lidocaine IV e. Procainamide IV

*B* Ventricular tachycardia (VT) originates from ectopic ventricular pacemakers and is usually a regular rhythm with rate greater than 100 beats per minute and wide QRS complexes. Treatment of VT is primarily dependent on whether or not the patient is stable. Evidence of acute altered mental status, hypotension, continued chest pain, or other signs of shock are signs of instability. Unstable patients, such as the passenger on this airplane, should receive immediate synchronized cardioversion. It is critical that the cardioverter be placed in the synchronized mode, which permits a search for a large R wave and a corresponding shock around the incidence of such a wave. A shock administered outside of this constraint can induce ventricular fibrillation (VF).

A husband and wife present to the ED with 1 day of subjective fever, vomiting, watery diarrhea, and abdominal cramps. They were at a restaurant a day before for dinner and both ate the seafood special, which consisted of raw shellfish. In the ED, they are both tachycardic with temperatures of 99.8°F and 99.6°F for him and her, respectively. Which of the following is responsible for the majority of acute episodes of diarrhea? a. Parasites b. Viruses c. Enterotoxin-producing bacteria d. Anaerobic bacteria e. Invasive bacteria

*B* Viral diarrheal diseases are responsible for the majority of all acute episodes of diarrhea. Rotavirus, typically a disease of young children and Norwalk virus are the most frequent etiologic agents. In addition, enteric adenovirus is a common cause of gastroenteritis. Though dehydration is a common complication, these illnesses are usually self-limited, requiring only supportive care.

A 59-year-old man presents to the emergency department (ED) complaining of new onset chest pain that radiates to his left arm. He has a history of hypertension, hypercholesterolemia, and a 20-pack-year smoking history. His electrocardiogram (ECG) is remarkable for T-wave inversions in the lateral leads. Which of the following is the most appropriate next step in management? a. Give the patient two nitroglycerin tablets sublingually and observe if his chest pain resolves. b. Place the patient on a cardiac monitor, administer oxygen, and give aspirin. c. Call the cardiac catheterization laboratory for immediate percutaneous intervention (PCI). d. Order a chest x-ray; administer aspirin, clopidogrel, and heparin. e. Start a β-blocker immediately.

*B*. (Rosen, pp 1039-1044.) The patient's presentation is classic for an ACS. He has multiple risk factors with T-wave abnormalities on his ECG. The most appropriate initial management includes placing the patient on a cardiac monitor to detect dysrhythmias, establish intravenous access, provide supplemental oxygen, and administer aspirin. If the patient is having active chest pain in the ED, sublingual nitroglycerin or morphine should be administered until the pain resolves. This decreases wall tension and myocardial oxygen demand. A common mnemonic used is MONA(Morphine, Oxygen, Nitroglycerin, Aspirin) greets chest pain patients at the door. (a) Although nitroglycerin is one of the early agents used in ACS, it is prudent to first rule out a right ventricular infarct, which if present, may lead to hypotension. (c) PCI is warranted if the patient's ECG showed STsegment elevation. (d) The patient will require a chest x-ray and most likely receive clopidogrel and heparin; however this is done only afterbeing on a monitor with oxygen and chewing an aspirin. (e) β-Blockers are usually added for tachycardia, hypertension, and persistent pain and only given once the patient is evaluated for contraindications. Relative contraindications to the use of β-blockers include asthma or chronic obstructive lung disease, CHF, and third-trimester pregnancy.

A 29-year-old man is brought to the ED by EMS for a syncopal episode that occurred during a basketball game. A friend states that the patient just dropped to the ground shortly after scoring a basket on a fast-break. On examination, you note a prominent systolic ejection murmur along the left sternal border and at the apex. An ECG reveals left ventricular hypertrophy, left atrial enlargement, and septal Q waves. You suspect the diagnosis and ask the patient to perform the Valsalva maneuver while you auscultate his heart. Which of the following is most likely to occur to the intensity of the murmur with this maneuver? a. Decrease b. Increase c. Remain unchanged d. Disappear e. The intensity stays the same but the heart skips a beat

*B*The patient has hypertrophic cardiomyopathy, which is characterized by left ventricular hypertrophy without associated ventricular dilation. The hypertrophy is usually asymmetric, involving the septum to a greater extent than the free wall. Patients are at increased risk of dysrhythmias and sudden death. Syncope is usually exertion-related and is caused by a dysrhythmia or a sudden decrease in cardiac output. The murmur associated with hypertrophic cardiomyopathy is a prominent systolic ejection murmur heard along the left sternal boarder and at the apex with radiation to the axilla. The murmur is a result of LV outflow obstruction and mitral regurgitation. It is increased with maneuvers that decrease left ventricular end-diastolic volume, such as the Valsalva maneuver, sudden standing, and exercise.

A 72-year-old man is in the ED for the evaluation of generalized weakness over the previous 24 hours. He has a past medical history of coronary artery disease with a CABG performed 5 years ago, diabetes mellitus, and arthritis. The nurse places the patient on a cardiac monitor and begins to get his vital signs. While the nurse is obtaining the vital signs, he notices the patient suddenly becomes unresponsive. You arrive at the bedside, look at the monitor, and see the following rhythm showing vfib. Which of the following is the most appropriate next step in management? a. Wait 5 minutes to see if he awakens on his own. b. Immediately defibrillate at 200 J (biphasic). c. Perform synchronized cardioversion at 100 J. d. Immediately intubate the patient. e. Insert an IV line and administer amiodarone.

*B*The rhythm is VF. Along with pulseless VT, these are nonperfusing rhythms that are treated identically because it is thought to be caused by the same mechanisms. The earlier a "shock" is administered in cardiac arrest, the more likely the patient will return to spontaneous circulation with a perfusing rhythm. If there is a delay to defibrillation (> 4 minutes), CPR should be administered for 60 to 90 seconds before defibrillation. If after defibrillation (200 J biphasic or 360 J monophasic) the patient's rhythm is still VF or pulseless VT, then assisted ventilation and chest compressions should be started. Intubation should be performed and IV access obtained for the administration of epinephrine or vasopressin. If the rhythm is unchanged after administration of vasopressor therapy, then another attempt at defibrillation at 360 J (or 200 J biphasic) with subsequent administration of an antidysrhythmic (eg, amiodarone) is recommended. Of note, monophasic defibrillation delivers a charge in only one direction. Biphasic defibrillation delivers a charge in one direction for half of the shock and in the electrically opposite direction for the second half. Biphasic defibrillation significantly decreases the energy necessary for successful defibrillation and decreases the risk of myocardial damage.

A 62-year-old woman presents to the ED with general weakness, shortness of breath, and substernal chest pain that radiates to her left shoulder. Her BP is 155/80 mm Hg, HR is 92 beats per minute, and RR is 16 breaths per minute. You suspect that she is having an acute MI. Which of the following therapeutic agents has been shown to independently reduce mortality in the setting of an acute MI? a. Nitroglycerin b. Aspirin c. Unfractionated heparin d. Lidocaine e. Diltiazem

*B.* Aspirin is an antiplatelet agent that should be administered early to all patients suspected of having an ACS, unless there is a contraindication. The ISIS-2 trial provides the strongest evidence that aspirin independently reduces the mortality of patients with acute MI.

A 62-year-old man presents to the ED with gradual dyspnea over the last few weeks. He reports that he is a daily smoker and has not seen a physician in years. Upon physical examination, there are decreased breath sounds on the right as compared to the left. A chest radiograph indicates blunting of the right costophrenic angle with a fluid line. A thoracentesis is performed. Given this patient's history, which of the following most likely describes his effusion? a. Transudative effusion b. Exudative effusion c. Transudative and exudative effusion d. Lactate dehydrogenase < 200 U e. Fluid-to-blood protein ratio < 0.5

*B.* Given this patient's longstanding history of tobacco use and having not seen a doctor for annual examinations, it is likely that the pleural effusion is exudative as a result of an underlying malignancy. Other causes of exudative effusions include the following: infection, connective tissue diseases, neoplasm, pulmonary emboli, uremia, pancreatitis, esophageal rupture, postsurgical, trauma, and drug-induced. Pleural fluid analysis includes LDH, glucose, protein, amylase, cell count, Gram stain, culture, and cytology. Effusions that have a LDH > 200 U, fluid-to-blood LDH ratio > 0.6 and a fluid-to-blood protein ratio > 0.5 are classified as exudative. Levels less than these are classified as transudative.

A 26-year-old woman presents to the ED with an acute onset of dyspnea after falling down a few steps. The patient denies any loss of consciousness and reports feeling short of breath. Her initial chest x-ray appears normal; however she continues to be symptomatic with stable vital signs. Which of the following procedures should be performed next? a. Repeat upright chest x-ray b. Inspiratory and expiratory chest radiographs c. Chest CT scan d. Chest thoracostomy e. Chest thoracotomy

*B.* Inspiratory and expiratory radiographs allow better visualization of the lung pleura and may help better elucidate the presence of a pneumothorax not initially visualized on the chest radiograph.

59-year-old woman presents to the ED complaining of worsening lower abdominal pain over the previous 3 days. She describes feeling constipated recently and some burning when she urinates. Her BP is 135/75 mm Hg, HR is 89 beats per minute, temperature is 101.2°F, and her RR is 18 breaths per minute. Her abdomen is mildly distended, tender in the LLQ, and positive for rebound tenderness. CT scan is consistent with diverticulitis with a 7-cm abscess. Which of the following is the most appropriate management for this condition? a. Reserve the OR for emergent laparotomy. b. Start treatment with ciprofloxacin and metronidazole and plan for CT-guided draining of the abscess. c. Give an IV dose of ciprofloxacin and have the patient follow-up with her primary physician. d. Start treatment with ciprofloxacin and metronidazole and plan for an emergent barium enema. e. Start treatment with ciprofloxacin and metronidazole and prep for an emergent colonoscopy.

*B.* Management for complicated acute diverticulitis involves admission and antibiotic treatment. Treatment is directed against both anaerobic and gram-negative bacteria. Intraabdominal abscess formation secondary to diverticulitis requires prompt surgical consultation and should be drained using CT or ultrasound-guided percutaneous draining. Abscesses less than 5 cm in diameter may be treated with antibiotics alone.

A 54-year-old woman presents to the ED because of a change in behavior at home. For the past 3 years she has end-stage renal disease requiring dialysis. Her daughter states that the patient has been increasingly tired and occasionally confused for the past 3 days and has not been eating her usual diet. On examination, the patient is alert and oriented to person only. The remainder of her examination is normal. An initial 12-lead ECG shows peaked T waves. Which of the following electrolyte abnormalities best explains these findings? a. Hypokalemia b. Hyperkalemia c. Hypocalcemia d. Hypercalcemia e. Hyponatremia`

*B.* Patients with end-stage renal disease, who require dialysis, are prone to electrolyte disturbances. This patient's clinical picture is consistent with hyperkalemia. The ECG can provide valuable clues to the presence of hyperkalemia. As potassium levels rise, peaked T waves are the first characteristic manifestation. Further rises are associated with progressive ECG changes, including loss of P waves and widening of the QRS complex. Eventually the tracing assumes a sinewave pattern, followed by ventricular fibrillation or asystole

An 18-year-old woman presents to the ED complaining of acute onset of RLQ abdominal pain. She also describes the loss of appetite over the last 12 hours, but denies nausea and vomiting. Her BP is 124/77 mm Hg, HR is 110 beats per minute, temperature is 102.1°F, RR is 16 breaths per minute, and oxygen saturation is 100% on room air. Abdominal examination reveals lower abdominal tenderness bilaterally. On pelvic examination you elicit cervical motion tenderness and note cervical exudates. Her WBC is 20,500/μL and β-hCG is negative. Which of the following is the most appropriate next step in management? a. Bring her to the OR for an appendectomy b. Begin antibiotic therapy c. Perform a culdocentesis d. Bring her to the OR for immediate laparoscopy e. Order an abdominal plain film

*B.* Pelvic inflammatory disease (PID) comprises a spectrum of infections of the female upper reproductive tract. Although N gonorrhoeae and C trachomatis are thought to cause the majority of infections, new evidence points to greater rates of polymicrobial infections. Most cases of PID are thought to start with a sexually transmitted disease of the lower genital tract and ascend to the upper tract. Women typically present with lower abdominal pain and may have vaginal discharge, vaginal bleeding, dysuria, and fever. The examination usually reveals lower abdominal tenderness and cervical motion tenderness or adnexal tenderness. Many patients are treated as outpatients with antibiotics. Considerations for admission include those women who are pregnant, failed outpatient therapy, are toxic appearing, have evidence for a tubo-ovarian abscess, or a surgical emergency cannot be ruled out. Long-term outcomes are improved if antibiotics are begun immediately.

Paramedics bring a 45-year-old man to the ED after being involved in a high-speed motor vehicle collision. His BP is 85/50 mm Hg and HR is 131 beats per minute after administering 2 L of normal saline. He is awake but slow in responding to questions. A right upper quadrant (RUQ) ultrasound image shows fluid in the Morison pouch. Which of the following is the most appropriate next step in management? a. Emergent abdominal computed tomographic (CT) scan b. Transfer to the operating room (OR) for laparotomy c. Perform a diagnostic peritoneal lavage (DPL) d. Observe until one more liter of crystalloid fluid is administered e. Serial abdominal examinations

*B.* The image reveals fluid in the space between the right kidney and liver, known as Morison pouch. Free fluid is seen as an anechoic area (black in color). The FAST examination is replacing the DPL in cases of abdominal trauma. It is inexpensive, noninvasive, and confirms the presence of hemoperitoneum in minutes. The minimum amount of intraperitoneal fluid needed for detection by ultrasound is approximately 70 cc. Patients who remain unstable despite volume resuscitation and have intraperitoneal fluid demonstrated by the FAST examination need to be taken to the OR for a laparotomy to identify and treat the source of bleeding.

A 55-year-old man presents to the ED with chest pain and shortness of breath. His BP is 170/80 mm Hg, HR is 89 beats per minute, and oxygen saturation is 90% on room air. Physical examination reveals crackles midway up both lung fields and a new holosystolic murmur that is loudest at the apex and radiates to the left axilla. ECG reveals ST elevations in the inferior leads. Chest radiograph shows pulmonary edema with a normalsized cardiac silhouette. Which of the following is the most likely cause of the cardiac murmur? a. Critical aortic stenosis b. Papillary muscle rupture c. Pericardial effusion d. CHF e. Aortic dissection

*B.* The patient's presentation is consistent with acute mitral valve regurgitation because of a ruptured papillary muscle in the setting of an AMI. Patients usually present with pulmonary edema in the setting of an AMI. Chest x-ray characteristically reveals pulmonary edema with a normal heart size. The characteristic murmur of mitral regurgitation is a holosystolic murmur that is loudest at the apex.

A 73-year-old man who is a 1-pack-per-day smoker and has a medical history of hypertension and peripheral vascular disease presents to the ED complaining of mid-abdominal and right flank pain. He states that he had this same pain 1 week ago and that it got so bad that he passed out. His BP is 125/75 mm Hg, HR is 85 beats per minute, temperature is 98.7°F, and his RR is 17 breaths per minute. Physical examination reveals a bruit over his abdominal aorta and a pulsatile abdominal mass. Which of the following is the most appropriate initial test to evaluate this patient? a. Angiography b. Ultrasound c. MRI d. Plain radiograph e. D-dimer

*B.* The patient's presentation is worrisome for an AAA. If the patient was hemodynamically unstable, he should be brought immediately to the OR for definitive repair. However, in the stable patient, imaging studies can aid in the diagnosis of an AAA. Ultrasound is almost 100% sensitive in detecting AAA, it is noninvasive, and can be performed rapidly at the patient's bedside. If the entire abdominal aorta is visualized and found to be of normal diameter, it is safe to say that the patient's symptoms are not from an AAA. An alternative to ultrasound in the stable patient is a CT scan, which is essentially 100% accurate in determining the presence of an AAA. The CT scan is less subject to technical and interpretation errors than an ultrasound.

A 23-year-old woman presents to the ED complaining of lower abdominal pain and vaginal spotting for 2 days. Her menstrual cycle is irregular. She has a history of ovarian cysts and is sexually active but always uses condoms. Her BP is 115/75 mm Hg, HR is 75 beats per minute, temperature is 98.9°F, and RR is 16 breaths per minute. Which of the following tests should be obtained next? a. Chlamydia antigen test b. β-Human chorionic gonadotropin (β-hCG) c. Transvaginal ultrasound d. Abdominal radiograph e. Observe her abdominal pain, if it resolves discharge her with a diagnosis of menstruation

*B.* β-Human chronic gonadotropin (β-hCG) should be obtained in all women of child-bearing age who present with abdominal pain or vaginal bleeding. Diagnostically, it is one of the most important tests in female patients. A positive pregnancy test in the setting of abdominal pain and vaginal bleeding demands that the physician rule out an ectopic pregnancy. Even though the patient states she always uses condoms during intercourse, there is still a small risk of pregnancy.

A 31-year-old kindergarten teacher presents to the ED complaining of acute onset substernal chest pain that is sharp in nature and radiates to her back. The pain is worse when she is lying down on the stretcher and improves when she sits up. She smokes cigarettes occasionally and was told she has borderline diabetes. She denies any recent surgeries or long travel. Her BP is 145/85 mm Hg, HR is 99 beats per minute, RR is 18 breaths per minute, and temperature is 100.6°F. Examination of her chest reveals clear lungs and a friction rub. Her abdomen is soft and nontender to palpation. Her legs are not swollen. Chest radiography and echocardiography are unremarkable. Her ECG shows diffuse ST elevation w/ PR depression. Which of the following is the most appropriate next step in management? a. Anticoagulate and CT scan to evaluate for a PE b. Prescribe a NSAID and discharge the patient c. Aspirin, heparin, clopidogrel, and admit for ACS d. Administer thrombolytics if the pain persists e. Prescribe antibiotics and discharge the patient

*B.*The classic presentation of pericarditis includes chest pain, a pericardial friction rub, and ECG abnormalities. A prodrome of fever and myalgias may occur. Pericarditis chest pain is usually substernal and varies with respiration. It is classically sharp or pleuritic in character. It is typically relieved by sitting forward and worsened by lying down or swallowing. The physical examination hallmark of acute pericarditis is the pericardial friction rub. The earliest ECG changes are seen in the first few hours to days of illness and include diffuse ST-segment elevation seen in leads I, II, III, aVL, aVF, and V2 to V6. Most patients with acute pericarditis will have concurrent PR-segment depression. The mainstay of treatment includes supportive care with anti-inflammatory medications (eg, NSAIDs).

A 32-year-old man is brought to the ED by paramedics after a diving accident. The lifeguard on duty accompanies the patient and states that he dove head first into the shallow end of the pool and did not resurface. On examination, the patient is speaking but cannot move his arms or legs and cannot feel pain below his clavicle. He is able to feel light touch and position of his four extremities. A cervical spine radiograph does not reveal a fracture. Which of the following is the most likely diagnosis? a. Spinal cord injury without radiographic abnormality (SCIWORA) b. Central cord syndrome c. Anterior cord syndrome d. Cauda equina syndrome e. Brown-Séquard syndrome

*C* Anterior cord syndrome results from cervical flexion injuries (eg, diving in shallow water) that cause cord contusion or protrusion of a bony fragment or herniated intervertebral disk into the spinal canal. It may also occur from vascular pathology, such as laceration or thrombosis of the anterior spinal artery. The syndrome is characterized by different degrees of paralysis and loss of pain and temperature sensation below the level of injury. Its hallmark is the preservation of the posterior columns, maintaining position, touch, and vibratory sensation.

A 33-year-old woman presents to the ED with a painful sprained ankle. She has a past medical history of depression for which she is taking phenelzine, a monoamine oxidase inhibitor. After you place an elastic wrap on her ankle, she asks you to prescribe her some pain medication. Which of the following medications is contraindicated in patients taking a monoamine oxidase inhibitor? a. Ibuprofen b. Acetaminophen c. Meperidine d. Oxycodone e. Hydrocodone

*C* Any patient taking a monoamine oxidase inhibitor (MAOI) is at risk for developing the serotonin syndrome if the individual congests a selective serotonin reuptake inhibitor (SSRI) or another drug that raises CNS serotonin levels. Some medications that can cause this interaction include indirect-acting and mixed-acting sympathomimetics (eg, cocaine, amphetamine), antidepressants (eg, TCA, SSRI), meperidine (eg, Demerol), and dextromethorphan, which is found in many nonprescription antitussives. In contrast, morphine and its derivatives lack serotonin-potentiating effects. Serotonin syndrome is characterized by altered mental status, hyperthermia, neuromuscular dysfunction, and autonomic dysfunction. Symptoms may also include shivering, trismus, akathisia, coma, and seizures.

A 43-year-old undomiciled man is brought to the ED after being found intoxicated on the street. He is currently rousable and expresses a request to be left alone. Initial vitals include a HR of 92 beats per minute, a BP of 125/80 mm Hg, and an RR of 14 breaths per minute with an oxygen saturation of 93% on room air. His rectal temperature is 101.2°F. A chest radiograph shows infiltrates involving the right-lower lobe. Given this clinical presentation, what initial antibiotic coverage is most appropriate for this patient? a. Gram-negative coverage only b. Gram-positive coverage only c. Broad-spectrum with anaerobic coverage d. PCP coverage e. Antifungal therapy

*C* Aspiration pneumonia occurs secondary to the inhalation of either oropharyngeal or gastric contents into the lower airways. Aspiration of gastric juices may cause a pulmonary inflammatory response. This type of mechanism of acquiring pneumonia is commonly seen in those with swallowing difficulties or a relaxed cardiac sphincter because of alcohol. Given these factors, this patient is in a high risk category for aspiration pneumonia. The small degree of angulation of the right mainstem bronchus makes the right lung at higher risk. Most particles easily travel down this route, ending up in the right middle or lower lobe of the lung. Antibiotic coverage should be broad, covering for both gram positive and gram-negative organisms including anaerobes, which are commonly present in the mouth. Given the severity, these patients may go on to develop ARDS, an inflammatory response to infection, and, subsequently, respiratory failure.

A 29-year-old woman presents to the ED complaining of worsening left wrist pain for 1 month. She states that approximately 3 months ago she fell from a ladder and landed on her outstretched hand. She never went to the hospital and just dealt with the pain for a while. On examination, there is no deformity of the wrist. Neurovascular status is normal, but there is tenderness when you palpate the snuffbox. What is the most likely reason for this patient's wrist pain? a. Fracture of the distal ulna b. Acute fracture of the scaphoid c. Avascular necrosis of the scaphoid d. Hematoma of the radial artery e. Fracture of the lunate

*C* Avascular necrosis of the scaphoid is seen in approximately 3% of scaphoid fractures. The typical presentation of a scaphoid fracture is a FOOSH. On examination patients have snuff box tenderness or tenderness with axial loading of the thumb. Blood supply to the scaphoid is provided by a single artery that flows into the distal portion of the bone leaving the proximal portion vulnerable in the setting of a fracture. This increases the likelihood of complications, particularly avascular necrosis, in the setting of a poorly healed fracture.

21-year-old girl presents to the ED complaining of diarrhea, abdominal cramps, fever, anorexia, and weight loss for 3 days. Her BP is 127/75 mm Hg, HR is 91 beats per minute, and temperature is 100.8°F. Her abdomen is soft and nontender without rebound or guarding. WBC is 9200/μL, β-hCG is negative, urinalysis is unremarkable, and stool is guaiac positive. She tells you that she has had this similar presentation four times over the past 2 months. Which of the following extraintestinal manifestations is associated with Crohn disease but not ulcerative colitis? a. Ankylosing spondylitis b. Erythema nodosum c. Nephrolithiasis d. Thromboembolic disease e. Uveitis

*C* Crohn disease is characterized by chronic inflammation extending through all layers of the bowel wall. Onset is generally between the ages of 15 and 40 years. Crohn disease should be suspected in any patient whose symptoms show a picture consistent with chronic inflammatory colitis. Extraintestinal manifestations are see in 25% to 30% of patients. The incidence is similar for Crohn disease and ulcerative colitis. They include aphthous ulcers, erythema nodosum, iritis or episcleritis, arthritis, and gallstones. Nephrolithiasis is seen as a result of hyperoxaluria because of increased oxalate absorption in patients with ileal disease. Because ulcerative colitis affects only the large bowel, this extraintestinal manifestation is seen only in patients with Crohn disease.

As a senior resident in the ED, an intern calls you over to see a patient he treated for a migraine headache. The patient is a 21-year-old woman with a history of poorly controlled migraines. The patient was using a number of migraine medications at home and several were administered intravenously. As you approach the patient, you note her tongue is protruding and her head is tilted to the left. She is grimacing. The intern is concerned that the patient is having an acute stroke and would like to obtain a head CT scan. You advise the intern that the symptoms are likely the result of a medication side effect. Which of the following medications is likely to have caused the patient's symptoms? a. Morphine sulfate b. Acetaminophen c. Metoclopramide d. Caffeine e. Sumatriptan

*C* Dystonic reactions may occur with the use of dopamine blocking agents. Medications classically associated with dystonic reactions are typical antipsychotics (eg, haloperidol) but can also occur with the antiemetics used to treat migraines. They are generally not life-threatening and respond almost immediately to administration of diphenhydramine (Benadryl) given intravenously or intramuscularly or benzodiazepines. Common dystonic reactions include oculogyric crises (eyes deviating in different directions), torticollis, tongue protrusion, facial grimacing, and difficulty speaking.

A 45-year-old woman presents to the ED complaining of 3 days of fever and worsening throat pain and painful odynophagia without cough or coryza. She sits on a chair, leaning forward with her mouth slightly open. She has a cup of saliva and a box of facial tissues at her side. Vitals are HR of 120 beats per minute, BP of 110/70 mm Hg, RR of 22 breaths per minute, oxygen saturation of 99% on room air, and temperature of 102°F. Her voice is hoarse, but she is able to open her mouth fully. Her posterior oropharynx is moderately hyperemic, without exudates or tonsillar enlargement. A soft tissue lateral cervical radiograph shows marked edema of the prevertebral soft tissues and absence of the vallecular space. Which of the following is the most likely diagnosis? a. Retropharyngeal abscess b. Peritonsillar abscess c. Epiglottitis d. Pharyngitis e. Laryngotracheitis

*C* Epiglottitis is a lifethreatening inflammatory condition, usually infectious, of the epiglottis and the aryepiglottic and periglottic folds. Since most children are immunized against Haemophilus influenzae type B (Hib), most cases of epiglottitis are now seen in adults, with an average age of 46 years. Signs and symptoms include a prodromal period of 1 to 2 days consisting of constitutional symptoms, then the patient exhibits high fever, dysphagia, odynophagia, drooling, and dyspnea. Stridor is primarily inspiratory and softer and lower-pitched than croup. The "thumbprint sign" seen on lateral cervical radiograph demonstrates a swollen epiglottis obliterating the vallecula.

A 74-year-old man presents to the ED after being involved in a motor vehicle collision. He states he was wearing his seat belt in the driver's seat when a car hit him from behind. He thinks his chest hit the steering wheel and now complains of pain with breathing. His RR is 20 breaths per minute, oxygen saturation is 98% on room air, BP is 145/75 mm Hg, and HR is 90 beats per minute. On examination, you notice paradoxical respirations. Which of the following best describes a flail chest? a. One rib with three fracture sites b. Two adjacent ribs each with two fracture sites c. Three adjacent ribs each with two fracture sites d. One fractured right sided rib and one fractured left-sided rib e. Two fractured right sided ribs and two fractured left-sided ribs

*C* Flail chest results when three or more adjacent ribs are fractured at two points, allowing a freely moving segment of the chest wall to move in a paradoxical motion. It is one of the most commonly overlooked injuries resulting from blunt chest trauma.

A 60-year-old woman with a history of diabetes is brought into the ED by EMS workers who state that the patient was found on a bus in a lethargic and diaphoretic. Her fingerstick glucose level at the scene was 35 mg/dL. EMS workers quickly administered dextrose through an IV line. The patient became alert and responsive and remained this way throughout her trip to the ED. However, in the ED you notice that the patient is again diaphoretic and is mumbling her speech. Her fingerstick glucose is now 47 mg/dL. You administer dextrose and she perks right up. Which of the following diabetes medications commonly causes hypoglycemia for which the patient is likely to require hospital admission? a. Regular insulin b. Metformin c. Glyburide d. Rosiglitazone e. Acarbose

*C* Glyburide is a commonly prescribed sulfonylurea. Sulfonylureas are oral agents that stimulate the beta cells of the pancreas to produce insulin. Many of the sulfonylureas have relatively long durations of action. Glyburide can act up to 24 hours after ingestion. Hypoglycemia secondary to sulfonylureas generally requires hospital admission to monitor for recurrent hypoglycemia.

A 67-year-old man presents to the ED for worsening confusion. His wife states that he received his first dose of chemotherapy for lung cancer 2 days ago. Over the last 24 hours, the patient became confused. His BP is 130/70 mm Hg, HR is 87 beats per minute, and temperature is 98.9°F. While in the ED, the patient seizes. You administer an antiepileptic and the seizure immediately stops. You compare his current electrolyte panel to one taken 2 days ago. Sodium (mEq/L) 139 -> 113 Potassium (mEq/L) 4.1 -> 3.9 Chloride (mEq/L) 105 -> 98 Bicarbonate (mEq/L) 23 -> 20 BUN (mg/dL) 13 -> 17 Creatinine (mg/dL) 0.4 -> 0.7 Glucose (mg/dL) 98 -> 92 Which of the following is the most appropriate treatment? a. 0.45% saline b. 0.9% saline c. 3% saline d. 5% dextrose e. 50% dextrose

*C* Hyponatremia is defined as a measured serum sodium less than 135 mEq/L. However, the development of symptoms secondary to hyponatremia is related more to the rate of change in the serum sodium than to the absolute value. Levels less than 120 mEq/L tend to cause symptoms regardless of the rate to reach this value. Symptoms can include confusion, lethargy, nausea, vomiting, anorexia, muscle cramps, and seizures. There are many causes of hyponatremia, including renal or GI losses, third-spacing, endocrine abnormalities, syndrome of inappropriate antidiuretic hormone release (SIADH), cirrhosis, CHF, and nephrotic syndrome. Many medications cause SIADH, in addition to pulmonary and CNS disease. This patient, in particular, just started chemotherapy for lung cancer. The treatment for hyponatremia is guided by the cause of the process. However, if a patient is symptomatic (eg, seizing), hypertonic saline (3%) should be carefully administered to raise the serum sodium to 120 mEq/L. A known complication of hypertonic saline when it is administered too fast and sodium levels rise rapidly is the development of central pontine myelinosis.

A 22-year-old woman with known IIH who is scheduled for a ventriculoperitoneal shunt in 2 weeks presents to the ED complaining of severe headache. She states the headache is similar to the normal headaches associated with her condition except that it is refractor to her regular medications, including triptans, and opiates. Her neurologist increased her dose of acetazolamide, but this also did not help. Her noncontrast head CT is unchanged from previous and she does not have papilledema. Which of the following is likely to provide prompt relief? a. IV corticosteroids b. Infusion of mannitol c. LP with removal of 15 cc of CSF d. IV metoclopramide e. Nonsteroidal anti-inflammatory drugs (NSAIDs)

*C* IIH is an idiopathic elevation of ICP. In the setting of a normal CT, the diagnosis is made by LP with an elevated opening pressure often between 250 and 450 mm H2O. Patients often experience complete relief of their symptoms with LP and return of their ICP to levels < 200 mm H2O.

A 32-year-old woman is brought to the ED by paramedics after being involved in a motor vehicle collision. The patient was the front-seat passenger of the car and was not wearing a seat belt. In the ED, the patient is speaking and complains of abdominal pain. Her breath sounds are equal bilaterally. You note a distended abdomen. A FAST eamination is positive for fluid in the left upper quadrant (LUQ). Her BP is 90/70 mm Hg and HR is 120 beats per minute. You administer 2 L of crystalloid solution. Her repeat BP is 80/60 mm Hg. Which of the following is the most appropriate next step in management? a. Administer a vasoconstrictor, such as epinephrine. b. Administer another 2 L of crystalloid. c. Administer type O, Rh-negative blood. d. Bring patient to the CT scanner for an emergent scan. e. Perform another FAST examination to see if the fluid is increasing.

*C* Initial fluid resuscitation usually begins with crystalloid fluids such as 0.9% normal saline or Ringer lactate. In general, if the patient remains hemodynamically unstable after 40 cc/kg of crystalloid administration (approximately 2-3 L), then a blood transfusion should be started. Fully cross-matched blood is preferable; however, this is generally not available in the early resuscitation period. Therefore, type specific blood (type O, Rh-negative or type O, Rh-positive) is a safe alternative and is usually ready within 5 to 15 minutes. Type O, Rh-negative blood is typically reserved for women in their childbearing years to prevent Rh sensitization. Type O, Rh-positive blood can be given to all men and women beyond their childbearing years.

A 50-year-old man presents to the ED complaining of fever, sore throat, and neck pain for 24 hours. He states that 1 week ago he had two molars extracted from his mouth. His BP is 145/75 mm Hg, HR is 102 beats per minute, temperature is 101.2°F, and his RR is 16 breaths per minute. On examination you notice that the patient is drooling. There is erythema and swelling of his submandibular area that gives the appearance of a "bull neck." His tongue is swollen and elevated and the floor of his mouth is tender. There is no fluctuant mass in his mouth. Which of the following is the most likely diagnosis? a. Acute mastoiditis b. Peritonsillar abscess c. Ludwig angina (LA) d. Acute necrotizing ulcerative gingivitis (ANUG) e. Streptococcus pharyngitis

*C* Ludwig angina is a potentially fatal disease that can progress to death within hours. It is a progressive cellulitis of the floor of the mouth and neck that begins in the submandibular space. A dental cause, such as an extraction, is present in approximately 90% of cases. The most common symptoms include dysphagia, neck pain, and swelling. Physical findings include bilateral submandibular swelling, tongue swelling, and protrusion. A tense edema and induration of the neck may occur that is described as a "bull neck." Management involves securing an airway and starting IV antibiotic therapy. immediately. There is debate on whether these patients should be managed surgically with incision and drainage or medically with antibiotics.

A 22-year-old woman presents to the ED by ambulance from a dance club. The paramedics report that the patient was agitated in the club and had a generalized seizure. Her BP is 165/100 mm Hg, HR is 119 beats per minute, temperature is 100.9°F, RR is 17 breaths per minute, oxygen saturation is 98% on room air, and fingerstick glucose is 92 mg/dL. On examination, the patient is hyperactive and appears to be hallucinating. Her pupils are dilated to 6 mm bilaterally and reactive. Her neck is supple. Examination of the heart is unremarkable except for tachycardia. Her lungs are clear and abdomen is soft and nontender. The patient moves all four extremities. Laboratory results are as follows: Sodium 109 mEq/L, WBC 12,000/mm3, Potassium 3.5 mEq/L, Hct 49%, Chloride 83 mEq/L, Platelets 350/μL, Bicarbonate 20 mEq/L, BUN 10 mg/dL, Creatinine 1 mg/dL, Glucose 103 mg/dL. Which of the following substances did this patient most likely consume? a. Cocaine b. Heroin c. 3,4-Methylenedioxymethamphetamine (MDMA) d. Ketamine (special K) e. PCP

*C* MDMA is currently one of the most widely abused amphetamines by college students and teenagers. It is commonly known as "ecstasy," "E," "XTC," and "M&M." MDMA is an entactogen, a substance capable of producing euphoria, inner peace, and a desire to socialize. Negative effects include ataxia, restlessness, confusion, poor concentration, and memory problems. MDMA, although classified as an amphetamine, is also a potent stimulus for the release of serotonin. MDMA can also cause significant hyponatremia. The increase in serotonin results in the excessive release of vasopressin (antidiuretic hormone [ADH]). Moreover, large free-water intake (increased thirst) combined with sodium loss from physical exertion (dancing) certainly contributes to the development of hyponatremia.

An 18-year-old man presents to the ED with nausea and vomiting complaining of testicular pain for the past hour that began while playing volleyball. He recalls having similar pain 1 week ago that resolved spontaneously after 10 minutes. He was recently well and reports no fever, diarrhea, urinary frequency, or dysuria. Physical examination reveals vital signs within normal limits. The patient appears in moderate discomfort, holding his scrotum. His abdomen is soft and nontender. His right hemiscrotum is swollen, erythematous, and diffusely tender. It is not possible to palpate the testis separate from the epididymis. The right cremasteric reflex is absent. His left testis has a horizontal lie and is nontender. You suspect testicular torsion. What is the correct way to attempt manual detorsion? a. Elevate the painful testis until there is pain relief. b. Rotate the testes in a lateral to medial direction as if you were closing a book. c. Rotate the testes in a medial to lateral direction as if you were opening a book. d. Rotate the testes in an inferior to superior direction. e. Rotate the testes in a superior to inferior direction.

*C* Manual detorsion is a maneuver used to untwist the spermatic cord to reestablish blood flow to the testis. This procedure should be performed in any patient with suspected torsion while the patient is being prepared for the OR. Most testes torse lateral to medial. If you were to stand at the foot of the patient's bed, you would perform detorsion for either testis just as you would open a book—rotating each testicle in a medial to lateral direction. Successful detorsion results in immediate reduction of pain. In many cases, detorsion is not successful or the testis twists again.

A 31-year-old woman with a history of schizophrenia presents to the ED for altered mental status. A friend states that the patient is on multiple medications for her schizophrenia. Her BP is 150/80 mm Hg, HR 121 beats per minute, RR 20 breaths per minute, and temperature 104.5°F. On examination, the patient is diaphoretic with distinctive "lead-pipe" rigidity of her musculature. You believe the patient has neuroleptic malignant syndrome. After basic stabilizing measures, which of the following medications is most appropriate to administer? a. Haloperidol b. Droperidol c. Dantrolene d. Diphenhydramine e. Acetaminophen

*C* Neuroleptic malignant syndrome (NMS) is a rare, but potentially fatal reaction commonly associated with the use of antipsychotic drugs. The classic triad for its clinical presentation includes altered mental status, hyperthermia, and muscle rigidity. The cornerstone of treatment is supportive care with rapid cooling, fluid and electrolyte repletion, and monitoring. Dantrolene, a nonspecific skeletal muscle relaxant, generally used in the treatment of malignant hyperthermia, is also effective for NMS. In addition, benzodiazepines are useful in the treatment of NMS. The offending agent should be discontinued.

A 32-year-old diabetic man steps on a nail through his running shoe. Two weeks later, he presents to the ED with fever and right foot pain. On physical examination, his heel is mildly erythematous and diffusely tender to palpation, with overlying warmth and edema. There is a small amount of purulent drainage through the puncture hole in his heel. A plain radiograph of his foot demonstrates a slight lucency of the calcaneus. He has decreased range of motion, but you are able to passively dorsiflex and plantarflex his ankle without difficulty. His vital signs include a temperature of 101.4°F, HR of 98 beats per minute, BP of 130/75 mm Hg, and RR of 16 breaths per minute. Which of the following is the most common causative organism of this condition? a. Salmonella sp. b. Pseudomonas aeruginosa c. Staphylococcus aureus d. Group B streptococci e. Pasteurella multocida

*C* Osteomyelitis is an infection or inflammation of a bone with an incidence following plantar puncture wounds of 0.1% to 2%. For patients overall, S aureus is the leading cause of osteomyelitis, followed by Streptococcus species. Pain, swelling, fever, redness, and drainage may all occur, but pain is the presenting complaint in most cases. Risk factors include trauma, surgery, soft tissue infections, and being immunocompromised (eg, HIV, diabetes, IV drug user, sickle-cell disease, alcoholism). Definitive diagnosis is made by bone scan which will demonstrate osteomyelitis within 72 hours of symptom onset. Radiographs may be normal early on, but will demonstrate periosteal elevation within 10 days. ESR is often elevated, but a normal or slightly elevated ESR does not rule out the diagnosis. The ESR is most valuable in following response to treatment, as the ESR should fall as the infection resolves. Blood cultures, which are positive in 50% of cases, should be used to guide antibiotic treatment. All patients with puncture wounds should receive tetanus prophylaxis.

A 32-year-old woman presents to the ED with a 1-month history of general malaise, mild cough, and subjective fevers. She states that she is human immunodeficiency virus (HIV) positive and her last CD4 count, 6 months ago, was 220. She is not on antiretroviral therapy or any other medications. Initial vitals include a HR of 88 beats per minute, a BP of 130/60 mm Hg, and an RR of 12 breaths per minute with an oxygen saturation of 91% on room air. Her chest radiograph shows bilateral diffuse interstitial infiltrates. Subsequent laboratory tests are unremarkable except for an elevated lactate dehydrogenase level. Given this patient's history and physical examination, which of the following is the most likely organism responsible for her clinical presentation? a. Coccidioides immitis b. Mycobacterium tuberculosis c. Pneumocystis jiroveci d. Mycoplasma pneumoniae e. Haemophilus influenzae

*C* PCP is a commonly seen opportunistic infection in the HIV/AIDS population. It typically presents with mild subjective symptoms of cough and general malaise. Objectively, patients are hypoxic and have a chest radiograph with a bilateral interstitial process. Risk factors include a CD4 count < 200. Serum lactate dehydrogenase (LDH) is also considerably higher in AIDS patients with PCP. In fact, greater the elevation in LDH, worse the prognosis. Despite the classic PCP radiograph demonstrating bilateral diffuse interstitial infiltrates, beginning in the perihilar region and extending into a "bat-wing" pattern, the chest radiograph may be normal in up to 30% of patients. In addition to Kaposi sarcoma involvement in the lungs, pulmonary infections, such as tuberculosis, cytomegalovirus, and fungal infections, should be considered.

An asymptomatic young adult was brought to the ED by a police officer after his home was raided. The patient swallowed five small packets of an unknown substance before being arrested. His BP is 125/75 mm Hg, HR is 85 beats per minute, temperature is 98.7°F, and RR is 16 breaths per minute. Physical examination is unremarkable. An abdominal radiograph confirms intraluminal small bowel densities. Which of the following is the most appropriate treatment? a. Magnesium citrate b. Gastric lavage c. Activated charcoal and polyethylene glycol d. Syrup of ipecac e. NAC

*C* Patients being arrested who swallow illegal drugs to conceal the evidence are referred to as "body stuffers." They commonly tend to ingest any and all the drugs they possess, potentially resulting in a polypharmaceutic overdose. Body stuffers are usually seen in the ED before symptoms have developed. Activated charcoal should be administered immediately and whole-bowel irrigation may be indicated. Sometimes there is radiographic evidence of the swallowed substances as seen in crack vials or staples on the packaging materials. Whole-bowel irrigation uses a polyethylene glycol electrolyte solution (eg, GoLYTELY), which is not absorbed and flushes drugs or chemicals through the GI tract. This procedure seems to be most useful when radiopaque tablets or chemicals, swallowed packets of street drugs, or sustained-released drugs have been ingested.

A 26-year-old woman presents to the ED with fever, malaise, and an evolving rash in the right axilla that she initially thought was from an insect bite that she received while hiking 1 week earlier. She complains of generalized fatigue, nausea, headache, and joint pain over the past several days. Her vitals are BP of 120/75 mm Hg, HR of 75 beats per minute, RR of 16 breaths per minute, and temperature of 101°F. On physical examination, she is awake and alert, with a nonfocal neurological examination. Her neck is supple, but she is diffusely tender over the shoulder, knee, and hip joints bilaterally without any distinct effusions. Her abdomen is soft and nontender. She has a 9-cm erythematous annular plaque with a central clearing under her right axilla. Which of the following is the next best step? a. Consult dermatology for a biopsy of the rash. b. Perform a LP and begin treatment with IV ceftriaxone. c. Prescribe doxycycline 21 days and arrange follow-up with her primary-care doctor. d. Prescribe hydrocortisone cream for the rash and acetaminophen for the headache and joint pain. e. Perform serologic testing for Borrelia burgdorferi and begin treatment only if positive.

*C* Recommended treatment for early Lyme disease includes doxycycline, amoxicillin (for children younger than 8 years or for pregnant or lactating women), or cefuroxime. Erythromycin and azithromycin can be used, but are less effective. The same drugs can be used for the second stage of disease, but their course of therapy needs to be longer.

Paramedics bring a 17-year-old high school football player to the ED on a backboard and with a cervical collar. During a football game, the patient "speared" another player with his helmet and subsequently experienced severe neck pain. He denies paresthesias and is able to move all of his extremities. A cervical spine CT scan reveals multiple fractures of the first cervical vertebra. Which of the following best describes this fracture? a. Odontoid fracture b. Hangman's fracture c. Jefferson fracture d. Clay-shoveler's fracture e. Teardrop fracture

*C* Spearing—hitting another player with the crown of the helmet—generates an axial loading force that is transmitted through the occipital condyles to the superior articular surfaces of the lateral masses of the first cervical vertebra (C1). This fracture is commonly referred to as a Jefferson fracture. It is considered an unstable fracture and is associated with C2 fractures 40% of the time. On plain radiograph, it is best seen on the open-mouth odontoid view as the lateral masses are shifted laterally. It is associated with diving accidents and in this scenario, "spearing" in a football game, which places an increased axial load to the cervical spine. Proper cervical spine precautions should remain in place throughout his management in the ED.

A 43-year-old man, who currently uses drugs intravenously (IV), presents to the emergency department (ED) with 2 weeks of fever, back pain, and progressive weakness in his arms bilaterally. He reports having a cough with whitish sputum. He denies any history of recent trauma. His blood pressure (BP) is 130/75 mm Hg, heart rate (HR) is 106 beats per minute, temperature is 103°F, and respiratory rate (RR) is 16 breaths per minute. On physical examination, he has tenderness to palpation in the mid-thoracic spine, and decreased strength in the upper extremities bilaterally, with normal range of motion. Laboratory results reveal a white blood cell (WBC) count of 15,500/μL, hematocrit 40%, and platelets 225/μL. Which of the following is the most likely diagnosis? a. Lung abscess b. Ankylosing spondylitis c. Spinal epidural abscess d. Vertebral compression fracture e. Spinal metastatic lesion

*C* Spinal abscesses are most commonly found in immunocompromised patients, IV drug users, and the elderly. Signs and symptoms of epidural abscess usually develop over a week or two and include fever, localized pain, and progressive weakness. An elevated WBC count is also commonly seen. MRI is the most useful diagnostic test. Staphylococcus aureus is the most common causative organism, followed by gram-negative bacilli and tuberculosis bacillus.

A 19-year-old man is brought into the trauma room by EMS after a head on cycling accident. The patient was not wearing a helmet. Upon presentation his BP is 125/75 mm Hg, HR is 105 beats per minute, RR is 19 breaths per minute, and oxygen saturation is 100% on mask. His eyes are closed, but open to command. He can move his arms and legs on command. When you ask him questions, he is disoriented but able to converse. What is this patient's GCS score? a. 11 b. 12 c. 13 d. 14 e. 15

*C* The GCS score, as seen below, may be used as a tool for classifying head injury and is an objective method for following a patient's neurologic status. The GCS assesses a person's eye, verbal, and motor responsiveness. This patient received a score of 3 for eye opening to verbal command, 4 for disorientation, but conversant, and 6 for obeying verbal commands.

A 29-year-old figure skater lands incorrectly after a jump and injures her right ankle. She stands up on her own but has difficulty bearing weight so she calls EMS. She presents to the ED with swelling and tenderness of her lateral malleolus. You palpate a dorsalis pedis and posterior tibialis pulse. She is able to plantar and dorsiflex without difficulty. You are unsure if the patient requires an ankle radiograph because you think she only sustained a sprain. In addition to pain near the malleoli, which of the following is an indication for a radiograph according to the Ottawa ankle and foot rules? a. Boney tenderness is present at the anterior edge of the distal 12 cm or at the tip of either malleolus. b. The patient is able to bear weight for at least four steps immediately after the injury but not at the time of evaluation. c. Boney tenderness is present at the navicular or at the base of the fifth metatarsal. d. The Achilles reflex is diminished on the side of the injury. e. An effusion is present on clinical examination.

*C* The Ottawa rules are a prospectively validated clinical decision tree for radiograph ordering in adults. By following these rules, emergency physicians can eliminate up to 30% of radiographs that are routinely ordered without missing clinically significant fractures. Radiographs are only required if there is bony pain in the malleolar or midfoot area, and any one of the following: bony tenderness at the navicular bone or at the base of the fifth metatarsal;bony tenderness is present at the posterior edge of the distal 6 cm or at the tip of either malleolus, and the patient is unable to bear weight for at least four steps immediately after the injury and at the time of evaluation

A 31-year-old homebuilder is cutting a piece of wood when the table saw backfires and slices through the base of his left thumb. In the ED his BP is 135/70 mm Hg, HR 87, RR 18, and pulse oxygen is 99%. You stabilize the patient and call the vascular surgeon. In the meantime, what is the best method of preserving the amputated finger? a. Irrigate it with 10% povidone-iodine solution to remove gross contamination, wrap it in sterile gauze moistened with normal saline and place it between two ice packs. b. Irrigate it with normal saline to remove gross contamination, wrap it in sterile gauze moistened with normal saline and then place it on ice. c. Irrigate it with normal saline to remove gross contamination, wrap it in sterile gauze moistened with normal saline, place it in a sterile water-tight container and store this container in ice water. d. Place it in a container of 10% povidone-iodine solution and store this container in ice water. e. Attempt to partially replant the part with 3-0 nylon sutures until the vascular surgeon arrives.

*C* The best way to preserve an amputated part is to rinse it with normal saline to remove gross contamination, wrap it in sterile gauze moistened with saline or lactated ringers, and place it in a sterile, water tight container. Then store this container in ice water.

A 71-year-old woman presents to the ED with 12-hours of emesis and abdominal pain. Her temperature is 101.2°F, BP is 100/79 mm Hg, and HR is 104 beats per minute. Physical examination reveals a tender (2 × 2)-cm bulge with erythema below the inguinal ligament and abdominal distension. An occasional high-pitched bowel sound is heard. After placing an IV line and nasogastric tube, which of the following is the most appropriate course of management? a. Administer broad-spectrum antibiotics and then obtain a CT scan of abdomen b. Administer broad-spectrum antibiotics and attempt reduction c. Administer broad-spectrum antibiotics and prepare the patient for the OR d. Administer broad-spectrum antibiotics and obtain a plain radiograph e. Administer broad-spectrum antibiotics and observe

*C* The clinical scenario is highly suspicious for a strangulated loop of bowel incarcerated in a femoral hernia. If the contents of a hernia can be returned to their natural cavity by manual reduction, the hernia is termed reducible; if they cannot, it is termed irreducible or incarcerated. Incarcerated hernias are subject to inflammatory and edematous changes and are at risk for strangulation, which refers to vascular compromise of the incarcerated contents. When strangulation is not emergently relieved, necrosis and gangrene develop. The treatment for an incarcerated hernia that cannot be manually reduced is surgical fixation. If strangulation is suspected or shock is present, broad spectrum antibiotics and fluid resuscitation are necessary.

A 78-year-old man with a history of atherosclerotic heart disease and congestive heart failure presents with increasing abdominal pain. The pain began suddenly a day ago and has progressively worsened since then. He denies nausea, vomiting, and diarrhea, but states that he had black tarry stool this morning. He denies any history of prior episodes of similar pain. Vitals are BP 120/65 mm Hg, HR 105 beats per minute, temperature 99°F. The patient is at high risk for which of the following conditions? a. Cholecystitis b. Cecal volvulus c. Mesenteric ischemia d. Perforated peptic ulcer e. Small bowel obstruction

*C* The complaint of abdominal pain in the elderly patient should prompt the emergency physician to lower his or her threshold for considering more serious intra-abdominal conditions. Patients with a history of arrhythmias (especially atrial fibrillation), low cardiac output (such as congestive heart failure), or who take particular medications, such as digoxin, are at high risk for the elusive condition of mesenteric ischemia. A history of sudden onset abdominal pain with increasing severity, but with a benign physical examination should prompt consideration of this entity. Mesenteric ischemia is associated with a high mortality rate, and initial diagnosis is often incorrect. The diagnostic study of choice is angiography.

A 58-year-old woman is brought in to the ED after a witnessed syncopal event. Upon arrival, the patient appears confused and agitated. Her vitals include HR of 89 beats per minute, BP of 145/70 mm Hg, RR of 16 breaths per minute, and oxygen saturation of 98% on room air. Within a few minutes, the patient is more alert and oriented. She denies any chest pain, headache, abdominal pain, or weakness preceding the event and is currently asymptomatic. She also states that she has not taken her antiepileptic medications in 2 days. The patient's examination is unremarkable including a nonfocal neurological examination. Given this patient's history and evolving examination, what is the most likely etiology of this patient's syncopal event? a. Cerebrovascular accident b. Transient ischemic attack c. Seizure d. Aortic dissection e. Pulmonary embolus

*C* The emergency medicine physician is often faced with differentiating whether the cause of a patient losing consciousness is a result of syncope or a seizure. The most likely etiology of this patient's symptoms is in the history that she gives. She tells you that she has not taken her antiseizure medications in 2 days. Also, given her evolving mental status and improvement in alertness, this patient most likely presented in a postictal state after she has seized. Without any focal deficit and further improvement in her mental status, one might be comfortable with this diagnosis. Serum testing of her antiepileptic drug levels must be performed to further investigate this suspicion. A CT of the head, ECG, and further investigation is warranted if these levels are normal and do not explain her loss of consciousness.

A 27-year-old woman presents to the ED complaining of headache lasting approximately 1 hour in duration that is unrelieved by aspirin and acetaminophen. She states the headache was not preceded by any visual phenomena, is left sided, pulsatile, and has occurred nearly monthly coinciding with her menstrual period for the past 6 months. She also complains of nausea and sensitivity to sound and light. Which of the following is the most appropriate therapy for this patient at the time of presentation? a. IV morphine sulfate b. Another trial of aspirin and acetaminophen c. Sumatriptan d. Topiramate e. Hydromorphone and acetaminophen

*C* The headache described is a menstrual migraine, a common variant of the migraine headache syndrome. Appropriate abortive therapies (this headache is just starting) are diverse and include IV ergot, triptans, and antiemetics. Sumatriptan (Imitrex) is a triptan which acts by blocking the 5-hydroxytryptamine (5-HT, serotonin) 1D receptor. It also has less associated nausea and vomiting than ergots. It may have a higher incidence of minor side effects (flushing, injection site reaction) and a higher relapse rate than ergots. Contraindications to triptans or ergots include pregnancy, hypertension, coronary artery disease, or use of either class of agent within the last 24 hours.

A 28-year-old male military recruit presents to the ED complaining of headache, fever, and neck stiffness. His temperature is 102.2°F and he refuses to move his neck. He is somewhat lethargic appearing and winces when the lights are turned on in the examining room. He has a nonfocal neurologic examination and you proceed with LP. Which of the following is the most specific finding for suspected bacterial meningitis? a. The presence of phonophobia, photophobia, and neck stiffness b. Fever higher than 102.2°F c. Elevated polymorphonuclear white blood cell (WBC) count on CSF analysis d. Elevated protein on CSF analysis e. Increased glucose on CSF analysis

*C* The normal number of WBCs in the CSF is 5 or fewer with 1 or less polymorphonuclear neutrophils (PMN). Numbers greater than these should be taken as evidence for CNS infection. In cases of acute bacterial meningitis, cell counts of 1000/μL to 20,000/μL WBCs are observed, often with neutrophil predominance. In cases of aseptic (viral) meningitis, cell counts are generally lower with lymphocyte predominance. Initial treatment with antibiotics prior to LP is unlikely to affect the cell count, after 6 hours though, the culture is less likely to return positive. It should be noted that a subset of patients with bacterial meningitis may present with lymphocytic predominance. Therefore, lymphocytic predominance does not rule out bacterial meningitis and antibiotics should be given.

After being fired from his job, a 35-year-old man attempts suicide by drinking from a bottle labeled "insecticide." Three hours later, emergency medical services (EMS) brings him in to the emergency department (ED) and you notice that he is extremely diaphoretic, drooling, and vomiting. He is awake, but confused. His vital signs include a blood pressure (BP) of 170/90 mm Hg, heart rate (HR) of 100 beats per minute, respiratory rate (RR) of 22 breaths per minute, temperature of 98.6°F, and oxygen saturation of 95% on room air. Physical examination demonstrates pinpoint pupils and crackles on lung examination. What is the treatment to reverse this patient's poisoning? a. Naloxone b. N-acetylcysteine (NAC) c. Atropine and pralidoxime (2-PAM) d. Flumazenil e. Physostigmine

*C* The patient drank insecticide, which is primarily composed of organophosphate compounds (eg, Malathion). These compounds inhibit acetylcholinesterase, the enzyme responsible for the breakdown of acetylcholine. The patient is having a "cholinergic crisis." Overstimulation of muscarinic and nicotinic receptors leads to his symptoms, commonly remembered by the mnemonics SLUDGE (salivation, lacrimation, urination, defecation, gastrointestinal [GI] upset, emesis) or DUMBBELS (defecation, urination, miosis, bronchospasm, bronchorrhea, emesis, lacrimation, salivation). The treatment for organophosphate toxicity is atropine and pralidoxime (2-PAM). Atropine is an anticholinergic, therefore it competitively inhibits the excess acetylcholine. Pralidoxime works to regenerate acetylcholinesterase, therefore also limiting the amount of acetylcholine available in neuronal synapses.

A 22-year-old woman presents to the ED complaining of headache. She states that while at home she experienced a headache that was associated with blurry vision in both eyes with a shimmering line in her vision. She subsequently lost her vision and felt uncoordinated, followed by increased pain at the base of her skull. Upon arrival in the ED, her vision returned to normal. A head CT scan and an LP are both negative. She now complains of a persistent, severe, pulsatile headache. She has had two similar episodes in the past year with the headache refractory to over-the-counter medications. Which of the following is likely to relieve her symptoms? a. Diazepam b. High flow oxygen c. Sumatriptan d. Acetaminophen e. LP with removal of 15 cc cerebrospinal fluid (CSF)

*C* The patient is experiencing a migraine variant known as a basilar migraine. Its onset is similar to other migraines in that it can begin with scotomata or aura. The visual symptoms are often bilateral and followed by a brief period of cortical blindness. Symptoms related to the basilar circulation then predominate including incoordination, dysarthria, vertigo, and numbness and tingling in the arms or legs. These symptoms generally last 10 to 30 minutes then resolve. Occasionally, transient coma or quadriplegia can develop but persist for only several hours. The resulting headache is occipital and pulsating. The symptoms may mimic a vertebrobasilar ischemic event. Treatment with first line agents for migraine is recommended. Threshold should be low to seek neurologic consultation unless the diagnosis is certain

A 69-year-old woman with a past medical history of hypertension, hypercholesterolemia, diabetes mellitus type 1, and alcohol abuse is brought to the emergency department (ED) by her daughter who states that her mom has been acting funny over the last hour. She states that the patient did not know where she was despite being in her own house. She also did not recognize her family and was speaking incomprehensibly. Her blood pressure (BP) is 150/80 mm Hg, heart rate (HR) is 90 beats per minute, temperature is 98.9°F, and her respiratory rate (RR) is 16 breaths per minute. On physical examination she is diaphoretic, agitated, and tremulous. Electrocardiogram (ECG) is sinus rhythm with normal ST segments and T waves. Which of the following is the most appropriate course of action for this patient? a. Administer a benzodiazepine to treat her ethanol withdrawal. b. Activate the stroke team and bring the patient directly to the computed tomographic (CT) scanner. c. Get a stat fingerstick and administer dextrose if her blood sugar is low. d. Request a psychiatric consult for probable sundowning. e. Administer haloperidol for sedation.

*C* The patient never received a fingerstick glucose at triage. Hypoglycemia can mimic a cerebrovascular accident or seizure. Therefore, it is critical that all patients who present with altered mental status get fingerstick glucose. Glucose level should be considered a vital sign. Hypoglycemia is a common problem in patients with type 1 diabetes. The clinical presentation of hypoglycemia is caused by increased secretion of epinephrine, as well as central nervous system (CNS) dysfunction. Symptoms include diaphoresis, nervousness, tremor, tachycardia, hunger, and neurologic symptoms ranging from confusion and bizarre behavior to seizures and coma.

A 25-year-old man is carried into the ED by two of his friends who state that he is not breathing. The patient has a history of heroin abuse. His vital signs are BP 115/70 mm Hg, HR 99 beats per minute, temperature 98.9°F, RR 3 breaths per minute, and oxygen saturation 87% on room air. You notice fresh needle marks and miotic pupils. You begin bag-valve-mask ventilation and his oxygen saturation increases to 99%. Which of the following is the most appropriate next step in management? a. Continue bag-valve-mask ventilation until he breathes on his own b. Endotracheal intubation of the patient c. Administration of naloxone d. Administration of flumazenil e. Place a nasogastric tube and administer activated charcoal

*C* The patient presents to the ED with central nervous system (CNS) and respiratory depression and miotic pupils. Along with his history of heroin abuse and fresh needle marks, this is most likely a heroin overdose. Opioid toxicity is associated with the toxidrome of CNS depression, respiratory depression, and miosis. Attention is always first directed at airway management in emergency medicine. The first action for this patient is to provide oxygen via bagvalve-mask ventilation. Because his respiratory depression is most likely secondary to opioid overdose, an opioid antagonist should be administered. Naloxone is the antidote most frequently used to reverse opioid toxicity. The goal of naloxone therapy is not necessarily complete arousal; rather, it is to reinstitute adequate spontaneous respiration, while attempting to avoid inducing acute opioid withdrawal.

A 78-year-old woman is transferred from a nursing home with altered mental status and fever. The nursing home reports that the patient was febrile to 102.3°F, disoriented, confused, and incontinent of urine. Her past medical history includes hypertension, a stroke with residual right-sided weakness, and nighttime agitation for which she was started on haloperidol 3 days ago. Her BP is 215/105 mm Hg, HR is 132 beats per minute, temperature is 102.8°F, and RR is 20 breaths per minute. On examination, the patient is oriented to name only, tremulous, diaphoretic and has marked muscular rigidity and three out of five right upper and lower extremity strength. What is the most likely diagnosis? a. Urinary tract infection b. Malignant hyperthermia c. Neuroleptic malignant syndrome d. Recurrent stroke e. Meningoencephalitis

*C* The patient presents with a rare but potentially life-threatening NMS. Antipsychotic drugs (eg, haloperidol) are the most common offending agents in the development of NMS, causing central dopamine depletion. The disorder is typically characterized by hyperthermia, muscle rigidity, altered mental status, and autonomic instability. Since NMS carries a high mortality, it is important to aggressively treat it with muscle relaxers, such as IV benzodiazepines, dantrolene, and dopamine agonists.

A 76-year-old woman is brought into the ED complaining of hip pain after a motor vehicle collision. She states that her knee slammed into the dashboard after her car struck another vehicle. On examination you note her right leg appears shortened, adducted, internally rotated, and flexed. She has no other injuries. The radiograph of her hip confirms your diagnosis of a hip dislocation. Which of the following statements regarding this type of injury is true? a. The most common presentation of this injury is an abducted, externally rotated, and flexed lower extremity. b. This type of injury accounts for 5% to 10% of all hip dislocations. c. The injury is commonly caused by a direct force applied to a flexed knee. d. The most common complication is injury to the femoral artery. e. The femoral nerve is almost always injured in this type of dislocation.

*C* The patient sustained a posterior hip dislocation. The most common mechanism for this to occur is during a motor vehicle collision in which a direct force is applied to the flexed knee (eg, the patient's knee hits the dashboard at high velocity). The limb appears shortened, adducted, internally rotated, and flexed. Because of the strong forces needed to cause such an injury, hip dislocations rarely occur in isolation. Life threatening injuries should always be addressed first, prior to treating hip dislocations. Nonetheless, early reduction is required since the likelihood of avascular necrosis of the femoral head increases in direct proportion to a delay in reduction. Early complications of posterior dislocations include sciatic nerve injury. Late complications include avascular necrosis of the femoral head, as mentioned above.

A 50-year-old man presents to the ED complaining of abrupt onset of epigastric pain that radiates to his back. He describes the pain as constant and associated with nausea and vomiting. The pain improves mildly if he leans forward. He has a 20-pack-year smoking history and has consumed 6 packs of beer daily for more than 5 years. His BP is 150/80 mm Hg, HR is 98 beats per minute, temperature is 100.1°F, and his RR is 18 breaths per minute. He is tender to palpation in the epigastric area. A chest x-ray reveals a small left-sided pleural effusion. His WBC count is 12,000/μL, hematocrit 39%, plasma glucose 225 mg/dL, AST and ALT are within normal limits, alkaline phosphatase 96 U/L, and lipase 520 U/L. His CT scan is seen below. What is the most likely diagnosis? a. Abdominal aortic aneurysm b. Mesenteric ischemia c. Pancreatitis d. Bowel perforation e. Cholecystitis

*C* The patient's clinical picture is consistent with acute pancreatitis, an inflammation and self-destruction of the pancreas by its digestive enzymes. There are many risk factors for pancreatitis, the most common being gallstones and alcohol, which account for more than 80% of the cases. Pancreatitis can be divided into mild and severe defined by the presence of organ failure or local complications, such as necrosis, pseudocyst, or abscess. It should be suspected in all patients with epigastric pain. Elevation in lipase, a pancreatic enzyme, is used to make the diagnosis of pancreatitis. At five times the upper limit of normal, the specificity of lipase approaches 100% for pancreatitis.

A 65-year-old man with a history of chronic hypertension presents to the ED with sudden-onset tearing chest pain that radiates to his jaw. His BP is 205/110 mm Hg, HR is 90 beats per minute, RR is 20 breaths per minute, and oxygen saturation is 97% on room air. He appears apprehensive. On cardiac examination you hear a diastolic murmur at the right sternal border. A chest x-ray reveals a widened mediastinum. Which of the following is the preferred study of choice to diagnose this patient's condition? a. Electrocardiogram (ECG) b. Transthoracic echocardiography (TTE) c. Transesophageal echocardiography (TEE) d. Computed tomography (CT) scan e. Magnetic resonance imaging (MRI)

*C* The patient's clinical picture of chronic hypertension, acute onset tearing chest pain, diastolic murmur of aortic insufficiency, and chest x-ray with a widened mediastinum is consistent with an aortic dissection. The preferred study of choice is a transesophageal echocardiogram (TEE), which is highly sensitive. It can be quickly performed at the bedside and does not require radiation or contrast. (b) A TTE is limited in diagnosing aortic dissections because wave transmission is hindered by the overlying sternum. It can be useful to see pericardial fluid as a result of proximal dissection.

A 35-year-old agitated man presents to the ED in police custody. He denies any past medical history and takes no medication. He admits to using some drugs today. His BP is 195/90 mm Hg, HR is 121 beats per minute, temperature is 100.1°F, RR is 18 breaths per minute, and oxygen saturation is 99% on room air. On examination, he is diaphoretic, and has pupils that are 8 mm in diameter, along with 3+ patella reflexes bilaterally. Electrocardiogram (ECG) reveals sinus tachycardia with a rate of 123. Which of the following toxic syndromes is this patient exhibiting? a. Anticholinergic b. Cholinergic c. Sympathomimetic d. Opioid e. Serotonin

*C* The sympathomimetic syndrome usually is seen after acute abuse of cocaine, amphetamines, or decongestants. Patients are usually hypertensive and tachycardic and exhibit mydriatic pupils. In massive overdoses, cardiovascular collapse can result in shock and wide-complex dysrhythmias. CNS effects include seizures. Sympathomimetic syndrome is sometimes difficult to distinguish from anticholinergic syndrome. The difference is that patients usually present with dry mucous membranes with an anticholinergic overdose, whereas patients are diaphoretic with sympathomimetics.

An 81-year-old woman is brought to the ED by her children who state that the patient is acting more tired than usual, has had fever for the last 2 days, and is more confused. Ordinarily, the patient is high functioning: she is ambulatory, cooks for herself, and walks on a treadmill 30 minutes a day. Her vital signs are BP 85/60 mm Hg, HR 125, RR 20, temperature 101.3°F, and pulse oxygenation 97% on room air. On examination, the patient has dry mucous membranes, but is otherwise unremarkable. She is oriented to person and place, but states that the year is 1925. Her laboratory results show a WBC 14,300/μL, hematocrit 31%, and platelets 350/μL. Her electrolytes are within normal limits. Blood glucose is 92 mg/dL. A chest radiograph does not show any infiltrates. Urinalysis reveals 2+ protein, trace ketones, WBC > 100/hpf, RBC 5 to 10/hpf, nitrite positive, and leukocyte esterase positive. After administering a 500 cc normal saline fluid bolus and broad-spectrum antibiotics through her peripheral IV line, the patient's BP is 82/60 mm Hg. You suspect that the patient is in septic shock. Which of the following is the next most appropriate course of action to manage this patient with early-goal-directed therapy? a. Start vasopressor therapy; repeat BP, if below a systolic of 90 mm Hg, increase the dose b. Check the hematocrit and if it is less than 30% prepare to transfuse packed red blood cells (RBC) c. Place a central venous line into the right internal jugular vein, measure a central venous pressure (CVP), administer normal saline boluses if the CVP is less than 8 mm Hg d. Place a central venous line into the left subclavian vein, measure a CVP, administer normal saline boluses if the CVP is less than 12 mm Hg e. Place a central venous line into the right femoral vein, measure a CVP, administer normal saline boluses if the CVP is less than 8 mm Hg

*C* The treatment for sepsis has evolved considerably over the past 10 years. Early recognition and early aggressive therapy for patients with sepsis have a significant impact on mortality.

A 32-year-old male construction worker reports standing on scaffolding before it suddenly gave way beneath him. In an attempt to catch his fall, he spontaneously grabbed and hung on to an overhead beam for approximately 30 seconds. Two hours later in the ED, the patient presents with right shoulder pain and decreased range of motion. He denies any loss of consciousness or other sustained trauma. Vital signs are within normal limits, except for the patient's pain scale of 10/10. He is holding his right arm with the contralateral hand. On physical examination, the patient's shoulder appears swollen with no skin breakage. The upper extremity is without obvious deformity. The patient has palpable brachial, radial, and ulnar pulses with capillary refill that is less than 2 seconds. He can wiggle his fingers, but cannot internally rotate his shoulder or raise his right arm above his head. Pinprick testing reveals paresthesias along the lateral deltoid of the affected arm. What is the most likely etiology of this patient's pain and paresthesias? a. Acromioclavicular joint sprain b. Posterior shoulder dislocation with axillary nerve impingement c. Anterior shoulder dislocation with axillary nerve impingement d. Anterior shoulder dislocation with median nerve impingement e. Posterior shoulder dislocation with ulnar nerve impingement

*C* This patient has an anterior shoulder dislocation. The glenohumeral joint is the most commonly dislocated joint in the body, mainly because of the lack of bony stability and its wide range of motion. Anterior dislocations account for 95% to 97% of cases and are most commonly seen in younger, athletic males and geriatric females. It usually happens by way of an indirect force that involves an abduction plus extension plus external rotation injury. Directly, it may occur as a result of a posterior blow that forces the humeral head out of the glenoid rim anteriorly. Radiographs obtained must include an axillary view to determine positioning of the humeral head. Patients usually present in severe pain, holding the affected arm with the contralateral hand in slight abduction. The lateral acromial process is prominent giving the shoulder a full or squared-off appearance. Patients typically cannot internally rotate their shoulder. Axillary nerve injuries can occur in up to 54% of anterior dislocations; however, these are neuropraxic in nature and tend to resolve on their own. Following the C5/C6 dermatome distribution, patients have a loss of sensation over the lateral aspect of the deltoid with decreased muscle contraction with abduction. After proper muscle relaxation with conscious sedation or intra-articular injection, closed reduction may be attempted using a variety of methods. After reduction, it is imperative to repeat a neurovascular examination and obtain confirmatory radiographs.

A 24-year-old man is brought into the ED by paramedics after being run over by a car. His systolic BP is 90 mm Hg by palpation, HR is 121 beats per minute, RR is 28 breaths per minute, and oxygen saturation is 100% on non-rebreather. The airway is patent and breath sounds are equal bilaterally. You establish large-bore access and fluids are running wide open. Secondary survey reveals an unstable pelvis upon movement with lateral to medial force. Bedside focused assessment by sonography for trauma (FAST) is negative for intraperitoneal fluid. Which of the following is the most appropriate immediate next step in management? a. Bilateral chest tubes b. Application of external fixator c. Application of pelvic binding apparatus d. Venographic embolization e. Angiographic embolization

*C* This patient is hemodynamically unstable with a pelvic fracture. The retroperitoneum can accommodate up to 4 L of blood after severe pelvic trauma. A few options are useful in managing hemorrhage from an unstable pelvic fracture. However, the initial and simplest modality to use in a patient in shock from a pelvis fracture is placement of a pelvic binding garment. This device can be applied easily and rapidly and is typically effective in tamponading bleeding and stabilizing the pelvis.

A 79-year-old woman with a history of coronary artery disease who underwent a coronary artery bypass graft (CABG) surgery in 2000 is brought to the emergency department (ED) by her family for 2 days of worsening shortness of breath. For the past 2 days, she has not gotten out of bed and is confused. She does not have chest pain, fevers, or cough. Her temperature is 98.1°F, blood pressure (BP) is 85/50 mm Hg, heart rate (HR) is 125 beats per minute, and respiratory rate (RR) is 26 breaths per minute. On examination, she is unable to follow commands and is oriented only to name. The cardiovascular examination reveals tachycardia with no murmurs. Her lungs have rales bilaterally at the bases. The abdomen is soft, nontender, and nondistended. Lower extremities have 2+ edema to the knee bilaterally. Which of the following is the most likely diagnosis? a. Hypovolemic shock b. Neurogenic shock c. Cardiogenic shock d. Anaphylactic shock e. Septic shock

*C* This patient is in cardiogenic shock from decreased cardiac output producing inadequate tissue perfusion. Support for this diagnosis includes an older patient with a history of coronary artery disease, and new mental status changes coupled with signs of volume overload. Common causes of cardiogenic shock include acute MI, pulmonary embolism, COPD exacerbation, and pneumonia. This patient should be stabilized with IV pressors since there is already pulmonary congestion evident on examination. A rapid workup including ECG, chest x-ray (CXR), laboratory tests, echocardiogram, and hemodynamic monitoring should help confirm the etiology and direct specific treatment of the underlying cause.

A 47-year-old man is brought to the ED by EMS after being persistently agitated at a business meeting. The patient's coworkers state that he has been working nonstop for a day-and-a-half and that he always seemed like a healthy guy who frequented bars every night. EMS administered 25 g of dextrose and thiamine with no symptom improvement. In the ED, the patient is anxious, confused, tremulous, and diaphoretic. He denies any medical problems, medications, or drug ingestions. His BP is 182/92 mm Hg, HR is 139 beats per minute, temperature is 100.4°F, RR is 18 breaths per minute, and fingerstick glucose is 103 mg/dL. An ECG reveals sinus tachycardia. Which of the following is the next best step? a. Administer acetaminophen b. Administer folate c. Administer diazepam d. Recheck fingerstick glucose e. Administer labetalol

*C* This patient presents with alcohol withdrawal. Signs and symptoms of this condition occur along a continuum ranging from simple shakes to delirium tremens (DTs) following a reduction or cessation of alcohol. Early symptoms usually appear 6 to 8 hours after cessation of drinking and involve tremulousness, anxiety, mild hypertension, and tachycardia. In more severe withdrawal, these symptoms worsen and paranoia, auditory, and visual hallucinations may develop proceeding to DTs with severe autonomic hyperactivity and profound altered mental status. DTs usually occur 3 to 5 days after alcohol cessation and carry 5% to 15% mortality even with supportive care. Additionally, alcohol withdrawal seizures may occur anywhere from 6 to 48 hours after cessation of alcohol. Benzodiazepines are the mainstay of therapy in alcohol withdrawal, as well as in sympathomimetic overdose and sedative-hypnotic withdrawal.

A 72-year-old man presents to the ED with worsening dyspnea. His initial vitals include a HR of 93 beats per minute, BP of 110/50 mm Hg, and RR of 20 breaths per minute with an oxygen saturation of 88% on room air. The patient appears thin and anxious. He is using accessory muscles to breathe. Despite distant breath sounds, you hear end-expiratory rhonchi and a prolonged expiratory phase. An ECG shows peaked P waves in leads II, III, and aVF. Given this patient's history and physical examination, which of the following conditions does this patient most likely have? a. Chronic bronchitis b. Asthma c. Emphysema d. Congestive heart failure (CHF) e. Pneumothorax

*C.* COPD is often referred to as a single disease but is in fact a triad of three distinct disease processes: emphysema, chronic bronchitis, and asthma. This clinical scenario paints a typical picture of emphysema. Individuals appear dyspneic, thin, and anxious. They generally have an increased anterior-posterior (AP) chest diameter and often use their accessory muscles to help with breathing. Lungs sounds are distant and associated with wheezes, rhonchi, and a prolonged expiratory phase. These individuals are often classified as "pink puffers" and use their pursed lips to push air that remains trapped in alveoli owing to the prolonged expiratory phase. Exacerbations should be treated with corticosteroids, anticholinergic inhalers, and intermittent β2-agonists to decrease inflammation, decrease mucous production, and relax smooth muscle in an effort to open up the distal airways. Patients with COPD are at higher risk for developing bacterial bronchitis and pneumonia. COPD is generally caused by smoking but may also result from air pollution, occupational exposure, and genetic factors, such as α1-antitrypsin deficiency. Patients may require supplemental outpatient oxygen to function and perform basic activities of daily living.

An 82-year-old woman becomes acutely short of breath while at rest on the rehabilitation unit. She is brought into the ED with an oxygen saturation of 86% on room air and in acute respiratory distress. Her initial ECG is within normal limits and unchanged from a recent previous examination. Her initial chest x-ray is also negative. Upon chest auscultation, there are equal bilateral breath sounds with some scattered rhonchi. Her nurse tells you that 2 days ago she underwent internal fixation of a right-femur fracture and has been on anticoagulant therapy. Given the history and presentation of this patient, what is the most likely etiology of her symptoms? a. Venous thromboembolism b. Air embolism c. Fat embolism d. Pulmonary hemorrhage e. Rib fracture

*C.* Fat embolism refers to the presence of fat globules within the lung parenchyma and peripheral circulation after a long bone fracture, major trauma, or orthopedic procedure. Symptoms usually appear 1 to 2 days after the event or intramedullary nailing. Unlike thromboemboli, fat emboli may pass through the pulmonary vasculature into the systemic arterial circulation where any organ may be affected. Respiratory distress is a common initial system with subsequent neurologic manifestations given that the cerebral circulation is at particular risk. Treatment is primarily supportive in an intensive care setting.

55-year-old man presents to the ED at 2:00 AM with left-sided chest pain that radiates down his left arm. He takes a β-blocker for hypertension, a proton-pump inhibitor for gastroesophageal reflux disease, and an antilipid agent for high cholesterol. He also took sildenafil the previous night for erectile dysfunction. His BP is 130/70 mm Hg and HR is 77 beats per minute. Which of the following medication is contraindicated in this patient? a. Aspirin b. Unfractionated heparin c. Nitroglycerin d. Metoprolol e. Morphine sulfate

*C.* It is contraindicated to administer nitroglycerin to individuals who have taken sildenafil in the previous 24 hours. The combination of nitroglycerin and sildenafil can lead to hypotension and death.

A 51-year-old man presents to the ED complaining of epigastric pain that radiates to his back. He states that he drinks a 6 pack of beer daily. You suspect he has pancreatitis. His BP is 135/75 mm Hg, HR is 90 beats per minute, temperature is 100.1°F, and his RR is 17 breaths per minute. Laboratory results reveal WBC 13,000/μL, hematocrit 48%, platelets 110/μL, amylase 1150 U/L, lipase 1450 IU, lactate dehydrogenase (LDH) 150 U/L, sodium 135 mEq/L, potassium 3.5 mEq/L, chloride 105 mEq/L, bicarbonate 23 mEq/L, BUN 15 mg/dL, creatinine 1.1 mg/dL, and glucose 125 mg/dL. Which of the following laboratory values are most specific for pancreatitis? a. Elevated amylase b. Hyperglycemia c. Elevated lipase d. Elevated LDH e. Leukocytosis

*C.* Lipase is a pancreatic enzyme that hydrolyzes triglycerides. In the presence of pancreatic inflammation it increases within 4 to 8 hours and peaks at 24 hours. At five times the upper limits of normal, lipase is 60% sensitive and 100% specific for pancreatitis. The diagnosis is usually made with a lipase of two times the normal limit, thereby increasing its sensitivity.

As you evaluate a patient with shortness of breath, you appreciate decreased breath sounds at the left-lung base. You suspect the patient has a small pleural effusion. Which of the following views of the chest is this small pleural effusion most likely to be detected? a. Supine b. Lateral decubitus right-side down c. Lateral decubitus left-side down d. Lateral e. Posterior-anterior (PA)

*C.* Pleural effusions are most easily detected on a lateral decubitus film with the affected side down, which in this case is the left side. Accumulations of 5 to 50 mL of fluid can be detected with this view

Which of the following patients has the *lowest* clinical probability for the diagnosis of pulmonary embolism? a. A 21-year-old woman 2 days after a cesarean delivery b. A 55-year-old woman on estrogen replacement therapy who underwent a total hip replacement procedure 3 days ago c. A 39-year-old man who smokes cigarettes occasionally and underwent an uncomplicated appendectomy 2 months ago d. A 62-year-old man with pancreatic cancer e. A 45-year-old man with factor V Leiden deficiency

*C.* Risk factors for venous thromboembolism were first described by Virchow triad: hypercoagulability, stasis, and endothelial injury. Hypercoagulability can be broadly classified into malignancy-related or nonmalignancy-related. Malignancies of primary adenocarcinoma or brain malignancy are the most likely to cause thrombosis. Some causes of nonmalignancy-related thrombosis are estrogen use, pregnancy, antiphospholipid syndromes, factor V Leiden mutation, and protein C and S deficiencies. Immobility such as paralysis, debilitating diseases, or recent surgery or trauma also place patients at risk.

A 30-year-old man is brought to the ED by emergency medical service (EMS) in respiratory distress. His initial vitals include a HR of 109 beats per minute, a BP of 180/90 mm Hg, and an RR of 20 breaths per minute with an oxygen saturation of 92% on room air. A chest x-ray shows a bilateral diffuse infiltrative process. A subsequent toxicologic screen is positive. Which of the following agents is most likely responsible for this patient's presentation? a. Cannabis b. Opioid c. Crack cocaine d. Methamphetamine e. Ethanol

*C.* The crystallized free base of cocaine is known as "crack cocaine." This form, smoked through a pipe, produces a highly lipid-soluble vapor that allows for rapid transport from the lungs to the brain for a quicker high compared to cocaine that is snorted or injected. As a result of this mechanism, the substance can be concentrated in high amounts in the lung parenchyma causing an infiltrative inflammatory process and pneumonitis referred to as "crack lung." This can subsequently result in respiratory failure.

A 55-year-old man with hypertension and a one-pack-per-day smoking history presents to the ED complaining of three episodes of severe heavy chest pain this morning that radiated to his left shoulder. In the past, he experienced chest discomfort after walking 20 minutes that resolved with rest. The episodes of chest pain this morning occurred while he was reading the newspaper. His BP is 155/80 mm Hg, HR 76 beats per minute, RR 15 breaths per minute. He does not have chest pain in the ED. An ECG reveals sinus rhythm with a rate of 72. A troponin I is negative. Which of the following best describes this patient's diagnosis? a. Variant angina b. Stable angina c. Unstable angina d. Non-ST-elevation MI e. ST-elevation MI

*C.* The patient exhibits unstable angina, which is defined as new-onset angina, angina occurring at rest lasting longer than 20 minutes, or angina deviating from a patient's normal pattern. Unstable angina is considered the harbinger of an acute MI and, therefore, should be evaluated and treated aggressively. Unstable angina is one of the three diagnoses that make up ACS, the other two being stable angina and acute MI (ST or non-ST elevation). Patients may be pain-free and have negative cardiac biomarkers with unstable angina. In general, unstable angina is treated with oxygen, aspirin, clopidogrel, low molecular weight or unfractionated heparin, and further risk stratification in the hospital.

A 22-year-old college student went to the health clinic complaining of a fever over the last 5 days, fatigue, myalgias, and a bout of vomiting and diarrhea. The clinic doctor diagnosed him with acute gastroenteritis and told him to drink more fluids. Three days later, the student presents to the ED complaining of substernal chest pain that is constant. He also feels short of breath. His temperature is 100.9°F, HR is 119 beats per minute, BP is 120/75 mm Hg, and RR is 18 breaths per minute. An ECG is performed revealing sinus tachycardia. A chest radiograph is unremarkable. Laboratory tests are normal except for slightly elevated WBCs. Which of the following is the most common cause of this patient's diagnosis? a. Streptococcus viridans b. Staphylococcus aureus c. Coxsackie B virus d. Atherosclerotic disease e. Cocaine abuse

*C.* The patient has myocarditis. The enteroviruses, especially the coxsackievirus B, predominate as causative agents in the United States. Coxsackie B virus usually causes infection during the summer months. Some other causes of myocarditis include adenovirus, influenza, HIV, Mycoplasma, Trypanosoma cruzi, and steroid abuse. Flu-like complaints, such as fatigue, myalgias, nausea, vomiting, diarrhea, and fever, are usually the earliest symptoms and signs of myocarditis. Tachycardia is common and can be disproportionate to the patient's temperature (ie, HR faster than what is expected). This may be the only clue that something more serious than a simple viral illness exists. Approximately 12% of patients also complain of chest pain. Cardiac enzymes may be elevated and the CBC and erythrocyte sedimentation rate (ESR) are nonspecific.

A 27-year-old woman presents to the ED complaining of an intensely pruritic rash all over her body, abdominal cramping, and chest tightness. She states that 1 hour ago she was at dinner and accidentally ate some shrimp. She has a known anaphylactic allergy to shrimp. Her BP is 115/75 mm Hg, HR is 95 beats per minute, temperature is 98.9°F, RR is 20 breaths per minute, and oxygen saturation is 97% on room air. She appears anxious, and her skin is flushed with urticarial lesions. Auscultation of her lungs reveals scattered wheezes with decreased air entry. Which of the following is the most appropriate next step in management? a. Administer oxygen via non-rebreather, place a large-bore IV, begin IV fluids, and administer methylprednisolone intravenously. b. Administer oxygen via non-rebreather, place a large-bore IV, begin IV fluids, and administer methylprednisolone and diphenhydramine intravenously. c. Administer oxygen via non-rebreather, place a large-bore IV, begin IV fluids, administer methylprednisolone and diphenhydramine intravenously, and give subcutaneous epinephrine. d. Administer oxygen via non-rebreather, place a large-bore IV, begin IV fluids, and start aerosolized albuterol. e. Administer oxygen via non-rebreather, place a large-bore IV, begin IV fluids, and start aerosolized epinephrine.

*C.* The patient is having an anaphylactic reaction to the shrimp she ate. Anaphylaxis refers to a severe systemic allergic reaction with variable features such as respiratory difficulty, cardiovascular collapse, pruritic skin rash, and abdominal cramping. Anaphylaxis is a hypersensitivity reaction caused by an IgE-mediated reaction. Foods are the major cause in cases of anaphylaxis in which a source can be determined. Common foods that cause anaphylaxis include nuts, shellfish, and eggs. In the ED, attention is focused on reversing cardiovascular and respiratory disturbances. Epinephrine is the first drug of choice for patients with anaphylaxis. The route of administration is chosen by the severity of the patient's presentation. In a patient with upper airway obstruction or hypotension, IV epinephrine should be administered. Patients with stable vital signs can receive subcutaneous epinephrine. Epinephrine should be used with caution in the elderly or any patient with coronary artery disease or dysrhythmias. Antihistamines, such as diphenhydramine and ranitidine, should be used in all cases. These drugs block the action of circulating histamines at target tissue receptors. Corticosteroids, such as methylprednisolone, have an onset of action approximately 4 to 6 hours after administration and, therefore are of limited value in the acute setting. However, since giving them early may blunt the biphasic reaction of anaphylaxis and therefore, it is advised to administer to patients in anaphylaxis.

A 78-year-old woman is brought to the ED by EMS complaining of vomiting and abdominal pain that began during the night. EMS reports that her BP is 90/50 mm Hg, HR is 110 beats per minute, temperature is 101.2°F, and RR is 18 breaths per minute. After giving her a 500-mL bolus of NS, her BP is 115/70 mm Hg. During the examination, you notice that her face and chest appear jaundiced. Her lungs are clear to auscultation and you do not appreciate a murmur on cardiac examination. She winces when you palpate her RUQ. An ultrasound reveals dilation of the common bile duct and stones in the gallbladder. What is the most likely diagnosis? a. Cholecystitis b. Acute hepatitis c. Cholangitis d. Pancreatic cancer e. Bowel obstruction

*C.* The patient's clinical picture is consistent with cholangitis, which is caused by an obstruction of the biliary tract leading to bacterial infection. Obstruction is commonly secondary to a stone, but may be because of malignancy or stricture. Cholangitis is a surgical emergency. The classic triad of physical findings described by Charcot is RUQ pain, fever, and jaundice. Sepsis is a common complication. Sonography may demonstrate intrahepatic or ductal dilation. The presence of stones in the gallbladder suggests obstruction as the etiology.

A 31-year-old man from Florida presents to the ED complaining of severe pain that starts in his left flank and radiates to his testicle. The pain lasts for about 1 hour and then improves. He had similar pain last week that resolved spontaneously. He noted some blood in his urine this morning. His BP is 145/75 mm Hg, HR is 90 beats per minute, temperature is 98.9°F, and his RR is 24 breaths per minute. His abdomen is soft and nontender. As you examine the patient, he vomits and has trouble lying still in his stretcher. Which of the following is the most appropriate next step in management? a. Call surgery consult to evaluate the patient for appendicitis. b. Order an abdominal CT. c. Start intravenous (IV) fluids and administer an IV nonsteroidal anti-inflammatory drug (NSAID) and antiemetic. d. Perform an ultrasound to evaluate for an abdominal aortic aneurysm (AAA). e. Perform an ultrasound to evaluate for testicular torsion.

*C.* The patient's history of colicky flank pain that radiates to the groin and hematuria is consistent with a ureteral stone. Adequate analgesia is critical in treating a patient with a ureteral stone. Intravenous ketorolac, an nonsteroidal anti-inflammatory drug (NSAID), is frequently administered as a first-line analgesic, but morphine may be necessary for continued pain. In addition to their analgesia, NSAIDs decrease ureterospasm and renal capsular pressure in the obstructed kidney. Antiemetics, such as metoclopramide, help with the nausea and vomiting.

While playing a match of tennis, a 56-year-old man with a medical history significant only for acid reflux disease starts to feel substernal chest pain that radiates into his left arm and shortness of breath. His pain feels better after drinking antacid, but since it is not completely resolved, his partner calls 911. Upon arrival, EMS administers aspirin and sublingual nitroglycerin. After 20 minutes, the man's symptoms resolve. He is brought to the ED for further evaluation where his ECG shows sinus rhythm without any ischemic abnormalities. You order a chest radiograph and send his blood work to the laboratory for analysis. Which of the following statements regarding the diagnosis of acute MI is most accurate? a. A normal ECG rules out the diagnosis of acute MI. b. One set of negative cardiac enzymes is sufficient to exclude the diagnosis of MI in this patient. c. Troponin may not reach peak levels for at least 12 hours. d. Relief of symptoms by antacids essentially rules out a cardiac cause of his chest pain. e. Epigastric discomfort and indigestion is a rare presentation of ACS.

*C.* The patient's presentation is concerning for a cardiac cause of his chest pain. Chest pain radiating to the left arm that is associated with shortness of breath is a classic presentation of ACS. On arrival to the ED, the patient should be placed on a monitor, receive oxygen by nasal cannula, have an IV placed and blood sent to the laboratory, and an ECG performed. Any abnormalities in his vital signs should also be addressed. Serum cardiac markers are useful in detecting MI. Troponin T and I appear in the serum within 6 hours of symptom onset and remain elevated for 1 to 2 weeks. Troponin I is the most specific cardiac marker available (almost 100%) and peaks between 12 and 18 hours. However, the disposition of patients with suspected acute coronary syndromemshould be based on the clinical examination and not the cardiac enzymes. Initial determination of these markers has a low sensitivity for detecting ischemia and cannot be used to reliably diagnose or exclude the presence of an ACS.

A 57-year-old man complains of chest palpitations and lightheadedness for the past hour. Five years ago he underwent a cardiac catheterization with coronary artery stent placement. He smokes half a pack of cigarettes daily, and drinks a glass of wine at dinner. His HR is 140 beats per minute, BP is 115/70 mm Hg, and oxygen saturation is 99% on room air. An ECG reveals a wide complex tachycardia at a rate of 140 that is regular in rhythm. An ECG from 6 months ago shows a sinus rhythm at a rate of 80. Which of the following is the most appropriate medication to treat this dysrhythmia? a. Digoxin b. Diltiazem c. Amiodarone d. Adenosine e. Bretylium

*C.* This patient has a ventricular tachycardia defined by a QRS complex greater than 120 ms and a rate greater than 100 beats per minute. Ventricular tachycardia is the result of a dysrhythmia originating within or below the termination of the His bundle. Most patients with ventricular tachycardia have underlying heart disease. Treatment begins with assessing whether or not the patient is stable. If the patient shows signs of instability, such as hypotension or altered mental status, then cardioversion should be performed. However, if the patient is stable, medications can be administered to treat the dysrhythmia. Amiodarone, a class III antidysrhythmic that has pharmacologic characteristics of all fourclasses, is considered a first-line agent in treating ventricular dysrhythmias.

A 42-year-old man presents to the ED via ambulance after activating EMS for dyspnea. He is currently on an oxygen face mask and was administered one nebulized treatment of a β2-agonist by the paramedics. His initial vitals include an RR of 16 breaths per minute with an oxygen saturation of 96% on room air. The patient appears to be in mild distress with some intercostal retractions. Upon chest auscultation, there are minimal wheezes localized over bilateral lower lung fields. The patient's symptoms completely resolve after two more nebulizer treatments. Which of the following medications, in addition to a rescue β2-agonist inhaler, should be prescribed for outpatient use? a. Magnesium sulfate b. EpiPen c. Corticosteroids d. Cromolyn sodium e. Ipratropium

*C.*Corticosteroids have been shown to improve asthma symptoms in subsequent days after an exacerbation and prevent acute recurrences in patients who are deemed suitable to be discharged from the ED. An acceptable dosage is 40- to 60-mg prednisone daily for 3 to 10 days after the initial event. Inhaled steroids may also be an alternative to prevent relapses in more intractable cases, and should be used daily with the guidance of the patient's primary-care provider. Spacers are available to ensure adequate delivery of the medications deep into the alveoli.

A 76-year-old man presents to the ED in acute respiratory distress, gasping for breath while on face mask. Paramedics state that he was found on a bench outside of his apartment in respiratory distress. Initial vitals include a HR of 90 beats per minute, a BP of 170/90 mm Hg, and an RR of 33 breaths per minute with an oxygen saturation of 90%. Upon physical examination, the patient is coughing up pink, frothy sputum, has rales two-thirds of the way up both lung fields, and has pitting edema of hislower extremities. A chest radiograph reveals bilateral perihilar infiltrates, an enlarged cardiac silhouette, and a small right-sided pleural effusion. After obtaining IV access and placing the patient on a monitor, which of the following medical interventions is most appropriate? a. Morphine sulfate only b. Nitroglycerin only c. Nitroglycerin and a loop diuretic d. Aspirin e. Antibiotics

*C.*Pulmonary edema can be divided into cardiogenic and noncardiogenic. Cardiogenic varieties are commonly seen in the ED and are usually a result of high hydrostatic pressures. It is seen in patients with MI or ischemia, cardiomyopathies, valvular heart disease, and hypertensive emergencies. Nitroglycerin acts to decrease the preload of the heart by venous dilation. This lowers the work of the heart so that it can function more effectively. A loop diuretic is used to induce diuresis and is also thought to act as a venous dilator. In conjunction with one another, these medications act to improve the overall functional capacity of the heart. If medical interventions are not stabilizing, preparation should be made for endotracheal intubation. Positive airway pressure devices (eg, BiPAP) may also be used as a temporizing measure for oxygen delivery.

A 57-year-old woman presents to the ED with a basin in her hand and actively vomiting. You insert an IV catheter, start IV fluids, and administer an antiemetic agent. The patient feels much better but also complains of severe crampy abdominal pain that comes in waves. You examine her abdomen and note that it is distended and that there is a small midline scar in the lower abdomen. Upon auscultation, you hear high-pitched noises that sound like "tinkles." Palpation elicits pain in all four quadrants but no rebound tenderness. She is guaiac negative. Which of the following is the most common cause of this patient's presentation? a. Travel to Mexico b. Ethanol abuse c. Hysterectomy d. Hernia e. Constipation

*C.*The patient presents with the clinical findings of SBO: vomiting, intermittent crampy abdominal pain, abdominal distention, hyperactive bowel sounds, and general tenderness. The most common cause of SBO in developed countries is postoperative adhesions, responsible for more than 50% of all SBO. There is a particularly high incidence of SBO after gynecologic surgeries, such as a hysterectomy

A 31-year-old woman with a known psychiatric history presents to the ED after ingesting an unknown quantity of pills from her medication vial. In the ED, she complains of nausea, abdominal cramping, and feels unsteady on her feet. On physical examination you observe that she is tremulous, ataxic, and exhibits dystonia. Which of the following substances will not be adsorbed by activated charcoal? a. Digoxin b. Diphenhydramine c. Amitriptyline d. Lithium e. Acetaminophen

*D* A major change has occurred in the approach of GI decontamination over the past decade. Previous recommendations indicated that the stomach should be emptied by either syrup of ipecac or by gastric lavage. Activated charcoal alone has demonstrated similar or superior results and now is the recommended GI decontaminant. The complications of gastric emptying procedures, primarily aspiration, are largely avoided when only activated charcoal is used. Most ingested drugs and chemicals are adsorbed to activated charcoal. The few agents that do not adsorb to charcoal include ions (eg, mineral acids and alkalis, lithium, borates, bromides), hydrocarbons (HCs), metals (eg, iron), and ethanol. The patient in question ingested lithium that is used to treat her bipolar disorder. A lithium level should be drawn and management decided based on those results. Potential treatments include whole-bowel irrigation and dialysis. The substances in the other answer choices are absorbed by activated charcoal.

165. Paramedics bring a 55-year-old woman to the ED after she was struck by a motor vehicle traveling at 30 mile/h. Her BP is 165/95 mm Hg, HR is 105 beats per minute, and RR is 20 breaths per minute. Upon arrival, she does not open her eyes, is verbal but not making any sense, and withdraws to painful stimuli. You assign her a GCS score of 8. As you prepare to intubate the patient, a colleague notices that her left pupil has become dilated compared to the right. Which of the following has the quickest effect to reduce ICP? a. Cranial decompression b. Dexamethasone c. Furosemide d. Hyperventilation e. Mannitol

*D* A unilateral dilated pupil in the setting of head trauma is an indicator of increased ICP. If ICP is not lowered immediately, the patient has little chance of survival. Hyperventilation to produce an arterial PCO2 of 30 to 35 mm Hg will temporarily reduce ICP by promoting cerebral vasoconstriction and subsequent reduction of cerebral blood flow. The onset of action is within 30 seconds. In most patients, hyperventilation lowers the ICP by 25%. PCO2 should not fall below 25 mm Hg because this may cause profound vasoconstriction and ischemia in normal and injured areas of the brain. Hyperventilation is a temporary maneuver and should only be used for a brief period of time during the acute resuscitation and only in patients demonstrating neurologic deterioration.

A 70-year-old woman with a history of hypertension, congestive heart failure, and atrial fibrillation presents to the ED with several hours of acute onset diffuse abdominal pain. She denies any nausea or vomiting. The pain is constant but she is unable to localize it. She was diagnosed with a renal artery thrombosis several years ago. Vital signs include HR of 95 beats per minute, BP of 110/70 mm Hg, and temperature of 98°F. Her abdomen is soft and mildly tender, despite her reported severe abdominal pain. Her WBC count is 12,000/μL, hematocrit 38%, platelets 250/μL, and lactate 8 mg/dL. The stool is trace heme-positive. You are concerned for acute mesenteric ischemia. What is the best way to diagnose this condition? a. Serum lactate levels b. Abdominal radiograph (supine and upright) c. CT scan d. Angiography e. Barium contrast study

*D* Angiography remains the "gold standard" in the diagnosis of mesenteric ischemia. Unlike any other diagnostic tools, it is capable of both diagnosing and treating the problem. It is capable of identifying all four types of acute mesenteric ischemia: (1) arterial embolus, (2) arterial or (3) venous thrombosis, and, under most circumstances, (4) nonocclusive mesenteric ischemia. Angiography should be obtained without delay when the diagnosis is suspected.

As you arrive for your ED shift, you are called to help with a "coding" patient. The senior resident has just intubated the patient and the nurses have established IV access and attached the cardiac monitor. An emergency medical treatment (EMT) student is performing chest compressions. You ask the EMT student to stop compressions. The monitor shows a flat line with no electrical activity. You are unable to detect any pulses. What is your next step in management? a. Defibrillate at 360 J b. Epinephrine 1-mg IV push c. Atropine 1-mg IV push d. Ask the nurse to run a rhythm strip in an additional lead e. Apply transcutaneous pacers

*D* Asystole is absent heart rhythm or more colloquially, "flat line." A common cause of asystole is a disconnected lead or malfunctioning equipment, so the AHA recommends confirmation of asystole by switching to another lead on the cardiac monitor. Confirmation can also be achieved with a 12-lead ECG if the equipment is readily available.

A 48-year-old man is brought to the ED by paramedics for generalized weakness. His medical history is significant for a CABG last month. He has been unable to get out of bed for the past day because of dizziness when changing position. He denies chest pain, shortness of breath, or syncope. His temperature is 98.9°F, BP is 86/60 mm Hg, HR is 44 beats per minute, RR is 18 breaths per minute, and oxygen saturation is 98% on room air. There is a well-healing midline sternotomy incision. Cardiac examination reveals a III/VI systolic ejection murmur. There are minimal rales at his lung bases. He is immediately attached to the cardiac monitor. His rhythm strip is shows _______. What is your initial treatment? a. Observe on monitor b. Transcutaneous pacing c. Transvenous pacing d. Atropine 0.5-mg IV e. Epinephrine IV drip at 2 μg/min

*D* Atropine is the initial treatment of choice for patients in second-degree, Mobitz I AV block. The majority of patients respond to atropine without further treatment. Mobitz I is commonly seen with acute inferior MI, digoxin toxicity, myocarditis, and after cardiac surgery.

55-year-old woman with a past medical history of diabetes presents to the ED with fevers, headache, vision complaints, and right-sided weakness. She was treated for otitis media 2 weeks ago with amoxicillin as an outpatient. CT shown in answer. What is the most likely diagnosis? a. Central nervous system (CNS) toxoplasmosis b. Subdural hygroma c. Glioblastoma multiforme d. Brain abscess e. SAH

*D* Brain abscesses are uncommon and their incidence has decreased over the past several years as a result of better antibiotic treatment of the remote infections that cause them. Today, the majority of brain abscesses in developed countries are the result of contiguous spread from otitis media, mastoiditis, paranasal sinusitis, or meningitis. They can also occur after trauma, classically after a basilar skull fracture. Presentation is often nonspecific, with almost half present with headache alone. Focal weakness, fevers, and nausea are other common presenting complaints. Antibiotic choice should be guided by suspected source and ability to penetrate the CNS. In general, MRI is more sensitive at detecting CNS infection than CT. With contrast CT, the walls of the abscess enhance and there is a central necrotic area of lower density. On this CT with IV contrast, there are multiple ring-enhancing lesions with surrounding edema in the left frontal lobe. The large extra-axial collections with enhancing margins represent emphysemas. There is also midline shift secondary to mass effect of the abscesses.

A 29-year-old man is brought to the ED by EMS after being stabbed in the left side of his back. His BP is 120/80 mm Hg, HR is 105 beats per minute, RR is 16 breaths per minute, and oxygen saturation is 98% on room air. On the secondary survey, you note motor weakness of his left lower extremity and the loss of pain sensation in the right lower extremity. Which of the following is the most likely diagnosis? a. Spinal shock b. Central cord syndrome c. Anterior cord syndrome d. Brown-Séquard syndrome e. Cauda equina syndrome

*D* Brown-Séquard syndrome or hemisection of the spinal cord, typically results from penetrating trauma, such as a gunshot or knife wound. Patients with this lesion have ipsilateral motor paralysis and contralateral loss of pain and temperature distal to the level of the injury. This syndrome has the best prognosis for recovery of all of the incomplete spinal cord lesions.

A 61-year-old man presents to the ED with low back pain after slipping on an icy sidewalk yesterday. He states that the pain started on the left side of his lower back and now involves the right and radiates down both legs. He also noticed difficulty urinating since last night. On neurologic examination, he cannot plantar flex his feet. Rectal examination reveals diminished rectal tone. He has a medical history of chronic hypertension and underwent a "vessel surgery" many years earlier. Which of the following is the best diagnosis? a. Abdominal aortic aneurysm b. Disk herniation c. Spinal stenosis d. Cauda equina syndrome e. Osteomyelitis

*D* Cauda equina syndrome is an injury to the lumbar, sacral, and coccygeal nerve roots, causing peripheral nerve injury that can lead to permanent neurologic defects if not recognized and corrected rapidly. Because of the central location of the disk herniation, symptoms are often bilateral and involve leg pain, saddle anesthesia, and impaired bowel and bladder function (retention or incontinence). On examination, patients may exhibit loss of rectal tone and display other motor and sensory losses in the lower extremities. Patients with suspected cauda equina syndrome require an urgent CT scan or MRI.

A 23-year-old man presents to the ED with left lower-abdominal pain and left testicular pain that started 1 to 2 weeks ago and has gradually worsened. He denies nausea and vomiting. His HR is 98 beats per minute, BP is 125/65 mm Hg, temperature is 100.9°F, and RR is 18 breaths per minute. Physical examination reveals a tender left testicle with a firm nodularity on the posterolateral aspect of the testicle. Pain is relieved slightly with elevation of the testicle and the cremasteric reflex in normal. Which of the following is the next best step? a. Prescribe pain medications and penicillin for coverage of syphilis. b. Recommend bed rest and scrotal elevation with urology follow-up. c. Attempt to untwist the left testicle by rotating it in a clockwise direction and order an immediate scrotal ultrasound. d. Give ceftriaxone 250 mg intramuscularly (IM), plus a 10-day course of oral doxycycline. e. Confirm the diagnosis with transillumination of the testicle, and then consult urology for surgical drainage.

*D* Epididymitis is an inflammation of the epididymis, often caused by age-dependent bacterial infection. Sexually transmitted diseases (STDs), such as gonorrhea and Chlamydia, are most common in patients younger than 35 years, while urinary pathogens, such as Escherichia coli and Klebsiella, are most common in patients older than 40 years. Unlike testicular torsion, the onset of pain in epididymitis is usually gradual and the cremasteric reflex is (usually?) intact. Scrotal elevation may transiently relieve pain (positive Prehn sign). Treatment includes bed rest, scrotal elevation or support when ambulating, avoidance of heavy lifting, and antibiotics for infection.

A 50-year-old man is brought to the ED by ambulance with significant hematemesis. In the ambulance, paramedics placed two large-bore IVs and began infusing normal saline. In the ED, his HR is 127 beats per minute, BP is 79/45 mm Hg, temperature is 97.9°F, RR is 24 breaths per minute, and oxygen saturation is 96%. On physical examination, his abdomen is nontender, but you note spider angiomata, palmar erythema, and gynecomastia. Laboratory results reveal WBC 9000/μL, hematocrit 28%, platelets 40/μL, aspartate transaminase (AST) 675 U/L, alanine transaminase (ALT) 325 U/L, alkaline phosphatase 95 U/L, total bilirubin 14.4 mg/dL, conjugated bilirubin 12.9 mg/dL, sodium 135 mEq/L, potassium 3.5 mEq/L, chloride 110 mEq/L, bicarbonate 26 mEq/L, blood urea nitrogen (BUN) 20 mg/dL, creatinine 1.1 mg/dL, and glucose 150 mg/dL. Which of the following is the most likely diagnosis? a. Perforated gastric ulcer b. Diverticulosis c. Splenic laceration d. Esophageal varices e. Ruptured AAA

*D* Esophageal varices develop in patients with chronic liver disease in response to portal hypertension. Approximately 60% of patients with portal hypertension will develop varices. Of those who develop varices, 25% to 30% will experience hemorrhage. Patients who develop varices from alcohol abuse have an even higher risk of bleeding, especially with ongoing alcohol consumption. This patient has evidence of chronic liver disease with thrombocytopenia and elevated bilirubin and liver enzymes. In alcoholic hepatitis, the AST is greater than the ALT by a factor of 2. Spider angiomata, palmar erythema, and gynecomastia further suggest underlying liver disease.

A 47-year-old man presents to the ED complaining of knee pain after slipping on ice on his way to work. On examination, you note significant swelling to his knee and palpate a large effusion. The patient has limited range of motion as a result of the swelling. You perform a palliative arthrocentesis and note blood and fat globules in the syringe. Which of the following findings is the presence of fat globules most suggestive of? a. ACL tear b. PCL tear c. Medial meniscus tear d. Fracture e. Vascular injury

*D* Frequently, there is a fracture to account for the presence of fat within the joint because the fat has entered from the bone marrow cavity. The presence of fat globules may be seen in the joint aspirate and even on the radiograph of the knee showing a joint effusion with a fat-fluid level.

A 45-year-old woman presents to the ED with 1 day of painful rectal bleeding. Review of systems is negative for weight loss, abdominal pain, nausea, and vomiting. On physical examination, you note an exquisitely tender swelling with engorgement and a bluish discoloration distal to the anal verge. Her vital signs are HR 105 beats per minute, BP 140/70 mm Hg, RR 18 breaths per minute, and temperature 99°F. Which of the following is the next best step in management? a. Recommend warm sitz baths, topical analgesics, stool softeners, a high-fiber diet, and arrange for surgical follow-up. b. Incision and drainage under local anesthesia or procedural sedation followed by packing and surgical follow-up. c. Obtain a complete blood cell (CBC) count, clotting studies, type and cross, and arrange for emergent colonoscopy. d. Excision under local anesthesia followed by sitz baths and analgesics. e. Surgical consult for immediate operative management.

*D* Hemorrhoids are dilated venules of the hemorrhoidal plexuses. They are associated with constipation, straining, increased abdominal pressure, pregnancy, increased portal pressure, and a low-fiber diet. Hemorrhoids can be either internal or external. Those that arise above the dentate line are internal and painless. Those that are below the dentate line are external and painful. Individuals commonly present with thrombosed external hemorrhoids. On examination, there is a tender mass at the anal orifice that is typically bluish-purple in color. If pain is severe and the thrombosis is less than 48 hours then the physician should excise the thrombus under local anesthesia followed by a warm sitz baths. This patient is suffering from an acutely thrombosed external hemorrhoid. If not excised, symptoms will most often resolve within several days when the hemorrhoid ulcerates and leaks the dark accumulated blood. Residual skin tags may persist. Excision provides both immediate and long-term relief and prevents the formation of skin tags

A 25-year-old man is brought into the trauma resuscitation room after his motorcycle is struck by another vehicle. EMS reports that the patient was found 20 ft away from his motorcycle, which was badly damaged. His vital signs include a BP of 90/60 mm Hg, HR of 115 beats per minute, RR of 2 breaths per minute, and pulse oxygenation of 100% on facemask Which of the following is the smallest amount of blood loss that produces a decrease in the systolic BP in adults? a. Loss of 5% of blood volume b. Loss of 10% of blood volume c. Loss of 15% to 30% of blood volume d. Loss of 30% to 40% of blood volume e. Loss of greater than 40% of blood volume

*D* Hypovolemia secondary to hemorrhage is the most common cause of shock in the trauma patient. The earliest signs of hemorrhagic shock are tachycardia and cutaneous vasoconstriction. The amount of blood loss present can be estimated based on the individual's initial clinical presentation. Class III hemorrhage is characterized by 30% to 40% blood loss (1500-2000 mL). This stage exhibits tachypnea, tachycardia (HR > 120), decrease in systolic BP, delayed capillary refill, decreased urine output, and a change in mental status.

A 27-year-old woman with known idiopathic intracranial hypertension (IIH) presents to the ED complaining of a bifrontotemporal headache several times a day for 6 weeks after running out of her medications. She complains of occasional pulsatile tinnitus but no visual disturbances. Funduscopic examination reveals no papilledema and normal venous pulsations. Which of the following factors determines the need for urgent treatment in patients with IIH? a. The presence of papilledema on funduscopic examination b. A history of pulsatile tinnitus c. Presence of an empty sella on CT scan d. Complaint of visual loss or visual disturbances e. A history of concomitant minocycline use

*D* IIH formerly pseudotumor cerebri, requires urgent treatment when there is a history of visual phenomena, particularly transient vision loss. Agents used to lower ICP in this setting include carbonic anhydrase inhibitors (ie, acetazolamide) and loop diuretics (ie, furosemide). Transient visual loss, pain, or blurring occurs frequently and can be permanent in up to 10% of patients. Treatment of the headache itself often employs the same agents used to treat migraines; ergots, antiemetics, and occasionally steroids. The headache is often refractory and difficult to manage. The condition is associated with obesity and weight loss is sometimes helpful.

A 65-year-old actively seizing woman is brought to the ED by EMS. She was found slumped over at the bus stop bench. EMS personnel state that when they found the woman she was diaphoretic and her speech was garbled. En route to the hospital, she started to seize. As you wheel her to a room, the nurse gives you some of her vital signs which are a BP of 150/90 mm Hg, HR of 115 beats per minute, and oxygen saturation of 96%. Which of the following is the next best step in managing this patient? a. Request a rectal temperature to rule out meningitis. b. Call the CT technologist and tell them you are bringing over a seizing patient. c. Ask for a stat ECG and administer an aspirin. d. Check the patient's fingerstick blood glucose level. e. Intubate the patient.

*D* In a patient who is actively seizing, it is essential to check the blood glucose level. Hypoglycemia is an easily reversible cause of seizure and is corrected with the administration of dextrose, not the usual anticonvulsants. Patients at both extremes of age are particularly susceptible to glucose stress during acute illness.

A 51-year-old man presents to the ED complaining of nausea and abdominal pain after drinking some "bitter stuff." He is considered one of the "regulars" who is usually at triage with ethanol intoxication. His temperature is 97.9°F, BP is 130/65 mm Hg, HR is 90 beats per minute, RR is 16 breaths per minute, and oxygen saturation is 97% on room air. Physical examination is unremarkable, except for slurred speech and the smell of acetone on the patient's breath. Laboratory results reveal serum sodium 138 mEq/L, potassium 3.5 mEq/L, chloride 105 mEq/L, bicarbonate 23 mEq/L, BUN 10 mg/dL, creatinine 1.7 mg/dL, glucose 85 mg/dL, arterial blood pH 7.37, and lactate 1.4 mEq/L. Urinalysis shows moderate ketones. Which of the following is the most likely diagnosis? a. Diabetic ketoacidosis (DKA) b. Ethanol intoxication c. Methanol intoxication d. Isopropyl alcohol intoxication e. Ethylene glycol intoxication

*D* Isopropyl alcohol is one of the toxic alcohols (ethylene glycol, methanol, and isopropyl alcohol). It is a clear, colorless liquid with a bitter taste. It is commonly used as a rubbing alcohol and as a solvent in haircare products, skin lotion, and home aerosols. Moreover, it is often ingested as an inexpensive and convenient substitute for ethanol. Clinically, GI and CNS complaints predominate. Its GI irritant properties cause patients to complain of abdominal pain, nausea, and vomiting. Pupillary size varies but miosis is commonly observed. Large ingestions can result in coma. Hypotension, although rare, signifies severe poisoning. Characteristically, metabolic acidosis, unlike the other toxic alcohols, is not present. This is because isopropyl alcohol is metabolized to acetone, a ketone, not an acid. It is also the cause for the presence of urinary ketones and the odor on the patient's breath. Isopropyl alcohol intoxication is often remembered by ketosis without acidosis. Another unique finding is the presence of "pseudo renal failure" or isolated false elevation of creatinine with a normal BUN. This results from interference of acetone and acetoacetate by the colorimetric method used to measure the creatinine level.

A 22-year-old college student presents to the ED with a painful, swollen finger. He states that he was playing with a roommate's dog and was bitten 3 days earlier. On physical examination, his heart rate (HR) is 70 beats per minute, blood pressure (BP) is 115/65 mm Hg, respiratory rate (RR) is 16 breaths per minute, and temperature is 101.6°F. Bilaterally, the radial pulses remain equal, and the hands and fingers are intact to sensation. His right second finger is held in flexion, and is symmetrically swollen from the distal tip to the MCP joint. Both your attempt to extend the digit passively and palpate the flexor tendon sheath produce great pain. Based on these findings you make your diagnosis and notify the hand surgeon immediately. What name is given to the criteria used to make the diagnosis? a. Finkelstein b. Trousseau c. Tinel d. Kanavel e. Phalen

*D* Kanavel criteria for flexor tenosynovitis are known as S.T.E.P.: (1) Symmetric swelling of the finger, (2) Tenderness over the flexor tendon sheath, (3) Extension (passive) of the digit is painful, and (4) Posture of the digit is flexed. A tenosynovitis is an infection of the flexor tendon sheath caused by penetrating trauma and dirty wounds (eg, a dog bite). Infection spreads along the tendon sheath; therefore, failure to diagnose and treat a flexor tenosynovitis may lead to loss of function of the affected digit and eventually the entire hand. It is, therefore, a surgical emergency. Treatment includes hand immobilization and elevation, immediate consultation with a hand surgeon, and IV antibiotics. Pain control and tetanus immunization should be provided, if not up-to-date.

You are called to the bedside of a hypotensive patient with altered mental status. The nurse hands you an ECG which shows atrial flutter at 150 beats per minute with 2:1 arteriovenous (AV) block. You feel that the patient is unstable and elect to perform emergency cardioversion. You attach the monitor leads to the patient. What is the critical next step in electrical cardioversion? a. Set the appropriate energy level b. Position conductor pads or paddles on patient c. Charge the defibrillator d. Turn on the synchronization mode e. Administer 25 μg of fentanyl IV

*D* Low-energy cardioversion is very successful in converting atrial flutter to sinus rhythm. Remember, cardioversion is different than defibrillation. Cardioversion is performed on patients with organized cardiac electrical activity with pulses, whereas defibrillation is performed on patients without pulses (VF and VF without a pulse). Patients with heart beats who receive electrical energy during their heart's relative refractory period are at risk for VF. Therefore, cardioversion is a timed shock designed to avoid delivering a shock during the heart's relative refractory period. By activating synchronization mode, the machine will identify the patient's R waves and not deliver electrical energy during these times. The key step when cardioverting is to activate the synchronization mode and confirm the presence of sync markers on the R waves prior to delivering electrical energy.

A 51-year-old diabetic man complains of intense right-ear pain and discharge. On physical examination, his BP is 145/65 mm Hg, HR 91 beats per minute, and temperature 101°F. He withdraws when you retract the pinna of his ear. The external auditory canal is erythematous, edematous, and contains what looks like friable granulation tissue in the external auditory canal. The tympanic membrane is partially obstructed but appears to be erythematous, as well. Which of the following statements regarding this patient's condition is true? a. It is a common complication of otitis externa that afflicts otherwise healthy patients. b. The mainstay of treatment is outpatient with oral antibiotics. c. Patients are usually afebrile and have minimal ear pain. d. It is caused by P aeruginosa. e. Hearing loss is the most common complication.

*D* Malignant or necrotizing otitis externa is a complication of otitis externa that occurs primarily in adult diabetics and debilitated and immunocompromised individuals. It is associated with a high mortality rate. The condition is better described as an osteitis of the underlying bone of the external auditory canal caused by P aeruginosa. It is distinguished by fever, intense ear pain, erythema, edema, and granulation tissue in the external canal. Cranial nerve palsies and trismus can also occur. These patients require hospitalization and treatment with IV antipseudomonal antibiotics and possible surgical debridement. CT and MRI are appropriate studies to evaluate for osteomyelitis. Patients also require immediate ENT consultation.

A 33-year-old woman presents to the ED complaining of fever, vomiting, and gradually worsening RUQ pain. She states that her pain radiates to her back. Her BP is 130/75 mm Hg, HR is 90 beats per minute, temperature is 100.9°F, and RR is 17 breaths per minute. While examining her abdomen you palpate her RUQ and notice that she momentarily stops her inspiration. What is the name of this classic sign? a. Grey-Turner sign b. Kernig sign c. McMurray sign d. Murphy sign e. McBurney sign

*D* Murphy sign is named after the Chicago surgeon, John B. Murphy. The patient is asked to take a deep breath while the examiner applies pressure over the area of the gallbladder. If the gallbladder is inflamed, the descending diaphragm forces it against the examiner's fingertips, causing pain and often a sudden pause to inspiration. A sonographic Murphy sign elicits the same response with an ultrasound probe over the gallbladder.

A 74-year-old lethargic woman is brought to the ED by her family. Her daughter states that the patient has been progressively somnolent over the last week and could not be woken up today. The patient takes medications for diabetes, hypertension, hypothyroidism, and a recent ankle sprain, which is treated with a hydrocodone/acetaminophen combination. In the ED, the patient is profoundly lethargic, responsive only to pain, and has periorbital edema and delayed relaxation of the deep tendon reflexes. Her BP is 145/84 mm Hg, HR is 56 beats per minute, temperature is 94.8°F, and RR is 12 breaths per minute. Which of the following is the most likely diagnosis? a. Hypoglycemia b. Opioid overdose c. Stroke d. Myxedema coma e. Depression

*D* Myxedema coma is a lifethreatening complication of hypothyroidism. Mortality in myxedema coma approaches 20% to 50% even with appropriate management. The patient exhibits classic signs and symptoms of the disease: lethargy or coma, hypothermia, bradycardia, periorbital and nonpitting edema, and a delayed relaxation phase of deep tendon reflexes (areflexia in more severe cases). Myxedema coma can be triggered by sepsis, trauma, surgery, congestive heart failure, prolonged cold exposure, or use of sedatives or narcotics (as seen in this example).

A 27-year-old man complains of chest palpitations and lightheadedness for the past hour. He has no past medical history and is not taking any medications. He drinks a beer occasionally on the weekend and does not smoke cigarettes. His HR is 180 beats per minute, BP is 110/65 mm Hg, and oxygen saturation is 99% on room air. An ECG reveals a HR of 180 beats per minute with a QRS complex of 90 msec with a regular rhythm. There are no discernable P waves. Which of the following is the most appropriate medication to treat this dysrhythmia? a. Digoxin b. Lidocaine c. Amiodarone d. Adenosine e. Bretylium

*D* Narrow-complex tachycardias are defined as rhythms with a QRS complex duration less than 100 ms and a ventricular rate greater than 100 beats per min. Although virtually all narrow-complex tachydysrhythmias originate from a focus above the ventricles, the term supraventricular tachycardia (SVT) is conventionally used to denote those rhythms aside from sinus tachycardia, atrial tachycardia, atrial fibrillation, and atrial flutter (eg, atrioventricular nodal reentry tachycardia and atrioventricular reentry tachycardia). Adenosine, an ultrashortacting AV-nodal blocking agent, is typically used to treat SVTs. Because it is so fast-acting, it must be delivered through a large vein (eg, the antecubital fossa) with a rapid intravenous fluid bolus. In addition to adenosine, maneuvers that increase vagal tone have been shown to slow conduction through the AV node. Some of these maneuvers include carotid sinus massage, Valsalva maneuver, and facial immersion in cold water. Lidocaine and amiodarone, although may be effective in treating narrow-complex tachycardias, are agents generally used to treat wide-complex or ventricular tachycardias.

A 30-year-old man is brought to the ED by police officers. The patient is agitated, vomiting, and complaining of body aches. He states that he is withdrawing from his medication. His vital signs are BP 160/85 mm Hg, RR 20 breaths per minute, HR 107 beats per minute, and temperature 99.7°F. On examination he is diaphoretic, has rhinorrhea, piloerection, and hyperactive bowel sounds. Which of the following substances is this patient most likely withdrawing from? a. Ethanol b. Cocaine c. Nicotine d. Methadone e. Clonidine

*D* Opioid withdrawal initially presents with drug craving, yawning, rhinorrhea, and piloerection and progresses to nausea, vomiting, diarrhea, hyperactive bowels, diaphoresis, myalgias, arthralgias, anxiety, fear, and mild tachycardia. Methadone withdrawal starts approximately 24 hours after the last dose and persists for 3 to 7 days. Heroin withdrawal begins about 6 hours after the last dose and usually fully manifests at 24 hours. Opioid withdrawal is not a life-threatening condition as long as adequate hydration and nutritional support is maintained.

A 25-year-old man is brought into the ED by two police officers because of suspected drug use. The patient is extremely agitated and is fighting the police officers. It takes three hospital staff members and the two police officers to keep him on the stretcher. His vital signs are: BP 150/80 mm Hg, HR 107 beats per minute, temperature 99.7°F, RR 18 breaths per minute, and oxygen saturation 99% on room air. Physical examination is unremarkable except for cool, diaphoretic skin, persistent vertical and horizontal nystagmus, and occasional myoclonic jerks. Which of the following is the most likely diagnosis? a. Cocaine intoxication b. Cocaine withdrawal c. Amphetamine intoxication d. PCP intoxication e. Opiate withdrawal

*D* PCP intoxication is characterized by a wide spectrum of findings. Behavior may be bizarre, agitated, confused, or violent. The hallmark of PCP toxicity is the recurring delusion of superhuman strength and invulnerability resulting from both the anesthetic and dissociative properties of the drug. Patients have broken police handcuffs, fracturing bones in doing so. The major cause of death or injury from PCP is behavioral toxicity leading to suicide and provoked homicide. Typical neurologic signs include nystagmus (horizontal, vertical, or rotary), ataxia, and altered gait. Pupils are usually midsized and reactive, but can be mydriatic or miotic. Bizarre posturing, grimacing, and writhing may be seen. Management is conservative. To prevent self-injury, the patient must be safely restrained. Antipsychotics or benzodiazepines are frequently administered for chemical sedation. PCP intoxication usually ranges from 8 to 16 hours, but can last longer in chronic users.

A 35-year-old woman presents to the ED for the second time. She is complaining of fever, neck stiffness, and photophobia. She was seen in your ED 2 days ago for the same symptoms. At that time, she had a normal neurologic examination, was otherwise well-appearing, and underwent diagnostic LP. The results of her CSF analysis were as follows: Glucose 82 mg/dL, Protein 60 mg/dL, WBC 150/ L (98% lymphocytes), Gram stain No organisms seen. The patient was sent home after a period of observation with presumed viral meningitis. She was told to return if her symptoms were not better in 48 hours. Since then, her fever increased (100.4°F-102.2°F). What is the next most appropriate step in management? a. Administration of acyclovir b. Aggressive antipyretic therapy and observation c. CT scan of the sinuses d. CT of the head and, if no contraindication, repeat LP e. Viral cultures and polymerase chain reaction (PCR) from previously obtained CSF

*D* Patients with CSF analysis consistent with viral meningitis can be managed as outpatients with close follow-up. These patients must be reliable for follow-up, not immunocompromised and otherwise well-appearing. They should be told to return for reevaluation if their symptoms do not improve within 48 hours. If they have not improved, reevaluation including neuroimaging, repeat LP, and treatment with antibiotics is indicated.

A 76-year-old woman with a history of congestive heart failure, coronary artery disease, and an "irregular heart beat" is brought to the ED by her family. She has been complaining of increasing abdominal pain over the past several days. She denies nausea or vomiting and bowel movements remain unchanged. Vitals are HR of 114 beats per minute, BP 110/75 mm Hg, and temperature 98°F. On cardiac examination, her HR is irregularly irregular with no murmur detected. The abdomen is soft, nontender, and nondistended. The stool is heme-positive. This patient is at high risk for which of the following conditions? a. Perforated gastric ulcer b. Diverticulitis c. Acute cholecystitis d. Mesenteric ischemia e. Sigmoid volvulus

*D* Patients with coronary artery disease, valvular heart disease, and arrhythmias, particularly atrial fibrillation, are at high risk for mesenteric ischemia. In addition, age greater than 50 years, congestive heart failure, recent myocardial infarction, critically ill patients with sepsis or hypotension, use of diuretics or vasoconstrictive medications, and hypercoagulable states place patients at higher risk. The most common cause of acute mesenteric ischemia is arterial embolus, which accounts for 50% of cases. The classic finding is "pain out of proportion to examination findings," that is, a patient complains of severe pain but is not particularly tender on examination. A high degree of suspicion for mesenteric ischemia in an elderly patient with abdominal pain is warranted.

A 19-year-old man suffers a single gunshot wound to the left chest and is brought in by his friends. He is complaining of chest pain. On examination, his temperature is 99°F, BP is 70/40 mm Hg, HR is 140 beats per minute, RR is 16 breaths per minute, and oxygen saturation is 96% on room air. He has distended neck veins, but his trachea is not deviated. Lungs are clear to auscultation bilaterally. Heart sounds are difficult to appreciate, but you feel a bounding, regular pulse. Abdomen is soft and nontender. Extremity examination is normal. Two large-bore IV lines are placed and the patient is given 2 L of normal saline. Chest radiograph shows a globular cardiac silhouette, but a normal mediastinum and no pneumothorax. What is the definitive management of this patient? a. Intubation b. Tube thoracostomy c. Pericardiocentesis d. Thoracotomy e. Blood transfusion

*D* Patients with penetrating trauma to the chest with possible cardiac injury and signs of hemodynamic instability need immediate operative thoracotomy. This patient has signs of cardiac tamponade, a collection of blood surrounding the heart and interfering with the heart's ability to contract. He has Beck triad of hypotension, distended neck veins, and muffled heart sounds. His CXR also shows an enlarged heart. An echocardiogram would be helpful in confirming the diagnosis, but since this patient is unstable and echocardiogram may not be readily available, the treatment is immediate thoracotomy in the operating room.

A 26-year-old woman, who was a belted front-seat driver in a headon motor vehicle collision, is brought to the ED. She is speaking but complains of progressively worsening shortness of breath and hemoptysis. Her BP is 135/75 mm Hg, HR is 111 beats per minute, RR is 24 breaths per minute, and oxygen saturation is 96% on non-rebreather. On examination, you note ecchymosis over the right side of her chest. Her breath sounds are equal bilaterally. There is no bony crepitus and the trachea is midline. After placing two large-bore IVs and completing the primary and secondary surveys, you view the chest radiograph shows frank consolidation. Which of the following is the most likely diagnosis? a. Diaphragm rupture b. Hemothorax c. Tension pneumothorax (PTX) d. Pulmonary contusion e. Acute respiratory distress syndrome (ARDS)

*D* Pulmonary contusions usually occur after a blunt traumatic force to the chest that causes injury to the lung parenchyma. This is followed by alveolar edema and hemorrhage. Pulmonary contusion is reported to be present in 30% to 75% of patients with significant blunt chest trauma, most often from automobile collisions with rapid deceleration. Typical radiographic findings begin to appear within minutes of injury and range from patchy, irregular, alveolar infiltrate to frank consolidation.

A 55-year-old man with a history of alcoholism and osteoarthritis developed left knee pain several days after a fall from standing height. The patient was brought to the ED by ambulance after being found on a park bench stating he was unable to walk because of the pain. On physical examination, his left knee is warm, diffusely tender, and swollen with a large effusion. He has pain on passive range of motion. His BP is 150/85 mm Hg, HR is 105 beats per minute, temperature is 102.7°F, RR is 16 breaths per minute, and fingerstick glucose is 89 mg/dL. Which of the following is the most appropriate diagnostic test? a. Knee radiographs b. Magnetic resonance imaging (MRI) c. Erythrocyte sedimentation rate (ESR) and C-reactive protein d. Arthrocentesis e. Bone scan

*D* Septic arthritis is an infection of a joint space, most commonly the knee, followed by the hip, shoulder, and wrist. Patient's present with a warm, tender, erythematous, swollen joint and pain with passive range of motion. Fever and chills are common. Arthrocentesis is diagnostic with joint fluid demonstrating a WBC count > 50,000/μL with > 75% granulocytes. Staphylococcus aureus remains the predominant pathogen for all age groups. In young adults gonococcal septic arthritis is common. Patients with septic arthritis should receive a first dose of antibiotics in the ED prior to admission. If a coincident cellulitis is present over the involved joint, arthrocentesis may need to be delayed.

A 61-year-old man presents to the ED with chest wall pain after a motor vehicle collision. He is speaking full sentences, breath sounds are equal bilaterally, and his extremities are well-perfused. His BP is 150/75 mm Hg, HR is 92 beats per minute, and oxygen saturation is 97% on room air. Chest radiography reveals fractures of the seventh and eighth ribs of the right anterolateral chest. He has no other identifiable injuries. Which of the following is the most appropriate treatment for this patient's rib fractures? a. Apply adhesive tape on the chest wall perpendicular to the rib fractures b. Insert a chest tube into the right thorax c. Bring the patient to the OR for surgical fixation d. Analgesia and incentive spirometry e. Observation

*D* Simple rib fractures are the most common form of significant chest injury. Ribs usually break at the point of impact or at the posterior angle, which is structurally the weakest area. The fourth through ninth ribs are most commonly involved. Rib fractures occur more commonly in adults than in children owing to the relative inelasticity of the adult chest wall compared to the more compliant chest wall of children. The presence of two or more rib fractures at any level is associated with a higher incidence of internal injuries. The treatment of patients with simple acute rib fractures includes adequate pain relief and maintenance of pulmonary function. Oral pain medications are usually sufficient for young and healthy patients. Older patients may require better analgesia with opioids, but care must be taken to avoid over sedation. Continuing daily activities and deep breathing is important to ensure ventilation and prevent atelectasis. If there is a question about the patient's ability to cough, breathe deeply, and maintain activity, particularly if two or more ribs are fractured, it is preferable to admit the patient to the hospital for aggressive pulmonary care.

A 75-year-old woman is transferred to your ED from the local nursing home for fever, cough, and increasing lethargy. Over the past 3 days, the nursing home staff noticed increasing yellow sputum and decreasing urine output from the patient. Her BP is 118/75 mm Hg, RR is 20 breaths per minute, HR is 105 beats per minute, temperature is 100.9°F, and pulse oxygenation is 94% on room air. On examination, the patient appears dehydrated. Auscultation of the lungs reveals bibasilar crackles. Laboratory results reveal WBC 14,500/μL, hematocrit 39%, platelets 250/μL, sodium 132 mEq/L, potassium 3.5 mEq/L, chloride 100 mEq/L, bicarbonate 18 mEq/L, BUN 27 mg/dL, creatinine 1.5 mg/dL, and glucose 85 mg/dL. Serum lactate is 4.7 mol/dL. Based on this patient's presentation, how would you classify her condition? a. Systemic inflammatory response syndrome (SIRS) b. Severe systemic inflammatory response syndrome c. Sepsis d. Severe sepsis e. Septic shock

*D* The basis of sepsis (c) is the presence of infection and the subsequent physiologic alterations in response to that infection, namely the activation of the inflammatory cascade. (a) SIRS is a term used to define this clinical condition, and is considered present if abnormalities in two of the following four clinical parameters exist: (1) body temperature, (2) HR, (3) RR, and (4) peripheral leukocyte count. These criteria include body temperature < 36°C (96.8°F) or > 38°C (100.4°F), rather > 90 beats per minute, RR > 20 breaths per minute or, on blood gas, a PaCO2 less than 32 mm Hg, and WBC count < 4000 cells/mm3 or > 12,000 cells/mm3, or greater than 10% band forms. This patient's condition is most consistent with severe sepsis, which is defined as sepsis with acute organ dysfunction. Some examples of end-organ dysfunction include acute lung injury, encephalopathy, confusion, progressive oagulopathy caused by inability to synthesize clotting factors, elevated unconjugated serum bilirubin, oliguria, anuria, and systolic and diastolic heart failure. Severe SIRS (b) is defined as end-organ dysfunction without evidence of infection. Septic shock (e) is defined as sepsis with persistent hypotension despite fluid resuscitation and resulting tissue hypoperfusion.

A 67-year-old man is brought to the ED by emergency medical service (EMS). His wife states that the patient was doing his usual chores around the house when all of a sudden he started complaining of severe abdominal pain. He has a past medical history of coronary artery disease and hypertension. His BP is 85/70 mm Hg, HR is 105 beats per minute, temperature is 98.9°F, and his RR is 18 breaths per minute. On physical examination, he is diaphoretic and in obvious pain. Upon palpating his abdomen, you feel a large pulsatile mass. An electrocardiogram (ECG) reveals sinus tachycardia. You place the patient on a monitor, administer oxygen, insert two large-bore IVs, and send his blood to the laboratory. His BP does not improve after a 1-L fluid bolus. Which of the following is the most appropriate next step in management? a. Order a CT scan to evaluate his aorta. b. Call the angiography suite and have them prepare the room for the patient. c. Order a portable abdominal radiograph. d. Call surgery and have them prepare the operating room (OR) for an exploratory laparotomy. e. Call the cardiac catheterization laboratory to prepare for stent insertion.

*D* The classic triad of a ruptured abdominal aortic aneurysm (AAA) is pain, hypotension, and a pulsatile abdominal mass. Sometimes patients have only one or two of the components and occasionally may have none. Most patients who are diagnosed with AAA are asymptomatic. However, rupture is often the first manifestation of an AAA. Most patients with a ruptured AAA experience pain in the abdomen, back, or flank. It is usually acute in onset and severe. Approximately 20% of the time, patients present to the ED with syncope. Patients with a ruptured AAA are unstable until their aorta is cross clamped in the OR. Therefore, any hemodynamically unstable patient with a diagnosed or strongly suspected AAA should be taken immediately to the OR

A 19-year-old college student presents to the ED complaining of headache, sore throat, myalgias, and rash that developed over the previous 12 hours. Her BP is 95/60 mm Hg, HR is 132 beats per minute, temperature is 103.9°F, RR is 19 breaths per minute, and oxygen saturation is 98% on room air. She is confused and oriented only to person. Physical examination is remarkable for pain with neck flexion, a petechial and purpuric rash on her extremities, and delayed capillary refill. Which of the following best describes the emergency physicians' priority in managing this patient? a. Collect two sets of blood cultures prior to antibiotic administration. b. Call the patient's parents and have them come immediately to the hospital. c. Call her roommate to gather more information. d. Begin fluid resuscitation, administer intravenous (IV) antibiotics, and perform a lumbar puncture (LP). e. Administer acetaminophen to see if her headache and fever resolve.

*D* The first question an emergency physician asks for each patient is whether a life-threatening process is causing the patient's complaint. Emergency medicine is primarily a complaint-oriented, rather than a disease-oriented specialty. Its emphasis rests on anticipating and recognizing a life-threatening process rather than seeking the diagnosis. The goal is to think about and plan to prevent the life-threatening things from happening or progressing in the patient. The patient in the vignette may have meningitis, a life-threatening condition, or a viral syndrome with dehydration. The initial approach is stabilization and treatment or prevention of a life-threatening process. This patient requires fluid resuscitation for her BP, altered mental status, and delayed capillary refill. Antibiotics should be started immediately and an LP performed once increased ICP is evaluated by funduscopic examination or CT scan. She should be placed in isolation. Her disposition is directed by her response to initial resuscitation and results of the LP.

A 45-year-old man is brought to the ED after a head-on motor vehicle collision. Paramedics at the scene tell you that the front-end of the car is smashed. The patient's BP is 130/80 mm Hg, HR is 100 beats per minute, RR is 15 breaths per minute, and oxygen saturation is 98% on room air. Radiographs of the cervical spine reveal bilateral fractures of the C2 vertebra. The patient's neurologic examination is unremarkable. Which of the following best describes this fracture? a. Colles fracture b. Boxer's fracture c. Jefferson fracture d. Hangman's fracture e. Clay-shoveler's fracture

*D* The hangman's fracture, or traumatic spondylolysis of C2, occurs when the head is thrown into extreme hyperextension as a result of abrupt deceleration, resulting in bilateral fractures of the pedicles. The name "hangman's fracture" was derived from judicial hangings, where the knot of the noose was placed under the chin which caused extreme hyperextension of the head on the neck, resulting in a fracture at C2. However, many hangings resulted in death from strangulation rather than spinal cord damage. Today, the most common cause of a hangman's fracture is the result of head-on automobile collisions.

A 78-year-old man presents to the emergency department (ED) complaining of left arm weakness that started 10 minutes ago in the clinic. The patient states that he has a history of hypertension and diabetes, but has never had similar symptoms in the past. He is feeling well otherwise. His blood pressure (BP) is 157/85 mm Hg, heart rate (HR) is 87 beats per minute, temperature is 98.8°F, and respiratory rate (RR) is 14 breaths per minute. His neurologic examination is unremarkable and the patient embarrassingly states that his left arm is no longer weak. Which of the following is the most likely diagnosis? a. Thrombotic stroke b. Conversion disorder c. Migraine with focal neurologic deficit d. Transient ischemic attack (TIA) e. Todd paralysis

*D* The patient had a TIA, which involves neurologic deficits that resolve within 24 hours of onset. TIAs often precede ischemic stroke; up to 50% of patients with a TIA will have a stroke in the next 5 years, with the highest incidence in the first month. It is important to recognize TIAs and to evaluate patients for cardiac or carotid arterial sources of emboli. Although the symptoms often resolve, many patients with a TIA will have evidence of infarction on CT/MRI.

A 47-year-old man presents to urgent care complaining of a 2-day history of sore throat and subjective fever at home. He denies cough or vomiting. His BP is 130/75 mm Hg, HR is 85 beats per minute, temperature is 101°F, and his RR is 14 breaths per minute. He has tonsillar swelling without exudates and bilaterally enlarged and tender lymph nodes of the neck. The rapid streptococcal antigen test is negative. Which of the following is the next best step in management? a. Administer penicillin and discharge the patient. b. Schedule a lymph node biopsy to rule out lymphoma. c. Observe for 6 hours. d. Perform a throat culture, symptomatic care, and treat if results are positive. e. Administer amantadine to patient and all contacts.

*D* The patient has a modified Centor score of 2 (history of fever, tender adenopathy, no cough, age > 45 years). (subtract 1 for being more than 45)

A 25-year-old man presents to the ED complaining of headache for 2 days. He describes the pain as pulsatile and occipital. The patient had an LP 3 days prior and was diagnosed with viral meningitis after 4 days of symptoms. Noncontrast head CT at that time was normal. He improved shortly thereafter with defervescence of his fever and resolution of his constitutional and nuchal symptoms. He states that his new headache is different than his previous in that it is exacerbated by standing or sitting upright and is relieved by sitting down and is not associated with photophobia or neck stiffness. The headache is not relieved by over-the-counter pain medications. He is afebrile and nontoxic appearing. Which of the following is definitive therapy for this patient's headache? a. A 1-L bolus of intravenous (IV) normal saline b. Treatment with standard pharmacologic agents for migraine c. Treatment with meclizine d. Consultation with anesthesia for a blood patch e. Repeat LP to improve symptoms

*D* The patient has a post-LP headache. The headache is thought to be caused by the removal of CSF during LP with a continued leakage of CSF. It is exquisitely sensitive to position and many patients will experience complete relief of pain after being placed in Trendelenburg position. A blood patch is placed by injecting an aliquot of the patient's blood in a sterile fashion just external to the dura mater at the same interspace where the LP occurred. The majority of patients have relief of symptoms with this procedure. Prevention of the post-LP headache includes using a 22-gauge or smaller needle, removing as little fluid as possible, and facing the bevel up when the patient is in the lateral position.

A 22-year-old man calls the ED from a local bar stating that he was punched in the face 10 minutes ago and is holding his front incisor tooth in his hand. He wants to know what is the best way to preserve the tooth. Which of the following is the most appropriate advice to give the caller? a. Place the tooth in a napkin and bring it to the ED b. Place the tooth in a glass of water and bring it to the ED c. Place the tooth in a glass of beer and bring it to the ED d. Pour some water over the tooth and place it immediately back into the socket e. Place the tooth in a glass of milk and bring it to the ED

*D* The patient has an avulsed tooth, which is a dental emergency. When a tooth is missing from a patient, the possibility of aspiration or entrapment in soft tissues should be considered. Avulsed permanent teeth require prompt intervention. The best environment for an avulsed tooth is its socket. Replantation is most successful if the tooth is returned to its socket within 30 minutes of the avulsion. A 1% chance of successful replantation is lost for every minute that the tooth is outside of its socket. The tooth should only be handled by the crown, so as not to disrupt the root. If the patient cannot replant the tooth, then he or she should keep the tooth under his or her tongue or in the buccal pouch so that it is bathed in saliva. If that cannot be achieved, then the tooth can be placed in a cup of milk (e) or in saline. The best transport solution is Hank solution, which is a buffered chemical solution. However, it is typically unavailable in this setting.

A 55-year-old woman with a past medical history of diabetes walks into the emergency department (ED) stating that her tongue and lips feel like they are swollen. During the history, she tells you that her doctor justnstarted her on a new blood pressure (BP) medication. Her only other medication is a baby aspirin. Her vitals at triage are: BP 130/70 mm Hg, heart rate (HR) 85 beats per minute, respiratory rate (RR) 16 breaths per minute, oxygen saturation 99% on room air, and temperature 98.7°F. On physical examination, you detect mild lip and tongue swelling. Over the next hour, you notice that not only are her tongue and lips getting more swollen, but her face is starting to swell, too. What is the most likely inciting agent? a. Metoprolol b. Furosemide c. Aspirin d. Lisinopril e. Diltiazem

*D* The patient has angioedema, a rare, but significant side effect of angiotensin-converting enzyme (ACE-I) inhibitors. This type of angioedema is usually limited to the lips, tongue, and face.

A 22-year-old man presents to the ED with a 3-day history of rash, fever, malaise, and mouth sores. He has been unable to eat because of mouth pain. He denies arthralgias, penile discharge, new medications, drug allergies, or prior similar episodes. On examination, the patient's BP is 100/60 mm Hg, HR is 110 beats per minute, RR is 20 breaths per minute, and temperature is 102°F. The patient appears alert but uncomfortable. He has multiple vesiculobullous lesions on his conjunctivae and mouth as seen in the image below. Visual acuity is 20/20. Target lesions are found on his palms and soles. What is the most appropriate next step in management? a. Discharge him with analgesics, antihistamines, and mouth rinses. b. Discharge him with acyclovir, analgesics, antihistamines, and mouth rinses. c. Discharge him after 1-L normal saline IV; and prescriptions for analgesics, antihistamines, oral prednisone, and mouth rinses. d. Admit him and administer 1- to 2-L normal saline IV, oral prednisone, analgesics, antihistamines, and mouth rinses. e. Admit him and administer 1- to 2-L normal saline IV; analgesics, and acyclovir IV.

*D* The patient has signs suggestive of Erythema Multiforme (EM), an acute inflammatory skin disease that ranges from a localized eruption (EM minor) to a severe multisystem illness (EM major) with extensive vesiculobullous lesions and erosion of the mucous membranes, known as Stevens-Johnson syndrome (SJS). It affects all age groups with the highest incidence in males 20 to 40 years of age. SJS has significant morbidity and a mortality rate of approximately 10%. Death is usually a result of infection and dehydration. As in this case, patients with severe disease should be admitted. Therapy consists of IV fluids, oral prednisone, analgesics, antihistamines, mouth rinses, and skin care. While no causative factor can be found in 50% of cases, known triggers include infection, especially Mycoplasma and herpes simplex virus, drugs, especially anticonvulsants and antibiotics, and malignancies.

A 23-year-old woman presents to the ED complaining of abdominal pain, nausea, and vomiting. She has a history of depression but is not currently taking any antidepressant medications. Upon further questioning, the patient states that she ingested a bottle of pills in her medicine cabinet approximately 3 hours ago. Her BP is 115/65 mm Hg, HR is 101 beats per minute, temperature is 100.1°F, RR is 29 breaths per minute, and oxygen saturation is 100% on room air. Physical examination is unremarkable except for mild diffuse abdominal tenderness. Laboratory results reveal a white blood cell (WBC) count of 10,300/μL, hematocrit 46%, platelets 275/μL, aspartate transaminase (AST) 70 U/L, alanine transaminase (ALT) 85 U/L, alkaline phosphatase 75 U/L, sodium 143 mEq/L, potassium 3.7 mEq/L, chloride 98 mEq/L, bicarbonate 8 mEq/L, blood urea nitrogen (BUN) 22 mg/dL, creatinine 0.9 mg/dL, and glucose 85 mg/dL. Arterial blood gas values on room air are pH 7.51, PCO2 11 mm Hg, and PO2 134 mm Hg. Which of the following substances did this patient most likely ingest? a. Diphenhydramine (Benadryl) b. Ibuprofen c. Acetaminophen d. Aspirin e. Pseudoephedrine

*D* The patient most likely ingested aspirin. Patients with an acute salicylate overdose may present with nausea, vomiting, tinnitus, fever, diaphoresis, and confusion. Salicylates are capable of producing several types of acid-base disturbances. Acute respiratory alkalosis, without hypoxia, is caused by salicylate stimulation of the respiratory center in the brainstem. If the patient is hypoxic, salicylate induced noncardiogenic pulmonary edema should be considered. Within 12 to 24 hours after ingestion, the acid-base status in an untreated patient shifts toward an anion gap metabolic acidosis because of interference with the Krebs cycle, uncoupling oxidative-phosphorylation, and increased fatty acid metabolism. A mixed respiratory alkalosis and metabolic acidosis is typically seen in adults.

A 35-year-old woman presents to the ED complaining of headache and blurry vision. She has had daily headaches for 3 months associated with blurry vision. She is afebrile, not losing weight, and has a normal neurologic examination, including fundoscopy. You ask when her last menstrual period was and she states she has not menstruated for 5 months and is not taking oral contraceptive pills. She also complains of galactorrhea. Noncontrast head CT is normal. An LP is performed and reveals a normal opening pressure. Which of the following is the most appropriate next step in managing the patient's headaches? a. Repeat head CT with administration of IV contrast. b. Initiation of therapy with bromocriptine. c. Evaluation of CSF for xanthochromia and RBCs. d. Treatment of her headache with analgesia and a MRI. e. Repeat LP with removal of 15-mL CSF for therapeutic benefit.

*D* The patient presents with symptoms consistent with a prolactin-secreting pituitary adenoma. The appropriate imaging modality to diagnose a pituitary adenoma is with high resolution MRI with thin cuts through the sella. Women often present with amenorrhea, infertility, and galactorrhea. Men will present with decreased libido. In both cases, extension beyond the sella may present with visual field defects or other mass-related symptoms.

A 23-year-old woman presents to the ED in moderate pain in her left lower quadrant (LLQ). She states that the pain began suddenly and is associated with nausea and vomiting. She had a bout of diarrhea yesterday. This is the second time this month that she experienced pain in this location, however, never with this severity. Her BP is 120/75 mm Hg, HR is 101 beats per minute, temperature is 99.5°F, and RR is 18 breaths per minute. She has a tender LLQ on abdominal examination and a tender adnexa on pelvic examination. Which of the following is the most appropriate diagnostic test for the patient? a. CT scan b. MRI c. X-ray d. Doppler ultrasound e. Laparoscopy

*D* The patient's clinical picture is consistent with ovarian torsion. This phenomenon is most common in women in their mid-twenties. It is caused by the ovary twisting on its stalk, which leads to occlusion of venous draining from the ovary. This leads to ovarian edema, hemorrhage, and necrosis. Most occur in the presence of an enlarged ovary (ie, as a result of cyst, abscess, or tumor). Patients may give a history of similar pain that resolved spontaneously. The first choice to diagnose ovarian torsion is with Doppler ultrasound to demonstrate decreased or absent blood flow to the ovary. It can also identify an ovarian mass. If suspicion is high for ovarian torsion, the patient may immediately undergo laparoscopy, which is diagnostic and potentially therapeutic.

A 41-year-old man is brought into the ED after having a witnessed tonic-clonic seizure. He is alert and oriented and states that he has not taken his seizure medication for the last week. His BP is 140/75 mmHg, HR is 88 beats per minute, temperature is 99.7°F, and his RR is 16 breaths per minute. On examination you notice that his arm is internally rotated and adducted. He cannot externally rotate the arm and any movement of his shoulder elicits pain. Which of the following is the most likely diagnosis? a. Humerus fracture b. Clavicular fracture c. Scapular fracture d. Posterior shoulder dislocation e. Anterior shoulder dislocation

*D* The patient's clinical presentation is consistent with a posterior shoulder dislocation. Posterior dislocations are rare and account for only 2% of all glenohumeral dislocations. Posterior dislocations are traditionally associated with seizure patients and lightening injuries. However, the most common dislocation seen in postseizure patients is an anterior dislocation. Classically, the patient holds the dislocated arm across the chest in adduction and internal rotation. Abduction is limited and external rotation is blocked. Radiographs may reveal a "light bulb" sign, which is the light bulb appearance when the humeral head is profiled in internal rotation.

A 49-year-old man presents to the ED with nausea, vomiting, and abdominal pain that began approximately 2 days ago. The patient states that he usually drinks a six-pack of beer daily, but increased his drinking to two six-packs daily over the last week because of pressures at work. He notes decreased appetite over the last 3 days and states he has not had anything to eat in 2 days. His BP is 125/75 mm Hg, HR is 105 beats per minute, and RR is 20 breaths per minute. You note generalized abdominal tenderness on examination. Laboratory results reveal the following: White blood cells (WBC) 9000/μL, Sodium 131 mEq/L, Hematocrit 48%, Potassium 3.5 mEq/L, Platelets 210/μL Chloride 101 mEq/L, AST 85 U/L, Bicarbonate 10 mEq/L, ALT 60 U/L, BUN, 9 mg/dL, Alkaline phosphatase 75 U/L Creatinine 0.5 mg/dL, Total bilirubin 0.5 mg/dL, Glucose 190 mg/dL, Lipase 40 IU, Nitroprusside test weakly positive for ketones Which of the following is the mainstay of therapy for patients with this condition? a. Normal saline (NS) solution b. Half normal saline (1/2 NS) c. Glucose solution (D5W) d. Solution containing both saline and glucose (D5/NS or D5 1/2 NS) e. The type of solution is irrelevant

*D* The patient's presentation is consistent with alcoholic ketoacidosis (AKA). This is an acute metabolic acidosis that typically occurs in people who (1) chronically abuse alcohol and have a recent history of binge drinking, (2) have had little or no recent food intake, and (3) have had persistent vomiting. AKA is characterized by elevated serum ketone levels and a high anion gap (↑AG = Na − [HCO3 + Cl] > 12). A concomitant metabolic alkalosis is common, secondary to vomiting and volume depletion. AKA is the result of (1) starvation with glycogen depletion and counterregulatory hormone production; (2) a raised NADH/NAD+ ratio (related to the metabolism of ethanol); and (3) volume depletion, resulting in ketogenesis. The typical symptoms and physical findings relate to volume depletion and chronic alcohol abuse and include nausea, vomiting, abdominal pain, and/or hematemesis. The fruity odor of ketones may be present on the patient's breath. The patient's mental status may be impaired. Also associated with the presentation are dyspnea, tremulousness, and dizziness. Rarely do patients present with muscle pain, fever, diarrhea, syncope, seizure, or melena. In AKA, the β-hydroxybutyrate (β-OH)/ acetyl acetate (AcAc) formation ratio is 5:1. The nitroprusside reaction (Acetest) may be negative or only weakly positive for serum ketones because nitroprusside reacts with AcAc, but not with β-OH. Therefore, ketosis may be more severe than would be inferred from a nitroprusside reaction alone. Once the diagnosis of alcoholic ketoacidosis is established, the mainstay of treatment is (d) hydration with 5% dextrose in normal saline (D5NS or 1/2 NS). With initial therapy, ketone formation shifts toward the production of AcAc so that measured ketone levels rise, although β-OH levels decrease. Carbohydrate and fluid replacement reverse the pathophysiologic derangements that lead to AKA by increasing serum insulin levels and suppressing the release of glucagon and other counterregulatory hormones. Dextrose stimulates the oxidation of NADH and aids in normalizing the NADH/NAD+ ratio. As additional treatment, thiamine supplementation should be given as prophylaxis against Wernicke encephalopathy. In general, exogenous insulin is contraindicated in the treatment of AKA because it may cause life threatening hypoglycemia in patients with depleted glycogen stores.

While playing in his family's annual Thanksgiving Day touch football game, a 41-year-old man fell onto his outstretched hand upon attempting to make the winning catch. He presents to the emergency department (ED) complaining of severe wrist pain. Which of the following carpal bones is most frequently injured in a fall on the outstretched hand? a. Triquetrum b. Lunate c. Capitate d. Scaphoid e. Pisiform

*D* The scaphoid is the most common carpal bone injured in a fall on the outstretched hand (a "FOOSH" injury). On examination, patients exhibit tenderness at the anatomic snuffbox and pain referred to the anatomic snuffbox with longitudinal compression of the thumb. In the setting of a tender thumb, a thumb splint should be applied until films can be repeated after 1 week since initial films may not reveal the fracture. The application of the splint is especially important, because of the scaphoid's high risk for avascular necrosis.

A 32-year-old firefighter presents to the ED in acute respiratory distress. He was taken to the ED shortly after extinguishing a large fire in a warehouse. His initial vitals include a HR of 90 beats per minute, a BP of 120/55 mm Hg, and an RR of 18 breaths per minute with an oxygen saturation of 98% on 2-L nasal cannula. An ECG shows a first-degree heart block. Upon physical examination, there are diffuse rhonchi bilaterally. The patient is covered in soot and the hairs in his nares are singed. Given this clinical presentation, which of the following may be responsible for this patient's respiratory distress? a. Reactive airway disease b. Foreign body aspiration c. Decompression sickness d. Thermal burns e. Pneumothorax

*D* The singed nares seen in this patient should give you a clue to the possibility of severe thermal burns. Although there is minimal external involvement, damage from the heat may extend deep into the pulmonary system through inspiration. This results in a severe inflammatory reaction causing pneumonitis. Early intubation is critical.

A 44-year-old woman is undergoing a diagnostic evaluation for 3 hours of abdominal pain. She had two similar episodes in the past 2 months. She is tolerating oral intake and is afebrile. As part of this evaluation, a diagnostic ultrasound is performed and is shown below. Which of the following is the most likely diagnosis? a. Nephrolithiasis b. Pancreatic pseudocyst c. Ovarian cysts d. Cholelithiasis e. Liver abscess

*D* The ultrasound shows a gallbladder with multiple echogenic gallstones associated with well defined acoustic shadows. This image is typical for cholelithiasis. If the patient was febrile and thought to have cholecystitis, then one may also observe gallbladder distention, wall thickening, and pericholecystic fluid in addition to gallstones. The most common clinical manifestation of cholelithiasis is biliary colic.

A 20-year-old man presents to the ED with multiple stab wounds to his chest. His BP is 85/50 mm Hg and HR is 123 beats per minute. Two large-bore IVs (intravenous) are established and running wide open. On examination, the patient is mumbling incomprehensibly, has good air entry on lung examination, and you notice jugular venous distension. As you are listening to his heart, the nurse calls out that the patient has lost his pulse and that she cannot get a BP reading. Which of the following is the most appropriate next step in management? a. Atropine b. Epinephrine c. Bilateral chest tubes d. ED thoracotomy e. Pericardiocentesis

*D* The vignette describes a traumatic arrest after penetrating chest trauma. The most likely cause is cardiac tamponade, which occurs in approximately 2% of anterior penetrating chest traumas. Clinically, patients present with hypotension, JVD, and muffled heart sounds. These three signs are called Beck triad. In addition, tachycardia is often present. JVD may not be present if there is marked hypovolemia. The most effective method for relieving acute pericardial tamponade in the trauma setting is thoracotomy and incision of the pericardium with removal of blood from the pericardial sac. The indications to perform an ED thoracotomy generally include blunt or penetrating trauma patients who lose their vital signs in transport to or in the ED. Patients with penetrating wounds have a significantly better chance of surviving with thoracotomy; also patients with stab wounds are more likely to do better than GSWs.

A 44-year-old man presents to the ED complaining of right foot pain. He states while playing basketball 2 weeks ago, he stepped on a nail that punctured through his sneaker and cut his great toe. He went immediately to the ED, where an x-ray was taken and confirmed negative, in addition to being administered tetanus prophylaxis. On physical examination, his toe is swollen, erythematous, and tender to palpation. There is no obvious break in the skin or abscess present. His BP is 120/75 mm Hg, HR is 80 beats per minute, temperature is 100.4°F, and RR is 16 breaths per minute. Which of the following organisms is the most likely pathogen? a. Neisseria gonorrhoeae b. Staphylococcus epidermidis c. Sporothrix schenckii d. Pseudomonas aeruginosa e. Salmonella sp.

*D* This is a typical scenario for osteomyelitis. Conventional radiography on the day of injury is insensitive to the detection of osteomyelitis; even 1 week after the injury x-ray diagnosis is limited. The most common pathogen in osteomyelitis is S aureus. However, Pseudomonas is responsible for bone and joint infections in three settings. First, a puncture wound through a shoe. Pseudomonas does not grow on the puncture object, but rather is associated with the shoe itself and may be inoculated into the bone as the sharp object passes through the colonized shoe into the wound. Second, prosthetic devices implanted for orthopedic surgery. Third is IV drug use.

An 84-year-old woman with a history of metastatic breast cancer presents to the ED with new-onset dyspnea and exercise intolerance for the past week. She denies fever, chest pain, or cough. On examination, her temperature is 100.3°F, BP is 70/50 mm Hg, HR is 110 beats per minute, RR is 20 breaths per minute, and oxygen saturation is 93% on room air. As your colleague is performing a physical examination, you place the portable ultrasound on the heart and see a thin echo-free area around the heart with right atrium and right ventricular collapse. Which of the following is the most likely diagnosis? a. Pulmonary embolism b. Congestive heart failure c. Massive myocardial infarction (MI) d. Cardiac tamponade e. Dehydration

*D* This patient has cardiac tamponade from metastatic breast cancer. Bedside ultrasound is often diagnostic. The echo-free area around the heart is a pericardial effusion. It is important to realize that the presence or absence of pericardial effusion is not diagnostic of tamponade; right atrial and ventricular collapse, on the other hand, are more specific ultrasound signs for tamponade. Clinical findings should always be considered in making the diagnosis. Patients with tamponade usually have tachycardia, low systolic BP, and narrow pulse pressure. Tamponade must always be considered with trauma to the chest, as well as patients with metastatic malignancy, pericarditis, uremia, and those patients on anticoagulation.

A 61-year-old woman with a history of congestive heart failure (CHF) is at a family picnic when she starts complaining of shortness of breath. Her daughter brings her to the ED where she is found to have an oxygen saturation of 85% on room air with rales halfway up both of her lung fields. Her BP is 185/90 mm Hg and pulse rate is 101 beats per minute. On examination, her jugular venous pressure (JVP) is 6 cm above the sternal angle. There is lower extremity pitting edema. Which of the following is the most appropriate first-line medication to lower cardiac preload? a. Metoprolol b. Morphine sulfate c. Nitroprusside d. Nitroglycerin e. Oxygen

*D* This patient has decompensated CHF with pulmonary edema. Nitroglycerin is the most effective and most rapid means of reducing preload in a patient with CHF. Nitrates decrease myocardial preload and, to a lesser extent, afterload. Nitrates increase venous capacitance, including venous pooling, which decreases preload and myocardial oxygen demand. It is most beneficial when the patient who presents with CHF is also hypertensive. It is administered sublingually, intravenously, or transdermally.

A 34-year-old woman with no known medical problems is having a sushi dinner with her husband. Halfway through dinner, she begins scratching her arms and her husband notices that her face is flushed. The itching intensifies and she begins to feel chest pain, shortness of breath, and dizziness. On arrival to the ED, she can barely talk. Her temperature is 100°F, BP is 85/50 mm Hg, HR is 125 beats per minute, and RR is 26 breaths per minute, and oxygen saturation is 91% on room air. Which of the following is the most likely diagnosis? a. Hypovolemic shock b. Neurogenic shock c. Cardiogenic shock d. Anaphylactic shock e. Septic shock

*D* This patient is in anaphylactic shock from a food allergy while dining. Anaphylaxis is a severe systemic hypersensitivity reaction leading to shock from hypotension and respiratory compromise. The diagnosis is made clinically. This patient's reaction began classically with urticarial symptoms of pruritus and flushing. She then progressed to shock with hypotension and respiratory edema. She should be treated immediately with oxygen, intramuscular or IV epinephrine, corticosteroids, diphenhydramine, and IV fluids. Supplies should also be ready for intubation and surgical cricothyrotomy.

An 82-year-old man with a history of COPD and hypertension presents with shortness of breath and fever. His medications include albuterol, ipratropium, prednisone, hydrochlorothiazide, and atenolol. His temperature is 102.1°F, BP is 70/40 mm Hg, HR is 110 beats per minute, RR is 24 breaths per minute, and oxygen saturation is 91% on room air. The patient is uncomfortable and mumbling incoherently. On chest examination, you appreciate rales on the left side of his chest. His heart is tachycardic, but regular with no murmurs, rubs, or gallops. His abdomen is soft and nontender. You believe this patient is in septic shock from pneumonia and start IV fluids, broad-spectrum antibiotics, and a dopamine drip. His BP remains at 75/50 mm Hg. Which of the following is the most appropriate next step in management? a. D5 normal saline IV bolus b. Phenylephrine IV drip c. Fludrocortisone IV d. Hydrocortisone IV e. Epinephrine IV drip

*D* This patient is in septic shock from pneumonia and also has adrenal crisis. Initial treatment with IV fluids, antibiotics, and dopamine is appropriate. Continued hypotension in a patient on maintenance steroid therapy should make you think of adrenal crisis. Exogenous glucocorticoids suppress hypothalamic release of corticotropin-releasing hormone (CRH) and subsequently anterior pituitary release of adrenocorticotropic hormone (ACTH). The adrenals subsequently atrophy from lack of stimulation. The patient is now faced with an acute stress from pneumonia and sepsis. His adrenals have atrophied and are unable to respond with increased cortisol secretion. Laboratory clues to adrenal crisis include hyponatremia and hyperkalemia caused by a lack of aldosterone. The treatment of adrenal crisis in the face of septic shock is hydrocortisone.

A 22-year-old woman is brought to the ED by ambulance complaining of sudden onset of severe abdominal pain for 1 hour. The pain is in the RLQ and is not associated with nausea, vomiting, fever, or diarrhea. On the pelvic examination you palpate a tender right adnexal mass. The patient's last menstrual period was 6 weeks ago. Her BP is 95/65 mm Hg, HR is 124 beats per minute, temperature is 99.8°F, and RR is 20 breaths per minute. Which of the following are the most appropriate next steps in management? a. Provide her oxygen via face mask and administer morphine sulfate. b. Administer morphine sulfate, order an abdominal CT with contrast, and call an emergent surgery consult. c. Send the patient's urine for analysis and order an abdominal CT. d. Bolus 2-L NS, order a type and crossmatch and β-hCG, and call gynecology for possible surgery. e. Provide oxygen via face mask, give morphine sulfate, and order a transvaginal ultrasound.

*D* You should be concerned about a ruptured ectopic pregnancy in this patient. She missed her last menstrual period, has severe pain in the lower abdomen, and is hypotensive. This is a life-threatening condition that needs to be managed aggressively. The patient requires fluid resuscitation with 2-L NS. If her BP does not respond to the bolus, then blood should be administered. The patient will most likely be taken to the OR for a salpingectomy/oophorectomy. Risk factors for an ectopic pregnancy include history of pelvic inflammatory disease, prior ectopic pregnancy, pelvic surgery, and intrauterine device (IUD) use.

A 29-year-old man presents to the ED after being stabbed in his neck. The patient is speaking in full sentences. His breath sounds are equal bilaterally. His BP is 130/75 mm Hg, HR is 95 beats per minute, RR is 16 breaths per minute, and oxygen saturation is 99% on room air. The stab wound is located between the angle of the mandible and the cricoid cartilage and violates the platysma. There is blood oozing from the site although there is no expanding hematoma. Which of the following is the most appropriate next step in management? a. Explore the wound and blind clamp any bleeding site. b. Probe the wound looking for injured vessels. c. Apply direct pressure and bring the patient immediately to the OR to explore the zone 1 injury. d. Apply direct pressure and bring the patient immediately to the OR to explore the zone 2 injury. e. Apply direct pressure and bring the patient immediately to the OR to explore the zone 3 injury.

*D* Zone I extends superiorly from the sternal notch and clavicles to the cricoid cartilage. Injuries to this region can affect both neck and mediastinal structures. Zone II is the area between the cricoid cartilage and the angle of the mandible. Zone III extends from the angle of the mandible to the base of the skull. Generally, Zone II injuries are taken directly to the operating room for surgical exploration. Injuries in Zones I and III may be taken to the operating room or managed conservatively using a combination of angiography, bronchoscopy, esophagoscopy, and CT scanning.

A 43-year-old woman presents to the ED with a 3-week history of intermittent headache, nausea, and fatigue. She was seen at her private doctor's office 1 week ago along with her husband and children, who also have similar symptoms. They were diagnosed with a viral syndrome and told to increase their fluid intake. She states that the symptoms began approximately when it started to get cold outside. The symptoms are worse in the morning and improve while she is at work. Her BP is 123/75 mm Hg, HR is 83 beats per minute, temperature is 98.9°F, and oxygen saturation is 98% on room air. Physical examination is unremarkable. You suspect her first diagnosis was incorrect. Which of the following is the most appropriate next step to confirm your suspicion? a. Order a mono spot test b. Perform a nasal pharyngeal swab to test for influenza c. Consult psychiatry to evaluate for malingering d. Order a carboxyhemoglobin (COHb) level e. Order a lead level

*D*The most useful diagnostic test obtainable in a suspected CO poisoning is a COHb level. Normal levels range from 0% to 5%, as CO is a natural by-product of the metabolism of porphyrins. COHb levels average 6% in one-pack-per-day smokers. CO poisoning should be suspected when multiple patients, usually in the same family, present with flu-like symptoms, and were exposed to products of combustion (eg, home heaters/generators). This most commonly occurs in colder, winter months. The mainstay of treatment is the delivery of oxygen. Hyperbaric oxygen is usually used for patients with COHb levels greater than 25%.

A 32-year-old man with no past medical problems presents to the ED with palpitations. For the past 2 days he has been feeling weak and over the last 6 hours he has noticed that his heart is racing. He has no chest pain or shortness of breath. He has never felt this way before. His temperature is 98.9°F, BP is 140/82 mm Hg, HR is 180 beats per minute, and RR is 14 breaths per minute. His physical examination is normal. You obtain the following rhythm strip. showing a narrow complex, regular tachycardia What is your first-line treatment for this patient? a. Synchronized cardioversion at 100 J b. Adenosine 6-mg intravenous (IV) push c. Adenosine 12-mg IV push d. Valsalva maneuver e. Verapamil 3-mg IV push

*D*This patient has supraventricular tachycardia (SVT), a narrow complex, regular tachycardia. It is caused by a reentry or an ectopic pacemaker in areas of the heart above the bundle of His, usually the atria. Regular P waves will be present, but may be difficult to discern owing to the very fast rate. The patient in this case has normal vital signs and examination, and is therefore stable. First-line treatment for a patient with stable SVT is vagal maneuvers to slow conduction and prolong the refractory period in the AV node. The Valsalva maneuver can be accomplished by asking the patient to bear down as if they are having a bowel movement and hold the strain for at least 10 seconds. Other vagal maneuvers include carotid sinus massage (after auscultating for carotid bruits) and facial immersion in cold water.

A 61-year-old woman with a history of diabetes and hypertension is brought to the ED by her daughter. The patient states that she started feeling short of breath approximately 12 hours ago and then noticed a tingling sensation in the middle of her chest and became diaphoretic. An ECG reveals ST-depression in leads II, III, and aVF. You believe that the patient had a non-ST-elevation MI. Which of the following cardiac markers begins to rise within 3 to 6 hours of chest pain onset, peaks at 12 to 24 hours, and returns to baseline in 7 to 10 days? a. Myoglobin b. Creatinine kinase (CK) c. Creatinine kinase-MB (CK-MB) d. Troponin I e. Lactic dehydrogenase (LDH)

*D.* myoglobin- rises w/ in 1-2 hrs, peaks at 4-6, baseline at 24hrs CK - rises 3-8 hrs, peaks at 12-24 hrs, normalizes at 3-4 days CK-MB rises 4-6 hrs, peaks at 12-36 hrs, normalizes at 3-4 days LDH rises 12 hrs, peaks at 24-48 hrs, returns to normal 10-14 days

A 28-year-old man presents to the ED complaining of constant vague, diffuse epigastric pain. He describes having a poor appetite and feeling nauseated ever since eating sushi last night. His BP is 125/75 mm Hg, HR is 96 beats per minute, temperature is 100.5°F, and his RR is 16 breaths per minute. On examination, his abdomen is soft and moderately tender in the right lower quadrant (RLQ). Laboratory results reveal a WBC of 12,000/μL. Urinalysis shows 1+ leukocyte esterase. The patient is convinced that this is food poisoning from the sushi and asks for some antacid. Which of the following is the most appropriate next step in management? a. Order a plain radiograph to look for dilated bowel loops. b. Administer 40 cc of Maalox and observe for 1 hour. c. Send the patient for an abdominal ultrasound. d. Order an abdominal CT scan. e. Discharge the patient home with ciprofloxacin.

*D.* Appendicitis is the most common cause of the acute surgical abdomen. It can occur at any age but is most prevalent in the teens and twenties. It is classically described as starting with the vague onset of dull periumbilical pain that migrates to the RLQ. It is associated with anorexia, nausea, and vomiting. A low-grade fever may develop. You should suspect appendicitis in any patient with RLQ pain. Approximately 90% of patients have a WBC > 10,000/mm3 although, the WBC can be normal in appendicitis. Urinalysis can help differentiate urinary tract disease from acute appendicitis, although a mild pyuria may be seen in appendicitis if the appendix is irritating the ureter. Abdominal CT with IV and oral or rectal contrast is reported to have a sensitivity of up to 100% and specificity of 95%. CT findings of appendicitis include an enlarged appendix (> 6 mm), pericecal inflammation, and the presence of an appendicolith.

While discussing a case presentation with a medical student, a nearby patient who just returned from getting an ankle radiograph done yells out in pain. You walk over to him and ask what is wrong. He states that since returning from the radiology suite, his automatic implantable cardioverter defibrillator (AICD) is discharging. You hook him up to the monitor and note that his rhythm is sinus. You observe a third shock while the patient is in sinus rhythm. Which of the following is the most appropriate next step in management? a. Send the patient back to the radiology suite for another radiograph to desensitize his AICD. b. Administer pain medication and wait until the device representative arrives at the hospital to power off the AICD. c. Admit the patient to the telemetry unit to monitor his rhythm and find the cause of his AICD discharge. d. Place a magnet over the AICD generator to inactivate it and thereby prevent further shocks. e. Make a small incision over his chest wall and remove the AICD generator and leads.

*D.* Automatic implantable cardioverter—defibrillators (AICD) are placed in patients who are at high risk for fatal dysrhythmias (eg, ventricular tachycardia and fibrillation) and sudden death. In these patients, AICDs decrease the risk of sudden death from approximately 40% per year to less than 2% per year. Occasionally, it may become necessary to temporarily deactivate an AICD, as in the case of inappropriate shock in the setting of a stable rhythm (such as seen with the patient in question). The patient in this scenario is receiving a shock while he is in sinus rhythm. Some potential causes of inappropriate shock delivery include false sensing of supraventricular tachycardias, muscular activity (eg, shivering), sensing T waves as QRS complexes, unsustained tachydysrhythmias, and component failure. AICDs are generally inactivated by placing a magnet over the AICD generator. Although, there is some variability depending on the generation of the AICD, most EDs have a special donut magnet that is reserved for this function. If the patient subsequently experiences a dysrhythmia in the setting of having his or her defibrillator tuned off the physician should use the bedside defibrillator to treat the patient.

For which of the following patients is an abdominal CT scan contraindicated? a. A 52-year-old man with abdominal pain after blunt trauma, negative focused assessment with sonography for trauma (FAST) examination, BP 125/78 mm Hg, and HR 109 beats per minute b. A 22-year-old female with RLQ pain, negative β-hCG, temperature 100.6°F c. A 45-year-old man with abdominal pain, temperature 100.5°F, WBC 11,200/μL, BP 110/70 mm Hg, HR 110 beats per minute, and lipase 250 IU d. A 70-year-old man with abdominal pain, an 11-cm pulsatile mass in the epigastrium, BP of 70/50 mm Hg, and HR of 110 beats per minute e. A 65-year-old woman with right flank pain that radiates to her groin, microhematuria, BP 165/85 mm Hg, and HR 105 beats per minute

*D.* CT scanning has become an integral part of the ED evaluation of a patient. It is useful to differentiate abdominal pathologies when the history and physical examination are nonspecific, or in confirming a diagnosis suspected by the clinical presentation. Contrast materials for CT examinations in the ED are usually administered via oral and IV routes, which improves visualization of abdominal organs. A patient must be relatively stable to undergo a CT scan. The patient with abdominal pain and a BP of 70/50 mm Hg is not hemodynamically stable. His clinical presentation is consistent with a ruptured abdominal aortic aneurysm. This is a surgical emergency and should be treated in the OR. Obtaining a CT scan first will delay definitive treatment and worsen the patient's hemorrhagic shock and ultimately decrease his chance of survival.

A 42-year-old man found vomiting in the street is brought to the ED by EMS. He has a known history of alcohol abuse with multiple presentations for intoxication. Today, the patient complains of acute onset, persistent chest pain associated with dysphagia, and pain upon flexing his neck. His BP is 115/70 mm Hg, HR is 101 beats per minute, RR is 18 breaths per minute, and oxygen saturation is 97% on room air. As you listen to his heart, you hear a crunching sound. His abdomen is soft with mild epigastric tenderness. The ECG is sinus tachycardia without ST-T-wave abnormalities. On chest x-ray, you note lateral displacement of the left mediastinal pleural. What is the most likely diagnosis? a. Aspiration pneumonia b. Acute pancreatitis c. Pericarditis d. Esophageal perforation e. Aortic dissection

*D.* Esophageal perforation is a potentially life-threatening condition that can result from any Valsalva-like maneuver, including childbirth, coughing, and heavy lifting. Alcoholics are at risk as a result of their frequent vomiting. The most common cause of esophageal perforation is from iatrogenic causes, such as a complication from upper endoscopy. The classic physical examination finding is mediastinal or cervical emphysema. This is noted by feeling air under the skin on palpation of the chest wall or by a *crunching sound heard on auscultation, also known as Hamman sign*. Radiographic signs of pneumomediastinum can be subtle. Lateral displacement of the mediastinal pleura by mediastinal air creates a linear density paralleling the mediastinal contour. On the lateral projection, mediastinal air can be seen in the retrocardiac space

A 79-year-old man was being commemorated at an awards dinner for his 50 years of service at the local bank. While eating a steak dinner, he felt food get stuck in his stomach. He drank a glass of water, but shortly thereafter vomited the water up. He is a bit anxious and decides to come to the ED for further evaluation. His BP is 155/70 mm Hg, HR is 98 beats per minute, RR is 18 breaths per minute, and oxygen saturation is 99% on room air. What is the most common area for an esophageal foreign body to lodge in an adult? a. Aortic arch b. Cricopharyngeus muscle c. Tracheal bifurcation d. Lower esophageal sphincter e. Pyloric sphincter

*D.* Foreign bodies tend to lodge or impact at sites where esophageal narrowing occurs. In the adult, the lower esophageal sphincter is the most common site for impactions. When impaction occurs, patients usually feel some discomfort, sometimes substernal chest pain, anxiety, and progressive dysphagia. Most of the time, the patient can accurately locate the position of the impacted foreign body.

A 71-year-old woman presents to the ED after a reported mechanical fall 2 days ago. Her initial vitals include a HR of 55 beats per minute, a BP of 110/60 mm Hg, an RR of 14 breaths per minute, and an oxygen saturation of 96% on room air. The patient does not appear to be taking deep breaths. Her physical examination is significant for decreased breath sounds bilaterally and tenderness to palpation along the right side of her chest. After initial stabilization, which of the following is the diagnostic test of choice for this patient's condition? a. Chest x-ray b. Chest CT scan c. ECG d. Rib radiographs e. Thoracentesis

*D.* Given the history of trauma, a rib fracture is the most probable etiology of the patient's symptoms in this clinical scenario. Rib fractures usually occur at the point of impact or at the posterior angle, which is the weakest part of the rib. It is important to note that the true danger of rib fractures involves not the rib itself, but the risk of penetrating injury to underlying structures. A rib series is the most effective way to visualize these fractures. Treatment of patients with simple acute rib fractures includes pain relief so that respiratory splinting does not occur, which increases the rate of atelectasis and pneumonia. Chest binders should not be used as they promote hypoventilation. For multiple rib fractures, intercostal nerve blocks may be a more effective means of analgesia. Most rib fractures heal uneventfully within 3 to 6 weeks. The patient should be encouraged to take deep breaths to avoid developing pneumonia.

A 30-year-old obese woman with no significant past medical history presents to the ED complaining of shortness of breath and coughing up blood-streaked sputum. The patient states that she traveled to Moscow a month ago. Upon returning to the United States, the patient developed a persistent cough associated with dyspnea. She was seen by a pulmonologist, who diagnosed her with bronchitis and prescribed an inhaler. However, over the following weeks, the patient's symptoms worsened, and she developed pleuritic chest pain. In the ED, she lets you know that she smokes half a pack per day. Her vitals include a temperature of 99°F, BP of 105/65 mm Hg, HR of 124 beats per minute, RR of 22 breaths per minute, and an oxygen saturation of 94% on room air. Physical examination is noncontributory, except for rales at the left-mid lung. Her ECG reveals sinus tachycardia with large R waves in V1 to V3 and inverted T waves. Given this patient's history and presentation, what is the most likely etiology of her symptoms? a. Mycoplasma pneumoniae ("walking" pneumonia) b. Q fever pneumonia c. Pneumocystis jiroveci pneumonia (PCP) d. PE e. ARDS

*D.* In the history there are details that lead you to suspect pulmonary embolus: obesity, recent travel, progressive dyspnea despite inhaler treatment, and blood-streaked sputum. Objectively, her ECG shows right heart strain (large R waves and inverted T waves in the precordial leads V1-V3), which is because of the heart beating against the high resistance of the pulmonary vasculature causing backflow resulting in right ventricular enlargement. She is tachycardic, tachypneic, and hypoxic, cardinal signs of cardiovascular distress. Either dyspnea, pleuritic chest pain, or tachypnea is present in 95% of patients with a PE. The classic triad of dyspnea, pleuritic chest pain, and hemoptysis is uncommon and present in less than 25% of patients.

A 61-year-old woman was on her way to the grocery store when she started feeling chest pressure in the center of her chest. She became diaphoretic and felt short of breath. On arrival to the ED by EMS, her BP is 130/70 mm Hg, HR is 76 beats per minute, and oxygen saturation is 98% on room air. The nurse gives her an aspirin and an EKG shows ST elevation in II, III, and aVF. Which of the following best describes the location of this patient's myocardial infarction (MI)? a. Anteroseptal b. Anterior c. Lateral d. Inferior e. Posterior

*D.* Inferior wall MIs are characterized by ST elevation in at least two of the inferior leads (II, III, aVF).

A 22-year-old man presents to the ED with a history consistent with an acute MI. His ECG reveals ST elevations and his cardiac biomarkers are positive. He has been smoking half a pack of cigarettes per day for the last 3 months. He drinks alcohol when hanging out with his friends. His grandfather died of a heart attack at 80 years of age. The patient does not have hypertension or diabetes mellitus and takes no prescription medications. A recent cholesterol check revealed normal levels of total cholesterol, lowdensity lipoprotein (LDL), and high-density lipoprotein (HDL). Which of the following is the most likely explanation for his presentation? a. Cigarette smoking b. Family history of heart attack at age 80 years c. Incorrectly placed leads on the ECG d. Undisclosed cocaine use e. Alcohol use

*D.* Often young people are afraid to disclose a history of drug use. Cocaine is well-known to cause acute MI in young, otherwise healthy individuals. Patients with cocaine-related MI often have fixed atherosclerotic lesions. Although these lesions may themselves be of clinical significance, cocaine-induced elevations in pulse and BP increase myocardial work. The additional metabolic requirements that result may convert an asymptomatic obstruction into one of clinical significance. Cocaine use combined with ethanol consumption produces cocaethylene, a longer-acting and more toxic by-product.

A 59-year-old man presents to the ED complaining of vomiting and sharp abdominal pain in the epigastric area that began abruptly this afternoon. He describes feeling nauseated and has no appetite. Laboratory results reveal WBC 18,000/μL, hematocrit 48%, platelets 110/μL, AST 275 U/L, ALT 125 U/L, alkaline phosphatase 75 U/L, amylase 1150 U/L, lipase 1450 IU, LDH 400 U/L, sodium 135 mEq/L, potassium 3.5 mEq/L, chloride 110 mEq/L, bicarbonate 20 mEq/L, BUN 20 mg/dL, creatinine 1.5 mg/dL, and glucose 250 mg/dL. Which of the following laboratory results correlate with the poorest prognosis? a. Amylase 950, lipase 1250, LDH 400 b. Lipase 1250, LDH 400, bicarbonate 20 c. Lipase 1250, creatinine 1.5, potassium 3.5 d. WBC 18,000, LDH 400, glucose 250 e. WBC 18,000, amylase 950, lipase 1250

*D.* The patient's clinical picture is consistent with acute pancreatitis. Ranson developed criteria that help predict mortality rates in patients with pancreatitis. The presence of more than three criteria equals 1% mortality, while the presence of six or more criteria approaches 100% mortality. Ranson criteria at admission are age > 55, WBC > 16,000, glucose > 200, LDH > 350, AST > 250. Within 48 hours of admission, hematocrit fall > 10%, BUN rise > 5, serum calcium < 8, arterial PO2 < 60, base deficit > 4, and fluid sequestration > 6 L. The patient in the case fulfills four of Ranson criteria and has approximately 15% mortality risk. Note that lipase and amylase are not part of Ranson criteria despite being relevant in the diagnosis of acute pancreatitis. The mnemonic GA LAW & CHOBBS can be used to remember this criteria. Stands for Glucose, Age, LDH, AST and WBC; Calcium, Hematocrit, Oxygen, BUN, Base, Sequestration.

A 41-year-old woman presents to the ED complaining of pain in her RUQ that is steady but gets worse with eating over the past 2 days. The pain also radiates to the right side of her midback. She denies vomiting. Her only medication is an oral contraceptive. Her BP is 140/75 mm Hg, HR is 80 beats per minute, temperature is 98.7°F, and RR is 16 breaths per minute. Laboratory tests are within normal limits. An abdominal ultrasound reveals stones in her gallbladder, but no thickened wall or pericholecystic fluid. What is the most likely diagnosis? a. Cholangitis b. Urolithiasis c. Cholecystitis d. Biliary colic e. Peptic ulcer disease

*D.* The patient's clinical picture is consistent with biliary colic because of the passage of small stones from the gallbladder through the cystic duct into the common bile duct. The term colic is a misnomer in that these patients usually have a steady pain rather than an intermittent pain. Pain is present in the RUQ and often is referred to the base of the scapula. Laboratory results are commonly normal. Ultrasound reveals stones in the gallbladder without other pathologies. Patients can usually be sent home with pain medications and are instructed to avoid fatty foods.

An 81-year-old diabetic woman with a history of atrial fibrillation is transferred to your emergency department (ED) from the local nursing home. The note from the facility states that the patient is complaining of abdominal pain, having already vomited once. Her vital signs in the ED are temperature 100.1°F, blood pressure (BP) 105/75 mm Hg, heart rate (HR) 95 beats per minute, and respiratory rate (RR) 18 breaths per minute. You examine the patient and focus on her abdomen. Considering that the patient has not stopped moaning in pain since arriving to the ED, you are surprised to find that her abdomen is soft on palpation. You decide to order an abdominal radiographic series. Which of the findings on plain abdominal film is strongly suggestive of mesenteric infarction? a. Sentinel loop of bowel b. No gas in the rectum c. Presence of an ileus d. Pneumatosis intestinalis e. Air-fluid levels

*D.* There is a high suspicion that this patient has mesenteric ischemia, secondary to a thromboembolism from her atrial fibrillation. The typical patient with mesenteric ischemia may initially present with "pain that is out of proportion to the examination" (ie, although the patient is in pain, the abdomen is neither rigid, nor significantly tender on physical examination). Abdominal distention and peritoneal signs are late findings and signal the presence of bowel infarction. Plain abdominal radiographs are usually obtained early on in the workup. Although rare, the finding of gas in either the bowel wall (pneumatosis intestinalis) or in the portal venous system is strongly suggestive of intestinal infarct. This is a surgical emergency! (a, b, c, and e) These are possible findings but less specific

A 63-year-old insurance agent is brought to the ED by paramedics for shortness of breath and a RR of 31 breaths per minute. The patient denies chest pain, fever, vomiting, or diarrhea. His wife says he ran out of his "water pill" 1 week ago. His BP is 185/90 mm Hg, HR is 101 beats per minute, oxygen saturation is 90% on room air, and temperature is 98.9°F. There are crackles midway up both lung fields and 2+ pitting edema midway up his legs. An ECG shows sinus tachycardia. The patient is sitting up and able to speak to you. After placing the patient on a monitor and inserting an IV, which of the following is the most appropriate next step in management? a. Obtain blood cultures, complete blood cell (CBC) count, and begin empiric antibiotic therapy. b. Order a STAT portable CXR c. Administer oxygen via nasal cannula and have the patient chew an aspirin. d. Administer oxygen via non-rebreather, furosemide, nitroglycerin, and consider noninvasive respiratory therapy. e. Rapid sequence endotracheal intubation.

*D.*The patient has acute CHF exacerbation with acute pulmonary edema (APE). Although not always apparent at presentation, it is important to find the cause of the exacerbation. This patient, for instance, has been noncompliant with his medications. Treatment begins by assessing the airway, breathing, and circulation (ABCs). Initial stabilization is aimed at maintaining airway control and adequate ventilation. Preload and afterload reduction is integral with nitroglycerin being the agent of choice, provided that the patient is not hypotensive. Volume reduction with diuretics is also critical to lower BP and cardiac-filling pressures. Noninvasive airway techniques (eg, bilevel positive airways pressure [BiPAP], continuous positive airway pressure [CPAP]) also aid in improving oxygen exchange, reducing the work of breathing, and decreasing left ventricular preload and afterload by raising intrathoracic pressure in the compromised but not agonal APE patients.

A 29-year-old woman presents to the ED for hyperventilation. Her initial vitals include an RR of 28 breaths per minute with an oxygen saturation of 100% on room air. She is able to speak in full sentences and tells you that she cannot breathe and that her hands and feet are cramping up. She denies any trauma, past medical history, or illicit drug use. Chest auscultation reveals clear breath sounds bilaterally. A subsequent chest radiograph is normal. Upon reevaluation, the patient reports that she is breathing better. Her vitals include an RR of 12 breaths per minute with an oxygen saturation of 100% on room air. Which of the following conditions is most likely the etiology of this patient's symptoms? a. Pneumothorax b. Hemopneumothorax c. Pleural effusion d. Anxiety attack e. Asthma exacerbation

*D.*The patient has normal vitals, a normal chest radiograph, and is stable. In fact, her symptoms self-resolve with time and without intervention. Only with this information can one be comfortable with making the diagnosis of an anxiety attack as the precipitant to the patient's symptoms. A history of a stressor may be helpful, but it is important to note that these symptoms are not under the voluntary control of the patient, and often patients may not even be able to identify a specific stressor. It is important to remember that the diagnosis of anxiety in the ED is a diagnosis of exclusion. Her extremity symptoms are typical of carpal-pedal spasm seen with tetany, a result of a transient decrease in calcium serum levels secondary to a respiratory alkalosis.

A 60-year-old man with a history of alcohol abuse presents to the ED with hematemesis for 1 day. He denies abdominal or chest pain. On physical examination, his eyes appear reddened which he attributes to having drunken heavily the night before (he also reveals vomiting several times after this recent binge). Vital signs are: HR 115 beats per minute, BP 130/85 mm Hg, RR 18 breaths per minute, and temperature 99.5°F. Chest radiograph is unremarkable. Laboratory results reveal a WBC 10,000/μL, hemoglobin 14 mg/dL, hematocrit 40%, and platelets 210/μL. Which diagnosis is endoscopic evaluation most likely to confirm? a. Esophageal varices b. Boerhaave syndrome c. Curling ulcer d. Perforated gastric ulcer e. Mallory-Weiss tear

*E* A Mallory-Weiss tear usually follows a forceful bout of retching and vomiting and involves a 1- to 4-cm area of the mucosa or submucosa of the GI tract; 75% of cases occur in the stomach with the remainder near the gastroesophageal (GE) junction. Bleeding is usually mild and self-limited. However, 3% of deaths from upper GI bleeds result from Mallory-Weiss tears.

A 23-year-old man presents to the ED complaining of left hand pain. He states that he was mad at a friend and punched the wall in his bedroom. Immediately after he punched the wall, he felt intense pain in his left hand. On physical examination, you note swelling and tenderness over the fifth metacarpal. When you ask him to make a fist, his fifth finger rotates to lie on top of his fourth finger. What is the name of this type of fracture? a. Colles fracture b. Smith fracture c. Scaphoid fracture d. Galeazzi fracture e. Boxer's fracture

*E* A boxer's fracture is a fracture of the neck of the fifth metacarpal. It is one of the most common fractures of the hand and usually occurs from a direct impact to the hand (eg, a punch with a closed fist).

A 43-year-old man presents to the ED complaining of progressively worsening abdominal pain over the past 2 days. The pain is constant and radiates to his back. He also describes nausea and vomiting and states he usually drinks 6 pack of beer daily, but has not had a drink for 2 days. His BP is 144/75 mm Hg, HR is 101 beats per minute, temperature is 99.8°F, and RR is 14 breaths per minute. He is lying on his side with his knees flexed. Examination shows voluntary guarding and tenderness to palpation of his epigastrium. Laboratory results reveal WBC 10,500/μL, hematocrit 51%, platelets 225/μL, and lipase 620 IU. An abdominal radiograph reveals a nonspecific bowel gas pattern. There is no free air under the diaphragm. Which of the following is the most appropriate next step in management? a. Observe in the ED b. Send home with antibiotic therapy c. Admit to the hospital for endoscopy d. Admit to the hospital for exploratory laparotomy e. Admit to the hospital for medical management and supportive care

*E* Abdominal pain that radiates to the back and is associated with nausea, vomiting, epigastric tenderness, and an elevated lipase in a patient with a history of ethanol abuse all point to a diagnosis of alcoholic pancreatitis. Although patients with mild pancreatitis, no evidence of systemic complications, and a low likelihood of biliary tract disease may be managed as outpatients, this patient requires admission for the potentially rapid progression of symptoms, severity of pain, and possible unreliability of the patient. The initial treatment for acute pancreatitis is supportive: bowel rest, fluid resuscitation, and analgesia. Ninety percent of patients recover without complications. Surgery is reserved for complications of alcoholic pancreatitis, such as pseudocysts, phlegmons, and abscesses.

An 18-year-old woman is brought to the ED by her mother. The patient is diaphoretic and vomiting. Her mom states that she thinks her daughter tried to commit suicide. The patient admits to ingesting a few handfuls of extra-strength Tylenol approximately 3 hours ago. Her temperature is 99.1°F, BP is 105/70 mm Hg, HR is 92 beats per minute, RR is 17 breaths per minute, and oxygen saturation is 99% on room air. On examination, her head and neck are unremarkable. Cardiovascular and pulmonary examinations are within normal limits. She is mildly tender in her right upper quadrant but there is no rebound or guarding. Bowel sounds are normoactive. She is alert and oriented and has no focal deficits on neurologic examination. You administer 50 g of activated charcoal. At this point, she appears well and has no complaints. Her 4-hour serum acetaminophen (APAP) concentration returns at 350 μg/mL. You plot the level on the nomogram seen below. Which of the following is the most appropriate next step in management? a. Discharge home with instructions to return if symptoms return b. Observe for 6 hours and if the patient still has no complaints discharge her home c. Admit to the hospital for serial abdominal examinations d. Admit to the psychiatry unit and keep on suicide watch while performing serial abdominal examinations e. Begin NAC and admit to the hospital

*E* Acetaminophen is one of the most commonly used analgesic-antipyretic medications and causes more hospitalizations after overdose than by any other pharmaceutical agent. Risk of hepatotoxicity is best established by plotting the acetaminophen concentration on the acetaminophen nomogram. Acetaminophen concentration must be measured between 4 and 24 hours after ingestion and then plotted on the nomogram. Patients with acetaminophen concentrations on or above the treatment line should be treated. This patient has a 4-hour serum acetaminophen concentration of 350 μg/mL. According to the nomogram, at 4 hours any concentration above 200 μg/mL should be treated. Therefore, the patient should be started on NAC and admitted to the hospital. During her admission, she should be evaluated by a psychiatrist regarding her attempted suicide.

A 24-year-old man is brought to the ED after being shot once in the abdomen. On arrival, his BP is 100/60 mm Hg, HR is 115 beats per minute, and RR is 22 breaths per minute. His airway is patent and you hear breath sounds bilaterally. On abdominal examination, you note a single bullet entry wound approximately 1 cm to the right of the umbilicus. During the log roll, you see a single bullet exit wound approximately 3 cm to the right of the lumbar spine. His GCS score is 15. The patient's BP is now 85/65 mm Hg and HR is 125 beats per minute after 2 L of fluid. Which of the following is the most appropriate next step in management? a. Probe the entry wound to see if it violates the peritoneum b. Perform a FAST examination c. Perform a DPL d. Take the patient directly to the CT scanner e. Take the patient directly to the OR

*E* An important concern with anterior abdominal GSW is to determine whether the missile traversed the peritoneal cavity. Patients with transabdominal GSWs nearly all have intra-abdominal injuries requiring surgery. Most of the time, this can be determined by approximating the trajectory. Therefore, a hole in both the anterior and posterior abdomen highly suggests a transabdominal trajectory. If there are a single or odd number of holes, a plain film may help estimate the trajectory. In cases of tangential or multiple GSWs, it may be impossible to determine trajectory with any certainty. In a patient with evidence of peritoneal penetration, a missile tract that clearlyenters the abdominal cavity, or has a positive diagnostic study (eg, DPL, FAST, CT) in a tangential wound, he or she should undergo exploratory laparotomy. The standard algorithm for penetrating abdominal trauma recommends that any patient with unstable vital signs be taken directly to the OR to undergo an exploratory laparotomy. If their vital signs are stable, they should undergo further diagnostic studies, such as a FAST examination, DPL, or CT scan.

A 42-year-old man who is actively seizing is brought to the ED by EMS after a massive ingestion of an unknown substance. The man is known to have a history of acquired immunodeficiency syndrome (AIDS). An intravenous (IV) line is established and anticonvulsant therapy is administered. After high doses of diazepam, phenobarbital, and phenytoin, it is determined that the seizures are refractory to standard anticonvulsant therapy. Which of the following substances did this patient most likely ingest? a. Cocaine b. Diphenhydramine c. Tricyclic antidepressant d. Camphor e. Isoniazid (INH)

*E* An overdose of any of these agents can lead to seizures. However, INH is notorious for causing seizures that are refractory to standard therapy. Marked acidosis and respiratory compromise may also be present. Pyridoxine (vitamin B6) is the treatment of choice for INH overdose. INH is used for the treatment of tuberculosis, which is seen with a greater incidence in patients with AIDS. All of the other substances listed as answer choices should respond to standard therapy with benzodiazepines.

A 55-year-old man with hypertension and end-stage renal disease requiring hemodialysis presents with 2 days of painless hematochezia. He reports similar episodes of bleeding in the past, which were attributed to angiodysplasia. He denies abdominal pain, nausea, vomiting, diarrhea, and fever. His vitals include HR of 90 beats per minute, BP of 145/95 mm Hg, RR of 18 breaths per minute, and temperature of 98°F. His abdomen is soft and nontender and his stool is grossly positive for blood. Which of the following statements are true regarding angiodysplasia? a. They are responsible for over 50% of acute lower GI bleeding. b. They are more common in younger patients. c. Angiography is the most sensitive method for identifying angiodysplasias. d. They are less common in patients with end-stage renal disease. e. The majority of angiodysplasias are located on the right side of the colon.

*E* Angiodysplasias, also known as arteriovenous malformations, are small ectatic blood vessels in the submucosa of the GI tract. More than half of angiodysplasias are located on the right side of the colon.

A 51-year-old man with a long history of hypertension presents to the ED complaining of intermittent chest palpitations lasting for a week. He denies chest pain, shortness of breath, nausea, and vomiting. He recalls feeling similar episodes of palpitations a few months ago but they resolved. His blood pressure (BP) is 130/75 mm Hg, heart rate (HR) is 130 beats per minute, respiratory rate (RR) is 16 breaths per minute, and oxygen saturation is 99% on room air. An ECG shows a fib. Which of the following is the most appropriate next step in management? a. Sedate patient for immediate synchronized cardioversion with 100 Joules b. Prepare patient for the cardiac catheterization laboratory c. Administer Coumadin d. Administer amiodarone e. Administer diltiazem

*E* Atrial fibrillation (AF) is a rhythm disturbance of the atria that results in irregular, chaotic, ventricular waveforms. This chaotic activity can lead to reduced cardiac output from a loss of coordinated atrial contractions and a rapid ventricular rate, both of which may limit diastolic filling and stroke volume of the ventricles. Atrial fibrillation may be chronic or paroxysmal, lasting minutes to days. On the ECG, fibrillatory waves are seen and accompanied by an irregular QRS pattern. The main ED treatment for atrial fibrillation is rate control. This can be accomplished by many agents, but the agent most commonly used is diltiazem, a CCB with excellent AV nodal blocking effects

A 34-year-old construction worker is brought to the ED by EMS after falling 30 ft from a scaffold. His vital signs are HR 124 beats per minute, BP 80/40 mm Hg, and oxygen saturation 93% on 100% oxygen. He has obvious head trauma with a scalp laceration overlying a skull fracture on his occiput. He does not speak when asked his name, his respirations are poor and you hear gurgling with each attempted breath. Auscultation of the chest reveals diminished breath sounds on the right. There is no jugular venous distension (JVD) or anterior chest wall crepitus. His pelvis is unstable with movement laterally to medially and you note blood at the urethral meatus. His right leg is grossly deformed at the knee and there is an obvious fracture of his left arm. Which of the following is the most appropriate next step in management? a. Insert a 32F chest tube into the right thoracic cavity. b. Perform a DPL to rule out intra-abdominal hemorrhage. c. Create two Burr holes into the cranial vault to treat a potential epidural hematoma. d. Immediately reduce the extremity injuries and place in a splint until the patient is stabilized. e. Plan for endotracheal intubation of the airway with in-line stabilization of the cervical spine.

*E* Based on the principles of advanced trauma life support (ATLS), injured patients are assessed and treated in a fashion that establishes priorities based on their presenting vital signs, mental status, and injury mechanism. The approach to trauma care consists of a primary survey, rapid resuscitation, and a more thorough secondary survey followed by diagnostic testing. The goal of the primary survey is to quickly identify and treat immediately life-threatening injuries. The assessment of the ABCDEs (airway, breathing, circulation, neurologic disability, exposure) is a model that should be followed in all patients.

A 27-year-old man presents to the ED extremely agitated complaining of mild chest pain and dyspnea. He states that he was snorting cocaine all afternoon. You place him on a monitor and get his vital signs. His BP is 215/130 mm Hg, HR is 112 beats per minute, temperature is 100.1°F, RR is 17 breaths per minute, and oxygen saturation is 98% on room air. An ECG reveals sinus tachycardia at a rate of 116. Which of the following is the most appropriate medication to administer? a. Haloperidol b. Labetalol c. Esmolol d. Diltiazem e. Diazepam

*E* Benzodiazepines (eg, diazepam) should be used as the first-line agent for nearly all cocaine toxicities. Many effects of cocaine are thought to be mediated through CNS stimulation by the release or inhibiting the reuptake of catecholamines. The effects that acute cocaine intoxication has on the heart include coronary vasoconstriction with increasing myocardial oxygen demand. The goal of lowering the BP in this patient is to reverse the vasoconstriction by norepinephrine at peripheral α-adrenergic receptors. Benzodiazepines restore the CNS inhibitory tone on the peripheral nervous system. The use of β-adrenergic antagonists should be avoided with cocaine because of paradoxical hypertension and coronary artery vasoconstriction because of unopposed α-adrenergic receptor stimulation.

A 35-year-old man who is employed as a forklift operator was found sitting outside of a warehouse. He came stumbling out complaining of dizziness and headaches. Coworkers in an adjoining warehouse also complained of headache and nausea. After collapsing outside, he regained consciousness immediately but appeared confused. In the ED, his BP is 100/54 mm Hg, HR is 103 beats per minute, temperature is 100°F, and RR is 23 breaths per minute. Physical examination is unremarkable. Laboratory results reveal WBC 10,500/μL, hematocrit 45%, platelets 110/μL, sodium 137 mEq/L, potassium 4 mEq/L, chloride 103 mEq/L, bicarbonate 21 mEq/L, BUN 8 mg/dL, creatinine 0.5 mg/dL, and glucose 89 mg/dL. Arterial blood gas results are pH 7.32, PCO2 32 mm Hg, and PO2 124 mm Hg. Which of the following is the most likely diagnosis? a. Methemoglobinemia b. Hypoglycemic syncope c. Hydrocarbon poisoning d. Opioid overdose e. CO poisoning

*E* CO poisoning is the leading cause of poisoning morbidity and mortality in the United States. People are exposed to CO through fires, vehicle exhaust, home generators, and the metabolism of methylene chloride. Workers also become symptomatic from use of propane-powered equipment indoors, such as forklifts and ice skating resurfaces. Often, other people in the area have similar complaints. The earliest symptoms are nonspecific and readily confused with other illnesses such as viral syndromes. Mild symptoms include headache, nausea, and dizziness. Severe symptoms include chest pain, palpitations, and seizures. Diagnosis is made by detecting elevated CO in the blood. Normal levels range from 0% to 5%, while smokers may be as high as 10%. Treatment includes immediate oxygen therapy. Hyperbaric oxygen is the treatment of choice for patients with significant CO exposures.

A 23-year-old woman presents to the ED complaining of pain with urination. She has no other complaints. Her symptoms started 3 weeks ago. During this time, she has been to the clinic twice, with negative urine cultures each time. Her condition has not improved with antibiotic therapy with sulfonamides or quinolones. Physical examination is normal. Which of the following organisms is most likely responsible for the patient's symptoms? a. Staphylococcus aureus b. Herpes simplex virus c. Trichomonas vaginalis d. Escherichia coli e. Chlamydia trachomatis

*E* Chlamydia trachomatis urethritis accounts for 5% to 20% of cases of dysuria, and its presence may be suspected when urine cultures are sterile. If clinical symptoms and urinalysis point to a UTI, urine cultures are sterile, and standard antibiotic regimens effective against most urinary pathogens fail, consider Chlamydia urethritis.

A 39-year-old woman is brought to the ED by EMS on a backboard and cervical collar after being involved in a motor vehicle collision. The primary survey is unremarkable. However, the secondary survey reveals a deformity of the right arm involving the upper arm and elbow. Which of the following associations are paired incorrectly? a. Olecranon fracture and ulnar nerve injury b. Posterior elbow dislocation and ulnar and median nerve injury c. Anterior elbow dislocation and brachial artery injury d. Supracondylar fracture and brachial artery and median nerve injury e. Humeral shaft fracture and axillary nerve injury

*E* Humeral shaft fractures most commonly occur from a direct blow to the mid-upper arm. The fracture usually involves the middle third of the humeral shaft. The most common associated injury is damage to the radial nerve that causes wrist drop and loss of sensation in the first dorsal web space. Ulnar and median nerve injuries may also occur but are much less common

A 27-year-old woman presents to the ED 6 hours after the onset of body aches, abdominal cramping, and diarrhea. She is currently visiting relatives and normally lives in another state. She regularly takes six to eight tablets daily of hydrocodone for chronic low-back pain, sumatriptan for migraines, and amitriptyline and paroxetine for bulimia nervosa. Her BP is 130/80 mm Hg, HR is 100 beats per minute, temperature is 98.6°F, RR is 16 breaths per minute, and oxygen saturation is 99% on room air. Examination shows diaphoresis, dilated pupils, and piloerection. Neurologically she is moving all four extremities and you do not note tremors. She is alert and cooperative but seems restless. She denies hallucinations or suicidal ideations. She becomes very angry when you ask her for the phone numbers of her regular physicians. Which of the following is the most likely explanation of her symptoms? a. Anticholinergic overdose b. TCA intoxication c. Ethanol withdrawal d. Serotonin syndrome e. Opiate withdrawal

*E* Hydrocodone is an opioid used for pain relief. Opioid withdrawal occurs in tolerant individuals when opioid exposure is discontinued or an antagonist is administered. The effects of withdrawal are secondary to increased sympathetic discharge, which is responsible for the clinical signs and symptoms. Although these can be significant, they are typically not life-threatening. Withdrawal is associated with CNS excitation, tachypnea, and mydriasis. Pulse and BP may be elevated. The patient may present with nausea, vomiting, diarrhea, abdominal cramps, myalgias, and insomnia. Examination often reveals piloerection, yawning, lacrimation, rhinorrhea, and diaphoresis. Neurologic manifestations include restlessness, agitation, and anxiety, but cognition and mental status are unaffected. Dysphoria and drug craving are usually prominent.

During a busy day working in the ED, five patients come in all of whom you believe require an LP to evaluate for meningitis. From the following list of patients below, which one can you forgo a CT scan and proceed directly to an LP? a. A 65-year-old man with fever and headache b. A 49-year-old woman with acquired immunodeficiency syndrome (AIDS) c. A 74-year-old man with new right lower extremity motor weakness d. A 19-year-old man with a fever who is lethargic and disoriented e. A 51-year-old woman who is febrile and complains of neck stiffness

*E* Indications for brain CT scanning prior to LP include: patients who are older than 60 years, patients who are immunocompromised, patients with known CNS lesions, patients who presented with a seizure or who had a seizure within 1 week of presentation, patients with abnormal level of consciousness, patients with focal findings on neurologic examination, and patients with papilledema seen on physical examination with clinical suspicion of elevated ICP. All of the patients in this scenario presented fit one of these contraindications except for the 51-year-old woman with fever and neck stiffness.

A 67-year-old man is brought to the ED in respiratory distress. His initial vitals include a HR of 112 beats per minute, a BP of 145/88 mm Hg, and an RR of 18 breaths per minute with an oxygen saturation of 92% on room air. He is also febrile at 102°F. After obtaining IV access, placing the patient on a monitor, and administering oxygen via nasal cannula, a chest radiograph is performed and shows patchy alveolar infiltrates with consolidation in the lower lobes. On review of systems, the patient tells you that he had five to six watery bowel movements a day for the last 2 days with a few bouts of emesis. Which of the following infectious etiologies is most likely responsible for the patient's presentation? a. Streptococcus pneumoniae b. Haemophilus influenzae c. Mycoplasma pneumoniae d. Chlamydophila pneumoniae e. Legionella pneumophila

*E* Legionella pneumophila is an intracellular organism that lives in aquatic environments. The organism may live in ordinary tap water and has probably been underdiagnosed in a number of community outbreaks. It is typically seen in the elderly and immunocompromised. Legionnaire disease is more common in the summer, especially in August. Patients often experience a prodrome of 1 to 2 days of mild headache and myalgias, followed by high fever, chills, and multiple rigors. Cough is present in 90% of cases. Other pulmonary manifestations include dyspnea, pleuritic chest pain, and hemoptysis. GI symptoms include nausea, vomiting, diarrhea, and anorexia. Neurologic symptoms include headache, altered mental status, and rarely, focal symptoms. Urine antigen testing is highly specific and sensitive and, if available, very rapid in making the diagnosis.

A 71-year-old man presents to the ED with diffuse, crampy abdominal pain that began 1 hour after eating lunch today. The pain is intermittent over the last 8 hours with increasing severity. He also complains of nausea and chills, and vomited once on his way to the ED. He has not had a bowel movement or flatus since the pain began. His past medical history includes prostate cancer, left total hip replacement, appendectomy 25 years ago, right iliac artery aneurysm repair 5 years ago, incisional hernia repair 4 years ago, and irritable bowel syndrome. Which of the following is the most common cause of SBO in adults? a. Bezoar b. Neoplasm c. Incarcerated hernia d. Gallstone ileus e. Adhesion

*E* Overall, adhesions, hernias, and cancer account for more than 90% of cases of SBO. Postoperative adhesions are the most common cause of SBO

A 79-year-old man presents to the ED by paramedics with the chief complaint of agitation and confusion over the previous 12 hours. He has a past medical history of schizophrenia and is not taking any of his antipsychotics. His BP is 135/85 mm Hg, HR is 119 beats per minute, RR is 18 breaths per minute, oxygen saturation is 97% on room air, and fingerstick glucose is 135 mg/dL. Because of his agitation at triage, he was placed in wrist restraints. At this time, he is calm but confused. Examination reveals warm and clammy skin and 4-mm pupils that are equal and reactive. His cardiac examination reveals tachycardia and no murmurs. His lungs are clear to auscultation and his abdomen is soft and nontender. He is able to move all of his extremities. Which of the following is the most appropriate next step in management? a. Administer haloperidol or lorazepam b. Consult psychiatry c. Order a CT scan of his head d. Send a urine toxicologic screen e. Obtain a rectal temperature

*E* Patients frequently present to the ED with agitation. It is important to discern what is causing their agitation; the range of etiologies is expansive, from ethanol intoxication to intracerebral bleeding. The approach to the emergency patient always begins with the ABCs (airway, breathing, and circulation). In addition, the vital signs must be obtained early in a patient's assessment in order to reveal potentially life-threatening conditions. The patient in the vignette presents with agitation and tachycardia. Although it is tempting to attribute his agitation to his untreated schizophrenia, doing this without investigating medical causes of agitation can be disastrous. Finding out that the patient's temperature is 103.1°F, for instance, will lead you down a different clinical path than if his temperature is 98°F. This patient was ultimately diagnosed with meningitis.

A 27-year-old man bought to the ED by paramedics after a motor vehicle collision. His RR is 45 breaths per minute, oxygen saturation is 89%, HR is 112 beats per minute, and BP is 115/75 mm Hg. You auscultate his chest and hear decreased breath sounds on the left. Which of the following is the most appropriate next step in management? a. Order a STAT chest radiograph b. Perform a pericardiocentesis c. Perform a DPL d. Perform an ED thoracotomy e. Perform a tube thoracostomy

*E* Patients with a traumatic PTX typically present with shortness of breath, chest pain, and tachypnea. The physical examination may reveal decreased or absent breath sounds over the involved side, as well as subcutaneous emphysema. Any patient with respiratory symptoms in the setting of a PTX should be treated with a tube thoracostomy (chest tube).The preferred site for insertion is the fourth or fifth intercostal space at the anterior or midaxillary line. The tube should be positioned posteriorly and toward the apex so that it can effectively remove both air and fluid.

A 55-year-old man presents to the ED complaining of mild diffuse abdominal pain. He states that he underwent a routine colonoscopy yesterday and was told "everything is fine." The pain began upon waking up and is associated with some nausea. He denies fever, vomiting, diarrhea, and rectal bleeding. His BP is 143/71 mm Hg, HR is 87 beats per minute, temperature is 98.9°F, and RR is 16 breaths per minute. His abdomen is tense but only mildly tender. You order baseline laboratory tests. His chest radiograph is shows air below R diaphragm. Which of the following is the most likely diagnosis? a. Ascending cholangitis b. Acute pulmonary edema c. Acute liver failure d. Pancreatitis e. Pneumoperitoneum

*E* Potential complications of colonoscopy include hemorrhage, perforation, retroperitoneal abscess, pneumoscrotum, pneumothorax, volvulus, and infection. Perforation of the colon with pneumoperitoneum is usually evident immediately, but can take several hours to manifest. Perforation is usually secondary to intrinsic disease of the colon (eg, diverticulitis) or to vigorous manipulation during colonoscopy. Most patients require immediate laparotomy. However, expectant management is appropriate in some patients with a late presentation (1-2 days later), or without signs of peritonitis. The radiograph in the figure demonstrates air under the diaphragm, which is pathognomonic for pneumoperitoneum.

An 85-year-old man is rambling incoherently and not eating at his nursing home. Records indicate that he has a past medical history of hypertension, diabetes, dementia, and benign prostatic hypertrophy. On arrival to the ED, the patient is combative and oriented only to name. His temperature is 101°F rectally, BP is 85/50 mm Hg, HR is 125 beats per minute, RR is 22 breaths per minute, and blood sugar is 154 mg/dL. He appears uncomfortable and cachectic. His lungs are clear to auscultation, with scant crackles at the bases, and his abdomen is soft, nontender, and nondistended. He has a Foley catheter in place draining cloudy, white urine. He has no peripheral edema. Which of the following is the most likely diagnosis? a. Hypovolemic shock b. Neurogenic shock c. Cardiogenic shock d. Anaphylactic shock e. Septic shock

*E* Septic shock is a clinical syndrome of hypoperfusion, hypotension, and multiorgan dysfunction caused by infection. This patient is clearly in shock with hypotension, tachycardia, tachypnea, and acute mental status changes. He also has fever and pus in his urine, making a urine infection the likely source of infection. He requires immediate hydration, broad-spectrum antibiotics, possible intubation, pressors, and admission to the intensive care unit. Remember, elderly patients with comorbid conditions, such as diabetes are more prone to developing sepsis. In addition, patients with indwelling lines, such as Foley catheters, are at an even higher risk for infection.

A 42-year-old IV drug user presents to the ED with fever, chills, pleuritic chest pain, myalgias, and general malaise. The patient's vitals includ a HR of 110 beats per minute, BP of 110/65 mm Hg, RR of 18 breaths per minute, and temperature of 103°F. Physical examination is notable for retinal hemorrhages, petechiae on the conjunctivae and mucus membranes, a faint systolic ejection murmur, and splenomegaly. Which of the following is the most likely diagnosis? a. Sick sinus syndrome b. Myocarditis c. Pericarditis d. Cardiac tamponade e. Endocarditis

*E* Symptoms of endocarditis are nonspecific and vary widely, but the most common include fever (85%) and malaise (80%). In IV drug users, fever is present 98% of the time. Other symptoms include weakness, myalgias, dyspnea, chest pain, cough, headache, and anorexia. Neurologic signs and symptoms (eg, confusion, personality changes, decreased level of consciousness, and focal motor deficits) are seen in 30% to 40% of patients. Vasculitic lesions, including petechiae, splinter hemorrhages, tender fingertip nodules (Osler nodes), and nontender palmar plaques (Janeway lesions) are seen in 35% of patients. Splenomegaly, new heart murmur, and retinal hemorrhages may also be detected on physical examination. Risk factors for infective endocarditis include rheumatic or congenital heart disease, calcific degenerative valve disease, prosthetic heart valve, mitral valve prolapse, a history of IV drug use, or a history of endocarditis. Although any valve can be affected, IV drug use is the most common cause of right-sided endocarditis. The recurrence rate in these patients is 41%, significantly higher than the rate of less than 20% in non-IV drug use patients.

A 25-year-old stockbroker presents to the ED complaining of 6 weeks of daily headaches. Her headaches are band-like in distribution and are not associated with nausea, vomiting, visual phenomena, or neurologic symptoms. Normally they respond to acetaminophen, but they have increased in frequency in the past week as she stopped taking a medication that had been prescribed to prevent them. What type of primary headache syndrome is the patient likely experiencing? a. Migraine headaches b. Cluster type headaches c. Trigeminal neuralgia d. Postherpetic neuralgia e. Tension headache

*E* Tension headaches often occur daily, and classically cause bilateral occipital pain that is described as a band tightening around the head. In general, people experience a constant, dull pain that is nonthrobbing and without the ancillary features associated with migraines (visual phenomena, aura, neurologic complaints, nausea, vomiting). There is often secondary contraction of the neck and scalp musculature. First-line treatment includes NSAIDs and acetaminophen. If the headaches occur frequently enough to cause dysfunction in daily activities, patients may benefit from preventive therapy, such as amitriptyline, desipramine, or propranolol.

An 87-year-old man is brought to the ED on a long board and in a cervical collar after falling down a flight of steps. He denies losing consciousness. On arrival, his vital signs include a HR of 99 beats per minute, BP of 160/90 mm Hg, and RR of 16 breaths per minute. He is alert and speaking in full sentences. Breath sounds are equal bilaterally. Despite an obvious right arm fracture, his radial pulses are 2+ and symmetric. When examining his cervical spine, he denies tenderness to palpation and you do not feel any bony deformities. Which of the following is a true statement? a. Epidural hematomas are very common in the elderly age population. b. Cerebral atrophy in the elderly population provides protection against subdural hematomas. c. Increased elasticity of their lungs, allows elderly patients to recover from thoracic trauma more quickly than younger patients. d. The most common cervical spine fracture in this age group is a wedge fracture of the sixth cervical vertebra. e. Despite lack of cervical spine tenderness, imaging of his cervical spine is warranted.

*E* The Canadian C-spine rule for radiography in alert and stable patients following blunt head or neck trauma identified age greater than 65 years as a high risk factor for C-spine injury, even among those with stable vital signs and a GCS score of 15. Therefore, C-spine imaging in all such elderly patients is warranted.

A 70-year-old woman presents to the ED with dark stool for 3 weeks. She occasionally notes bright red blood mixed with the stool. Review of systems is positive for decreased appetite, constipation, and a 10-lb weight loss over 2 months. She denies abdominal pain, nausea, vomiting, and fever, but feels increased weakness and fatigue. She also describes a raspy cough with white sputum production over the previous 2 weeks. Examination reveals a pale female with a supine BP of 115/60 mm Hg, HR of 90 beats per minute. Standing BP is 100/50 mm Hg, with a pulse of 105 beats per minute. Which of the following is the most likely diagnosis? a. Hemorrhoids b. Diverticulitis c. Mallory-Weiss tear d. Diverticulosis e. Adenocarcinoma

*E* The combination of lower GI bleed with weight loss and decreased appetite points toward carcinoma, most likely adenocarcinoma of the colon. The lack of esophageal, abdominal, or rectal pain makes the other choices unlikely, as does the lack of associated symptoms (nausea, vomiting, or fever). Anemia or rectal bleeding in an elderly person should be assumed to be malignancy until proven otherwise.

A 22-year-old man presents to the ED after being ejected from his vehicle following a high-speed motor vehicle collision. Upon arrival, his BP is 85/55 mm Hg and HR is 141 beats per minute. Two large-bore IVs are placed in the antecubital veins and lactated ringers solution is being administered. After 3 L of crystalloid fluid, the patient's BP is 83/57 mm Hg. Which of the following statements is most appropriate regarding management of a hypotensive trauma patient who fails to respond to initial volume resuscitation? a. It is important to wait for fully cross-matched blood prior to transfusion b. Whole blood should be used rather than packed red blood cells (RBCs) c. Blood transfusion should begin after 4 L of crystalloid infusion d. Type O blood that is Rh-negative should be transfused e. Type O blood that is Rh-positive should be transfused

*E* The decision to begin blood transfusion in a trauma patient is based on the initial response to crystalloid volume resuscitation. Blood products should be administered if vital signs transiently improve or remain unstable despite resuscitation with 2 to 3 L of crystalloid fluid. However, if there is obvious major blood loss and the patient is unstable, blood transfusion should be started concomitantly with crystalloid administration. The main purpose in transfusing blood is to restore the oxygen carrying capacity of the intravascular volume. Fully cross-matched blood is preferable (eg, type B, Rh-negative, antibody negative); however, this process may take more than 1 hour, which is inappropriate for the unstable trauma. patient. Type-specific blood (eg, type A, Rh negative, unknown antibody) can be provided by most blood banks within 30 minutes. This blood is compatible with ABO and Rh blood types, but may be incompatible with other antibodies. If type-specific blood is unavailable, type O packed cells are indicated for patients who are unstable. Men should be administered type O, Rh-positive blood. To reduce sensitization and future complications, type O, Rh-negative blood is reserved for women of childbearing age.

A 54-year-old man is brought to the ED by his wife for bizarre behavior. The wife complains that her husband has not been acting like his usual self over the last several days. She states that he has not had any change in sleep, appetite, or activity level. She also recalls that her husband complained of morning headaches for the last 2 months. The patient is otherwise in good health and does not take any medications. His BP is 135/87 mm Hg, HR is 76 beats per minute, temperature is 98.9°F, and RR is 14 breaths per minute. His examination is otherwise unremarkable. Which of the following is the most likely diagnosis? a. Migraine headache b. Tension headache c. Subarachnoid hemorrhage d. Pseudotumor cerebri e. Frontal lobe mass

*E* The most likely diagnosis in this patient is a space-occupying lesion in the frontal lobe of the brain. Brain tumors can present with morning headaches associated with nausea and vomiting. Neurologic examination is normal in most patients. Papilledema might provide an important clue of increased ICP and the presence of a brain mass. Frontal lobe tumors typically involve personality changes as seen in this patient.

A 42-year-old diabetic man presents to the ED with 3 days of rapidly worsening scrotal and perineal pain. His HR is 115 beats per minute, BP is 135/90 mm Hg, RR is 18 breaths per minute, and temperature is 102.9°F. Laboratory results are notable for a WBC of 18,000/μL. Physical examination demonstrates a necrotizing infection of the scrotum and perineal subcutaneous fascia. Which of the following is the most appropriate next step in management? a. 14-Day course of levofloxacin and early urology follow-up. b. Oral metronidazole and cephalexin with early urology follow-up. c. Pain medications, daily warm soaks, and follow-up with an urologist. d. Aggressive fluid resuscitation, bedside incision and drainage, and urology consult. e. Surgical debridement, broad-spectrum IV antibiotics, and hospital admission.

*E* The patient has Fournier gangrene, a polymicrobial necrotizing fasciitis of the perineal subcutaneous fascia and male genitalia. Treatment is aggressive fluid resuscitation, surgical debridement, broad-spectrum antibiotics, and possibly hyperbaric oxygen therapy. These patients require hospital admission. If not promptly treated, the overall mortality rate of these patients is approximately 20%.

A 23-year-old man presents to the ED complaining of finger pain. He states that while playing football, he went to catch a pass and the ball hit the tip of his finger and bent his finger backward. He thinks his finger is just "jammed." On examination, you notice that the distal phalanx is flexed and there is swelling and tenderness over the distal interphalangeal (DIP) joint, as seen below. In addition, he cannot extend his distal finger at the DIP joint. An x-ray does not reveal a fracture. Which of the following is the most appropriate way to manage this injury? a. Place a dorsal splint so that the proximal interphalangeal (PIP) and DIP joints are immobile, remove splint in 1 to 2 weeks. b. Place a dorsal splint so that the PIP and DIP joints are immobile, remove splint in 1 week. c. Buddy tape the finger. d. Place a dorsal splint to immobilize the DIP joint, remove splint in 1 to 2 weeks. e. Place a dorsal splint to immobilize the DIP joint, remove splint in 6 to 8 weeks.

*E* The patient has a mallet finger or a rupture of the extensor tendon that inserts into the base of the distal phalanx. This type of injury occurs often from a sudden forceful flexion of an extended finger when an object, such as a football, strikes the tip of the finger. This is the common mechanism among athletes. Clinically, patients present with pain and swelling over the DIP joint, which is held in flexion of up to 40 degrees because of loss of the extensor mechanism. The most important aspect in managing these injuries is to keep the DIP joint in continuous extension until healing occurs. Therefore, a splint should be applied so that only the DIP is immobilized in extension for 6 to 8 weeks. The PIP and MCP joints should be mobile. Any disruption of the immobile DIP joint can result in improper healing.

A 65-year-old man presents to the ED with a headache, drowsiness, and confusion. He has a history of long-standing hypertension. His BP is 230/120 mm Hg, pulse 87 beats per minute, RR 18 breaths per minutes, and oxygen saturation 97% on room air. On examination, you note papilledema. A head CT scan is performed and there is no evidence of ischemia or hemorrhage. Which of the following is the most appropriate method to lower his BP? a. Administer propofol for rapid reduction in BP. b. Administer mannitol for rapid reduction in BP and intracranial pressure (ICP). c. Administer a high-dose diuretic to reduce preload. d. Administer labetalol until his BP is 140/80 mm Hg. e. Administer labetalol until his BP is 180/100 mm Hg.

*E* The patient has hypertensive encephalopathy, which is defined by a rapid rise in BP that is accompanied by neurologic changes. Patients typically present with a systolic BP > 220 mm Hg and diastolic BP > 110 mm Hg. Neurologic findings include severe headache, vomiting, drowsiness, confusion, seizure, blindness, focal neurologic deficits, or coma. Hypertensive emergency is a medical emergency. The goal of therapy is to stop and reverse the progression of end-organ dysfunction while maintaining organ perfusion and avoiding complications. Reduction in BP should be done rapidly but carefully. It is important to avoid dropping the pressure too low as this may lead to cerebral ischemia. The immediate goal is to reduce the mean arterial BP by 20% to 30% of pretreatment levels over the first hour of therapy. This can be accomplished by labetalol, a β1-, β2-, and α1-receptor blocker. Another useful medication is nitroprusside, which is a better choice if the patient's BP is being monitored through an intra-arterial line. Nitroprusside can cause a reflex tachycardia.

A 43-year-old man presents to the ED after falling approximately 6 ft from the roof of his garage. The patient states that he landed on his feet but then fell to the ground. You assess his ABC (airway, breathing, and circulation), which are all normal. Vital signs are stable. Your secondary survey reveals a swollen and tender right heel. A radiograph reveals a fractured calcaneus. About 6 hours after the initial fall, the patient starts complaining of a constant burning in his right foot. You examine the foot and elicit pain with passive movement. There is decreased two-point discrimination. His dorsalis pedis pulse is 2+. Which of the following is the most appropriate next step in management? a. Place ice on the foot and administer analgesia. b. Order another radiograph to look for an occult fracture. c. Elevate the leg and place an elastic bandage around the foot. d. Order a duplex ultrasound for suspicion of a deep vein thrombosis (DVT). e. Measure the intracompartmental pressure of the foot.

*E* The patient is showing signs and symptoms of compartment syndrome. The syndrome occurs as a result of an increase in pressure within a confined osseofascial space that impedes neurovascular function. The end result is necrosis and damage to tissues. It can occur after crush injuries, circumferential burns, hemorrhage, edema, or any process that increases compartment pressure. Clinically, the patient complains of pain that is out of proportion to the injury. Physical examination may reveal swelling, sensory deficits, and pain with passive motion. The presence of a pulse does not rule out compartment syndrome. Late findings include pallor of the skin, diminished or absent pulses, and a cool extremity. The only way to diagnose compartment syndrome is to measure intracompartmental pressure with a Stryker device. A pressure greater than 30 mm Hg is considered diagnostic and requires a fasciotomy to avoid tissue damage.

A 29-year-old woman presents to the ED complaining of a generalized headache over the last 2 months. She has seen many doctors for it but has yet to get a correct diagnosis. She describes the headache as moderate in intensity and worse with eye movement. Occasionally, it awakes her from sleep and is worse when tying her shoes. She is scared because her vision gets blurry for a few minutes everyday. Her only medication is acetaminophen and an oral contraceptive. Her BP is 140/75 mm Hg, HR is 75 beats per minute, temperature is 98.9°F, and RR is 16 breaths per minute. On physical examination you appreciate papilledema. Which of the following is the most appropriate next step in management? a. Consult neurosurgery. b. Administer 2 g of ceftriaxone then perform an LP rule out meningitis. c. Order a magnetic resonance imaging (MRI) to look for a carotid artery dissection. d. Diagnose a migraine headache and prescribe her a triptan. e. Perform a CT scan and if negative perform an LP specifically to measure the opening pressure.

*E* The patient most likely has IIH (pseudotumor cerebri), a neurologic disease seen primarily in young obese women of childbearing age. Clinically, patients complain of a generalized headache of gradual onset and moderate severity. It may worsen with eye movements or with the Valsalva maneuver. Visual complaints are common and may occur several times a day and can become permanent in 10% of patients. Patients typically have papilledema and visual field defects on physical examination. Diagnosis is made by a normal neuroimaging scan (eg, head CT scan) and an elevated intracerebral pressure (> 200 mm H2O) measured by the opening pressure from an LP. This is also therapeutic.

A 36-year-old woman presents to the ED with sudden onset of leftsided chest pain and mild shortness of breath that began the night before. She was able to fall asleep without difficulty but woke up in the morning with persistent pain that is worsened upon taking a deep breath. She walked up the stairs at home and became very short of breath, which made her come to the ED. Two weeks ago, she took a 7-hour flight from Europe and since then has left-sided calf pain and swelling. What is the most common ECG finding for this patient's presentation? a. S1Q3T3 pattern b. Atrial fibrillation c. Right-axis deviation d. Right-atrial enlargement e. Tachycardia or nonspecific ST-T-wave changes

*E* The patient most likely has a pulmonary thromboembolism (PE) that embolized from a thrombus in her left calf. The diagnosis of PE is usually made with a CT angiogram, echocardiogram, or a ventilation-perfusion scan. The most common ECG abnormalities in the setting of PE are tachycardia and nonspecific ST-T- wave abnormalities.

A 68-year-old man presents to the ED 4 hours after an upper endoscopy was performed for 5 months of progressive dysphagia. During the procedure, a 1-cm ulcerated lesion was found and biopsied. Now, the patient complains of severe neck and chest pain. His vitals are as follows: BP 135/80 mm Hg, HR 123 beats per minute, RR 26 breaths per minute, and temperature 101°F. On physical examination, he appears diaphoretic and in moderate distress with crepitus in the neck and a crunching sound over the heart. You obtain an electrocardiogram (ECG), which is notable for sinus tachycardia. After obtaining a surgical consult, which of the following is the next best step in management? a. Perform an immediate bronchoscopy. b. Give aspirin 325 mg and obtain a cardiology consult for possible cardiac catheterization. c. Repeat the endoscopy to evaluate the biopsy site. d. Perform an immediate thoracotomy. e. Order an immediate esophagram with water-soluble agent.

*E* The patient most likely has an esophageal perforation, a serious, life-threatening complication of endoscopy that must be identified and treated quickly. Although sometimes reported as a result of forceful vomiting (eg, Boerhaave syndrome), the most common cause is iatrogenic. These usually occur as a complication of GI procedures, including upper endoscopy, dilation, sclerotherapy, and even NG tube placement or endotracheal intubation. The signs and symptoms may include chest pain near the rupture site, fever, respiratory distress, hoarseness, or dysphagia. Most patients have mediastinal or cervical emphysema, which may be noted by palpation or by a crunching sound heard during auscultation (ie, Hamman sign). An immediate esophagram with a water-soluble agent (eg, Gastrografin) is indicated

A 33-year-old carpenter was working on a construction project to build a new house. While he was using a high-pressure paint gun, he inadvertently injected his left index finger. On arrival to the ED, he complains of intense hand pain. On examination, you note a 2-mm wound over the second proximal phalange. He has full range of motion and brisk capillary refill. Radiographs of the finger show soft tissue swelling, a small amount of subcutaneous air, but no fracture. His tetanus is up to date. Which of the following is the most appropriate disposition for this patient? a. Place the hand in a radial gutter splint and have the patient follow up with an orthopedic surgeon in 1 week. b. Discharge home with pain medication and have the patient return for repeat radiographs in 1 week. c. Order a CT scan of the finger to confirm that there is no occult fracture before discharging the patient home. d. Place the hand in a radial gutter splint, prescribe a 10-day course of antibiotics, and have the patient follow up with an orthopedic surgeon in 1 week. e. Place the hand in a radial gutter splint, administer broad-spectrum antibiotics, and admit the orthopedic service for operative debridement.

*E* The patient sustained a high-pressure injection injury of his finger—this is a surgical emergency. These injuries may involve extensive tissue loss and are associated with a high infection rate. Most of these injuries involve grease, paint, or other industrial toxins. Paint generates a large, early inflammatory response resulting in a high percentage of amputations. Within several hours after the digit has been injected, the extremity becomes painful and swollen. Initially, there may be anesthesia and even vascular insufficiency of the extremity. In the late stages, marked breakdown of the skin occurs resulting in ulcers and draining sinuses. If the material injected into the extremity is radiopaque, it is possible to determine its degree of spread. Management involves splinting and elevating the extremity, administration of antibiotics, tetanus prophylaxis updated as indicated, analgesia, and immediate orthopedic consultation.

A 45-year-old man is on his way to work and loses his footing while walking up a flight of steps. He feels excruciating pain at the back of his ankle and cannot ambulate. He arrives to the ED and states that he felt a snap in his leg during the injury. He has a past medical history of hypertension and hypercholesterolemia. He spends most of his free time in front of the television watching horror movies. On examination, you note swelling of the distal calf. Which of the following is likely to be positive in this individual? a. Homan sign b. Lachman test c. McMurray test d. Ballotable patella e. Thompson test

*E* The patient's diagnosis is an Achilles tendon rupture. The individual gives a history of a sudden excruciating pain and having heard or felt a pop or snap. This entity is most common in sedentary, middle-aged men or in athletes. The diagnosis can be made with the Thompson test. The patient is placed in the prone position. With normal function, squeezing the calf produces plantar flexion of the foot. With a complete tear of the Achilles tendon, plantar flexion will not occur. If doubt remains, magnetic resonance imaging (MRI) or ultrasound can be used to confirm the diagnosis.

You are notified that emergency medical service (EMS) is bringing in a patient who collapsed 5 minutes ago in his house and was intubated at the scene by paramedics. On arrival to the ED, you confirm ET placement and continue cardiopulmonary resuscitation (CPR). You connect the patient to the cardiac monitor and see the rhythm below. Which of the following is the most appropriate next step in management? a.Perform synchronized cardioversion at 100 J. b. Immediately defibrillate at 200 J. c. Confirm the rhythm in two leads, begin CPR, then defibrillate at 200 J. d. Confirm the rhythm in two leads, begin CPR, then administer amiodarone. e. Confirm the rhythm in two leads, begin CPR, then administer epinephrine and atropine.

*E* The rhythm is asystole. This rhythm represents complete cessation of myocardial electrical activity. Although asystole may occur early in cardiac arrest because of progressive bradycardia, it generally represents the end-stage rhythm after prolonged cardiac arrest secondary to VF or PEA. Because the potential exists for an organized rhythm or VF to appear as asystole in a single lead, asystole should always be confirmed in at least two limb leads. It may be difficult to distinguish between extremely fine VF and asystole. Treatment includes CPR, intubation, IV access, and the administration of epinephrine or vasopressin and atropine.

A 75-year old man complaining of chest pain is brought in to the ED by paramedics. He is barely able to speak to you because he is short of breath. The nurse immediately attaches him to the monitor, starts an IV, and gives him oxygen. His temperature is 98.9°F, BP is 70/40 mm Hg, HR is 140 beats per minute, RR is 28 breaths per minute, and oxygen saturation is 95% on room air. On examination, he is in mild distress. His heart is irregular and tachycardic. His lungs are clear to auscultation, with rales at the bases, bilaterally. An electrocardiogram (ECG) is shown below. What is your first-line treatment for this patient? a. Heparin drip b. Diltiazem 10 mg IV push c. Metoprolol 5 mg IV push d. Digoxin 0.5 mg IV e. Synchronized cardioversion at 100 J

*E* This ECG shows atrial fibrillation with rapid ventricular response (RVR). Normally, one area of the atria depolarizes and causes uniform contraction of the atria. In atrial fibrillation, multiple areas of the atria continuously depolarize and contract, leading to multiple atrial impulses and an irregular ventricular response. Atrial fibrillation reduces the effectiveness of atrial contractions and may lead to or worsen heart failure in patients with left ventricular failure. Treatment of atrial fibrillation is dependent on whether or not the patient is stable or not. This patient is clinically unstable; the atrial fibrillation with RVR has pushed him into heart failure and he is hypotensive and tachypneic. Unstable patients like this should undergo synchronized cardioversion. Synchronized cardioversion is performed at 100 J and then at 200 J if the first attempt fails.

An 82-year-old man presents to the ED feeling weak and dizzy. He has a past medical history of hypertension and diabetes and both are well controlled on hydrochlorothiazide, benazepril, atenolol, and metformin. On review of systems, he denies chest pain, gastrointestinal (GI) bleeding, and syncope, but states that he feels short of breath. His temperature is 98.6°F orally, BP is 86/60 mm Hg, HR is 44 beats per minute, RR is 18 breaths per minute, oxygen saturation is 98% on room air, and glucose is 116 mg/dL. He is immediately connected to the cardiac monitor. Which of the following choices best describes the ECG that shows p waves marching through? a. Normal sinus rhythm b. First-degree AV block c. Second-degree Mobitz I (Wenckebach) AV block d. Second-degree Mobitz II AV block e. Third-degree AV block

*E* This ECG shows third degree, or complete AV block. Note that there is no relationship between the P waves and QRS complexes. The P waves occur regularly, but since there is no AV conduction, the ventricles do not respond to the P waves. An escape pacemaker at a rate slower than the atrial rate drives the ventricles, producing regular QRS complexes independent of the P waves

A 26-year-old man with a long history of epilepsy is brought to the ED for a recent seizure. While in the ED, he is rhythmically moving his right leg and is unresponsive. Which of the following best describes this seizure pattern? a. Petit mal seizure b. Generalized tonic-clonic seizure c. Partial seizure with secondary generalization d. Simple partial seizure e. Complex partial seizure

*E* This is a complex partial seizure, also known as temporal seizure; although it does not necessarily originate in the temporal lobe. It is characterized by focal electrical discharges (partial seizure), such as clonic leg activity and alteration of consciousness. --petit mal = abscence seizure

A car pulls up to your ED and drops off a 19-year-old man who was shot in the chest. The man tells you his name and complains of right-sided chest pain and difficulty breathing. On primary survey, his airway is patent and his oropharynx has no blood or displaced teeth. He is breathing at 32 beats per minute with retractions and an oxygen saturation of 88% on 15 L of oxygen. There is a bullet wound to his right mid-chest with another wound in his back. His trachea is deviated to the left. On auscultation, he has diminished breath sounds on the right side. Which of the following is the most appropriate next step in management? a. Stat portable chest x-ray b. Intubation c. Perform ED thoracotomy d. Call the surgical service e. Needle decompression

*E* This patient has a tension pneumothorax. Secondary to the gunshot, air has entered the pleural space, secondary to the gunshot, and caused the right lung to collapse. This air cannot escape and pressure continues to increase, pushing the right lung into the mediastinum, causing the trachea to shift to the left. If this process is not corrected, venous return and cardiac output can be compromised and the patient will die. Classic symptoms of tension pneumothorax include dyspnea, tachypnea, tracheal deviation to the uninjured side, absent breath sounds on the injured side, and hypotension. Treatment of a tension pneumothorax is immediate needle decompression using a large 14- or 16-gauge IV catheter inserted into the pleural space (ie, second intercostals space in the midclavicular line). Air should come out of the catheter and the patient's clinical condition should improve. A tube thoracostomy should be performed after the needle decompression.

A 48-year-old man with a medical history of cirrhosis caused by hepatitis C has been vomiting bright red blood for 1 day. On arrival to the ED, the patient is confused and unable to provide more information. His family states that he has been vomiting large amounts of bright red blood every 4 hours and has no prior history of GI bleeding. The nurses hook him up to the monitor and start two large-bore IV lines. His BP is 75/43 mm Hg, HR is 130 beats per minute, RR is 24 breaths per minute, and oxygen saturation is 98% on room air. His abdomen is soft with no masses. His rectal examination reveals bright red blood. Which type of fluid is most critical in his resuscitation? a. 7% Sodium chloride b. 0.9% Sodium chloride c. Type and cross-matched blood d. Type-specific blood e. Type O, Rh-positive blood

*E* This patient has significant hypotension and tachycardia with mental status changes and needs type O, Rh-positive blood now. His hypovolemic shock is most likely caused by a brisk upper GI bleeding, secondary to variceal bleeding. While his blood is being processed in the laboratory, including type and cross, complete blood count, chemistry panel, liver function tests, and coagulation profile, a nasogastric tube should be placed and octreotide administered. A gastroenterologist and surgeon should also be consulted. Type O, Rh-negative blood is reserved for women in their childbearing years to prevent potential Rh sensitization.

An 18-year-old college student with no past medical history presents to the ED with a diffuse rash. She also describes having a headache, fever, and arthralgias for 3 days. On examination, her temperature is 101.2°F, BP is 120/63 mm Hg, HR is 110 beats per minute, RR is 24 breaths per minute, and oxygen saturation is 98% on room air. The patient is alert and oriented to person, place, and time. She has nuchal rigidity and photophobia. Her gums are oozing blood. Her abdomen is soft and nontender and her skin has a diffuse, petechial rash. You are concerned about meningococcemia and immediately start ceftriaxone and vancomycin. Her laboratory results reveal a white blood cell (WBC) count of 13,400/μL, hematocrit 36%, platelets 80/μL, PTT 60 seconds, international normalized ratio (INR) 1.9, and fibrinogen 250 g/L. Which of the following is the most appropriate next step in management? a. IV heparin b. Transfuse cryoprecipitate c. Transfuse packed red blood cells (RBC) d. Transfuse platelets e. Transfuse fresh frozen plasma (FFP)

*E* This patient has signs of disseminated intravascular coagulation (DIC) secondary to meningococcemia. She should be treated with FFP for elevated coagulation times (INR and PTT) with signs of active bleeding (oozing gums). FFP will provide lost clotting factors and help to control the bleeding.

A 58-year-old man presents to the ED with progressive dyspnea over the course of 1 week. Upon arrival, he is able to speak in full sentences and states that he stopped taking all of his medications recently. Initial vitals include a HR of 92 beats per minute, a BP of 180/100 mm Hg, and an RR of 16 breaths per minute with an oxygen saturation of 94% on room air. Upon physical examination, the patient has bibasilar crackles, jugular venous distention, and pedal edema. Which of the following medication regimens was the patient most likely on? a. Loop diuretic only b. Aspirin only c. Loop diuretic and β-blocker d. Calcium channel blocker e. Loop diuretic, β-blocker, and angiotensin-converting enzyme (ACE) inhibitor

*E* This patient is showing signs and symptoms of CHF, which is classified as either right- or left-sided. Right-sided heart failure manifests as jugular venous distention, ascites, and peripheral edema. Left-sided heart failure manifests as pulmonary edema or shock. This patient has both as left-sided failure often leads to right-sided failure. Outpatient management for CHF includes a β-blocker to decrease cardiac stress and improve contractility, a loop diuretic to aid in diuresis of excess fluid; and an ACE inhibitor for both BP management and renal protective effects.

A 35-year-old man presents to the ED with right hand swelling, pain, and erythema that began 3 days ago. He denies any trauma, sick contacts, insect bites, or recent travel. The patient's vitals are significant for an oral temperature of 101°F. Upon physical examination, you note an area of erythema surrounding multiple punctate lacerations over the right third and fourth metacarpophalangeal (MCP) joints with localized tenderness. The patient is neurovascularly intact with limited flexion caused by the swelling and pain. Given the above presentation, what is the most appropriate disposition for this patient? a. Suture and close follow-up with a hand surgeon b. Suture and prescription for oral antibiotics c. Wound irrigation and prescription for oral antibiotics d. Wound irrigation and tetanus prophylaxis e. Admission for intravenous (IV) antibiotics

*E* This patient most likely sustained a closed-fist injury, which have high infection rates and evidence of poor wound healing. Wounds sustained by punches to the jaw and human bites, also known as "fight bites," are classically over the metacarpal joints. Penetration deep into the joint space and infection is common given the positioning of the hand during the injury, human oral flora, and delay in seeking treatment. Infected wounds are polymicrobial and specifically include Eikenella corrodens, a facultative anaerobic gram-negative rod harbored in human dental plaque. It acts synergistically with aerobic organisms to increase the morbidity of these injuries. The joint spaces must be examined under full range of motion to detect any tendon lacerations or presence of foreign bodies. Hand radiographs should also be obtained to examine for any bony involvement. IV antibiotics and admission is the appropriate disposition. The antibiotics of choice are penicillin and second-generation cephalosporins with broader coverage in the immunocompromised. The wounds should be left open with a sterile dressing, splinted in the position of function (hand-holding-glass position) and elevated. Human bites have resulted in the transmission of hepatitis B, hepatitis C, syphilis, and herpes. Although HIV is present in human saliva, it is in relatively small amounts and considered a low risk for transmission. Appropriate antivirals and testing should be considered in these patients.

A 21-year-old college student is brought to the ED by her roommate who states that the patient has been very sleepy today. She has a history of diabetes and has not refilled her medication in over a week. Her BP is 95/61 mm Hg, HR is 132 beats per minute, temperature is 99.7°F, and RR is 20 breaths per minute. Her fingerstick glucose is 530 mg/dL. Which of the following choices most closely matches what you would expect to find on her arterial blood gas with electrolytes and urinalysis? a. pH 7.38, anion gap 5, normal urinalysis b. pH 7.57, anion gap 21, presence of glucose and leukocytes in urine c. pH 7.47, anion gap 12, presence of glucose and ketones in urine d. pH 7.26, anion gap 12, presence of glucose and ketones in urine e. pH 7.26, anion gap 21, presence of glucose and ketones in urine

*E* This patient presents with an anion gap metabolic acidosis, glucosuria, and ketonuria, which is consistent with diabetic ketoacidosis (DKA). DKA is an acute, life-threatening disorder occurring in patients with insulin insufficiency. It results in hyperglycemia, ketosis, and osmotic diuresis and clinically presents with gastrointestinal (GI) distress, polyuria, fatigue, dehydration, mental confusion, lethargy, or coma. When the diagnosis of DKA is clinically suspected and hyperglycemia is confirmed by elevated fingerstick glucose, the results of a blood gas and urinalysis confirm the diagnosis. In DKA, the liver metabolizes free fatty acids into ketone bodies for alternative fuel in the setting of cellular glucose underutilization. The result is ketonuria and anion gap metabolic acidosis (pH < 7.4 and HCO3 < 24). Glucosuria, the result of hyperglycemiarelated osmotic diuresis, is another manifestation of DKA. The anion gap is calculated by subtracting Cl− and HCO3 − from Na. A normal anion gap is 8 to 12 mEq/L. An elevated gap is a result of an increased concentration of unmeasured anions. In DKA, the elevated anion gap is caused by the production of ketones.

A 46-year-old woman is brought to the ED by her husband for 1 day of worsening confusion. The patient has a history of systemic lupus erythematosus (SLE) and takes chronic oral steroids. She has not been feeling well for the last few days. Her BP is 167/92 mm Hg, HR is 95 beats per minute, temperature is 100.3°F, and RR is 16 breaths per minute. On examination the patient is oriented to name and has diffuse petechiae on her torso and extremities. Laboratory results reveal hematocrit 23%, platelets 17,000/μL, BUN 38 mg/dL, creatinine 1.9 mg/dL. Which of the following is the most likely diagnosis? a. Henoch-Schönlein purpura b. Disseminated intravascular coagulopathy c. Von Willebrand disease d. Idiopathic thrombocytopenic purpura e. Thrombotic thrombocytopenic purpura (TTP)

*E* This patient presents with four of the five symptoms classically associated with TTP. These include thrombocytopenia, hemolytic anemia, neurologic deficits, renal impairment, and fever. TTP develops with fibrin-strand deposition in small vessels that attract platelets leading to platelet thrombi and thrombocytopenia. Passing RBCs get sheared in occluded vessels resulting in microangiopathic hemolytic anemia. Renal and neurologic impairment occur as a result of the lodging of thrombi in respective circulations. Plasmapheresis decreases TTP mortality from 90% to 10%. Adjunct therapies include fresh frozen plasma infusion and steroids. It is important to realize that although patients may be severely thrombocytopenic, platelet infusion is contraindicated since it exacerbates the underlying cycle of thrombogenesis. Risk factors for TTP include pregnancy, autoimmune disorders, drugs, infection, and malignancy. Hemolytic-uremic syndrome (HUS) is a closely related entity usually seen in children. There is pronounced renal dysfunction without altered mentation.

A 67-year-old woman presents to the ED complaining of a 2-day history of general malaise, subjective fevers, chills, diffuse headache, and rightsided jaw pain. She also notes diminished vision in her right eye. Her symptoms are minimally relieved with acetaminophen. She denies any sick contacts. The patient's vitals include an oral temperature of 100.6°F, HR is 95 beats per minute, BP is 130/75 mm Hg, and RR is 16 breaths per minute with oxygen saturation of 99% on room air. She is tender on the right side of her scalp. You initiate empirical treatment. Which of the following tests will confirm your diagnosis? a. Influenza A/B assay b. Rapid strep test c. Erythrocyte sedimentation rate (ESR) d. Complete blood count (CBC) e. Temporal artery biopsy

*E* This patient's clinical picture is consistent with temporal arteritis (TA). Patients are usually middle aged females (age > 50) who present with malaise, fevers, and headache. A complete physical examination would have revealed temporal artery tenderness to palpation. This patient also complains of symptoms consistent with polymyalgia rheumatica, a general achiness that may become confused with influenza. Temporal or giant-cell arteritis is a granulomatous inflammation that involves the large- and medium-sized arteries of the body, commonly the carotid artery and its branches. Symptoms are produced as a result of ischemia to the organs fed by the branches of the artery. Visual loss in one eye, transient diplopia and jaw claudication are common symptoms when the branches of the internal and external carotid are affected. A temporal artery biopsy is the diagnostic test of choice and will confirm the diagnosis. However, it is important to note that TA is segmental in nature and false-negatives do occur. Treatment up until the time of biopsy should include high-dose glucocorticoids, namely prednisone or methylprednisolone. This does not alter biopsy results and may prevent progression of the disease. Hospitalization is warranted in patients with severe debilitation or impending visual loss and may require high-dose steroids.

You are notified by the EMS dispatcher that there is a multiple-car collision on the local highway with many injuries. He states that there are two people dead at the scene, one person is critically injured and hypotensive, and three people have significant injuries, but with stable vital signs. Which of the following is the leading cause of death and disability in trauma victims? a. Abdominal injury b. Thoracic injury c. Back injury d. Cervical injury e. Head injury

*E* Trauma is the leading cause of death between the ages of 1 and 44 years. Many of these injuries are treatable mainly because the patients are young and otherwise healthy. The primary role of the emergency physician is to assess, resuscitate, and stabilize the trauma patient by priority. There are three peak times for trauma deaths. The first, classified as immediate death, is the period with the greatest number of fatalities. This occurs within seconds to minutes of the injury and these patients generally die at the scene. The cause is most commonly because of massive head injury, followed by high cervical spine injury with spinal cord disruption, cardiac and great vessel rupture, and airway obstruction. The second peak period, classified as early death, occurs within minutes to a few hours of injury. This is the period called the "golden hour" where intervention is critical and significantly reduces the morbidity and mortality rate in these patients. Death in these patients is generally secondary to subdural and epidural hematomas. Other causes of death in this group include ruptured spleen, lacerated liver, hypovolemic shock, fracture of pelvis or multiple long bones, hemopneumothorax, tension pneumothorax, cardiac tamponade, and aortic dissection or rupture. The third peak period, classified as delayed death, occurs days to weeks following the initial injury. Death in these patients is usually a result of multisystem organ failure and sepsis.

A 27-year-old pregnant woman, in her third trimester, is brought to the ED after being involved in a low-speed motor vehicle collision. The patient was wearing a seat belt in the back seat of a car that was struck in the front by another car. Her BP is 120/70 mm Hg and HR is 107 beats per minute. Her airway is patent, breath sounds equal bilaterally, and skin is warm with 2+ pulses. FAST examination is negative for free fluid. Evaluation of the fetus reveals appropriate fetal HR and fetal movement. Repeat maternal BP is 120/75 mm Hg. Which of the following is the most appropriate next step in management? a. Perform an immediate cesarean section in the OR. b. Perform an immediate cesarean section in the ED. c. CT scan of the abdomen and pelvis to rule out occult injury. d. Discharge the patient if laboratory testing is normal. e. Monitor the patient and fetus for a minimum of 4 hours.

*E* Trauma occurs in up to 7% of all pregnancies and is the leading cause of maternal death. It is important to focus the primary examination on the patient and evaluate the fetus in the secondary examination. The ABCs are followed in usual fashion. Once the patient is deemed stable, the fetus should be evaluated. Fetal evaluation focuses on the fetal HR and fetal movement. Minor trauma to the patient does not rule out injury to the fetus. Therefore, it is important to monitor the fetus. Cardiotocographic observation of the viable fetus is recommended for a minimum of 4 hours to detect any intrauterine pathology. The minimum should be extended to 24 hours if, at any time during the first 4 hours, there are more than three uterine contractions per hour, persistent uterine tenderness, a non-reassuring fetal monitor strip, vaginal bleeding, rupture of the membranes, or any serious maternal injury is present.

A 47-year-old man is brought into the ED after falling 20 ft from a ladder. His HR is 110 beats per minute, BP is 110/80 mm Hg, RR is 20 breaths per minute, and oxygen saturation is 100% on face mask. He is able to answer your questions without difficulty. His chest is clear with bilateral breath sounds, abdomen is nontender, pelvis is stable, and the FAST examination is negative You note a large scrotal hematoma and blood at the urethral meatus. Which of the following is the most appropriate next step in management? a. Scrotal ultrasound b. Kidney-ureter-bladder (KUB) radiograph c. IV pyelogram d. Retrograde cystogram e. Retrograde urethrogram

*E* Urethral injuries make up approximately 10% of genitourinary trauma. Anterior urethral injuries are most often attributed to falls with straddle injuries or a blunt force to the perineum. Approximately 95% of posterior urethral injuries are secondary to pelvic fractures. Signs and symptoms of urethral injury include perineal pain, inability to void, gross hematuria, blood at the urethral meatus, perineal or scrotal swelling or ecchymosis, and an absent, high-riding, or boggy prostate. A retrograde urethrogram is the study of choice when there is suspicion of a urethral injury. This procedure is performed by inserting an 8F urinary catheter 2 cm into the meatus and inflating the catheter balloon with 2 cc saline to create a seal. Then, 30 cc of radiopaque contrast is administered and a radiograph is obtained looking for extravasation of contrast from the urethra.

A 24-year-old woman presents to the ED after being sexually assaulted. She is a college student with no past medical history. Her BP is 130/75 mm Hg, HR is 91 beats per minute, temperature is 98.6°F, and RR is 16 breaths per minute. On physical examination you observe vaginal trauma and scattered bruising and abrasions. Which of the following medications should be offered to the patient in this scenario? a. Ceftriaxone, azithromycin, metronidazole, antiretrovirals, emergency contraception b. Ceftriaxone, tetanus, metronidazole, antiretrovirals, emergency contraception c. Ceftriaxone, azithromycin, tetanus, metronidazole, emergency contraception d. Ceftriaxone, azithromycin, tetanus, antiretrovirals, emergency contraception e. Ceftriaxone, azithromycin, tetanus, metronidazole, antiretrovirals, emergency contraception

*E* When a sexual assault patient is evaluated in the ED, the EP not only has the standard responsibility to care for the patient's immediate physical and psychological health, but he/she must also consider how the encounter may affect the patient's life considerably once discharged from the ED. Once life-threatening injuries are addressed, EPs are responsible for collecting physical evidence necessary for prosecuting the assailant by conducting a sexual assault or "rape kit" with the patient's consent. Most medications provided to sexually assaulted patients are provided as prophylaxis against sexually transmitted infections (STIs), pregnancy, and tetanus.

A 61-year-old man with a history of depression and hypertension is brought to the ED by EMS for altered mental status. The patient's wife states that he stopped taking his fluoxetine 1 month ago and now only takes metoprolol for his hypertension. The patient's BP is 75/40 mm Hg, HR is 39 beats per minute, RR is 14 breaths per minute, oxygen saturation is 99% on 100% oxygen, and fingerstick glucose is 81 mg/dL. The patient is awake and moaning, responding only to deep stimuli. You suspect an overdose of metoprolol. You endotracheally intubate the patient for airway control. Which of the following is the most appropriate next step in management? a. Syrup of ipecac, normal saline bolus, epinephrine b. Cardioversion with 200 J then administer epinephrine c. Cardioversion with 200 J then administer atropine d. Normal saline bolus, atropine, epinephrine, and activated charcoal e. Normal saline bolus, atropine, glucagon, and activated charcoal

*E* β-Adrenergic receptor blockers (eg, Metoprolol) are commonly prescribed medications for hypertension. β-Adrenergic antagonist overdose is often benign, with about 33% of patients remaining asymptomatic. This is partially explained by the fact that β-adrenergic antagonism is often well tolerated in healthy persons who do not rely on sympathetic stimulation to maintain cardiac output. Conversely, those with cardiac abnormalities may rely on sympathetic stimulation to maintain HR or cardiac output. The hallmark of β-adrenergic receptor blocker toxicity is bradycardia with hypotension. Patients may also exhibit conduction and rhythm abnormalities. Onset of toxicity usually occurs within 4 hours of ingestion. If a patient remains asymptomatic after 4 hours there is a low risk for subsequent morbidity unless a delayed-release preparation is involved.

A 27-year-old man who is otherwise healthy presents to the ED with a laceration on his thumb that he sustained while cutting a bagel. You irrigate and repair the wound and are about to discharge the patient when he asks you if he can receive an ECG. It is not busy in the ED so you perform the ECG which shows prolonged PR interval Which of the following is the most appropriate next step in management? a. Admit the patient for placement of a pacemaker. b. Admit the patient for a 24-hour observation period. c. Administer aspirin and send cardiac biomarkers. d. Repeat the ECG because of incorrect lead placement. e. Discharge the patient home.

*E*. The patient's ECG shows a sinus rhythm at a rate of 70 with first-degree heart block. First-degree heart block is defined as prolonged conduction of atrial impulses without the loss of any impulse. On an ECG this translates to a PR interval greater than 200 ms with a narrow QRS complex (less than 120 msec). First-degree heart block is often a normal variant without clinical significance, occurring in 1% to 2% of healthy young adults. This variant requires no specific treatment

A 41-year-old man is brought into the ED by paramedics in cardiopulmonary arrest. A friend states that the patient is a long-time user of IV heroin. You look at the monitor and see that the patient has pulseless electrical activity (PEA). Cardiopulmonary resuscitation is being performed and the patient is intubated. You decide to administer epinephrine to the patient, but realize that he does not have IV access. Which of the following drugs is ineffective when administered through an endotracheal (ET) tube? a. Atropine b. Naloxone c. Lidocaine d. Epinephrine e. Sodium bicarbonate

*E*Endotracheal administration of drugs is indicated whenever there is a need for emergent pharmacologic intervention in the absence of other access routes, such as IV orintraosseous. There are a limited number of emergency medications that can be administered safely and effectively via the endotracheal route. These include naloxone, atropine, versed, epinephrine, and lidocaine. This is remembered by the mnemonic NAVEL. Specific medications shown to be unsafe include sodium bicarbonate, isoproterenol, and bretylium. The endotracheal dosage of a medication should be at least equivalent to the IV route, and is usually 2 to 2.5 times the IV dose is administered. The patient in the scenario should receive epinephrine via the ET while IV access is established.

A 55-year-old man presents to the ED with fever, drooling, trismus, and a swollen neck. He reports a foul taste in his mouth since a tooth extraction 2 days ago. On physical examination, the patient appears anxious. He has bilateral submandibular swelling and elevation and protrusion of the tongue. He appears "bull-necked" with tense and markedly tender edema and brawny induration of the upper neck, and he is tender over the lower second and third molars. There is no cervical lymphadenopathy. His vital signs are: HR 105 beats per minute, BP 140/85 mm Hg, RR 26 breaths per minute, and temperature 102°F. Which of the following is the most appropriate next step in management? a. Administer a dose of IV antibiotics and obtain a soft-tissue radiograph of the neck. b. Administer a dose of IV antibiotics then perform an incision and drainage at the bedside. c. Begin steroids to decrease inflammation and obtain an ear, nose, and throat (ENT) consult. d. Discharge the patient with oral antibiotics and ENT follow-up. e. Secure his airway, start IV antibiotics, and obtain an ENT consult.

*E*Ludwig Angina (LA) is a potentially life-threatening cellulitis of the connective tissue of the floor of the mouth and neck that begins in the submandibular space. An infected or recently extracted tooth is present in most cases. Typically, it is a polymicrobial infection involving aerobic and anaerobic bacteria of the mouth. The most commonly isolated organisms include streptococci, staphylococci, and bacteroides species. The most common physical findings in LA are bilateral submandibular swelling and tongue protrusion or elevation. A tense edema and brawny induration of the neck above the hyoid may be present and is described as a "bull neck." Marked tendernes to palpation of the neck and subcutaneous emphysema may be noted. Trismus and fever are often present, but usually no palpable fluctuance or cervical lymphadenopathy. The involved teeth may be tender to palpation. Patients with LA should never be left alone as airway impairment can suddenly occur. Signs of impending airway compromise include stridor, tachypnea, dyspnea, drooling, and agitation. The upper airway may be distorted making endotracheal intubation difficult or impossible. Cricothyrotomy may also be difficult and increases the risk of spreading infection into the mediastinum. Fiberoptic nasotracheal intubation is preferred.

A tall, thin 18-year-old man presents to the ED with acute onset of dyspnea while at rest. The patient reports sitting at his desk when he felt a sharp pain on the right side of his chest that worsened with inspiration. His past medical history is significant for peptic ulcer disease. He reports taking a 2-hour plane trip a month ago. His initial vitals include a HR of 100 beats per minute, a BP of 120/60 mm Hg, an RR of 16 breaths per minute, and an oxygen saturation of 97% on room air. On physical examination, you note decreased breath sounds on the right side. Which of the following tests should be performed next? a. Electrocardiogram (ECG) b. D-dimer c. Ventilation perfusion scan (V/Q scan) d. Upright abdominal radiograph e. Chest radiograph

*E.* A spontaneous pneumothorax typically presents with ipsilateral pleuritic chest pain and dyspnea while at rest. Physical findings tend to correlate with the degree of symptoms. Mild tachycardia, decreased breath sounds to auscultation, or hyperresonance to percussion are the most common findings. It typically occurs in healthy young men of taller than average stature without a precipitating factor. Mitral valve prolapse and Marfan syndrome are also associated with pneumothoraces. The most common condition associated with secondary spontaneous pneumothorax is COPD. Although suggested by this patient's symptoms, the diagnosis of pneumothorax is generally made with a chest radiograph. The classic sign is the appearance of a thin, visceral, pleural line lying parallel to the chest wall, separated by a radiolucent band that is devoid of lung markings. If clinical suspicion is high with a negative initial chest x-ray, inspiratory and expiratory films, or a lateral decubitus film may be taken to evaluate for lung collapse.

An 81-year-old woman presents to the ED with acute onset of shortness of breath just before arrival. She refuses to answer questions for the interview, but repeatedly states that she is feeling short of breath. Her initial vitals include a HR of 89 beats per minute, a BP of 168/76 mm Hg, and an RR of 18 breaths per minute with an oxygen saturation of 89% on room air. A portable chest x-ray appears normal. Her physical examination is unremarkable, except for a systolic ejection murmur. Intravenous (IV) access is successfully obtained. After placing the patient on oxygen and a monitor, which of the following should be performed first? a. Evaluation of troponin level b. Evaluation of D-dimer level c. Rectal temperature d. Repeat chest x-ray e. ECG

*E.* All patients with chest pain and shortness of breath should receive an ECG. It is a quick, noninvasive test that often provides substantive information. An ECG will show that this patient is having a large anterolateral wall myocardial infraction (MI) affecting much of her left ventricle, the reason for her heart murmur. An ECG must be performed in those crucial first moments so that the proper care can be delivered. This example reminds us of the importance of keeping the differential diagnosis broad in patients that present with respiratory distress. The other procedures may be done in a timely manner, but do not necessarily need to be performed as the next most critical step.

A 48-year-old man with a past medical history of hepatitis C and cirrhosis presents to the ED complaining of acute onset abdominal pain and chills. His BP is 118/75 mm Hg, HR is 105 beats per minute, RR is 16 breaths per minute, temperature is 101.2°F rectally, and oxygen saturation is 97% on room air. His abdomen is distended, and diffusely tender. You decide to perform a paracentesis and retrieve 1 L of cloudy fluid. Laboratory analysis of the fluid shows a neutrophil count of 550 cells/mm3. Which of the following is the most appropriate choice of treatment? a. Metronidazole b. Vancomycin c. Sulfamethoxazole/trimethoprim (SMX/TMP) d. Neomycin and lactulose e. Cefotaxime

*E.* Analysis of abdominal fluid and clinical presentation are consistent with spontaneous bacterial peritonitis (SBP). It is recommended to start antibiotic treatment for SBP if the neutrophil count is greater than 250 cells/mm3. Causative organisms include gram-negative enterococcus such as E coli and Klebsiella, as well as Streptococcus sp., and Streptococcus pneumoniae. Therefore, the most appropriate antibiotic for treatment is a third-generation cephalosporin, such as cefotaxime

A 22-year-old woman is brought to the ED by paramedics who state that they found the patient hunched over on a park bench barely breathing. The patient is rousable only to painful stimuli. Her initial vitals include a HR of 78 beats per minute, a BP of 125/58 mm Hg, and a respiratory rate of 6 breaths per minute with an oxygen saturation of 94% on 2-L nasal cannula. Upon physical examination, the patient has clear breath sounds bilaterally and no signs of trauma. Her pupils are 2 mm bilaterally and reactive to light. Which of the following agents may be used to restore this patient's respirations? a. Oxygen b. Flumazenil c. Anticholinergic inhaler treatment d. β2-Agonist nebulized treatment e. Naloxone

*E.* Attention to airway and breathing is of particular importance in opioid intoxication, as indicated by the patient's pinpoint pupils, because respiratory and central nervous system (CNS) depression are the most common life-threatening complications. Naloxone is a μ-opioid receptor competitive antagonist and its rapid blockade of those receptors reverses the depressive effects of opioids.

A 56-year-old woman with a history of ovarian cancer presents to the ED with acute onset of right-sided chest pain, shortness of breath, and dyspnea. Her BP is 131/75 mm Hg, HR is 101 beats per minute, respirations are 18 breaths per minute, and oxygen saturation is 97% on room air. You suspect this patient has a pulmonary embolism (PE). Which of the following tests is most likely to be abnormal? a. Arterial blood gas b. Oxygen saturation c. ECG d. Chest radiograph e. D-dimer

*E.* D-dimer is a degradation product produced by plasmin-mediated proteolysis of cross-linked fibrin. There are two types of D-dimer assays. Those with greatest sensitivity are the enzyme-linked immunosorbent assays and the turbidimetric assays. Because of their high negative predictive value, D-dimer levels are typically used to rule out the diagnosis of PE. In conjunction with a low pretest probability, a negative D-dimer is predictive of not having a PE. Therefore, in this patient with a high probability of PE (eg, exhibiting dyspnea, chest pain, tachycardia, malignancy), it is likely that the D-dimer will be abnormal

A 32-year-old man is brought into the ED by EMS with fever, shortness of breath, and stridor. The patient was treated yesterday in the ED for a viral syndrome. His BP is 90/50 mm Hg, HR is 110 beats per minute, temperature is 101.2°F, and his RR is 28 breaths per minute. A chest radiograph reveals a widened mediastinum. The patient is endotracheally intubated, given a 2-L bolus of normal saline, and started on antibiotics. His BP improves to 110/70 mm Hg and he is transferred to the intensive care unit (ICU). You see a friend that accompanied the patient to the hospital and ask him some questions. You find out that the patient is a drum maker and works with animal hides. What is the most likely organism that is responsible for the patient's presentation? a. Streptococcus pneumoniae b. Corynebacterium diphtheriae c. Coxiella burnetii d. Haemophilus influenzae e. Bacillus anthracis

*E.* Inhalation anthrax is a rare, but life-threatening disease, with mortality rates exceeding 90%. It is caused by inhaling B anthracis spores into the lungs. Initially, the patient develops flu-like symptoms. Within 24 to 48 hours, the clinical course may abruptly deteriorate to septic shock, respiratory failure, and mediastinitis. Chest x-ray may reveal a widened mediastinum. Death usually results within 3 days. Anthrax is normally a disease of sheep, cattle, and horses. As there is no evidence for human-to-human transmission, disease in humans occurs when spores are inhaled. Working with untreated animal hides increases the risk for anthrax exposure.

A 24-year-old man woke up from sleep 1 hour ago with severe pain in his right testicle. He states that he is sexually active with multiple partners. On examination, the right scrotum is swollen, tender, and firm. You cannot elicit a cremasteric reflex. His BP is 145/75 mm Hg, HR is 103 beats per minute, RR is 14 breaths per minute, temperature is 98.9°F, and oxygen saturation is 99% on room air. Which of the following is the most appropriate next step in management? a. Administer one dose of ceftriaxone and doxycycline for 10 days and have him follow-up with a urologist. b. Swab his urethra, send a culture for gonorrhea and Chlamydia, and treat if positive. c. Send a urinalysis and treat for a urinary tract infection (UTI) if positive. d. Treat the patient for epididymitis and have him return if symptoms persist. e. Order a statim (STAT) color Doppler ultrasound and urologic consultation.

*E.* Testicular torsion is a surgical emergency. There are two peak periods in which torsion is likely to occur, the first year of life and at puberty. Manual detorsion can be attempted in most cases while arranging for definitive care. After appropriate analgesia, the anterior testicle should be twisted laterally, like opening a book. Color Doppler ultrasound is the best test of choice in most hospitals. Immediate evaluation and referral to a urologist is essential.

A 75-year-old man goes out to shovel snow from his driveway. After 5 minutes of shoveling, he feels short of breath, chest pain, and then passes out. He awakens minutes later to his wife shaking him. In the ED, he denies chest pain or dyspnea. His BP is 160/85 mm Hg, HR is 71 beats per minute, and oxygen saturation is 97% on room air. On examination, you hear a harsh systolic ejection murmur. An ECG reveals a sinus rhythm with left ventricular hypertrophy. Which of the following is the most likely diagnosis? a. Asystolic cardiac arrest b. Brugada syndrome c. Subclavian steal syndrome d. PE e. Aortic stenosis

*E.* The classic triad of aortic stenosis is dyspnea, chest pain, and exertional syncope. Syncope is a result of either inadequate cerebral perfusion or occasional dysrhythmias. The classic auscultory finding is a harsh systolic ejection murmur that is best heard in the second right intercostal space with radiation to the carotid arteries. Syncope in the setting of a new systolic murmur always should raise the suspicion for aortic stenosis as the etiology. The ECG usually reveals left ventricular hypertrophy.

A 19-year-old woman presents to the ED with 1 hour of acute onset progressively worsening pain in her RLQ. She developed nausea shortly after the pain and vomited twice over the last hour. She had similar but less severe pain 2 weeks ago that resolved spontaneously. Her BP is 123/78 mm Hg, HR is 99 beats per minute, temperature is 99.1°F, and her RR is 16 breaths per minute. On physical examination, the patient appears uncomfortable, not moving on the gurney. Her abdomen is nondistended, diffusely tender, worst in the RLQ. Pelvic examination reveals a normal sized uterus and moderate right-sided adnexal tenderness. Laboratory results reveal WBC 10,000/μL, hematocrit 38%, and a negative urinalysis and β-hCG. Pelvic ultrasound reveals an enlarged right ovary with decreased flow. Which of the following is the most appropriate management for this patient? a. Admit to the gynecology service for observation b. Administer IV antibiotics and operate once inflammation resolves c. Attempt manual detorsion d. Order an abdominal CT e. Immediate laparoscopic surgery

*E.* The differential diagnosis in a woman with RLQ pain is expansive and includes GI pathology, such as appendicitis, inflammatory bowel disease, diverticulitis, and hernia. Gynecologic pathology includes ectopic pregnancy, tubo-ovarian abscess, ruptured corpus luteum cyst, and ovarian torsion. It is often difficult to initially distinguish between gastrointestinal (GI) and gynecologic (GYN) pathology and which diagnostic test, abdominal CT, or a pelvic ultrasound, is warranted. Often, the decision is based on the pelvic examination. The patient in the question exhibits adnexal tenderness and therefore received a pelvic ultrasound that revealed a unilateral enlarged ovary with decreased flow, indicative of ovarian torsion. Ovarian torsion is a gynecologic emergency and conservative management has no place in the treatment decision of suspected torsion even if pain improves in the ED. Failure to surgically correct this entity may result in ischemia and subsequent necrosis of the involved ovary. Therefore, the mainstay of therapy is laparoscopy or laparotomy.

A 71-year-old obese man is brought to the ED complaining of constant left middle quadrant abdominal pain with radiation into his back. His past medical history is significant for hypertension, peripheral vascular disease, peptic ulcer disease, kidney stones, and gallstones. He smokes a pack of cigarettes and consumes a pint of vodka daily. His BP is 145/80 mm Hg, HR is 90 beats per minute, temperature is 98.9°F, and RR is 16 breaths per minute. Abdominal examination is unremarkable. An ECG is read as sinus rhythm with a HR of 88 beats per minute. An abdominal radiograph reveals normal loops of bowel and curvilinear calcification of the aortic wall. Which of the following is the most likely diagnosis? a. Biliary colic b. Nephrolithiasis c. Pancreatitis d. Small bowel obstruction (SBO) e. Abdominal aortic aneurysm

*E.* The patient presents with multiple risk factors for an AAA: age > 60, male gender, hypertension, cigarette smoking, and peripheral artery disease. Classically, AAA presents with constant abdominal pain, often localizing to the left middle or lower quadrant with radiation to the back. Physical examination may reveal a pulsatile abdominal mass. Patients can present unstable if the aneurysm leaks or ruptures requiring emergent management in the OR. Evidence of an AAA is seen on plain radiograph approximately 66% to 75% of the time. The most common findings are curvilinear calcification of the aortic wall or a paravertebral soft tissue mass. Ultrasound and CT are the best diagnostic tools for the stable patient.

An undomiciled 41-year-old man walks into the ED complaining of abdominal pain, nausea, and vomiting. He tells you that he has been drinking beer continuously over the previous 18 hours. On examination, his vitals are BP 150/75 mm Hg, HR 104 beats per minute, RR 16 breaths per minute, oxygen saturation 97% on room air, temperature of 99.1°F rectally, and finger stick glucose 81 mg/dL. The patient is alert and oriented, his pupils anicteric. You notice gynecomastia and spider angiomata. His abdomen is soft but tender in the RUQ. Laboratory tests reveal an AST of 212 U/L, ALT 170 U/L, alkaline phosphatase of 98 U/L, total bilirubin of 1.9 mg/dL, international normalized ratio (INR) of 1.3, WBC 12,000/μL. Urinalysis shows 1+ protein. Chest x-ray is unremarkable. Which of the following is the most appropriate next step in management? a. Place a nasogastric tube in the patient's stomach to remove any remaining ethanol b. Order a HIDA scan to evaluate for acute cholecystitis c. Administer hepatitis B immune globulin d. Send viral hepatitis titers e. Supportive care by correcting any fluid and electrolyte imbalances

*E.* The patient's clinical presentation is consistent with alcoholic hepatitis, which is a potentially severe form of alcohol-induced liver disease. Most people remain subclinical, but the presentation ranges from nausea and vomiting to fulminant hepatitis and liver failure. Laboratory tests usually reveal moderate elevations of AST and ALT. Usually in alcoholic hepatitis, the AST is greater than the ALT (think "scotch" and "tonic" for AST > ALT). The patient exhibits stigmata of chronic alcohol disease seen by gynecomastia and spider angiomata.

As you palpate the right upper quadrant (RUQ) of a 38-year-old woman's abdomen, you notice that she stops her inspiration for a brief moment. During the history, the patient states that over the last 2 days she gets pain in her RUQ that radiates to her back shortly after eating. Her vitals include a temperature of 100.4°F, HR of 95 beats per minute, BP of 130/75 mm Hg, RR of 16 breaths per minute. What is the initial diagnostic modality of choice for this disorder? a. Plain film radiograph b. Computed tomography (CT) scan c. Magnetic resonance imaging (MRI) d. Radioisotope cholescintigraphy (HIDA scan) e. Ultrasonography

*E.* The patient's history and physical examination is consistent with acute cholecystitis. Because of the poor predictive value of the history, physical and laboratory findings in cholecystitis, the most important test for diagnosis is a strong clinical suspicion and ultrasound imaging. It is rapid and noninvasive. Ultrasound may show the presence of gallstones as small as 2 mm, gallbladder wall thickening and distention, and pericholecystic fluid

A 59-year-old man presents to the ED with left-sided chest pain and shortness of breath that began 1 hour ago. Initial vital signs are BP 85/45 mm Hg, HR 105 beats per minute, RR 20 breaths per minute, and oxygen saturation 94% on room air. An ECG is shows ST elevation in leads V1-V4. Which of the following is the most appropriate definitive treatment? a. Administer metoprolol or diltiazem b. Electrical cardioversion c. Administer calcium gluconate d. Thrombolytic therapy e. Percutaneous angioplasty

*E.* The preferred treatment for an ST-elevation MI is primary percutaneous intervention (eg, angioplasty, coronary stent). It has shown to improve long term mortality over (d) thrombolytic therapy

As you are examining the patient described in the previous question (Mobitz II ekg), he starts to complain of chest discomfort and shortness of breath and has another syncopal episode. His ECG is shown below. Which of the following is the most appropriate next step in management? a. Call cardiology consult b. Cardiovert the patient c. Administer metoprolol d. Administer amiodarone e. Apply transcutaneous pacemaker

*E.* The rhythm strip findings are consistent with third-degree AV block, also called complete heart block. It is characterized by absent conduction through the AV node, resulting in the dissociation of atrial and ventricular rhythms. The ECG shows independent P waves and QRS complexes. Mobitz type II often progresses to third degree heart block, as seen in this case. The immediate step in managing complete heart block is applying a transcutaneous pacemaker for ventricular pacing as a temporizing measure. However, patients need implantable ventricular pacemakers for definitive management. In addition, the underlying cause of the block needs to be addressed.

A 67-year-old woman is brought to the ED by paramedics complaining of dyspnea, fatigue, and palpitations. Her BP is 80/50 mm Hg, heart is 139 beats per minute, and RR is 20 breaths per minute. Her skin is cool and she is diaphoretic. Her lung examination reveals bilateral crackles and she is beginning to have chest pain. Her ECG shows a narrow complex irregular rhythm with a rate in the 140s. Which of the following is the most appropriate immediate treatment for this patient? a. Diltiazem b. Metoprolol c. Digoxin d. Coumadin e. Synchronized cardioversion

*E.* This patient is hypotensive and exhibits signs and symptoms of heart failure (dyspnea, fatigue, respiratory crackles, and chest pain) and is in atrial fibrillation (irregular, narrow complex). Any patient with unstable vital signs with a tachydysrhythmia should receive a dose of sedation and undergo synchronized cardioversion starting at 100 J.

A 59-year-old man presents to the ED with left-sided chest pain and shortness of breath that began 2 hours prior to arrival. He states the pain is pressure-like and radiates down his left arm. He is diaphoretic. His BP is 160/80 mm Hg, HR 86 beats per minute, and RR 15 breaths per minute. ECG reveals 2-mm ST-segment elevation in leads I, aVL, V3 to V6. Which of the following is an absolute contraindication to receiving thrombolytic therapy? a. Systolic BP greater than 180 mm Hg b. Patient on Coumadin and aspirin c. Total hip replacement 3 months ago d. Peptic ulcer disease e. Previous hemorrhagic stroke

*E.* Thrombolytic therapy (clotbusters) can be administered to patients having an acute ST-elevation MI that is within 6 to 12 hours from symptom onset. Contraindications to fibrinolytic therapy are those that increase the risk of hemorrhage. The most catastrophic complication is intracranial hemorrhage. Absolute contraindications include: • Previous hemorrhagic stroke • Known intracranial neoplasm • Active internal bleeding (excluding menses) • Suspected aortic dissection or pericarditis

A 23-year-old man is brought to the ED after collapsing during a basketball game. His BP is 80/palp. His father died from a "heart-condition" at a young age. He is placed on a monitor and the rhythm shows wide complex QRS. Which of the following choices best describes this rhythm? a. Ventricular fibrillation b. Atrial fibrillation c. Wolf-Parkinson-White syndrome d. Supraventricular tachycardia e. Torsades de pointes

*E.* Torsades de pointes ("twisting of the points") is a life-threatening uncommon variant of ventricular tachycardia (VT). The ventricular rate can range from 150 to 250 beats per minute. It may occur secondary to medications that prolong the QT interval, such as some antipsychotics. It is also caused by electrolyte disturbances and congenital prolonged-QT syndrome. This congenital form typically presents in childhood or early adulthood and is precipitated by catecholamine excess, such as released during exercise or with the administration of certain medications.

A 29-year-old man presents to the ED complaining of RLQ pain for 24 hours. He states that the pain first began as a dull feeling around his umbilicus and slowly migrated to his right side. He has no appetite, is nauseated, and vomited twice. His BP is 130/75 mm Hg, HR is 95 beats per minute, temperature is 100.9°F, and his RR is 16 breaths per minute. His WBC is 14,000/μL. As you palpate the LLQ of the patient's abdomen, he states that his RLQ is painful. What is the name of this sign? a. Blumberg sign b. Psoas sign c. Obturator sign d. Raynaud sign e. Rovsing sign

*E.*Rovsing sign is the referred tenderness to the RLQ when the LLQ is palpated. It is seen with acute appendicitis.

An 82-year-old woman is brought to the ED by her daughter for worsening fatigue, dizziness, and lightheadedness. The patient denies chest pain or shortness of breath. She has not started any new medications. Her BP is 140/70 mm Hg, HR is 37 beats per minute, and RR is 15 breaths per minute. An IV is started and blood is drawn. An ECG shows P waves marching throughout. Which of the following is the most appropriate next step in management? a. Bed rest for the next 48 hours and follow-up with her primary-care physician. b. Administer aspirin, order a set of cardiac enzymes, and admit to the cardiac care unit (CCU). c. Place a magnet on her chest to turn off her pacemaker. d. Admit for Holter monitoring and echocardiogram. e. Place on a cardiac monitor, place external pacing pads on the patient, and admit to the CCU.

*E.*The patient's ECG reveals third-degree complete heart block. It is a disorder of the cardiac conduction system, where there is no conduction through the AV node. This may occur secondary to MI, drug intoxication, infection, or infiltrative diseases. On the ECG complete heart block is represented by QRS complexes being conducted at their own rate and independent of the P waves. Individuals with second-degree type II or third-degree complete heart block are considered unstable. External pacing pads should be placed on them, followed by a transvenous pacer if their BP is unstable. They may require a permanent pacemaker for irreversible complete heart block.

A 24-year-old woman is brought to the ED after being found on a nearby street hunched over and in mild respiratory distress. Upon arrival, she is tachypneic at 24 breaths per minute with an oxygen saturation of 97% on face mask oxygen administration. Upon physical examination, the patient appears to be in mild distress with supraclavicular retractions. Scattered wheezing is heard throughout bilateral lung fields. Which of the following medications should be administered first? a. Corticosteroids b. Magnesium sulfate c. Epinephrine d. Anticholinergic nebulizer treatment e. β2-Agonist nebulizer treatment

*E.*This patient is suffering from an acute asthma attack. This is a reversible bronchospasm initiated by a variety of environmental factors that produce a narrowing and inflammation of the bronchial airways. The first-line treatment in order to open the airways includes a β2-agonist, which acts to decrease bronchospasm of the smooth muscle.

Epidural Hematoma

*Lucid Interval* MMA Biconvex May be relieved by burr hole

presentation of bronchial carcinoid tumor and carcinoid syndrome

*persistent coughing* with *hemoptysis* and focal wheezing. As these tumors begin to secrete hormonal mediators, they may cause carcinoid syndrome during which symptoms can include *diarrhea, flushing, head and neck edema, bronchospasm, or hives.* suspect in patients with recurrent pneumonia most dx in central bronchi CT surgical excision

What are clinical findings of a spontaneous abortion?

+/- Vaginal bleeding Uterine size does not correlate with LMP Fundus of uterus may be boggy or tender Abdominal cramping

What is a threatened abortion?

+Vaginal bleeding Cervix is closed Products of conception have not passed

Causes of Pulseless Electrical activity (PEA)

- 6 H's and 6 T's - Hyperkalaemia -------Tamponade - Hypoxia - ---------Tension Pneumo - Hypothermia --------- Thrombosis - Hydrogen Ion ------ Toxins - Hypovolemia ------ Trauma - Hypoglycemia

Positive test for Diagnostic peritoneal lavage

- >100,000 RBC/mm3 - > 500 WBC's/mm3 - Bacteria on gram stain

Tx of Spinal Cord injury

- ABCs - High dose methylprednisolone in blunt trauma pts

Tx of Spinal cord injury

- AIRWAY - High-dose methyprednisolone for blunt trauma pts

Indications of Posterior urethral injury

- Abdominal pain, perineal pain and inability to void - Blood at meatus

Options for STABLE Blunt Trauma pt

- Admit with serial exams; vitals every 15-30 min - FAST Scan and U/A - CT scan abdomen/pelvis - Labs

Dx of Aortic Disruption

- CXR ( Screening tool) - CT scan ( Highly sensitive, readily available)

Dx of Pulmonary contusion and hematoma?

- CXR (May be negative initially) - CT scan (More sensitive)

Conditions in which Pulsus Paradoxus is found

- Cardiac Tamponade - Pericarditis - Cardiogenic shock - Asthma/COPD - PE - Tension PTX - Hypovolemia - Pregnancy

Types of Obstructive shock

- Cardiac tamponade - Massive PE - Tension PTX

S&S of Cardiac Tamponade

- Chest pain - Dsypnea - Beck's Triad (Decreased BP, Muffled heart sounds, JVD) - Kussmaul sign

Overall Tx for abdominal trauma

- Crystalloids (IVF) - pRBC - Factor VIIa if there is major bleeding -Tranexamic acid

Mac blade

- Curved blade goes into vallecula and allows visualization of trachea by moving epiglottis

Risk factors for necrotizing skin infections

- ETOH - DM - Prolonged bed rest

ECG findings in Cardiac Tamponade

- Electrical Alternans (Alternating high/low voltage QRS due to changing distance of heart from electrodes)

Signs of Pancreas injury

- Epigastric or back pain - Fluid around pancreas - serum enzyme levels - Grey-turner or Cullen sign

Components of Glasgow coma scale

- Eye response - Motor Response - Verbal Response - Lowest total is 3, highest is 15

Dx of Cardiac Tamponade

- FAST exam is first choice - Transthoracic or Transesophageal echo (when you have more time to dx pt)

Options for UNSTABLE Blunt Trauma pt

- Follow ATLS - FAST Scan - Operating Room if pt is unstable - labs

Le Fort II Fx

- Fracture extends through the maxilla, maxillary sinuses, nasal bones and infraorbital ridge - Facial tugging moves the upper palate and the nose but NOT THE EYES

Signs of Duodenal injury

- Frontal impact - Bloody NGT output - Retroperitoneal air - Feeling full, pressure, pain, inability to eat

Indications for CT imaging of brain in head trauma

- GCS <15 - Loss of consciousness - AMS - Vomiting - Seizure - Amnesia - age >65 years

Cervical Spine Sprain (Whiplash)

- Hx of abrupt hyperextension - Pts have neck pain/ spasm and headache - NO evidence of neurologic deficit and NO fx on C-spine films - Tx- Immobilization with cervical collar from hours to days

How is the dysautonomia of neurogenic shock manifested?

- Hypotension - Bradycardia - Peripheral vasodilation

Class III Hemorrhagic Shock

- Loss of 30-40% of blood volume (2000 cc's) - Measurable drop in BP - Narrowing Pulse pressure - Marked Tachycardia and Tachypnea - Mental status change - Decrease in urine output

Class IV Hemorrhagic Shock

- More than 40% blood lost; loss of > 2000 ml's - Unconscious - Skin is cold, and clammy, diaphoretic - No urine output - Heading to irreversible shock

What drugs should be avoided when a pt is in cardiogenic shock?

- Nitrates - Beta blockers - Calcium channel blockers

Epidural Hematoma

- Occur from blunt trauma to the temporal or temporoparietal area - Rupture of middle meningeal artery - Loss of consciousness followed by a lucid interval - Looks like a football on imaging

Management of tongue laceration

- Pack with gauze and pressure to visualize source - Lidocaine with 1% epi or cauterization may be necessary

Sxs of flail chest

- Pain and Respiratory distress - Paraodoxical chest wall movement - Crepitus - Respiratory Failure

How to assess quality of Chest Xray

- Penetration - Inspiration - 8-10 ribs - Rotation - Angulation

If you clinically Dx a Tension Pneumothorax, what should your next step be?

- Proceed directly to management and Tx

Ellis IV fractures

- Root fractures that involve the alveolar bone - Require stabilization of adjacent teeth with dental resin or ligature wire

Commotio Cordis

- Sudden cardiac death or near sudden cardiac death after blunt, low impact chest wall trauma - Impact occurs 10-30 ms prior to peak of T wave

Signs of Bowel injury

- Sudden deceleration injury - Free fluid in absence of solid organ injury - Free air

Central Transtentorial herniation

- Symptoms may progress from pinpoint pupils and increased muscle tone to hyperventilation and decorticate posturing

Spinal Shock

- Temporary loss of spinal reflex activity that occurs below a complete or incomplete spinal cord lesion - Loss of all neurologic function below the level of the lesion

Spinal Cord injury without obvious radiologic abnormality (SCIWORA)

- Transient or persistent neurologic symptoms in a pt without evidence of fracture or dislocation on plain films and/or CT

Labs to order in Abdominal trauma

- Type and screen - CBC -I-stat - Chemistry - ABG - Lactate - AMylase -HCG (ALWAYS FOR FEMALES) - ETOH

3 impacts in an MVA

- Vehicle Impact - Body Impact - Organ Impact

Lethial Triad in trauma

-Acidosis - Hypothermia - Coagulopathy

Primary Survey

-Airway -Breathing -Circulation -Disability -Exposure

Constraint of disposition

-As an EM practitioner you must do something for each pt that presents to you whether they receive diagnostic studies or lab work

Cerebral contusion

-Blood vessels break and leak causing pooling of blood in brain tissues - non-space occupying lesion within the brain itself - Less likely to cause brainstem hernation

Tx of Extraperitoneal rupture of the bladder

-Foley drainage for 7-15 days with 20Fr or greater sized catheter

Tx of Septal Hematoma

-Incision and drainage is requires to prevent necrosis of the septal cartilage and resultant saddle deformity

What causes Acute Labyrinthitis & Vestibular Neuritis?

-Infection or inflammation of the inner ear, usually due to latent virus -Neuritis = only semicircular canals affected -Labyrinthitis = vertigo + hearing loss

Appropriate solutions to transport a tooth?

-Milk - Saliva - Saline - Hank's or Viaspan solution

Signs of Hypoperfusion

-Obtunded - Cool skin temp - Delayed capillary refull - Decreased Pulses

What are the s/s of retinal detachment?

-Sudden painless loss of vision "like a curtain covering the eye" -May see flashes or floaters or cuts or lines in vision

In a massive hemothorax what will you see on CXR?

-White out on CXR - Blunting of costophrenic angle

Tripod fracture

-fracture of the zygomatic arch -fractures of the inferior orbital rim and anterior and posterior maxillary sinus walls -fracture of the lateral orbital rim

Pneumonia empiric treatment: 1. Augmentin, Macrolide (azithromycin, clarithromycin, erythromycin), doxycycline, second-generation cephalosporins ("a furry fox..") 2. Fluoroquinolone, ceftriaxone, cefotaxime. 3. Clindamycin, Pen G 4. Fluoroquinolone + Metronidazole, Ceftriaxone + metronidazole, Ticarcillan-Clav, Piperacillin-Tazo

1. CAP, inpatient 2. CAP, outpatient 3. Aspiration, community 4. Aspiration, hospital

What does it mean if a patient has the 1. HepB surface antigen/HBsAg 2. HepB surface antibody/ANTIHBs 3. Hep B antibody to core antigen/ANTIHBc

1. HBsAg - patient has Hep B infection 2. ANTIHBs - patient has been immunized or has recovered from acute infection 3. ANTIHBc - indicates an acute infection

What are three major risk factors for CHF?

1. Hypertension - causes chambers of heart enlarge and weaken 2. CAD - ischemia damages cardiac muscle 3. Valvular heart disease - dysfunctional valves cause the heart to have to work harder which causes failure over time

What are the four areas evaluated in the FAST exam?

1. Perihepatic region --- Morison pouch 2. Perisplenic region 3. Pericardium region 4. Pelvis - Douglas pouch

What is the NEXUS criteria?

1. absence of intoxication 2. absence of distracting injury 3. absence of midline cervical tenderness 4. absence of focal neurologic impairment

Stroke: Which vascular supply is affected with the following common manifestations? 1. contralateral extremity weakness - lower > upper; altered reasoning; bowel & bladder incontinence 2. contralateral face and arm weakness greater than leg; contralateral sensory deficits; dysphasia 3. contralateral visual field deficits; altered mentation; cortical blindness 4. vertigo/nystagmus; dysarthria; dysphagia; contralateral pain and temperature sensory deficits; syncope

1. anterior cerebral artery 2. middle cerebral artery 3. posterior cerebral artery 4. vertebrobasillar arteries

CSF Characteristics - Bacterial or Viral? 1. Opening pressure >300mmHg, >1000/microliter WBC; <40mg/dL of glucose 2. Opening pressure <300mmHg, <1000/microliter WBC; >40mg/dL of glucose

1. bacterial 2. viral

Atrial flutter - sawtooth pattern in II, III, aVF - what three treatments are used?

1. cardioversion if no contraindications 2. acute rate control tx w BB, CCB - amiodarone, sotalol, quinidine, or procainamide 3. If site of reentrant is known, catheter ablation

when a patient presents with an S3 gallop, particularly when accompanied by pulsus alterans (arterial pulse waveform showing alternating strong and weak beats) - what are the following? (reduced or elevated) 1. LV pressure 2. ejection fraction 3. BNP

1. elevated LV filling pressure 2. reduced EF 3. elevated BNP

What are the three strategies for reducing intraocular pressure in acute angle closure glaucoma? BY DEFINITION, IOP SHOULD BE ABOVE 20 mmHg.

1. increasing aqueous humor outflow - pilocarpine 2. inhibiting aqueous humor production/IOP - Timolol, Acetazolamide 3. reducing aqueous or vitreous humor volume - IV mannitol

Pediatric Fever Algorithm: 1. For babies ___in age and fever at ____; admit to hospital for blood cx, urine cx, lumbar puncture, possible CXR, IV Ampicillin & Gentamicin. 2. For children ___in age and fever at ___; do blood cx, urine cx, possible CXR, ceftriaxone 50mg/kg, d/c home if cx neg, follow up in 24 hr 3. For children ___in age and fever at ___; urine cx, possible CXR, stool culture; close follow up

1. less than 28 days old, temp >/=38C/100.4 2. 28 days to 3 months; temp >/=38C/100.4 3. 91 days to 3 years; non-toxic appearing; temp >/=39C/102.2, FULLY IMMUNIZED

tetralogy of fallot 4 abnormalities?

1. pulmonary stenosis 2. right ventricular hypertrophy 3. overriding aorta 4. ventricular septal defect

Timeline: Reperfusion should take place before ___hours of symptom onset. Door to needle time for fibrinolysis is ____min. Door to balloon time for PCI is ___min.

12; 30min; 90min

Viral Exanthems ___presents with cough, coryza, conjunctivitis, Koplik's spots, and a macular rash w head to toe progression. ___presents w strawberry tongue and sandpaper rash. Chest, arm pits, behind ears, and groin. ___presents with maculopapular rash that spreads from face, "3-day measles" - dangerous during pregnancy.

1st disease - measles 2nd disease - scarlet fever 3rd disease - german measles; rubella

Generally pts with a mild TBI can be Discharged after _____ hours of observation

2 hours

HR range for atrial flutter

250-350bpm

Clinical features of pericarditis? What are the three classic features of pericarditis?

3 classic features: Pleuritic, sharp chest pain (relieved by sitting upright/leaning forward), Diffuse ST elevation, Friction rub Constrictive pericarditis causes dyspnea, fatigue, weakness, edema, hepatomegaly, ascites

tPA should be considered in patients presenting with ischemic strokes of less than ____ hours of symptom onset -- and without evidence of hemorrhage on CT of the head. What is the time for "expanding the window".

3 hours -- up to 4.5 hours according to AHA

Diagnostic labs for endocarditis?

3 sets of blood cultures at least 1 hour apart before starting antibiotics

What is minimal urine output for burn patients?

30 ccs

Permanent neuronal injury can ensue within ____minutes of continuous seizure activity, even in the absence of apparent convulsions.

30-60

Hypothermia is defined as a core body temperature less than ___. The risk of cardiac dysrhythmia does not occur until severe hypothermia ensues (less than 30/86). ____blebs are less favorable than clear blebs. Tx:

36/95F; Hemorrhagic: rapid active warming with circulating 37-40C water.

What is the 3 to 1 rule for fluid resuscitation?

3mL of crystalloid replacement for every 1mL of blood lost

Outer skin on a lip laceration should be closed with?

6-0 Nylon

What is the prevalence of endometriosis?

6-10% of women in the US

Hypotension is defined as a systolic blood pressure less than _____mm Hg or a decrease from baseline by more than 30mmHg. What are the 3 treatments for improving blood pressure?

90mmHg; 1. IV Fluids 2. Vasopressors - dopasmine, dobutamine (risk is aggravation of arrhythmias and increase myocardial oxygen demand) 3. intra-aortic balloon pump

What is Mastitis? Who does it most commonly occur in? What bug causes it?

A breast infection that occur primarily in lactating women. Caused by S. Aureus

Tx for type A aortic dissection? B?

A is treated with excision of tear and a portion of the ascending aorta, replacement with aortic graft, and repair or replacement of the aortic valve B is treated medically with tight BP control with BB and afterload reducers (Nitroprusside). May need surgery.

Initial assessment and resuscitation of the burn patient?

ABCDEs Check urine output Check for eschar and compartment syndromes

everyone w/ HF needs to be on at least two meds... waht are these meds (classes)

ACEI, BB; also often add a diuretic (diuretics are sulfa drugs) such as fureosemide, may subsequently add spironolactone

What is Scarlet fever?

AKA "SCARLETINA". Most commonly affects children between 5 and 12 years of age. Caused by GAS which release erythrogenic exotoxin.

What are endometriomas?

AKA "chocolate cysts". Caused by endometriosis. They are pelvic masses with thick fibrotic walls and filled with old blood.

What is Erythema Infectiousum?

AKA 5th disease Caused by Human Parvovirus B19

What is stable angina?

AKA Angina pectoris Episodic angina that persists for years in some patients Lasts 5-20 minutes Relieved by nitro, O2, rest ST depression/T wave inversion

What is Roseola?

AKA Exanthem Subitum Caused by human herpesvirus 6 (HHV-6) Occurs in infants ages 6 months - 3 yo

What is prinzmetal angina?

AKA Variant angina. Spasm of the epicardial vessels in patients with normal coronary arteries. Typically occurs at rest and is brought on by use of tobacco or cocaine. EKG shows ST elevations during an attack.

What are the different patterns of fetal heart rate?

Accelerations - increase of 15bpm for 15 seconds above normal rate. Indicates fetal well being Early decelerations - decrease in heart rate before contractions begin. Considered benign. Variable decelerations - rapid drops in fetal heart rate with a return to baseline with variable pattern. If mild and infrequent considered benign. If not, could be caused by cord compression. Late decelerations - fetal heart rate drops during 2nd half of contractions. Denotes fetal distress

What should be on your DDx with vaginal bleeding and a negative pregnancy test?

Acute menorrhagia Truama GYN infections Ruptured ovarian cyst Ovarian torsion FB Coagulopathy Cancer DUB UTI

What is acute arterial occlusion?

Acute occlusion of an artery usually by embolization or thrombosis

WHat is the tx for myxedema coma?

Administer loading dose T4 followed by daily dose -Also give T3 (more rapid onset) until pt is stabilized with clinical improvement *Passive rewarming

What is the tx for orbital cellulitis?

Admit for IV antibiotics (amoxicillin or ceftriaxone + vanco for severe cases)

Which of these structures can sense pain in the brain: extracranial structures proximal venous sinuses and brances (just beyond circle of Willis) dura overlying the base of brain

All of them -- most intracranial structures can not sense pain though (brain parenchyma, arachnoid, pia, upper portions of dura)

What are management strategies for the first stage of labor?

Ambulation if tolerated If patient wants to be supine, best in lateral position to prevent compression of SVC IV fluids Labs - CBC, T&S Consider rupture of membranes Provide analgesia

What are common symptoms of pregnancy? (7)

Amenorrhea N/V Breast tenderness Quickening Easy fatigability Change in appetite Urinary frequency, nocturia, infection

ST Elevations in V1, V2, V3, V4, V5

Anterioseptal MI, left anterior descending artery

Tx for Zoster?

Antivirals within 72 hours of outbreak - Acyclovir, valacyclovir Steroids may prevent postherpetic neuralgia Prevention with zostavax!

_________is considered the gold standad test for aortic dissection but is also the most invasive of all the radiographic studies. What is more normally done in the ED?

Aortography; CT with contrast

Management post-MI?

BB Statin ACE-I/ARB CCB

How are frequent PVCs treated?

BB/CCBs

Thrombolytic Therapy -- used for MI, stroke, and PE -- what are the absolute contraindications? BHTARC

BHTARC 1. active bleeding 2. hemorrhagic stroke, CVA, TIA in last year 3. intracranial tumor 4. suspected aortic dissection 5. diabetic retinopathy 6. altered level of consciousness

What does the murmur of AR sound like?

BLOWING, faint, high pitched, systolic and diastolic decrescendo murmur in the 2-4th LICS. Radiates to apex and RSB. Have patient sit, lean forward, with full exhalation.

How do you manage preterm labor/delivery?

Bed rest Oral or IV hydration Antibiotics Steroids Tocolytics if indicated Cerclage if indicated

What are you hearing if there is an S3?

Blood rushing in the LV and hitting the wall ("gallop")

Flail Chest

Blunt force trauma causing chest wall injury with loss of bony continuity to the thoracic cage

What imaging modality would be most appropriate for lower airway injury?

Bronchoscopy

Cervical and Lumbar Radiculopathies C6 - motor weakness? decreased reflex? decreased sensation?

C6 motor - biceps, wrist extensors reflex - biceps sensation - dorsolateral aspect thumb and index finger

Pharm therapy for prinzmetal angina?

CCB

What are the most common causes of 2ndary amenorrhea in hypoestrogenic women?

CNS tumor Stress Hyperprolacinemia Hypophysitis Sheehan syndrome Premature ovarian syndrome

In which population is Klebsiella pneumonia most commonly seen in?

COPD, Alcoholics and the elderly. currant jelly sputum bulging fissures

71yo male presents to ED for sudden onset blurry vision (reports seeing things in strange colors). has been drinking, bumped head last night; per son pt isn't acting like himself and speech is slurred (has been drinking). pt has hx of TIA, MI, smoking. PE reveals double vision which persists w/ one eye closed, slurred speech, otherwise unremarkable. what do you do?

CT w/o contrast to check for (hemorhaggic) stroke. this patient was also on blood thinners and therefore less likely to have ischemic. double vision is not concerning unless it persists w/ one eye closed in which cause it suggests a cerebral tumor of some sort. this patient was on blood thinners, which makes hemorrhagic more concerning, also rules out the possibility of using TPA. be sure to keep hypoglycemia in the ddx for altered MS (this guy had a BS of 68).

Clinical features of PACs

Can occur at any age, often in absence of heart disease May feel like palpitations or "fluttering" in the chest

How is pericardial effusion different from cardiac tamponade?

Cardiac tamponade is where there is a pericardial effusion that causes a significant degree of compression of the heart resulting in decreased blood being pumped around the body.

What causes a pericardial effusion?

Caused by pericarditis, lung cancer, uremia, trauma

What are recommended outpatient antibiotic regimens for treatment of PID? How long is duration of therapy?

Ceftriaxone (IM) + Doxy with or without Metronidazole OR Cefoxitin (IM) + Probenecid + Doxy with or without Metronidazole *Total duration of treatment will still be 14 days

____is infection of the deep dermis and subcutaneous tissue. Empiric antibiotic therapy should be directed against staph and strep organisms. Dicloxacillin, augmentin, first generation cephalosporin

Cellulitis

____syndrome typically occurs with cervical hyperextension when the ligamentum flavum buckles into the spinal canal and pinches the cord. Usually manifests as weakness - upper extremities more than lower. Prognosis good.

Central cord syndrome

Head Injuries

Concussion: Repetitive questioning, brief LOC Skull Fracture: Battle sign (bruising of mastoid process), Raccoon's eyes, Cerebrospinal fluid leak, blood in EAC EDH, SDH, ICH: Seizure, Altered Mental Status, Pain, Focal Deficits In the elderly, clinical acumen cannot reliably exclude significant injury

What tracts are located on the anterior part of the spinal cord?

Corticospinal and spinothalamic tracts

Appendicitis Work-up and Treatment

DDx: PID, gonadal torsion, acute gastroenteritis, diverticulitis, UTI, kidney stone, ectopic pregnancy IV, pain control, NPO, fluids CBC, UA, BMPm GC/Chlamydia, hCG CT or ultrasound Surgical Consult +/- Antibiotics

Diagnostic studies for PID?

DNA probes for G&C TV ultrasound Culdocentesis or laparoscopy

What are characteristics of fourth degree burn?

Damage to fascia, muscle, bone Tissue is necrotic Most common causes: Hot and heavy objects (molten metal, extended exposure, electrical injuries)

Clinical features of hemorrhagic shock?

Decrease in BP Narrow pulse pressure Tachycardia Dilation of pupils

Vfib

Defibrillation is most important ACLS pulseless and unconscious

How are symptoms and the extent of endometriosis related?

Degree of endometriosis does not correlate with symptoms. Pain is more related to the depth of disease.

What is premature delivery?

Delivery of a viable infant before 37 weeks. Occurs in 8-10% of births. Most common cause of neonatal death not resulting from a congenital cause.

What will the EKG show with pericarditis?

Diffuse ST elevation without reciprocal changes or PR depression

Pericarditis EKG

Diffuse ST segment elevation-upward curves (smile) p-r depression

What are signs/symptoms of Trichomoniasis?

Diffuse malodorous yellow-green vaginal discharge with vulvar irritation Vaginal pH>4.5 Colpitis macularis (Strawberry cervix) Many women are asymptomatic

What should you avoid with PROM?

Digital exam

What is contraindicated with placenta previa?

Digital exam - can cause hemorrhage

What are signs/symptoms of cervicitis?

Discharge which is yellow or green +/- odor Friable, quick to bleed cervix Can progress to PID if untreated

what is the tx for menieres?

Diuretics -Low salt diet -Anti-vertigo meds

What do you need to do if a patient is on a GnRH agonist for >6 months?

Do add back therapy - give low dose OCP or medroxyprogesterone to replete hormones.

Antiarrhythmic therapy in patients with chronic A flutter?

Dofetilide is DOC

Most common rupture point in bladder?

Dome

name the ONE technique/image to check for suspected testicular torsion

Doppler US (emphasis here on the doppler; I knew it was US but didn't know the flavor)

Diagnostic studies for PAD? What is the gold standard?

Doppler ultrasound Ankle-brachial index (ABI) - uses Doppler measures to compare BP in upper and lower extremities. <0.9 indicates disease Angiography <----gold standard CT/MRI Elevated homocysteine

Drug Hypersensitivity Syndrome

Drug reaction occurring in the first 2 months after initiation of drug Morbilliform drug eruption, facial edema Symmetric distribution trunk/extremities

What are characteristics of allergic contact dermatitis?

Due to type IV cell mediated or delayed hypersensitivity reaction Must be sensitized to offending agent Can spread beyond contact area or generalize Signs: Erythema, papules, vesicle, erosions, crusts, scales Occurs within 12-72 hours LINEAR FORMATIONS

Clinical features of thrombophlebitis/DVT?

Dull pain Erythema Tenderness Induration of involved vein Swelling of involved area with heat and redness Homan's sign (unreliable)

Diagnostic test for DVT?

Duplex ultrasound <---preferred study Venography <--- most accurate study D-dimer (neg is <500) Angiography if PE is suspected

How is Trichomonas diagnosed? Treatment?

Dx: yellow-green d/c, elevated pH, saline prep shows flagellated trichomonads Tx: Metronidazole

what is THE first thing that needs to be ordered for a pt presenting w/ chest pain decided to be angina?

EKG

Work up for chest pain? Which test is most reliable?

EKG <---most reliable Cardiac markers Echo Stress test

Prevention of chronic venous insufficiency?

Early aggressive treatment of venous reflux states: (acute thrombophlepbitis, varicose veins, use of compression hose, weight reduction)

Topical Analgesics should not be used on what type of ellis fracture?

Ellis type III

____is infection of the brain parenchyma. CSF: Gram stain usually negative, WBC majority is mononuclear leukocytes, protein >100, CRP normal, glucose normal to slightly decreased.

Encephalitis

____is the presence of endometrial tissue outside the endometrial cavity. S/S: dyspareunia, infertility, abnormal bleeding, and chronic pelvic pain. Dx: direct visualization w laparoscopy

Endometriosis

What are causes of secondary dysmenorrhea?

Endometriosis Adenomyosis Uterine fibroids PID IUD

What should be on your Ddx for gynecologic pelvic pain?

Endometriosis Gynecologic malignancies PID Pelvic adhesions Adenomyosis Chronic endometritis Symptomatic pelvic relaxation (prolapse pain)

Diplopia on upward gaze indicates?

Entrapment of the rectus muscle

______hematoma often presents with spinal fluid rhinorrhea and unconsciousness followed by resolution and then later unconsciousness AFTER a skull fracture.

Epidural

What type of brain injury is characterized by loss of consciousness followed by a lucid interval?

Epidural Hematoma

Blunt trauma to the temporal or temporoparietal area can result in?

Epidural hematoma

_____is inflammation or infection of supraglottic structures ie. tongue, vallecula, arythenoid, and tonsils. What age is most commonly affected?

Epiglottitis; between 2 and 6 years old

what test can be done to differentiate Prinzmetal angina from true angina?

Ergonovine challenge

Hemophilia A is treated with ________. Hemophilia B is treated with ________. vonWillebrand disease is treated with ___or___.

Factor VIII; Factor IX; vW factor or DDAVP

Shock Definition and Types

Failure to adequately deliver blood, oxygen, or nutrients to tissue to meet metabolic demands 1. Hypovolemic (not enough blood) 2. Cardiogenic (heart is not working well enough) 3. Distributive (Sepsis, anaphylaxis, cord injury) 4. Obstructive (PE, Tamponade)

What drugs are used for chemical conversion of A.fib?

Flecainide Propafenone Amiodarone - DOC Dronedarone Ibutilide

Simple wedge fracture

Flexion and compression causes fracture of the anterior vertebral body between two other vertebral bodies - Stable fracture unless there is significant posterior ligamentous disruption

Parkland formula

Formula widely used to estimate the volume of crystalloid necessary for initial resuscitation of burn patient: 3-4cc of LR x Kg x BSA 1/2 given in first 8 hours Last 1/2 given over 16 hours

Jefferson Fracture

Fracture of C1 (atlas) caused by a compressive downward force from the occipital condyles

Odontoid fracture

Fracture of the dens of C2 - Type II and III are unstable and these may present with neurologic impairment

Open skull fracture

Fracture of the skull with an overlying laceration or open wound

Moderate TBI

GCS 9-13

name four common (in my opinion) causes of chest pain in a relatively young patient (i.e. not ischemic)

GERD, Prinzmetal, pericarditis, costochondritis

How do you mange a stable and non-pregnant patient with vaginal bleeding?

Get a good history Pelvic exam Labs - CBC, CMP, TSH, PT/PTT, UA, T&S U/S - if there is pain Iron supplementation If post-menopausal, have them follow up with GYN

What happens with AS when you squat?

Gets louder because squatting increases blood flow to the heart. Decreases with Valsalva (which decreases blood flow).

______is an uncommon cause of acute renal failure due to immune complex deposits in the kidney. Dysmorphic RBC and RBC CASTS are common.

Glomerulonephritis

What is the criteria for diagnosing DKA? Glucose, HCO3, pH, Sodium, Potassium

Glucose >250; HCO3<15; pH<7.2; sodium low; potassium may show as normal or slightly elevated but total body potassium is usually depleted by renal losses

STD antibiotic treatment Gonorrhea? Chlamydia? Syphillis? Genital Herpes? Genital Warts?

Gonorrhea - ceftriaxone; cefixime Chlaymydia - azithromycin; doxycycline Syphillis - Penicillin G; doxycycline Genital Herpes - acyclovir; valcyclovir Genital Warts - Imiquod cream, Gardisil VACCINE

Epididymitis or infection of the epididymis is often divided into two groups based on age distribution: 1. <40 years old is usually from ____ 2. >40 years old is usually from ____.

Gonorrhea or Chlamydia; gram negative rod from UTI or prostatitis

general indication for fioricet

HA

What is the MC cause of aortic dissection?

HTN

What are clinical features of Roseola?

High, abrupt FEVER--->febrile seizures. Fever lasts 3-7 days. Rash on face, neck, arms, legs (pinkish-red flat or raised rash which turn white when touched) Defervescence (fever goes away) occurs before rash appears

Digoxin inhibits the membrane-bound sodium potassium-ATPase ion exchanger. Dig toxicity is made clinically and typically relies on history. ______greater than 5mEg/L and significant dysrhythmias warrant administration of Digibind. What EKG findings?

Hyperkalemia; PVC's, atrial tachycardia w AV block, a fib

Clinical features of neurogenic shock?

Hypotension without reflex tachycardia because there is a disconnect between the brain and cardiovascular system

Most important clinical sign in Pulmonary contusion or Hematoma?

Hypoxia

When should you not insert a foley catheter?

If there is blood at the meatus

When do you chemically convert A.fib?

If they have been in A.fib <48 hours

Tx of scalp laceration

If underlying skull and intracranial injury is ruled out, simple closure may be performed - Staples,sutures and tissue adhesive are all acceptable closure methods

What are outbreaks of shingles precipitated by?

Illness Stress Advancing age

Where do most EPs occur?

In the fallopian tubes (95%). 55% of these occur in the ampulla of the tubes. Due to lack of sub-mucosal layer in the fallopian tubes.

Definition of Shock

Inadequate tissue perfusion

What is the major risk for PROM and PPROM?

Infection (chorioamnionitis and endometritis). This risk increases with time and hastens delivery. Cord prolapse

CVA Signs and Symptoms

Ischemic: Abrupt, loss of function, Multiple syndromes based on anatomy, HTN Hemorrhagic: Abrupt, Loss of Function, Vomiting, Headache, Altered mental status (coma), HTN (>200 systolic often)

CVA Treatment

Ischemic: Activate stroke team, TPA vs Intervention, Modest BP control (no more than 25% decrease) <185/110 Hemorrhagic: Activate Stroke Team, Contact neurosurgeon, Modest BP control (variable practice, Nimodipine), Consider FFP, rVIIA, platelets, Reverse anti-coagulation

____sign refers to pain in the neck or back that occurs when a pt with meningitis attempts to extend the leg at the knee while the thigh is held in 90 degrees of flexion. ____sign refers to spontaneous flexion of the hips during attempted passive flexion of the hips during attempted passion flexion of the neck.

Kernig; Brudzinski

which type of breathing is associated w/ DKA... Kussmaul respirations, or Cheyne Stokes?

Kussmaul. this occurs w/ any metabolic acidosis. these are deep and regular breaths.

Cervical and Lumbar Radiculopathies L5 - motor weakness? decreased reflex? decreased sensation?

L5 motor - extensor hallicus longus reflex - none sensation - top of foot, first web space

____is acute unilateral infection or inflammation of the vestibular system - typically due to a recent viral infection. CP: rotational vertigo, nystagmus, n/v. What kind of nystagmus? Tx?

Labyrinthitis; horizontal - rotary away from affected side; diazepam, meclizine, dimenhydrinate

How do you treat TEN?

Like a burn victim - admit to burn unit Emergent derm consult

Shock Treatment

Look for underlying cause All bleeding stops....Eventually IV access (Central) -2 Large bore IVs Fluids first then blood Antibiotics early, source control Pressors after fluids and blood Ionotrops Epinephrine Consider Steroids (Stress dose steroids)

Clinical features of PID?

Lower abdominal and pelvic pain which is bilateral Nausea HA Lower back pain +/- Fever Chandelier sign Purulent discharge

Where does a DVT most commonly occur?

Lower extremities (Long saphenous vein)/pelvis

What can be used for refractory cases of asthma exac?

Mag Sulfate

Risk factors for acute MI?

Male Increasing age Smoking HTN HLD Diabetes Family Hx Menopause Cocaine use

____is a regional infection of the breast typically seen in lactating women; caused by patient's skin flora or oral flora of infant. Tx: dicloxacillin; complications include development of _____.

Mastitis; breast abscess which would be I&D; usually caused by Staph Aureus

Clinical features of pericardial effusion?

May be painful or painless. Cough and dyspnea are common.

Counseling for breast feeding with mastitis?

May continue because the source is likely from the infant Breast feeding may continue on unaffected breast Use of a breast pump may reduce congestion of the infected breast

Medical tx of EP?

Methotrexate treats 80% of EPs

What are clinical features of Erythema Infectiosum?

Mild flu like illness Rash at 10-17 days (not contagious with rash) SLAPPED CHEEKS LACEY arms and legs Arthralgias in older patient Can cause fetal death in 1st trimester

Renal Lithiasis

Mineral precipitation in collection system, Abrupt & sharp, Flank to groin pain, Writhing pain, N/V, diaphoresis DDx: Cholecystitis, splenic rupture, appendicitis, torsed gonad, shingles, trauma, diverticulitis, pyelonephritis IV, fluids, pain control (ketorolac) UA: expect hematuria, causion pyuria Non-contrast CT vs US +/- CBC, BMP Urology follow up, Tamsulosin, Opiate, Strainer

What is Bullous Pemphigoid?

Most common bullous autoimmune disease! Autoantibodies, complement fixation, neutrophils, and eosinophils cause bullous formation

Uncal Transtentorial herniation

Most common herniation - Usually caused by an expanding lesion in the temporal lobe or lateral middle fossa -May cause fixed and dilated ipsilateral pupil, with progression will cause contralateral motor paralysis

Clinical features of A. fib?

Multiple, small areas of the atrial myocardium are continuously discharging in a disorganized fashion. Irregularly irregular without p waves

Cardiac Markers: _____is detectable within 1-2 hours after acute MI. Duration <1 day. Low specificity.

Myoglobin

What are the cardiac biomarkers for diagnosis of MI? Which is the most specific? Which is the most sensitive?

Myoglobin <---most sensitive Troponin I Troponin T <----most specific CK-MB

Patients with long standing hypothyroidism may develop this. Myxedema coma is a syndrome of hypothermia, AMS, respiratory insufficiency, and myxedema. ____is a nonpitting, dry, waxy swelling of the skin caused by deposition of mucopolysaccharides in the dermis. What is the cornerstone of treatment?

Myxedema; IV levothyroxine, supportive care, and hydrocortisone to avoid adrenal insufficiency

What are expectant management treatments for endometriosis?

NSAIDs Prostaglandin synthetase inhibitors COCs Progestins (Depo-Provera, IUD) Narcotics - try to avoid Danazol GnRH agonist

How do you treat primary dysmenorrhea?

NSAIDs before menses and continue for 2-3 days OCPs Vit B Magnesium Acupuncture Heat Regular exercise

___is used for opiate and heroin overdose. ___is used for acute benzodiazepine overdose. ___is used for tricyclic antidepressant overdose.

Naloxone; Flumazenil; Sodium Bicarbonate

What medications are recommended for BP control in aortic dissection?

Nitroprusside and a B-blocker

What are PE findings with hypothalamic-pituitary insufficiency? Labs? Tx?

No breast development Labs: Low FSH and LH Tx: Cyclic estrogen and progesterone

What is contractility?

Normal ability of muscle to contract at a given force for a given stretch. It is independent of preload/afterload

What are causes of vasovagal syncope?

Occurs when your body overreacts to certain triggers, such as the sight of blood or extreme emotional distress. Vasodilation and reflex bradycardia occur.

Burst Fracture

Occurs with AXIAL LOADING and causes the vertebral body of C3-C7 to shatter outward from the compressive force - Disrupts the anterior longitudinal ligament

What will you find on PE with endometriosis?

Often normal pelvic exam May have tender nodularity of the cul-de-sac and uterine ligaments and a fixed, retroverted uterus Adenexal masses

What are complications from ulcers?

Osteomyelitis Bacteremia Sepsis

The most common pathogen in ____is Pseudomonas Aeroginosa but can also be caused by Staph aureus, GAS, and aspergillus. Tx usually consists of a mixture of polymyxin, neomycin, and hydrocortisone.

Otitis Externa

What should be in your Ddx for acute pelvic pain?

Ovarian torsion or rupture of ovarian or fallopian tube cyst Functional ovarian cyst Endometritis/PID Ectopic pregnancy Appendicitis Acute cystitis Ureteral stone formation

Where is endometriosis usually found? What is the most common site?

Ovaries (60%) Uterine cul-de-sac Uterosacral ligaments Posterior surfaces of uterus Pelvic peritoneum Look at bladder, bowel, appendix, ureter

What are clinical features of abruptio placentae?

PAINFUL vaginal BLEEDING Uterine, abdominal, or back pain Uterus becomes hypertonic, tender Fetal distress depending on separation

How do you treat scabies?

PERMETHRIN applied from chin to bottom of feet and left overnight (8 hours) then washed off in morning. Repeat tx in 7 days.

What is the site of a arterial insufficiency ulcer vs. a venous stasis ulcer?

PVD arterial insufficiency ulcer is usually on toes/feet Venous stasis ulcer is usually over medial malleolus

What is rest pain?

Pain in the foot, usually over the distal metatarsals. This pain arises at rest, classically at night

How does erysipelas present?

Painful localized redness, heat, and swelling. Raised, WELL-DEMARCATED, indurated border Often affects face and legs NO SITE OF SOURCE or port of entry, unlike cellulitis

What is primary dysmenorrhea?

Painful menstruation caused by excess prostaglandin and leukotriene levels. Onset is usually within 2 years of menarche. There is NO pathologic abnormality.

What will PE reveal with pilonidal disease?

Painful, fluctuant area the sacrococcygeal cleft Tuft of hair emerging from the midline opening in the natal cleft

Diagnostic procedures/imaging for DUB?

Pap Endometrial biopsy Pelvic US Hysterosalpingography Hysteroscopy D&C

____is optic disc swelling caused by increased intracranial pressure.

Papilledema

Diagnostic studies for ovarian cysts?

Pelvic US - will be mobile, fluid filled, and simple Abdominal and Pelvic CT

____is a serious complication of STIs because of increased risk of infertility and ectopic pregnancy. Lower abdominal tenderness, Chandelier's sign, and purulent cervical discharge. ABX: broad spectrum cephalosporins, clinda/gent if allergic

Pelvic inflammatory disease (PID)

When is renal imaging indicated?

Penetrating trauma Pediatric trauma Deceleration injury Adult blunt trauma

How do you treat mastitis?

Penicillinase-resistant antibiotic (Cloxacillin, dicloxacillin, nafcillin) Cephalosporin Hot compresses Keep breastfeeding/pumping

What is the surgical option of treating AS in patients who are poor surgical candidates?

Percutaneous balloon aortic "valvuloplasty"

How are recurrent pericardial effusions treated?

Pericardial window

What is Dressler's syndrome (post MI syndrome?)

Pericarditis, fever, leukocytosis, pericardial or pleural effusions

What is Sick Sinus Syndrome?

Physiologically inappropriate sinus bradycardia, sinus pause, sinus arrest, or episodes of alternating sinus tachycardia and sinus brady. Occurs most often in elderly due to scarring of the heart's conduction system or infants who have had heart surgery

Most common complication of Pulmonary contusion and hematoma?

Pneumonia

35yo female presents w/ CP that started this morning and has been constant ever since; otherwise asymptomatic. Troponin negative but EKG shows ST elevation... diagnose

Prinzmetal angina... she's relatively young (tends to occur in those under 50), more common in females, tends to start in the morning, may cause ST elevation but have negative troponins

Elements of history to obtain in a patient with ACS symptoms?

Prior CABG, PCI? CAD, angina, previous MI? Risk factors? Family hx? PMHx? Substance use? Previous cardiac tests (EKG, Echo, Stress test, Cath)?

A Fx of the 1st and 2nd rib are associated with?

Pulmonary, Cardiac, Vascular injury

What is preterm labor?

Regular uterine contractions (>4 in 6 hours) between 20-36 weeks of gestation and presence of one or more of the following signs: 1. Cervical dilation of 2cm+ on presentation 2. Cervical dilation of 1cm+ on serial exams 3. Cervical effacement of greater than 80%

Most common injury from blunt thoracic trauma?

Rib Fracture

What is the EKG changes in a PE?

S1Q3T3 - with or without a RBBB

on which anatomical structure of the heart do epinephrine and norephinephrine work

SA node

What defines orthostatic hypotension?

SBP drop of >20 and DBP drop of >10 Rise in pulse of >15

Infectious diarrhea: ____ is usually caused by contaminated eggs, dairy products, or poulty. ____is usually spread by the fecal-oral route. _____is acquired from eating undercooked poultry or contaminated natural water sources.

Salmonella; Shigella; Campylobacter jejuni

Hypertensive urgency

Severe HTN (>180/110) without signs of end organ damage

Hypertensive emergency

Severe HTN with end-organ damage

What are risk factors for PID?

Sex Age 15-25 STI in partner Previous PID Not using condoms

What are PE findings with Turner syndrome? Labs? Tx?

Short webbed neck No breast development Labs: high FSH Tx: Cyclic estrogen and progesterone

What kind of murmurs are usually benign?

Short, soft, systolic murmurs that are asymptomatic often don't require further investigation

What are the current recommendations for screening for AA?

Single abdominal US in men >65 who have ever smoked followed by contrast CT

Supraventricular Rhythms

Sinus Arrhythmia Sinus Brady Sinus Tachy PACs PSVT SVT due to accessory pathways A fib A flutter Junctional rhythm PJC MAT

What are risk factors for spontaneous abortion?

Smoking Infection Systemic disease Immunologic parameters Drug use

What is key to treatment of arrhythmias?

Stable patients are treated with medicine Unstable patients (dyspnea, AMS, hypotension) are treated with electricity

four etiological agents of pneumonia

Strep pneumo (most common), M. catt, H. influ, Staph aureus (typically following a viral infection)

With pharyngitis, what bacteria is found in rapid strep test? Centor score: absence of cough, fever, tonsillar exudate, tendor/swollen anterior cervical lymph nodes

Strep pyrogenes

What are PACs associated with?

Stress Fatigue Alcohol/tobacco use Caffeine COPD Dig toxicity CAD

_____hematoma involves injury to a vein and symptoms develop later after head injury with headache, confusion, coma, and hemiparesis. Often after acceleration-deceleration injuries.

Subdural

Most common type of Cardiac contusion?

Subendocardial

What is the primary pharmacotherapy for angina?

Sublingual nitroglycerine

Paralytic agent used in Rapid Sequence intubation?

Succinylcholine

How do you treat Measles?

Supportive care - Measles is self-limiting (lasts 7-10 days)

Torsed Gonad Male

Swollen, Firm, high-riding testicle Tansverse ie *Loss of cremasteric reflex* Pain Control Urology consult before ultrasound Reduction

How do you treat Rubella?

Symptomatic treatment

What are clinical signs/symptoms of endometriosis?

Symptoms are related to menses: Dysmenorrhea Dyspareunia Infertility Dyschezia (difficulty passing bowel movement) Intermittent spotting Chronic pelvic pain

WHat are the s/s of central vertigo?

Symptoms with gradual onset but are constant -May have nausea or diaphoresis -Vertical nystagmus

Tx for unstable patient with V. tach who has a pulse?

Synchronized cardioversion

What are clinical features of right sided heart failure?

Systemic vascular congestion JVD Tender or nontender hepatic congestion Decreased appetite/nausea Pitting edema Hepatomegaly

Describe what is happening to valves in Systole vs Diastole?

Systole - Mitral, Tricuspid close (S1), Ventricles contract Aortic and pulmonic valves close (S2), Ventricles fill

which class of antidepressant is amitryptyline (Effexor)

TCAD

T/F All tongue laceration need prophylactic ABX

TRUE

T/F Brown-Sequard lesions affect all three major neural tracts

TRUE

T/F Cardiac enzymes have not predictive value in Dx Commotio Cordis

TRUE

T/F For a partial tear of the urethra 1 attempt at a foley placement may be done

TRUE

T/F In central cord syndrome there is greater motor weakness in the upper extremities than in the lower extremities

TRUE

T/F In the absence of other trauma, sexual assault must always be considered with injuries to the female genitalia

TRUE

T/F Kussmaul sign is seen in pts with Cardiac Tamponade

TRUE

T/F Lumbar puncture is indicated with increased suspicion for subarachnoid hemorrhage and early normal CT scan

TRUE

T/F MRI is often more sensitive to diffuse axonal injury

TRUE

T/F Most intraoral lesions will heal on their own within 2-3 days

TRUE

T/F the GCS is often predictive of outcomes

TRUE

T/F you can pass a foley catheter in a female with a pelvic fx

TRUE

T/F Foreign bodies, such as knives or bullets, should be left in place

TRUE (May be providing local tamponade)

T/F Foreign bodies such as knives or bullets should be left in place

TRUE (may be providing local tamponade)

What are additional diagnostic criteria for PID?

Temp >38.3 (101) Abnormal cervical or vaginal mucopurulent discharge Presence of many WBCs on wet prep Elevated ESR Elevated CRP Gonorrhea or chlamydia test +

How do progestins treat endometriosis?

They inhibit endometrial tissue growth

How do OCPs treat dysmenorrhea?

They inhibit proliferation of endometrial tissue--->less prostaglandins

First Trimester Vaginal Bleeding: ____abortion has closed cervical os with no POCs expelled. ____abortion has open cervical os with no POC's expelled. ____abortion has open cervical os with some POC's expelled. ____is has closed cervical os with all POC's expelled.

Threatened; Inevitable; Incomplete; Complete

How do we define V.Tach?

Three or more PVCs in a row. Can be stable or unstable. Frequent complication of acute MI and dialated cardiomyopathy

What is the best scoring system for deciding when a patient with ACS should undergo aggressive tx?

Thrombolysis In Myocardial Infarction (TIMI) score

DVT

Thrombus embedded in one of the major deep veins of the lower legs,thighs, or pelvis. Unilateral pain, *edema*, tenderness. massive clots may cause ischemia (compression) or arterial spasm Prior DVT, Malignancy, age >40, obesity, estrogen, pregnancy, trauma, catheters, orthopedic surgery are all risk factors

How do you treat resistant primary dysmenorrhea?

Tocolytics CCBs Progestogens

name one arrhythmia that QT prolongation can lead to (this is the reason that QT prolongation is worrisome)

Torsades

Toxicology

Toxin and Antidote Salicylates- Sodium Bicarbonate Acetaminophen- N-acetyl cystine Opiates- Narcan Carbon Monoxide- O2, Hyperbaric oxygen Organophosphates- Atropine, 2-PAM Beta Blockers- Glucagon TCA's- Sodium Bicarbonate

What is syncope?

Transient loss of consciousness accompanied by loss of postural tone, followed by complete resolution without intervention. 50% that present to the ED have no definite etiology.

Transudates vs. Exudates: ______occur when systemic factors that control formation and absorption of pleural fluid are altered. Left sided heart failure/cirrhosis. ______occur when local factors that control formation and absorption of pleural fluid are altered - pneumonia, malignancy, viral infex, PE

Transudates; Exudates

What studies are helpful for diagnosing endometriosis? What is the gold standard?

U/S MRI Laparoscopy with excisional biopsy and confirmatory histology - gold standard

How is NSTEMI/UA managed different from STEMI?

UA/NSTEMI you calculate a TIMI score to determine how aggressive to treat. Conservative management (ASA, Plavix, anticoagulation) is appropriate in low risk patients

a leading cause of morbidity for paralyzed patients and the implicaitons of this

UTI... order a UA on all paralyzed patients

Clinical features of mastitis?

Unilateral tenderness and heat Fever/chills/flu-like symptoms Usually one quadrant or lobule of breast is affected

Vtach

Unstable- cardioversion stable- meds or cardioversion

Virchow's Triad

Venostasis Hypercoagulability Vessel Wall injury

Management of placenta previa?

Watchful waiting in a stable pt Blood transfusion as needed Patients should abstain from sex C section is the preferred method of delivery

What are the special signs associated with AR?

Water-hammer (corrigan) pulse Quincke sign Duroziez sign Musset sign Austin-Flint murmur

Signs of PVD?

Weak femoral or pedal pulses Aortic, iliac, or femoral bruit Skin changes (atrophy, hair loss, pallor) ED with iliac artery disease (Leriche syndrome) Numbness and tingling

Aortic Aneurysm

Weakened and bulging area in the aorta, true aneurysm (all 3 layers), Genetic, structural, metabolic milieu Age >60, atherosclerosis, HTN, smoking, lipids, other vascular disease *97% Infrarenal*

What is an Aortic aneurysm?

Weakness and subsequent dilation of the vessel wall

What is Pilonidal disease?

When hair follicle becomes distended with keratin, folliculitis develops which causes edema and occlusion. The follical ruptures and forms a pilonidal abscess which results in a sinus tract that leads to a deep, subcutaneous cavity

What are common causes of PVCs?

When the myocardium is irritated by factors such as ischemia, electrolyte abnormalities

When do you need surgical management for mastitis?

When there are abscesses or duct ectasia

When should you screen for valvular disease?

When there is a high degree of suspicion when a patient presents with chest pain, heart failure, arrhythmias, congenital abnormalities (ex: Marfans), or hx of rheumatic fever.

What are characteristics of second degree deep burns?

White and dry Tender with or without blisters Most common cause: Hot liquid, steam, flame Heals 14-21 days Some scar May need graft

Tx for witness arrest vs unwitnessed arrest?

Witnessed: Defibrillation first Unwittnessed: Begin CPR first

HTN Work up and Treatment

Work-up: CBC, CMP (creatinine), UA, EKG, Trop, CXR, head CT (based on complaint) Exotica Tx: Arrange F/U in majority of pts Lower SBP 25% in 24 hours Labetolol (IV form is fast onset, oral is spontaneous) Nitrates, Hydralazine, ?benzos

Infectious diarrhea: ____produces a clinic syndrome similar to appendicitis. Spread via fecal-oral route. ___is most commonly acquired from eating undercooked beef. Can cause hemolytic uremic syndrome and thrombotic thrombocytopenic purpura. A protozoa, _______, is one of the principal agents of traveler's diarrhea.

Yersinia; E Coli; Giardia lamblia

Hypotension, tracheal deviation, and elevated jugular venous pressure indicates that a simple pneumothorax has progressed to what? What does CXR show for this?

a tension pneumothorax; lack of lung vascular markings at periphery

the goal of the FAST exam is to assess whether or not there is free fluid in certain locations/compartments... name these locations/compartments.

abdomen, pericardium, thorax; it also assess whether or not there is a pneumothorax

APAP causes hepatotoxicity through an intermediate metabolite when glutathione stores are depleted. Treatment with ___, ___, and supportive care is required when the level is 150mg/dL at 4 hours after ingestion.

activated charcoal and NAC - N-acetylcysteine

65yo caucasian female presents to the ED; she was referred by her PCP d/t a sudden spike in BUN/Cr, otherwise asymptomatic. She has a recent hx of squamous cell carcinoma on her anus and started two chemo drugs recently (cisplatin, 5-FU). BUN/Cr was normal two weeks ago, now 65/4.57. Given this info, what is your dx?

acute kidney injury. I say this to highlight a few things: AKI can be asymptomatic (and one definition of this condition is a sudden spike in BUN/Cr). She was referred to a hospitalist which would likely refer to nephrology (I say this to highlight that this is a condition that needs to be tx'd inpatient). Cisplatin in particularly is supposedly known to be hard on the kidneys

in few words, what is the mgmt of acute supraglotitis

airway mgmt (artificial airway), empiric abx therapy they say that the ADRs of glucocorticoids may outweigh the risks in this condition, bronchodilators (epi) have no utility

mgmt for epiglotitis

airway mgmt is the first thing to consider, usually supplemental oxygen is sufficient may need steroids (i don't think they need to be intubated but i may be wrong); administer abx (etiological agents are Strep or Staph if they have the Hib or HIB vaccine otherwise H. influ); this condition requires hospital admission

27yo presents w/ sinus congestion dx'd clinically as sinusitis. technical terminology is acute rhinosinusitis (i forget to distinguish acute from chronic, as told by a 4 week cut off). this has been persisting for the past week or so and thus was deemed to be bacterial. name the tx of choice for this

amox-clav (not just amox)

why is a tension pneumothorax considered an emergency (so much so of an emergency that it is decompressed presumptively, no time for a CXR)

and quickly progress to respiratory distress

name the types of pneumonia

another slide may say otherwise, trust this one. community acquired, hospital acquired, ventillator acquired, aspiration, PCP

how to tx pneumonia in the ICU

antipneumococcal beta-lactam (ceftriaxone or cefotaxime) + either azythromycin or a respiratory tract fluoroquinolone (RTF)

Pediatrics Cardiovascular topic list

atrial septal defect coarctation of the aorta patent ductus arteriosus tetralogy of fallot ventricular septal defect acute rheumatic fever kawasaki disease hypertrophic cardiomyopathy syncope

abx to use for PNA

azithromycin is the go to. for more complicated cases consider adding augmentin. alternatievly can use levaquin or doxycycline

alcohol stimulates GABA and when ceased get cathecholamine release. consequently, which class of drug can be used to tx alcohol withdrawal, given its similar mechanism of action?

benzos... there is no specific one (although i'm guessing most use lorazepam or diazepam), and you can give any amount, don't need to worry about an overdose

What two diagnostic tools are used to help rule out an ectopic pregnancy? What remains the gold standard for the diagnosis of ectopic pregnancy?

betaHcg & transvaginal ultrasound; Laporoscopy

ask about this medication in any patient that is bleeding

blood thinners to include things like coumadin, heparin, NSAIDS, Eliquis, Xarelto

What does CXR show with pleural effusion? What is the tx?

blunting of costophrenic angle - free pleural fluid on lateral decubitis film; thoracentesis

what does low magnesium do to an ECG

causes QT prolongation (>440ms until proven otherwise) which can lead to cardiac failure

pt presents w/ new onset seizure. you suspect illicit drug use. which illicit drug can cause seizures?

cocaine

one of the most important facets in evaluation of a patient that has had a seizure is to figure out why they have had a seizure. consequently, inquire about a hx of seizures. if they have a hx, non-compliance can be a common cause. similarly, overdose on certain drugs/compounds can cause seizures. name four of these drugs/compounds

cocaine, theophylline, INH, organophosphates; tx like you would any other seizure

three major types of pneumonia

community acquired, hospital acquired, ventillator acquired (I keep forgetting about this one)

hallmark symptoms of concussion

confusion and amnesia, less commonly loss of consciousness

this condition may produce a "seal-like" cough

croup

epiglotitis and croup have a similar presentation and tends to occur in young patients... name some features that distinguishes one from the other

croup produces a barking cough which is absent in epiglotitis, croup tends to produce an inspiratory stridor, epiglotitis tends to produce drooling and pt may assume the tripod position

What is the treatment for retinal detachment to correct the tear?

cryotherapy or photocoagulation

Pulsus Paradoxus

decrease in systolic BP of more than 10mmHg with normal inspiration; palpated as weakened pulse with inspiration along with more heart contractions to pulse beats

utility of general impression

don't forget about the power of this in directing your interview... a 36yo presented to the ED w/ chest pain; I was thinking along the lines of costochondritis or pericarditis d/t age but need to first consider whether or not the patient really looks sick

definition of a positive exercise stress test

drop in BP of 10mmHg systolic, at least 2mm of ST depression (dunno in how many leads)

what is one of the most common causes of coma?

drug overdose. thus when you see a comatose patient, you need to suspect that they overdosed on some drugs

With dysfunctional uterine bleeding, it is important to determine if ovulation has occurred. What is the gold standard to determine if ovulation is occurring?

endometrial biopsy

name two common drugs/hormones that stimulate the SA node

epinephrine, norepinephrine

What will peak flow be in someone with an acute exacerbation of asthma?

exac if < 80% baseline, severe exac if < 50% baseline

Blepharitis is inflammation of the anterior or posterior ____. Treatment is good hygiene and sulfonamide/antistaphylococcal eye ointment or systemic antibiotics.

eyelids

all innocent murmurs are ___ (grade) and ____(timing)

faint: grad 1 - 2/6 early-mid systolic in timing

acute rheumatic fever minor Jones criteria

fever arthralgia elevated acute phase reactants (ESR, CRP) EKG showing heart block (prolonged PR interval)

kawasaki disease Sx (diagnostic criteria)

fever of 5 days duration w/ at least 4 of following: changes in extremities (erythema/swelling of hands/feet, periungal peeling) polymorphous truncal rash non-purulent bilateral conjunctivitis mucositis (strawberry tongue) changes in lips/oral cavity (erythema, cracking lips) cervical LAD > 1.5 cm

ssx of right heart failure

fluid symptoms in the periphery--JVD, hepatosplenomegaly, DOE, peripheral edema

end organ damage can entail four organs; which four?

heart, brain, kidneys, liver

there are four organs that can be assessed for end organ damage, namely in regards to the sepsis continuum. name those four organ and what features indicate end organ damage

heart, brain, liver, kidney heart demonstrated by some sort of cardiomyopathy or vessel involvement such as hypotension brain demonstrated by altered mental status liver demonstrated by LFTs Kidney demonstrated by

which organ systems are primarily affected by electrical shocks

heart, lungs, brain---arrhythmias, respiratory arrest, seizures

For severe barbiturate toxicity, ____may be indicated but is ineffective for benzodiazepines. ____may be used to revere acute benzodiazepine overdose in patients without risk of seizure of benzodiazepine withdrawal.

hemodialysis; Flumazenil

The ____anemias occur from premature destruction of the RBCs in the reticuloendothelial system (extrinsic) or in the blood vessels (intrinsic ). Name some examples of intrinsic and extrinsic factors.

hemolytic Intrinsic: sickle cell, G6PD deficiency Extrinsic: immune, infection, hypersplenism

i have a vague understanding that syncope patients need to be evaluated for cause, namely neurogenic versus cardiogenic. name one neurogenic issue that can cause syncope

hemorrhagic stroke

What is the emergent treatment for arterial embolism/thrombosis (pain, pallor, pulselessness, paresthesias, paralysis)?

heparin, emergency embolectomy-thrombectomy to restore blood flow

What is the tx for retinal detachment?

immediate surgical repair

acute rheumatic fever etiology

immunologic reaction that is a delayed sequela of GABHS infection of the pharynx

overdose patients may be conscious and may have altered mental status... if they do, what is the immediate tx that may reduce this?

in this population need to think about glucose and thiamine, so give em' these

A ____hernia cannot be reduced and a ____hernia occurs when bowel edema compromises the blood flow to an incarcerated hernia. Both can give rise to a small bowel obstruction.

incarcerated; strangulated

Bladder -- the four type of ______are total, stress, urge, and overflow.

incontinence

Low calcium stimulates PTH release then VitD release and then what? High calcium causes PTH decrease then Calcitonin release and then what?

increased absorption to bone; enhances bone deposition and renal excretion of calcium

physiology of transudative pleural effusion

increased hydrostatic pressure or decreased oncotic pressure of the pulmonary vasculature causes the translocation of fluid pleural:serum protein <0.5 pleural:resum LDH <0.6 pleural fluid LDH <2/3 upper limit of normal

coarctation of the aorta Tx

infant in shock > give PGE1 immediately surgical resection transcatheter repair

what causes aspiration pneumonitis

inflammatory chemical injury of the tracheobronchial tree and pulmonary parenchyma produced from the inhalation of regurgitated gastric contents can lead to aspiration pneumonia: n alveolar space infection resulting from the inhalation of pathogenic material from the oropharynx.

25yo male presents to the ED after having a stool broken over his head. obvious laceration on forehead. evaluate this patient (include everything)

inquire about vision disturbances (blurry, double vision), HA (progressive suggestive of hemorrhage which is the thing that you need to be concerned w/ here), changes in motility/speech/memory (amnesia is pathonomonic for concussion), loss of consciousness, be sure to do a neuro exam (cardio and respiratory for all), check orientation to person place time, need to do a CT w/o contrast to rule in/out hemorrhage, need to repair that laceration (should give pain killer for the actual procedure, don't discharge w/ any), need to inquire about tetanus status for all laceration/cut/bite patients (inquire about tingling sensation at site of trauma)

there are a number of severe pediatric airway conditions. when you see a kid in respiratory distress, it is important to differentiate if the noise (wheeze, stridor, etc) is inspiratory or expiratory as this helps to make a differential. name one inspiratory and two expiratory conditions

inspiratory suggests upper airway obstruction such as croup (i'm also guessing acute epiglotitis); expiratory suggests asthma or bronchiolitis

GCS < ______ you should intubate

less than 8 - "Less than eight, Intubate"

approach to mgmt of stable angina pts

look at the modifiable risk factors... lipid panel, fasting glucose, stress test

i know that benzos are the first line of tx for seizures. namely, which benzo?

lorazepam

How should BP be lowered in someone with malignant HTN?

lower DBP by 10% first hour, then 15% next 3-12 hours, IV nitroprusside or lebatalol

ventricular septal defect; muscular vs membranous which is more common?

membranous

HR range for each: mild, moderate, severe bradyarrhythmia

mild is 40-60, moderate 20-40, severe is less than 20

how is an opioid overdose treated?

narcan/naloxone. you know that it works when patients comes out of coma and may be stuporous, pupils will dilate

46yo white male w/ hx of DM and chronic pancreatitis presents w/ episodes of vomiting that started yesterday, seems suspicious for opioid withdrawal, DKA or exacerbation of pancreatitis. given this information, what is the indication for ordering a troponin?

need to keep a high index of suspicion MI in DM patients as they can have an atypical presentation. this also applies to females and elderly

how is a wide complex Vtach defined

need to look at the start of the R (just before the upward deflection begins) to the nadir of the S (aka the peak); if this is greater than 100ms in one precordial lead (v1-v6) then it's a wide complex

hemorrhagic stroke mgmt

need to stop bleeding so get a prompt neurosurgery consult and they'll handle it. if they're on blood thinners (warfarin) need to give some vitamin K and FFP

What are the classic EKG findings in hyperkalemia? Stabilize the cardiac membrane with what? Remove potassium with Kayexalate or furosemide. Shift potassium into cells with insulin/glucose, sodium bicarb, and albuterol

peaked T waves, prolonged PR, loss of P waves, widening QRS; calcium chloride or calcium gluconate

Pt comes in w/ complaint of recent onset CP. Troponin negative, EKG demonstrates peaked T waves and PR depression in aVR. what does this indicate?

pericarditis... can present w/ peaked T waves, PR elevation/depression in aVR

The treatment of isopropyl alcohol ingestion and ethanol ingestion is largely ____. The hallmark of isopropyl alcohol ingestion is ketosis without acidosis.

supportive

tetralogy of fallot Tx

surgical correction for "tet spells" -holding baby in knee chest position (vagal maneuver) -morphine -oxygen -beta-blocker -general anesthesia -sodium bicarbonate

what are some common symptoms of a hemorrhagic stroke

syncope, seizure, severe HA (I saw one of these in the ED, the guy was feeling dizzing and had a really bad HA, he also had a SBP of 210)

acute epiglotitis tends to occur in pediatric patients and looks like a number of other respiratory conditions... name at least one key feature that may distinguish it from others

tends to produce drooling as these kids can't swallow; may also expect to see tripod position

everything about epiglotitis (according to Tintinalli text)

the kid will look anxious, tripod position, may be drooling, will not be coughing (this may differentiate it from Croup, diphtheria). it is acceptable to try to view the epiglotits (some people will say don't do think b/c it can injure them but threres no data to support this claim). get an xray to check for thumbprint sign (the white part of the xray will look like a thumb), if this is negative but clinical suspicion remains can directly visualize the epiglotis. airway management is the most important thing, put the kid in an upright position, give oxygen, nebulized racemic epi, intubate, second or third generation cephalosporin (cefuroxime, ceftriaxone) for 7-10 days, steroids

what is the utility of pulmonary angiography in PE?

this is considered the gold standard for PE but it is more invasive and a spiral CT does a pretty good job of detection em', so if the spiral CT comes back negative but clinical suspicion remains high then consider pulmonary angiography

two things about general impression

this is important and should be the first thing that you consider to guide your diagnosis (i.e. patient has CP and looks toxic start thinking MI right away rather than asking about other things), get an idea for age because elderly patients may have an abnormal presentation

word of caution for interpretting QT prolongation

this is rate dependent and can thus be masked. algorithms exist to check for it and if i recall correctly this is the corrected QT. this all goes to say that you need to keep a high index of suspicion especially for tachycardia or anything close to it

what is the Centor criteria? What is it good for?

this is used to predict whether or not a sore throat is bacterial in origin, has a terrible positive predictive value (thus if they have these things you should still order a rapid strep to be sure) but a good negative predictive value (thus if they don't have these things they probably do not have strep) criteria includes hx of fever, absence of cough, tonsilar exudate (exudate is not the same is ulcers), anterior cervicular lymphadenopathy

all about ARDS

this looks a lot like CHF--SOB, low oxygen sat, crackles on lungs. this is giong to occur in patients that are sick w/ something serious such as sepsis or some other major insult such as nearly drowning. get a CXR which demonstrates fluid. PCWP will be normal (around 10) but realistically don't need to check this, instead get BNP and echo both of which will be normal which differentiates it from CHF. tx is aimed at resolving the underlying etiology, need to get oxygen sat up using PEEP

pt involved in a MVC comes to ED; hypotensive, afebrile, you notice JVD and hear muffled heart sounds... what do you need to do next?

this sounds a lot like a cardiac tamponade (Beck's triad of hypotension, muffled heart sounds, JVD; typically accompanied by a hx of trauma). next step is to get an echo

kawasaki disease Dx

thrombocytosis elevated ESR note: seen in 2nd week

utiliity of head CT in pt suspected of having a stroke

to check for things that would contraindicate tPA use such as tumors, bleeds, aneurysms... it's role is not to rule in a stroke as many cannot be seen on CT, thus diagnosis is more so clinical

atrial septal defect Sx

usually asymptomatic exercise intolerance when older paradoxical embolism > stroke arrhythmias pulmonary obstructive vascular dz in 3rd or 4th decade systemic ejection murmur in the LUSB (pulmonic area) middiastolic rumble in the RLSB "fixed" wide split S2 right ventricular heave

traditionally the leading etiological agent of epiglotitis but thanks to modern medicine there's a vaccine against it

was Haemophilus influenza, but now there's an HIB vaccine and now other causes are Staph and Strep

keep a high index of suspicion for SIRS in the ED... what is the SIRS criteria?

whacky temp (less than 36 or greater than 38), HR greater than 90, tachypnea of greater than 20, whacky WBC of less than 4k or greater than 12k or greater than 10% immature neutrophils aka bands

a note about back pain

while gathering the HPI be sure to inquire about hx of CA (regardless of nature of complaint) b/c back is most common site of mets... could also inquire about TB

What two meds should be given to all ACS patients that do not have contraindications?

BB - unless brady or severe COPD - then do NDCCB (verapamil/diltiazem) ACEi - if cough, use ARB

Multifocal atrial tachycardia - noted in patients with COPD or severe systemic illness - EKG shows multiple shaped P waves and differing PR intervals. ____are agents of choice?

CCB

BLOCKS 1. ____=prolonged PR interval 2.____=progressive increase in PR until Pwave is blocked. 3._____=sudden block in P wave w no change in PR 4._____=atrial and ventricular rhythm are independent of each other.

First degree; Wenckebach Mobitz type I; Mobitz type II, Third degree block

Acute bronchitis is usually caused due to ____ie. Adenovirus, Influ A or B, Coronavirus, Rhinovirus, RSV. ______causes include H flu, Mycoplasma pneumonia, M cat, Chlamydia pneumo, or Strep pneumo.

Viruses; Bacterial

Hypertrophic cardiomyopathy: is due to hypertrophy of the _____. PE reveals mitral regurgitation, a ____heart sound, and prominent left ventricular impulse. EKG reveals LVH

cardiac septum; S4


Set pelajaran terkait

Information Retention and Access

View Set

Study For AP World History Final

View Set

ch 12 ( nursing care of patients having surgery)

View Set